You are on page 1of 505

American board of family medicine QS (2014 - 20)

&
Pretest of family medicine QS (2019)

• This collection is maid as an aid to help you study the WADA topics.
• It is IMPORTANT & A MUST to review the recent guidelines as
some of the questions are 4-5 years back.
• Done by the trainees of MOH family medicine program in Taif city.

"The third edition"


‫"الشكر لكل من ساهم وشارك في إعداد هذا العمل وهم‪"- :‬‬
‫‪ )1‬د ‪.‬محمد السفياني (جميع اإلصدارات)‬
‫‪ )2‬د‪ .‬هاجر الوقداني (جميع اإلصدارات)‬
‫‪ )3‬د ‪.‬جواهر الثمالي (جميع اإلصدارات)‬
‫‪ )4‬د ‪.‬داليا القثامي (جميع اإلصدارات)‬
‫‪ )5‬د ‪.‬نوره السميري (جميع اإلصدارات)‬
‫‪ )6‬د ‪.‬وداد الثبيتي (اإلصدار الثاني والثالث)‬
‫‪ )7‬د ‪.‬غالي الحازمي (اإلصدار الثاني)‬
‫‪ )8‬د‪ .‬أسرار الزهراني (اإلصدار الثاني)‬
‫‪ )9‬د ‪.‬مي القثامي (اإلصدار الثاني)‬
‫‪ )10‬د ‪.‬سماهر الصخيري (اإلصدار الثاني)‬
‫‪ )11‬د ‪.‬أفراح اللهبي (اإلصدار الثاني)‬
‫‪ )12‬د‪ .‬أحمد اللهيبي (اإلصدار الثالث)‬
‫‪ )13‬د‪ .‬ساره بن باز (اإلصدار الثالث)‬
‫‪ )14‬د‪ .‬وعد مليباري (اإلصدار الثالث)‬
‫‪ )15‬د‪ .‬رغد مندورة (اإلصدار الثالث)‬
‫‪ )16‬د‪ .‬غيث الحازمي (اإلصدار الثالث)‬
‫‪ )17‬د‪ .‬رائد المالكي (اإلصدار الثالث)‬

‫للمالحظات واالقتراحات‪moh.fm2018@gmail.com :‬‬


Anemia ............................................................................................................................................................ 9
ABFM 2020 ................................................................................................................................................ 9
ABFM 2019 .............................................................................................................................................. 13
ABFM 2018 .............................................................................................................................................. 16
ABFM2017 ............................................................................................................................................... 26
ABFM 2016 .............................................................................................................................................. 28
ABFM2014 ............................................................................................................................................... 31
Bronchial asthma .......................................................................................................................................... 33
Pretest2109 ............................................................................................................................................... 33
ABFM 2020 .............................................................................................................................................. 41
ABFM2019 ............................................................................................................................................... 42
ABFM2018 ............................................................................................................................................... 44
ABFM2017 ............................................................................................................................................... 46
ABFM2014 ............................................................................................................................................... 50
COPD ............................................................................................................................................................ 54
PRETEST2019 ......................................................................................................................................... 54
ABFM 2020 .............................................................................................................................................. 58
ABFM2019 ............................................................................................................................................... 61
ABFM2018 ............................................................................................................................................... 63
ABFM2017 ............................................................................................................................................... 67
ABFM2015 ............................................................................................................................................... 69
ABFM2014 ............................................................................................................................................... 71
Depression .................................................................................................................................................... 74
PRETEST2019 ......................................................................................................................................... 74
ABFM 2020 .............................................................................................................................................. 77
ABFM2019 ............................................................................................................................................... 78
ABFM2017 ............................................................................................................................................... 80
ABFM2016 ............................................................................................................................................... 84
ABFM2014 ............................................................................................................................................... 87
Dermatological problems.............................................................................................................................. 89
ABFM 2020 ............................................................................................................................................ 108
ABFM2019 ............................................................................................................................................. 113
ABFM2018 ............................................................................................................................................. 120
ABFM2107 ............................................................................................................................................. 130
ABFM2016 ............................................................................................................................................. 142
ABFM2015 ............................................................................................................................................. 149
ABFM2014 ............................................................................................................................................. 155
DM .............................................................................................................................................................. 163
RETEST2019 .......................................................................................................................................... 163
ABFM 2020 ............................................................................................................................................ 173
ABFM2019 ............................................................................................................................................. 177
ABFM2017 ............................................................................................................................................. 191
ABFP2015 .............................................................................................................................................. 196
ABFM2014 ............................................................................................................................................. 203
Dyslipidemia ............................................................................................................................................... 209
PRETEST2019 ....................................................................................................................................... 209
ABFM 2020 ............................................................................................................................................ 214
ABFM2016 ............................................................................................................................................. 215
ABFM2015 ............................................................................................................................................. 217
Dyspepsia .................................................................................................................................................... 219
ABFM 2020 ............................................................................................................................................ 219
ABFM2019 ............................................................................................................................................. 220
ABFM2017 ............................................................................................................................................. 223
GAD ............................................................................................................................................................ 224
ABFM 2020 ............................................................................................................................................ 224
ABFM2017 ............................................................................................................................................. 225
ABFM2014 ............................................................................................................................................. 226
Headache..................................................................................................................................................... 227
ABFM 2020 ............................................................................................................................................ 227
ABFM2019 ............................................................................................................................................. 228
ABFM2017 ............................................................................................................................................. 229
Heart failure ................................................................................................................................................ 231
Pretest 2019 ............................................................................................................................................ 231
ABFM 2020 ............................................................................................................................................ 235
ABFM2019 ............................................................................................................................................. 237
ABFM2018 ............................................................................................................................................. 239
ABFM2016 ............................................................................................................................................. 241
ABFM2015 ............................................................................................................................................. 242
ABFM2014 ............................................................................................................................................. 242
Hypertension ............................................................................................................................................... 245
PRETEST2019........................................................................................................................................ 245
ABFM 2020 ............................................................................................................................................ 252
.ABFM2019 ............................................................................................................................................ 257
ABFM2018 ............................................................................................................................................. 261
ABFM2017 ............................................................................................................................................. 268
ABFM2016 ............................................................................................................................................. 272
ABFM 2015 ............................................................................................................................................ 277
Coronary Artery Disease ............................................................................................................................ 281
Pretest 2019 ............................................................................................................................................ 281
ABFM 2020 ............................................................................................................................................ 285
ABFM2019 ............................................................................................................................................. 289
ABFM2018 ............................................................................................................................................. 291
ABFM2016 ............................................................................................................................................. 301
ABFM2015 ............................................................................................................................................. 307
Nocturnal enuresis ...................................................................................................................................... 312
Pretest 2019 ............................................................................................................................................ 312
ABFM2016 ............................................................................................................................................. 315
ABFM2015 ............................................................................................................................................. 316
Obecity ........................................................................................................................................................ 317
Pretest 2019 ............................................................................................................................................ 317
ABFM 2020 ............................................................................................................................................ 322
ABFM 2018 ............................................................................................................................................ 323
ABFM 2016 ............................................................................................................................................ 324
ABFM 2015 ............................................................................................................................................ 326
ABFM 2014 ............................................................................................................................................ 327
STD ............................................................................................................................................................. 330
Pretest2019 ............................................................................................................................................. 330
ABFM2018 ............................................................................................................................................. 333
ABFM2015 ............................................................................................................................................. 334
Pnuemonia .................................................................................................................................................. 336
Pretest2019 ............................................................................................................................................. 336
ABFM 2020 ............................................................................................................................................ 337
ABFM2019 ............................................................................................................................................. 338
ABFM 2018 ............................................................................................................................................ 343
ABFM2017 ............................................................................................................................................. 344
ABFM 2015 ............................................................................................................................................ 347
ABFM 2014 ............................................................................................................................................ 352
Thyroid ....................................................................................................................................................... 358
Pretest2019 ............................................................................................................................................. 358
ABFM 2020 ............................................................................................................................................ 361
ABFM2019 ............................................................................................................................................. 364
ABFM2018 ............................................................................................................................................. 366
ABFM2017 ............................................................................................................................................. 368
ABFM2015 ............................................................................................................................................. 371
ABFM2014 ............................................................................................................................................. 374
URTIs ......................................................................................................................................................... 378
Pretest 2019............................................................................................................................................. 378
ABFM 2020 ............................................................................................................................................ 385
ABFM2019 ............................................................................................................................................. 387
ABFM 2018 ............................................................................................................................................ 392
ABFM2016 ............................................................................................................................................. 397
ABFM2015 ............................................................................................................................................. 398
ABFM2014 ............................................................................................................................................. 402
LRTI+ COVID-19 ...................................................................................................................................... 407
ABFM 2018 ............................................................................................................................................ 407
ABFM 2016 ............................................................................................................................................ 410
Vaginal discharge+ Abnormal Vaginal Bleeding: ...................................................................................... 411
ABFP 2020 ............................................................................................................................................. 411
ABFP 2018 ............................................................................................................................................. 412
ABFP 2017 ............................................................................................................................................. 413
ABFP 2016 ............................................................................................................................................. 413
Menstrual cycle + Menstrual abnormalities ............................................................................................... 414
ABFP 2020 ............................................................................................................................................. 414
ABFP 2018 ............................................................................................................................................. 415
ABFP 2017 ............................................................................................................................................. 418
ABFP 2016 ............................................................................................................................................. 418
Autism: ....................................................................................................................................................... 421
ABFM 2020 ............................................................................................................................................ 421
ABFM 2018 ............................................................................................................................................ 421
ABFM 2017 ............................................................................................................................................ 422
ABFM 2016 ............................................................................................................................................ 422
Abdominal Pain Approach ......................................................................................................................... 423
2019 pretest ............................................................................................................................................. 423
ABFM 2020 ............................................................................................................................................ 424
ABFM 2018 ............................................................................................................................................ 426
ABFM 2016 ............................................................................................................................................ 430
Irritable bowel disease ................................................................................................................................ 432
ABFM 2020 ............................................................................................................................................ 432
ABFM 2018 ............................................................................................................................................ 432
ABFM 2016 ............................................................................................................................................ 433
Contraception .............................................................................................................................................. 434
ABFM 2020 ............................................................................................................................................ 434
ABFM 2018 ............................................................................................................................................ 434
ABFM 2017 ............................................................................................................................................ 435
ABFM 2016 ............................................................................................................................................ 437
Irritable bowel syndrome: ........................................................................................................................... 439
ABFM 2018 ............................................................................................................................................ 439
ABFM 2017 ............................................................................................................................................ 440
ABFM 2016 ............................................................................................................................................ 441
Well Baby Care + Infantile Colic ............................................................................................................... 442
ABFM 2020 ............................................................................................................................................ 442
ABFM 2018 ............................................................................................................................................ 444
ABFM 2016 ............................................................................................................................................ 446
Gastroenteritis ............................................................................................................................................. 446
ABFM 2020 ............................................................................................................................................ 446
ABFM 2017 ............................................................................................................................................ 448
ABFM 2016 ............................................................................................................................................ 448
Adrenal gland disorders .............................................................................................................................. 450
2019 pretest ............................................................................................................................................. 450
ABFM 2020 ............................................................................................................................................ 450
ABFM 2018 ............................................................................................................................................ 451
ABFM 2017 ............................................................................................................................................ 453
Obsessive – compulsive disorder................................................................................................................ 454
Peripheral vascular disease ......................................................................................................................... 454
ABFM 2020 ............................................................................................................................................ 454
Palpitation ................................................................................................................................................... 455
ABFM 2020 ............................................................................................................................................ 455
Rheumatoid Arthritis .................................................................................................................................. 457
ABFM 2019 ............................................................................................................................................ 457
Approach to head truma with CT SS .......................................................................................................... 458
ABFM 2020 ............................................................................................................................................ 458
Approach to Limping Child ........................................................................................................................ 459
ABFM 2020 ............................................................................................................................................ 459
Failure to thrive and Short Stature .............................................................................................................. 460
ABFM 2017 ............................................................................................................................................ 460
Children Emergencies ................................................................................................................................. 463
2019 pretest ............................................................................................................................................. 463
ABFM 2020 ............................................................................................................................................ 463
CSF analysis ............................................................................................................................................... 466
None ........................................................................................................................................................ 466
Antenatal care ............................................................................................................................................. 466
ABFM 2020 ............................................................................................................................................ 466
Infertility ..................................................................................................................................................... 468
None ........................................................................................................................................................ 468
Menopause .................................................................................................................................................. 468
ABFM 2020 ............................................................................................................................................ 468
Semen analysis............................................................................................................................................ 470
ABFM 2019 ............................................................................................................................................ 470
Nail disorders .............................................................................................................................................. 471
None ........................................................................................................................................................ 471
Hair disorders.............................................................................................................................................. 471
ABFM 2020 ............................................................................................................................................ 471
Liver function test interpretation + Hepatitis markers ................................................................................ 472
ABFM 2020 ............................................................................................................................................ 472
ABFM 2017 ............................................................................................................................................ 474
Stool Analysis & Cultures .......................................................................................................................... 475
None ........................................................................................................................................................ 475
Dizziness ..................................................................................................................................................... 476
Pretest2019............................................................................................................................................. 476
ABFM2020 ............................................................................................................................................. 479
ABFM2019 ............................................................................................................................................. 480
ABFM2018 ............................................................................................................................................. 481
Eyelid & Lacrimal disorders ....................................................................................................................... 482
ABFM2020 ............................................................................................................................................. 482
ABFM2019 ............................................................................................................................................. 482
Eyelid & Lacrimal disorders ....................................................................................................................... 484
ABFM2020 ............................................................................................................................................. 484
Otitis media ................................................................................................................................................. 487
ABFM2018 ............................................................................................................................................. 489
ABFM2015 ............................................................................................................................................. 490
ABFM2014 ............................................................................................................................................. 491

x RedABFM2020
Eye ....................................................................................................................................................... 492
............................................................................................................................................. 492
Pretest2019 ............................................................................................................................................. 492
ABFM2019 ............................................................................................................................................. 497
ABFM2018 ............................................................................................................................................. 497
ABFM2017 ............................................................................................................................................. 498
ABFM2016 ............................................................................................................................................. 499
ABFM2015 ............................................................................................................................................. 500
ABFM2014 ............................................................................................................................................. 501
Anemia
ABFM 2020
177. A68-year-old female with a history of diabetes mellitus, hypertension and heart failure
with preserved ejection fraction presents with along-standing diabetic foot ulcer. The patient
reports no signs of any gastrointestinal bleeding, no blood in her urine, no bleeding gums, and
no vaginal bleeding. Her medications include metformin (Glucophage),insulin
glargine(Lantus), lisinopril (Prinivil, Zestril), atorvastatin (Lipitor), and furosemide(Lasix).
A CBC reveals a WBC count of 7600/mm3 (N 4500–11,000), a hemoglobin level of 9.7 g/dL
(N 14.0–17.5), a mean corpuscular volume of 89 m3 (N 80–100), and a platelet count of
412,000/mm3 (N 150,000–400,000).
To further assess the patient’s anemia, you obtain the following laboratory results:
• Ferritin 293 ng/mL (N 22–275)
• Serum iron 43 ng/dL (N 50–175)
• Transferrin 190 mg/dL (N 177–264)
• Reticulocyte count 3.2% (N 0.5–1.5)
• VitaminB12 564 pg/mL (N 230–1050)
• Haptoglobin 198mg/dL(N63–273)
Which one of the following is the most likely cause of her anemia?
1. Anemia of chronic disease
2. Bone marrow suppression
3. Hemolysis
4. Iron deficiency
5. Vitamin B12 deficiency
ANSWER: A
This patient has findings most consistent with anemia of chronic disease, also known as
anemia of inflammation. This condition is thought to be primarily a disorder of iron
distribution in response to systemic inflammation, which also biases hematopoiesis toward
myeloid cell production rather than erythropoiesis and shortens the erythrocyte lifespan.
Anemia of chronic disease is a normocytic and normochromic anemia. Iron studies typically
show evidence of iron restriction without systemic iron deficiency. A common challenge in
diagnosis is when true iron deficiency coexists with anemia of chronic disease.
This patient’s normal WBC and platelet counts make bone marrow suppression less likely.
The normal haptoglobin level and low reticulocyte count are not consistent with hemolysis.
She has a normocytic rather than microcytic anemia and her ferritin level is elevated. These
two factors make iron deficiency less likely despite her low serum iron level. The low normal
transferrin level is also consistent with anemia of chronic disease rather than iron deficiency.
Her normal vitamin B12 level makes a deficiency unlikely. Her history of a chronic foot ulcer
and elevated inflammatory markers (ferritin and platelets) are consistent with anemia of
chronic disease.
Which one of the following oral iron preparations is most effective for the treatment of iron
deficiency anemia in a patient with non–dialysis-dependent chronic kidney disease?
A. Ferric citrate (Auryxia)
B. Ferrous fumarate
C. Ferrous gluconate
D. Ferrous sulfate
ANSWER: A Oral iron supplements, including ferrous fumarate, ferrous gluconate, and
ferrous sulfate, are generallyineffective when used by hemodialysis patients and are only
modestly effective when used by patients with non–dialysis-dependent chronic kidney disease.
The one exception is ferric citrate, which is highly efficacious in all patients with chronic
kidney disease. Intravenous iron preparations such as iron dextran are similarly effective in
both groups and are considered the gold standard for the treatment of iron deficiency in
patients on chronic hemodialysis.

141. A35-year-old female presents with fatigue. She follows a strict vegan diet and does not
take any medications. There is no history of any abnormal bleeding. Although she has had
fatigue for 2 years, she now notices that she has a craving for ice. Results of a laboratory
evaluation are shown below.
WBCs 5100/mm3 (N 3400–10,800)
Basophils 0.0 × 103/ L (N 0.0–0.2)
Immature granulocytes 0%
Immature granulocyte count 0.0 × 103/ L (N 0.0–0.1)
Platelets 271,000/mm3 (N 150,000–379,000)
RBCs 4.75 million/mm3 (N 3.77–5.28)
Reticulocytes 1.2% (N 0.6–2.6)
Hemoglobin 9.2 g/dL (N 11.1–15.9)
Hematocrit 32.3% (N 34.0–46.6)
Mean corpuscular volume 68 m3 (N79–97)
Mean corpuscular hemoglobin 19.4 pg/cell (N 26.6–33.0)
Mean corpuscular hemoglobin concentration 28.5g/dL(N31.5–35.7)
RDW 18.1% (N 12.3–15.4)
Total iron binding capacity 519 g/dL (N 250–450)
Unsaturated iron binding capacity 482 g/dL (N 131–425)
Iron 37 g/dL (N 27–159)
Iron saturation 7% (N 15–55)
Transferrin 405mg/dL(N200–370)
Ferritin 4 ng/mL (N 15–150)
Vitamin B12 353 pg/mL (N 160–950)
Given these results, which one of the following would be the next step in the management of
this patient?
A. Recheck her hemoglobin level in 3 months
B. Start ferrous sulfate, 325 mg orally
C. Order an intravenous iron infusion
D. Order a blood transfusion
E. Refer for colonoscopy
ANSWER: B
This patient has symptoms and laboratory findings consistent with iron deficiency. Because
she is premenopausal, excessive menstrual bleeding is a possible etiology, as is insufficient
iron intake, given her strict vegan diet. The best first step is to replace her iron stores with an
oral replacement. Although a hemoglobin level should be rechecked in 3 months, this should
happen after the initiation of iron therapy. Intravenous iron infusion is reserved for patients
who cannot tolerate oral therapy. This patient does not require a blood transfusion at this time.
If she does not respond to iron therapy, and common causes such as dietary restriction and
menstrual bleeding are excluded, then she should be referred for colonoscopy to search for a
gastrointestinal cause of blood loss.

142. In a patient on chronic warfarin (Coumadin) therapy who has a stable INR in the
therapeutic range, which one of the following antibiotics would be most likely to elevate the
INR?
A. Cephalexin (Keflex)
B. Clindamycin (Cleocin)
C. Penicillin G
D. Rifampin (Rifadin)
E. Trimethoprim/sulfamethoxazole (Bactrim)

ANSWER: E
Trimethoprim/sulfamethoxazole is one of the antimicrobials most likely to increase the INR of
a patient taking warfarin. If trimethoprim/sulfamethoxazole is used in a patient on warfarin,
reducing the warfarin dosage by 25%–40% is recommended, with close monitoring of the
INR. The patient’s INR should be checked within 3–5 days of starting or stopping any
antimicrobial. First generation cephalosporins such as cephalexin, fourth generation
cephalosporins, clindamycin, and penicillin G have a lower likelihood of affecting the INR.
Rifampin decreases the INR and the warfarin dosage should be increased if rifampin is started.
Other antimicrobials that significantly affect the INR include metronidazole and fluconazole.
Azithromycin, ciprofloxacin, clarithromycin, and levofloxacin may impact the INR with a
variable patient-specific effect.
128. A 24-year-old male with sickle cell disease (HbSS) presents to the emergency department
(ED) with a 5-day history of severe pain in his lower back, legs, and shoulders. The review of
systems is positive for jaundice and scleral icterus, but is negative for headache, chest pain,
shortness of breath, and priapism. He reports that his symptoms are typical of his previous
vaso-occlusive pain, and he believes this episode was triggered by the recent cold weather. He
tried to avoid coming to the ED by using oral medication at home, including naproxen, 500
mg twice daily, and immediate-release oxycodone (Roxicodone), 10 mg every 4 hours as
needed. He takes no other medications and reports no known drug allergies. He is pain free on
approximately 75% of days, and he can typically manage his pain at home during the
remaining times. The patient’s vital signs are unremarkable except for a mildly elevated pulse
rate of 102 beats/min. His WBC count is 17,000/mm3 (N 5000– 10,000) and his hemoglobin
level is at its baseline of 8.0 g/dL. The patient is admitted to the hospital for intravenous fluids
and pain medications via patient-controlled analgesia. This is his second admission of the year
for a vaso-occlusive pain crisis. He responds well over the next few days and is discharged to
home in good condition.
Which one of the following should be discussed with the patient at follow-up?
A. Daily hydroxyurea therapy
B. Oral penicillin prophylaxis
C. Switching from immediate-release oxycodone to long-acting oxycodone (OxyContin) twice
daily
D. Annual transcranial Doppler ultrasonography
E. Scheduled blood transfusions to maintain a hemoglobin level >10 g/dL
ANSWER: A In a clinical trial of patients with sickle cell disease, those taking hydroxyurea
had two fewer severe vaso-occlusive pain crises per year, and they also had a decreased need
for blood transfusions. Hydroxyurea is also the only medication that has been shown to
prevent acute chest syndrome (ACS) in sickle cell disease, with a number needed to treat of 6
to prevent an episode of ACS over a 21-month period. Hydroxyurea therapy has traditionally
been recommended for patients who have three or more severe vaso-occlusive pain crises per
year, or for those who have daily pain that affects their quality of life. However, given the
risk-to- benefit profile of the medication, current guidelines now recommend offering
hydroxyurea to reduce complication rates for all patients older than 9 months of age with
sickle cell anemia (SOR B). Oral penicillin prophylaxis to prevent pneumococcal sepsis is
indicated for young children (typically up until age 5), not young adults. Similarly,
transcranial Doppler ultrasonography to screen for stroke is performed in children and
adolescents. This adult patient, who has no headache or other worrisome symptoms, does not
require screening. This patient does not have daily pain or pain on the majority of days, so he
does not need daily long-acting opioid therapy. Blood transfusions in asymptomatic patients
can lead to iron overload in patients with sickle cell disease. There is no consensus regarding
the hemoglobin level that should automatically prompt a blood transfusion in these patients.
ABFM 2019
Q1. Which one of the following is the most common cause of iron deficiency anemia in
premenopausal women?
A. Blood donation
B. Peptic ulcer disease
C. Hematuria
D. Abnormal uterine bleeding
E. Colon cancer
ANSWER: D
Abnormal uterine bleeding is the most common cause of iron deficiency anemia in
premenopausal women, accounting for 20%–30% of cases. Gastrointestinal causes are less
common but should be considered if the gynecologic evaluation is normal or the anemia fails
to resolve with iron supplementation. Blood donation and hematuria are less common causes.

Q2- A 45-year-old African-American male with a several-month history of fatigue and


anorexia with weight loss is found to be anemic, with a hemoglobin level of
9.2 g/dL (N 14.0–18.0). Which one of the following microscopic findings on a peripheral
blood smear would be most characteristic of vitamin B12 deficiency anemia in this patient?
A. Microcytic, hypochromic RBCs
B. Crescent- or sickle-shaped RBCs
C. Basophilic stippling of RBCs
D. Hyper-segmented polymorphonuclear WBCs
E. Schistocytes
ANSWER: D
Vitamin B12 deficiency, which is most commonly related to a deficiency of intrinsic factor that
is produced by the gastric mucosa and is necessary for absorption of vitamin B12 in the
terminal ileum, is manifested as a megaloblastic macrocytic anemia. In addition to an elevated
mean corpuscular volume, the classic finding on microscopy of the peripheral smear is the
presence of multiple (usually five or more) segments in the WBC nuclei. Vitamin B12 levels
are low and a high methylmalonic acid level helps to confirm the diagnosis. A methylmalonic
acid level is recommended in patients who have borderline low vitamin B12 levels but are at
risk for deficiency. Risk factors include chronic proton pump inhibitor use; chronic metformin
therapy; chronic malnutrition due to alcoholism, chronic gastritis, or peptic ulcer disease; and
diseases of the terminal ileum, such as Crohn’s disease. People who have had gastric bypass
surgery are also at risk for vitamin B12 deficiency, which is why supplementation should be
provided in these patients. Microcytic, hypochromic RBCs are typical of iron deficiency
anemia but may also be seen in anemia of chronic disease. Sickled or crescent-shaped cells are
seen with sickle cell anemia, and basophilic stippling may be seen in anemia associated with
lead toxicity. Schistocytes are seen in hemolytic anemia.

Q3-A 67-year-old male presents for a Medicare wellness visit. He underwent basic laboratory
work prior to the office visit. He is feeling well and does not have any concerns or symptoms.
His blood pressure is 127/76 mm Hg, his heart rate is 64 beats/min, and he is afebrile. A
comprehensive metabolic panel is unremarkable. A CBC shows the following results:
WBCs 7500/mm3 (N 4100–10,900)
RBCs 4.05 million/mm3 (N 4.70–6.10)
Hemoglobin 12.9 g/dL (N 14.0–18.0)
Hematocrit 39% (N 42–52)
Mean corpuscular volume 82 m3 (N 80–95) Platelets 197,000/mm3 (N 130,000–448,000)
Which one of the following would be the most appropriate next step in the workup of this
patient?

A. A serum ferritin level


B. A serum transferrin receptor–ferritin index
C. Oral iron supplementation, and a repeat CBC in 4 weeks
D. Referral to a gastroenterologist
ANSWER: A
Anemia is often diagnosed incidentally on laboratory testing and is often asymptomatic. It is
associated with increased morbidity and mortality in older adults, and is often caused by
nutritional deficiencies, chronic kidney disease, occult blood loss from gastrointestinal
malignancies, or chronic inflammation. However, in many patients the cause remains
unknown. A detailed history and physical examination are indicated. In patients with
normocytic or microcytic anemia, a serum ferritin level should be ordered. A low serum
ferritin level is associated with iron deficiency and should be further evaluated so the
underlying cause can be addressed. A serum transferrin-receptor–ferritin index should be
determined for patients with a serum ferritin level between 46 and 100 ng/mL to distinguish
between iron deficiency anemia and other types of anemia. Referring this patient to a
gastroenterologist would not be indicated at this time.
ABFM 2018
Q5-You have diagnosed a 32-year-old female with moderate iron deficiency anemia,
presumed to be due to chronic menstrual blood loss. She has no gastrointestinal or
genitourinary symptoms, and no bruising or bleeding other than menstrual bleeding. Her vital
signs are normal and a physical examination is unremarkable. You initiate a trial of oral iron
therapy. Which one of the following would be the best way to assess the patient’s response to
oral iron?

A) A reticulocyte count in 1–2 weeks


B) A repeat hematocrit in 2 weeks
C) A peripheral smear to look for new RBCs in 4 weeks
D) A serum total iron binding capacity and ferritin level in 6 weeks

ANSWER: A
The reticulocyte count is the first and best indicator of iron absorption and bone marrow
response to oral iron therapy in the treatment of iron deficiency anemia. An increase in
reticulocytes is seen as early as 4 days, peaking at 7–10 days. The rate of production of new
RBCs slows thereafter due to a compensatory decrease in erythropoietin as more iron becomes
available. It typically takes 4–6 weeks before seeing recovery in the hematocrit, and for the
RBC count and indices to normalize. However it is usually 4–6 months before iron stores are
fully restored to normal levels, so treatment should continue for at least that long.
Q6-A 2-year-old African-American male with a history of sickle cell disease is brought to
your office for a well child check. Which one of the following would be most appropriate for
screening at this time?
A) A chest radiograph
B) A DXA scan
C) Abdominal ultrasonography
D) Renal Doppler ultrasonography
E) Transcranial Doppler ultrasonography

ANSWER: E
Individuals with sickle cell disease are at increased risk for vascular disease, especially stroke.
All sickle cell patients 2–16 years of age should be screened with transcranial Doppler
ultrasonography (SOR A). A chest radiograph, abdominal ultrasonography, a DXA scan, and
renal Doppler ultrasonography are not recommended for screening patients with sickle cell
disease.

Q7-A 48-year-old female with GERD treated with a proton pump inhibitor for the past 2 years
sees you for a routine visit. She reports that she has paresthesia and numbness in both feet. Her
hemoglobin A1c is 5.8%, her hemoglobin level is 10.4 g/dL (N 12.0–16.0), and her mean
corpuscular volume is 102 m3 (N 81–99).
Microfilament testing shows decreased sensation in both feet. Which one of thefollowing is the
most likely cause of her peripheral neuropathy?
A) Charcot-Marie-Tooth disease
B) Diabetic peripheral neuropathy
C) Hyperthyroidism
D) Tarsal tunnel syndrome
E) Vitamin B12 deficiency

ANSWER: E
This patient has polyneuropathy, macrocytic anemia, and a history of chronic proton pump
inhibitor use. The most likely cause is vitamin B12 deficiency and a serum level is indicated.
Her hemoglobin A1c is 5.8%, which puts her at risk of developing diabetes mellitus but is not
indicative of diabetes. Charcot-Marie- Tooth disease is a rare cause of polyneuropathy and
unlikely in this case.
Hypothyroidism, and not hyperthyroidism, is associated with polyneuropathy. Tarsal tunnel
syndrome causes a mononeuropathy.

Q8-You see a 53-year-old female with diabetes mellitus, hypertension, mixed hyperlipidemia,
and GERD. Recent laboratory studies include an incidental finding of thrombocytopenia. The
patient has no other significant past medical history, and she does not use tobacco or drink
alcohol. Her current medications include metformin (Glucophage), lisinopril (Prinivil, Zestril),
omeprazole (Prilosec), calcium citrate, and pravastatin (Pravachol). A physical examination is
notable for a BMI of
31.3 kg/m2. Her skin, heart, lungs, abdomen, and extremities are normal. Results of a CBC and
a comprehensive metabolic panel are normal with the following exceptions Platelets 70,000
(N 150,000-379,000 ) Glucose 108mg/dL Bilirubin 0.4mg/dL(N0.0-0.4 ) Alkaline
phosphatase 175U/L(N38-126 ) ALT(SGPT) 52 U/L (N 10-28 )

A peripheral smear is normal except for reduced platelets. Tests for hepatitis B, hepatitis C, and
HIV are negative . The most likely etiology of this patient's thrombocytopenia is
A) a hematologic malignancy
B) chronic liver disease
C) drug-induced thrombocytopenia
D) immune thrombocytopenic purpura (ITP )
E) primary bone marrow failure

ANSWER: B
This patient presents with a typical example of nonalcoholic steatohepatitis (NASH)
progressing toward cirrhosis, with multiple risk factors including diabetes mellitus,
hyperlipidemia, obesity, and mildly elevated hepatic transaminases. Abnormalities of other
cell lines would likely occur if a hematologic malignancy or bone marrow failure were present.
While immune thrombocytopenic purpura is a diagnostic consideration, it is much less
common than NASH and requires other causes to be ruled out. This patient is not taking any
medications that have been frequently reported to cause drug-induced thrombocytopenia.
Q9-Which one of the following malignancies is associated with hereditary hemochromatosis?
A) Biliary carcinoma
B) Chronic myeloid leukemia
C) Hepatocellular carcinoma
D) Multiple myeloma
E) Pancreatic cancer

ANSWER: C
Hereditary hemochromatosis is a genetic disorder of iron regulation and subsequent iron
overload. Possible end-organ damage includes cardiomyopathy, cirrhosis of the liver, and
hepatocellular carcinoma. Symptoms are often nonspecific early on, but manifestations of iron
overload eventually occur. The diagnosis should be suspected in patients with liver disease or
abnormal iron studies indicative of iron overload. A liver biopsy can confirm the diagnosis
and the degree of fibrosis. Identification of such patients and proper ongoing treatment with
phlebotomy may prevent the development of hepatocellular carcinoma and other
complications of this disease. There is some data that suggests an association of breast cancer
with hereditary hemochromatosis but not with any of the other malignancies listed.

Q10-A 66-year-old female with a previous history of hypertension, stable angina, and carotid
endarterectomy presents with acute upper abdominal pain, which has developed over the past 3
hours. A physical examination reveals epigastric tenderness without guarding or rebound, but
does not reveal a cause for the level of pain reported by the patient. Initial laboratory findings
are within normal limits,
including a CBC, glucose, lactic acid, amylase, lipase, liver enzymes, and kidneyfunction tests.
You suspect acute mesenteric ischemia.

Which one of the following diagnostic imaging tests is the preferred initial evaluation for this
problem?

A) Duplex ultrasonography
B) CT angiography
C) Catheter angiography
D) Magnetic resonance angiography (MRA) E) Upper and lower GI endoscopy
ANSWER: B
CT angiography (CTA) is the recommended imaging procedure for the diagnosis of acute
mesenteric vascular disease. The procedure can also identify other possible intra-abdominal
causes of pain. Duplex ultrasonography is also accurate, especially for proximal lesions, but
can be difficult to perform in patients with obesity, bowel gas, and marked calcification of the
vessels, and may be problematic in patients presenting acutely, due to the length of the study
and the abdominal pressure required. It is more useful in cases of suspected chronic mesenteric
ischemia. Endoscopy is often normal in acute ischemia and may not reach the ischemic section
of bowel. MR angiography may be useful, but it takes longer to perform than CTA and lacks
the necessary resolution. Catheter angiography is required for endovascular therapies such as
thrombolysis or angioplasty with or without stenting, but is usually not performed for making
the initial diagnosis in the acute setting.

Q11-The novel anticoagulants (NOACs) include apixaban (Eliquis), dabigatran (Pradaxa),


edoxaban (Savaysa), and rivaroxaban (Xarelto). Which one of the following should be
considered when starting or adjusting the dosage of a NOAC?
A) Serum albumin
B) INR
C) Liver enzymes
D) Partial thromboplastin time
E) Renal function
ANSWER: E
The novel anticoagulants (NOACs) require dosage adjustments based on renal function. There
are no dosing recommendations for NOACs based on liver function or albumin level. The INR
is used to adjust warfarin dosing and the partial thromboplastin time is used to adjust heparin
dosing.

Q12-A 68-year-old female presents with a 3-month history of low back pain and fatigue. She
has unintentionally lost 15 lb. A physical examination is positive for vertebral point tenderness
over the third and fourth lumbar vertebrae. Initial laboratory testing reveals a normocytic
anemia, elevated total protein, and a mild decrease in renal function. You order a lumbar spine
radiograph and additional diagnostic testing. Which one of the following would be most
appropriate at this point?
A) A serum ferritin level and iron studies
B) TSH and vitamin B12 levels
C) Serum protein electrophoresis
D) MRI of the lumbar spine
E) A bone marrow biopsy

ANSWER: C
This patient’s presentation is concerning for hematologic malignancy, in particular multiple
myeloma. Along with radiography, the next appropriate step is serum protein electrophoresis.
If laboratory work shows a monoclonal spike or if a skeletal survey indicates lytic lesions,
referral to an oncologist is indicated for a bone marrow biopsy. MRI of the lumbar spine
would be premature and obtaining iron studies, a TSH level, or a vitamin B12 level would not
adequately address the initial abnormal laboratory studies or facilitate making the diagnosis of
multiple myeloma.

Q13-An elderly male presents with a shallow, irregularly shaped ulceration over the medial
aspect of his right lower leg between the lower calf and medial malleolus. There is some
surrounding edema with pigment deposition over the lower leg. He reports aching and burning
pain in the lower leg with daytime swelling. His symptoms improve with leg elevation. You
make a diagnosis of venous stasis ulcer. Which one of the following would be the most
appropriate management?
A) The use of foam dressings rather than other standard dressings
B) The use of silver-based antiseptic products even if there is no infection
C) Compression therapy
D) A 3-week course of systemic antibiotics
ANSWER: C
This patient likely has a venous stasis ulceration. The use of compression therapy with a
pressure of 30–40 mm Hg is the mainstay of treatment. There is no evidence for the use of
systemic antibiotics for lower-extremity ulcerations.
Likewise, there is no evidence to support the use of either silver-based or honey- based
preparations in ulcerations with no infection. Foam dressings are no more effective than other
standard dressings.

Q14-A 67-year-old female who was recently diagnosed with colon cancer presented to the
emergency department 2 days ago with acute shortness of breath and was diagnosed with a
pulmonary embolism. She was started on enoxaparin (Lovenox) and was hemodynamically
stable during her stay in the hospital. Her shortness of breath has improved and her oxygen
saturation is currently 95% on room air.
Which one of the following would be most appropriate for this patient?
A) Continue enoxaparin upon discharge
B) Discontinue enoxaparin and start rivaroxaban (Xarelto)
C) Discontinue enoxaparin and start warfarin (Coumadin)
D) Start warfarin and continue enoxaparin until the INR is 2.0

ANSWER: A
Enoxaparin and other low molecular weight heparins are effective and are the preferred agents
for acute and long-term anticoagulation in patients with an active malignancy (SOR B).
Warfarin has been shown to be less effective in cancer patients and is not recommended to treat
venous thromboembolic disease in this setting (SOR B). The novel oral anticoagulants
including rivaroxaban have not been studied in the setting of malignancy and are not
recommended.
Q15-A 7-year-old female is brought to your office by her mother for follow-up of an urgent
care visit. The child has a 5-day history of abdominal pain and low- grade fevers to 100.1°F.
Her mother took her to an urgent care clinic last night when the patient developed the rash
shown below. The rash is not pruritic or painful. She does not have any sick contacts, urinary
symptoms, or changes in bowel habits.
A physical examination is normal except for the rash and minimal diffuse abdominal
tenderness. A CBC and basic metabolic panel are normal and a urinalysis is notable only for
microhematuria (30–40 RBCs/hpf) and mild proteinuria (30 mg/dL).
The following laboratory studies were obtained at the urgent care clinic. Basic metabolic panel
: normal
Urinalysis Color : yellow/clear
Leukocyte esterase : negative
Nitrite :negative
Protein: 30 mg/dL (normal negative)
Glucose : negative (normal)
Bilirubin : negative (normal)
RBCs : 34/hpf (N <4)
WBCs: 4/hpf (N <5)
Bacteria : none
Squamous epithelialcells : <1(normal)
Ketones :negative (normal)
Blood large (normal negative)
Urine Gram stain : nobacteria, no PMNs
Urine culture : negative × 24 hours
In addition to close follow-up, which one of the following is the next appropriate step in the
management of this child?

A) Supportive care only


B) Amoxicillin for 10 days
C) Prednisone tapered over 10 days
D) A biopsy of a skin lesion
E) Referral to a nephrologist for consideration of a renal biopsy

ANSWER: A
This patient most likely has Henoch-Schönlein purpura. In addition to close observation, the
only treatment is supportive care, including adequate oral hydration. There is no indication for
antibiotics, and oral corticosteroids have not been shown to be beneficial. In patients with
progressive renal impairment,
referral to a nephrologist is warranted, but given this patient’s normal renal function at this
time it is not indicated. A skin biopsy of the purpura would most likely show a leukocytoclastic
vasculitis and would not help in the diagnosis.

Q16-A 55-year-old male sees you for an annual health maintenance visit. He is a former
smoker and has a history of type 2 diabetes mellitus, hypertension, and hyperlipidemia. He
had a normal colonoscopy at age 50, and had an ST-elevation myocardial infarction 2 years
ago treated with a drug-eluting stent. He is currently asymptomatic and does not have any chest
pain, hypoglycemia, dyspepsia, melena, or rectal bleeding. His medications include metformin
(Glucophage), 2000 mg daily; glimepiride (Amaryl), 2 mg daily; bisoprolol (Zebeta), 5 mg
daily; losartan/hydrochlorothiazide (Hyzaar), 50 mg/12.5 mg daily; rosuvastatin (Crestor), 20
mg daily; clopidogrel (Plavix), 75 mg daily; and aspirin, 81 mg daily.
His blood pressure is 128/76 mm Hg and his heart rate is 63 beats/min. A physical examination
is unremarkable. His hemoglobin A1c is 6.4%.

You recommend that the patient stop taking


A) aspirin
B) clopidogrel
C) aspirin and clopidogrel
D) metformin

ANSWER: B
Patients with drug-eluting stents should be on dual antiplatelet therapy with aspirin plus a
thienopyridine such as clopidogrel for a minimum of 1 year. At the time of this patient’s visit,
2 years after the stent placement, there is no indication to continue clopidogrel, but aspirin
therapy should be continued indefinitely. All of the patient’s other medications have current
active indications and should be continued, although if the patient experiences hypoglycemia,
the sulfonylurea could be decreased or discontinued.
Q17-A 65-year-old male brings in results from a health fair screening. He was advised to see
you because he had a hemoglobin level of 10.2 g/dL (N 14.0–18.0) and a mean corpuscular
volume of 80 m3 (N 80–94). A review of systems is unremarkable except for recent fatigue,
and a physical examination is also unremarkable. You order laboratory testing, with the
following results:
Ferritin 15 ng/mL (N 20–150)
Vitamin B12 420 pg/mL (N 200–900)
Folate 12 ng/mL (N 2–20)
Reticulocyte index ------------------0.3% (N 0.5–1.0) The most likely diagnosis is
A) iron deficiency anemia
B) vitamin B12 deficiency
C) anemia of chronic disease
D) hemolysis
E) myelodysplastic anemia

ANSWER: A
This patient most likely has iron deficiency anemia. The low normal mean corpuscular
volume, low serum ferritin, and low reticulocyte index are all consistent with iron deficiency.
Vitamin B12 deficiency would be indicated by low vitamin B12 and a macrocytic anemia.
Serum ferritin would be higher with anemia of chronic disease and myelodysplastic anemia.
The reticulocyte index would be high with hemolysis.

Q18-An obese 37-year-old white female sees you because of fatigue. She is otherwise
asymptomatic and has normal vital signs. A complete physical examination is unremarkable
with the exception of obesity. A CBC shows no anemia, but her WBC count is 12,500/mm3
(N 4500–11,000). A TSH level and a comprehensive metabolic panel are normal. She does not
take any medications and has not had any recent illnesses.
Which one of the following would be most appropriate at this point?
A) Reassurance that her leukocytosis is likely caused by her obesity and counseling about
weight loss
B) A repeat CBC with differential and a review of the peripheral smear
C) A blood culture
D) Flow cytometric testing
E) Referral to a hematologist for further workup
ANSWER: B
Leukocytosis is a relatively common finding with many possible etiologies. For most cases
without a clear cause, a repeat CBC with differential and a peripheral smear review are
indicated to confirm leukocytosis, determine subtypes, and look for concerning abnormalities
on the smear. Given this patient’s fatigue, a hematologic referral may be indicated if
leukocytosis is confirmed on repeat testing. Similar recommendations would apply to flow
cytometry testing. Blood cultures are not necessary in cases of leukocytosis without evidence
of infection. Obesity can cause leukocytosis, but because of the patient’s fatigue it would not
be appropriate at this time to attribute the leukocytosis to obesity alone.

ABFM2017
Q19- A 45-year-old premenopausal female with a BMI of 34.0 kg/m2 presents to your office
with
increasing dyspnea on exertion. She has a recent history of iron deficiency anemiaand wonders
if her iron level is low again. Her history is negative for heavy menses, bloody or melanotic
stools, abdominal pain, and unusual bleeding or bruising. Her evaluation for anemia has
included upper and lower endoscopy with normal findings. Last year the patient underwent
gastric bypass surgery and has been on a fairly restricted diet since that time. She uses a
levonorgestrel IUD (Mirena) for contraception.
A physical examination is unremarkable. Her serum ferritin level is 6 ng/mL (N 10– 120) and
her hemoglobin level is 8.0 g/dL (N 12.0–16.0) with microcytic indices. You determine that
she has iron deficiency anemia.
Which one of the following would be the most appropriate management?
A) Oral iron replacement
B) Intravenous iron replacement
C) Blood transfusion
D) Urgent referral to a hematologist
ANSWER: B
This patient has symptomatic iron deficiency anemia. Because she has had a gastric bypass,
she is not able to absorb iron adequately and therefore needs intravenous iron replacement. A
blood transfusion may help temporarily but will not restore her iron stores, so it is not
indicated. She will not absorb oral iron adequately. If she does not respond to intravenous
iron, consultation with a hematologist may be indicated.
Q20- A 45-year-old premenopausal female with a BMI of 34.0 kg/m2 presents to your office
with increasing dyspnea on exertion. She has a recent history of iron deficiency anemia and
wonders if her iron level is low again. Her history is negative for heavy menses, bloody or
melanotic stools, abdominal pain, and unusual bleeding or bruising. Her evaluation for anemia
has included upper and lower endoscopy with normal findings. Last year the patient
underwent gastric bypass surgery and has been on a fairly restricted diet since that time. She
uses a levonorgestrel IUD (Mirena) for contraception. A physical examination is
unremarkable. Her serum ferritin level is 6 ng/mL (N 10–120) and her hemoglobin level is 8.0
g/dL (N 12.0–16.0) with microcytic indices. You determine that she has iron deficiency
anemia.
Which one of the following would be the most appropriate management?
A) Oral iron replacement
B) Intravenous iron replacement
C) Blood transfusion
D) Urgent referral to a hematologist

ANSWER: B
This patient has symptomatic iron deficiency anemia. Because she has had a gastric bypass,
she is not able to absorb iron adequately and therefore needs intravenous iron replacement. A
blood transfusion may help temporarily but will not restore her iron stores, so it is not
indicated. She will not absorb oral iron adequately. If she does not respond to intravenous
iron, consultation with a hematologist may be indicated.
ABFM 2016
Q21-A 16-year-old female is brought to your office by her mother, who is concerned that her
daughter has seemed tired lately. The patient denies any specific health concerns or recent
illnesses. She is taking an oral contraceptive and reports that her menstrual bleeding is light in
flow. Recent laboratory findings
include a TSH level of 1.44 :U/mL (N 0.5–5.0), a hematocrit level of 38% (N 36–46), a mean
corpuscular volume of 71 :m3 (N 78–102), an RBC count of 5.7 million/mm3 (N 4.10–5.10),
and ovalocytes on a peripheral smear.
Which one of the following is most likely to explain this patient’s initial laboratory
abnormalities and lead to a diagnosis?
A) A vitamin B12 level
B) A ferritin level
C) A free T4 level
D) Hemoglobin electrophoresis
E) A fluorescent spot test

ANSWER: D
This presentation is consistent with $-thalassemia minor trait in a generally asymptomatic
patient. Hemoglobin electrophoresis will be abnormal, with HbA2 increased and HbA
decreased. The free T4 level is likely to be normal in a patient with a normal TSH level. A
ferritin level is also likely to be normal given the normal levels of hemoglobin and hematocrit.
A fluorescent spot test is used to screen for G-6-PD deficiency, which would be associated
with bite cells and Heinz bodies. A vitamin B12 level would be useful for evaluating
macrocytosis, which is not present in this patient.

Q22- An orthopedic surgeon contacts you about abnormal preoperative laboratory results for
one of your patients. The patient is a 58-year-old male who takes acetaminophen and over-the-
counter naproxen for pain but is otherwise healthy and has no symptoms.
Laboratory Findings
WBCs 3000/mm3 (N 3000–11,600)
Hemoglobin ……………………8.9 g/dL (N 13.0–18.0)
Hematocrit 26.9% (N 38.5–52.0)
Platelets 118,000/mm3 (N 130,000–140,000)
Mean corpuscular volume 89 :m3 (N 80–98)
Red cell distribution width ……. 12.6% (N 11.0–14.5) Serum vitamin B12 457 pg/mL (N
220–960)
Ferritin 258 :g/L (N 24–336)
Reticulocytes …………………… 1.5% (N 0.8–2.1)

Which one of the following is most consistent with these results?


A) Folate deficiency
B) Iron deficiency
C) Hemolysis
D) Acute blood loss
E) Bone marrow suppression

ANSWER: E
This patient’s test results suggest a moderate anemia that is associated with a mild reduction of
platelets. The mean corpuscular volume is solidly in the normal range, so this is a normocytic
anemia and is unlikely to be due to iron or folate deficiency, which usually result in microcytic
and macrocytic anemias, respectively. A normal ferritin level also suggests a diagnosis other
than iron deficiency.
Acute blood loss and hemolysis are two common causes of normocytic anemia. However,
with both of these conditions a high reticulocyte count would be expected. The fact that the
reticulocyte count is in the normal range despite significant anemia suggests that this patient
has decreased bone marrow production of red blood cells, which is at least contributing to his
anemia. Bone marrow response to anemia is often evaluated by using the reticulocyte index,
which is calculated by multiplying the measured reticulocyte percentage (1.5 in this patient)
by the ratio of the patient’s hematocrit to a normal hemotocrit based on the person’s age and
sex (26.9 divided by 45 in this case). If the patient has a normal bone marrow, the reticulocyte
index should be 2%–3% or higher. In this case the result is 0.9%, suggesting a profound
suppression of bone marrow.
Q23-In a healthy full-term infant who is exclusively breastfed, iron supplementation should
begin at what age in order to prevent iron deficiencyanemia?
A) 1 month
B) 4 months
C) 6 months
D) 9 months
E) 12 months

ANSWER: B
Healthy full-term infants receive 60%–80% of their iron stores from their mothers during the
third trimester of pregnancy. Thus, even an exclusively breastfed infant will not typically be at
risk of developing iron deficiency anemia until the age of 4 months. Iron supplementation
should then be started and continued until the child is eating foods containing sufficient
dietary iron. Preterm babies who are exclusively breastfed should begin iron supplementation
at 1 month of age due toa foreshortened third trimester.

Q24- Which one of the following is routinely required to establish a diagnosis of chronic
lymphocytic leukemia?
A) A lymph node biopsy
B) A bone marrow biopsy and aspirate
C) CT of the chest, abdomen, and pelvis
D) Flow cytometry and immunophenotyping of peripheral blood
E) Weight loss of more than 10%, unexplained fever, and night sweats.

ANSWER: D
Flow cytometry and immunophenotyping of peripheral blood demonstrate clonality of the
circulating lymphocytes in chronic lymphocytic leukemia (CLL). Absolute lymphocytosis is
defined as >5000 cells/mm3.
Ninety percent of CLL patients are asymptomatic at the time of diagnosis. Imaging procedures
are not necessary for establishing the diagnosis. While a bone marrow biopsy or aspiration
and/or a lymph node biopsy may be necessary as the disease progresses, they are not routinely
needed to establish a CLL diagnosis.
ABFM2014

Q25-A 30-year-old female sees you because of increasing fatigue. She has no chronic medical
problems and reports no recent acute illnesses. She recalls being told that she was mildly
anemic after the birth of her daughter 3 years ago. The anemia resolved after 3 months of oral
iron supplementation. The patient’s menstrual periods are regular and last approximately 6
days, with heavy bleeding for the first 3 days then moderate to mild flow for approximately 3
days. She denies epistaxis, black stools, or other signs of bleeding.
On examination her temperature is 36.7°C (98.1°F), pulse rate 93 beats/min, respiratory rate
16/min, and blood pressure 116/58 mm Hg. The remainder of her physical examination is
unremarkable. A CBC is notable for a hemoglobin level of
10.9 g/dL (N 12.0–16.0) and a mean corpuscular volume of 70 :m3 (N 78–102(

Which one of the following serum levels would be most appropriate for further evaluating her
microcytic anemia at this point?
A) Ferritin
B) Folate
C) Erythropoietin
D) Hemoglobin A1c
E) TSH

ANSWER: A
After confirmation of anemia and microcytosis on a CBC, a serum ferritin level is
recommended (SOR C). If the ferritin level is consistent with iron deficiency anemia,
identifying the underlying cause of the anemia is the priority. A common cause of iron
deficiency anemia in premenopausal adult women is menstrual blood loss. If the serum ferritin
level is not consistent with iron deficiency anemia, the next stage of the evaluation should
include a serum iron level, total iron- binding capacity (TIBC), and transferrin saturation
(SOR C). Iron deficiency anemia is still probable if the serum iron level and transferrin
saturation are decreased and TIBC is increased. It is more likely anemia of chronic disease if
the serum iron level is decreased and the TIBC and transferrin saturation are decreased or
normal. Other laboratory tests that may help in differentiating the cause of microcytosis include
hemoglobin electrophoresis, a reticulocyte count, and peripheral blood smears
Q26-A 29-year-old female presents with a 1-week history of a rash on her legs. A full review
of systems is significant only for regular borderline-heavy periods that lasted for 7 days during
her last two menstrual cycles. She has not had any recent illness or hospitalization, and takes
no medications. Her examination shows nonblanching purple macules on her upper legs.
A comprehensive metabolic panel reveals normal renal function and liver enzyme tests, and a
urine pregnancy test is negative. A CBC reveals a platelet count of 27,000/mm3 (N 150,000–
400,000) but is otherwise normal
.
Which one of the following is the most likely cause of the rash?
A) Acute leukemia
B) Congenital thrombocytopenia
C) Immune thrombocytopenic purpura
D) Thrombotic thrombocytopenic purpura
E) Henoch-Schönlein purpura

ANSWER: C
The rash described in this patient with significant thrombocytopenia is consistent with
purpura. Purpura from vasculitic causes such as meningococcal infection, disseminated
intravascular coagulation, or Henoch-Schönlein purpura (also known as IgA nephropathy) is
typically palpable rather than macular as in this case. Immune thrombocytopenic purpura is a
relatively common cause of isolated thrombocytopenia. The lack of systemic symptoms or
other abnormal laboratory findings make acute lymphoproliferative disorders such as leukemia
unlikely.
Likewise anemia, neurologic changes, fever, and renal failure are seen with thrombotic
thrombocytopenic purpura. The acute onset of purpura and heavy periods makes congenital
thrombocytopenia unlikely.
Bronchial asthma
Pretest2109
Q1-A 33-year-old healthy nonsmoking man presents to you for evaluation of his cough. He
says the cough has been present for about 8 weeks. After a few days, he went to an urgent care
where he received antitussives and a bronchodilator. Those did not help, and he returned 2
weeks later and was given a course of azithromycin. His cough has persisted. He complains of
an associated sore throat, but no fever. He reports worsening nighttime symptoms, and
drinking coffee seems to precipitate the cough.

Which of the following treatments is most likely to help?


a. A proton pump inhibitor
b. An antihistamine
c. An angiotensin-converting enzyme (ACE)-inhibitor
d. A steroid inhaler
e. Levofloxacin.

The answer is a.
The most common causes of a chronic cough are asthma, postnasal
drainage, smoking, and GERD. Given that he did not respond to a bronchodilator, asthma is
an unlikely diagnosis. Sore throat, combined with symptoms that are worse when lying down
or with ingestion of caffeine or alcohol make GERD a likely diagnosis. Therefore, treatment
with a proton-pump inhibitor would be most likely to help his symptoms. Antihistamines
could treat asthma or post-nasal drip due to allergies. A steroid inhaler would help with
asthma, and an antibiotic might be helpful if an infection were the underlying cause.
ACEinhibitors have cough as a side effect, and would not be used in this case. If the cough
were acute, the differential diagnosis would include asthma exacerbation, acute bronchitis,
aspiration, exposure to irritants (cigarette smoke, pollutants), allergic rhinitis, uncomplicated
pneumonia, sinusitis with postnasal drip, and viral upper respiratory infection. Of these usual
causes, viral upper respiratory infection is by far the most common cause. Viral upper
respiratory infection is the most frequent illness in humans with a prevalence of up to 35%.
Q2-You are treating a 52-year-old woman with a 40-pack-year history of smoking. She reports
a productive cough that has been present for the last 3 to 4 months, beginning in the fall. She
remembers having the same symptoms last year in the fall, and attributed it to a “cold that she
just couldn’t kick.” She does not have fevers, reports mild dyspnea when walking up stairs,
and denies hemoptysis.
Which of the following is the most likely diagnosis?
a. Irritation of airways from cigarette smoke
b. Chronic bronchitis
c. Postnasal drainage due to seasonal allergies
d. Lung cancer
e. Asthma

The answer is b.
Because the patient reports a productive cough for at least 3 months of the year for at least 2
consecutive years, she meets the criteria for chronic bronchitis. This is a common cause of
chronic cough in smokers. Chronic bronchitis is a useful clinical designation, but falls under
the broad category of chronic obstructive pulmonary disease (COPD). While it is true that her
smoking may cause irritation of her airways, it wouldn’t explain why the cough isn’t present
year-round (since she continues to smoke throughout the year). The most common cause of
chronic cough in nonsmokers is postnasal drainage, but since this patient has a significant
smoking history, chronic bronchitis is more likely. Lung cancers rarely present solely with
cough. Associated signs and symptoms include weight loss and hemoptysis. Asthma is less
likely to present with a productive cough.

Q3-Four weeks ago, you treated a 22-year-old woman for acute bronchitis with supportive
care. Although she feels much better, the cough has persisted. She has used bronchodilators,
antihistamines, and antitussives without relief. Of the choices listed below, which of the
following is the best course of treatment at this time?
a. A 10-day course of amoxicillin
b. A 5-day course of azithromycin
c. A steroid nasal spray
d. An NSAID
e. An inhaled steroid

The answer is e.
The Centers for Disease Control and Prevention published guidelines for treating acute
bronchitis. The guidelines state that antibiotics are not indicated for uncomplicated acute
bronchitis, regardless of the duration of the cough.
Antibiotics should be reserved for patients with significant COPD and CHF, those who are
very ill-appearing, or the elderly. This patient likely has hyper-responsive airways, sometimes
called postinfectious cough. In this case, the best treatment would be an inhaled steroid and a
bronchodilator or antihistamine. If the cough persists, an oral steroid taper can be used. Anti-
inflammatory medications and nasal steroids are not effective.

For questions 4 to 7, use the following scenario:


You are seeing a 13-year-old patient in the office for the first time. She has had recent
episodic shortness of breath and her mother is concerned that she has developed asthma. As
you explore this patient’s history, you learn that she has been having 2 to 3 months of daytime
symptoms, including coughing, wheezing, and shortness of breath more than 2 days per week
but not daily. She wakes uponce weekly at night with coughing spells and the teacher at school
just told her mother that the patient is often not participating in her normal recess activities
because of her symptoms. She has never been to the emergency department (ED) or
hospitalized for these symptoms and has not had any workup at this point.

Q4-. Which of the following features, if present, is the strongest predisposing factor in the
development of asthma in children?
a. Family history of asthma
b. History of atopy
c. Caucasian race
d. Exposure to cigarette smoke
e. Exposure to environmental pollution.

The answer is b.
Asthma is common, affecting approximately 5% of the population. While there is a genetic
component to its development, the strongest identified predisposing factor for its development
in children is atopy. In fact, 80% of children with asthma are atopic. The same does not
necessarily hold true in adulthood with less than half of asthmatic adults aged 30 years old with
evidence of atopy. Obesity is increasingly being recognized as a risk factor as well. Living in
poverty and non- white racial group are also important risk factors. Nonspecific predictors
include exercise, upper respiratory infections, pneumonia, gastroesophageal reflux disease,
changes in weather, stress, and exposure to environmental smoke.
Q5-Based on this patient’s history, how would you categorize her symptoms?
a. Intermittent asthma
b. Mild persistent asthma
c. Moderate persistent asthma
d. Severe persistent asthma
e. Cough variant asthma

The answer is c.
It is important to categorize asthma symptoms prior to initiation of treatment based on severity
of symptoms. Terms used to describe asthma are intermittent or persistent, and mild,
moderate, or severe impairment. Clinicians should assess the degree of daytime symptoms,
nighttime symptoms, impairment of activities, and, if on therapy, how often they are using
their short-acting β2-agonist therapy. The recommended reading at the end of this section
includes a publication from the National Heart, Lung and Blood Institute that describes
classification and treatment guidelines for an initial visit, follow up visits and a step-wise
approach for long term management of asthma. This can be accessed at:
https://www.nhlbi.nih.gov/files/docs/guidelines/asthma_qrg.pdf. Based on this information,
the patient in this question has moderate symptoms in all categories, but is not yet
onmedication. Therefore, her asthma would be considered moderate persistent.

Q6--In order to more fully understand this patient’s symptoms, you order pulmonary function
testing (PFT). What finding is consistent with moderate persistent asthma?

a. Peak flow 80% of personal best


b. Forced expiratory volume in 1 second/forced vital capacity (FEV1/FVC) 88% predicted
c. FEV1 less than 75% predicted
d. FVC less than 80% predicted
e. Volume-time curve of less than 4 seconds
The answer is c.
PFT is essential to the diagnosis and management of asthma. The FEV1 is reduced in patients
with obstructive defects. The FVC, or the amount of air a patient can exhale during the entire
test, is also low in patients with obstructive defects. An FEV1 of 60% to 80% of predicted or
an FEV1/FVC of 75% to 80% are consistent with moderate persistent asthma. Peak flow
meters use the peak expiratory flow rate (PEFR) to parallel the FEV1 and they are quite useful
in the home management of asthma, but they are not a component of the in-office PFT. A
reading of 60% to 80% would be consistent with moderate persistent asthma. The volume-time
curve is used to measure the validity of the patient’s efforts on the PFT and should be at least
6 seconds to ensure a valid test attempt.

Q7-The history and PFTs for this patient are consistent with moderate persistent asthma. What
is the best first course of action for this patient?
a. Initiate a low-dose inhaled corticosteroid alone
b. Initiate a short-acting bronchodilator alone
c. Initiate a medium-dose inhaled corticosteroid and a long-acting bronchodilator
d. Initiate a low-dose inhaled corticosteroid, a long-acting bronchodilator, and a short-acting
bronchodilator rescue inhaler
e. Initiate a low-dose inhaled corticosteroid and a leukotriene receptor antagonist

The answer is d.
For patients more than 12 years of age with moderate persistent
asthma, therapy should be initiated at step 3. These patients need to have daily control of
inflammation. The preferred regimen is maintenance therapy with either a low-dose inhaled
corticosteroid plus a long-acting β2-agonist or a medium-dose inhaled corticosteroid alone.
All patients should be given a short- acting β2-agonist rescue inhaler for acute/quick relief.
The alternative regimen for this step would be to use a low-dose inhaled corticosteroid plus a
leukotriene receptor antagonist.

For questions 8 to 10, use the following scenario:


You are caring for a 30-year-old woman who has had asthma since childhood. Currently, she
reports symptoms 2 to 3 times a week, but never more than once a day. Sometimes, her
symptoms cause her to skip her usual exercise regimen. She wakes in the night approximately
3 or 4 times a month to use her inhaler and return to bed.
Q8- Which of the following classifications best characterizes her asthma?
a. Mild intermittent
b. Moderate intermittent
c. Mild persistent
d. Moderate persistent
e. Severe persistent.

The answer is c.
Asthma is classified by its severity, and by assessing daytime and nighttime symptoms.
Patients with symptoms less than twice a week, with brief exacerbations, and with nighttime
symptoms less than twice a month are classified as having “mild intermittent” asthma. There
is no “moderate intermittent” classification. The “mild persistent” classification refers to
symptoms more than twice a week but less than once a day, with symptoms that sometimes
affect usual activity. Nighttime symptoms occur more than twice a month. The “moderate
persistent” classification is characterized by daily symptoms and use of short-acting inhaler,
with exacerbations that affect activity and may last for days. Nighttime symptoms occur at
least weekly. “Severe persistent” asthma is characterized by continual symptoms that limit
physical activities, with frequent exacerbations and nighttime symptoms.

Q9- The patient has updated PFTs consistent with her above asthma categorization and she is
only using a short-acting β2-agonist rescue inhaler to treat her asthma. Given her severity of
asthma, what would be the next best course of action for controlling her symptoms?

a. Addition of a low-dose inhaled corticosteroid


b. Addition of a leukotriene receptor antagonist
c. Addition of a medium-dose inhaled corticosteroid
d. Addition of a medium-dose inhaled corticosteroid plus a leukotriene receptor antagonist
e. Addition of a low-dose inhaled corticosteroid plus a long-acting bronchodilator
. The answer is a.
For a patient with mild persistent asthma, intensification to step 2 therapy is indicated. The
preferred therapy for step 2 is the addition of a low-dose inhaled corticosteroid, the alternative
regimen is the use of a leukotriene receptor antagonist, cromolyn, or theophylline. A medium-
dose inhaled corticosteroid or a low-dose inhaled corticosteroid plus a long-acting
bronchodilator would be indicated for step 3 therapy. A medium-dose inhaled corticosteroid
plus a leukotriene receptor antagonist would be indicated for step 4 therapy.

Q 10-You have developed an asthma action plan for this patient, including the use of home
peak flows. When is it recommended to reassess this patient’s condition?
a. 1 week
b. 2 weeks
c. 2 months
d. 3 months
e. When her peak flow readings fall in the green zone

The answer is b.
The goal of asthma control therapy is to maintain long-term control of symptoms with the
least amount of medication to prevent remodeling of the
lungs and increased risks of morbidity/mortality. When patients have a step-up or step-down to
their therapy, it is recommended they receive follow-up within 2 to 6 weeks or if their
condition worsens. The peak flow readings are used to direct the patient’s care using a home
device that measures PEFR. Peak flow measurements parallel the FEV1 and are an easy and
inexpensive way to monitor asthma control. Readings fall in either the green (PEFR 80% to
100%), yellow
(PEFR 50% to 80%), or red zone (PEFR ≤ 50%). PEFR in the green zone indicates a patient
should continue their current course of therapy and does not trigger follow-up care.
Yellow zone readings are a warning and should precipitate additional medicine or a call to their
physician. Red zone readings are more worrisome and the patient should use their inhaler and
either call their physician or go to the ED.

Q 11-You are caring for a 19-year-old man who has been treated for mild intermittent asthma
since childhood. He has been controlled using a short-acting bronchodilator as needed. Over
the past month, he has been using his inhaler more than 4 times a week, and has had to wake
up in the middle of the night to use his inhaler on three occasions. In the past, he was intolerant
of the side effects associated with an inhaled corticosteroid. Which of the following is the
most appropriate treatment option?
a. Long-acting β-agonist
b. Leukotriene receptor antagonist
c. Cromolyn (Intal)
d. Theophylline
e. Oral corticosteroids.

The answer is b.
In this case, the patient has had mild intermittent asthma, but is
becoming persistent and requires a step-up in therapy. Since he is intolerant of inhaled
steroids, a leukotriene modifier is the best choice. They have been shown to improve lung
function and reduce the need for rescue therapy. Long-acting β- agonists should not be used as
monotherapy since they have been shown in studies to have a small but statistically significant
increased risk of severe or fatal asthma attacks. Cromolyn therapy has been replaced by newer
agents, mainly because of compliance issues. Theophylline and oral steroids would not be
indicated in this case.

Q12-You are caring for a 22-year-old with moderate persistent asthma who has been
wellcontrolled for several months. He developed an upper respiratory infection and his control
worsened. He has not had a fever, but is coughing up sputum. In addition to stepping up his
therapy, which of the following is true?
a. You should begin a course of amoxicillin.
b. You should begin a course of amoxicillin/clavulanate.
c. You should begin a course of azithromycin.
d. You should begin a course of ciprofloxacin.
e. No antibiotics are necessary.

The answer is e.
Multiple studies have shown that infections with viruses and bacteria predispose to acute
asthma exacerbations. However, the use of empiric antibiotics is not recommended. There is
no consistent evidence to support improved clinical outcomes. Antibiotics should be
considered in cases where there is a high likelihood of acute bacterial respiratory infection, as
in the case of high fever, purulent sputum production, or radiographic evidence of lower
respiratory or sinus infection.
ABFM 2020
1- A 45-year-old female returns to your clinic after pulmonary function testing for dyspnea.
Her prebronchodilator
FEV1/FVC ratio was 0.65, which improved to 0.9 post bronchodilator.
These findings are most consistent with which one of the following?
A. Asthma
B. COPD
C. Interstitial lung disease
D. Pulmonary hypertension
Answer: a
A diagnosis of COPD is established by an FEV1/FVC that is consistent with obstruction and
is not significantly reversible with bronchodilator treatment. The American Thoracic
Society/European Respiratory Society guidelines define reversibility as an improvement of
more than 12% in adults. This patient’s FEV1/FVC increased by more than that with
bronchodilation, so her results are most consistent with asthma. Spirometry is not used to
diagnose interstitial lung disease or pulmonary hypertension. Interstitial lung disease is
diagnosed using high-resolution CT. Echocardiography is the recommended first step in the
evaluation of suspected pulmonary hypertension, but confirmation by right heart
catheterization is often required.

2-. A 40-year-old female presents for follow-up of a recent diagnosis of severe persistent
asthma. Skin testing for common inhalant allergens is negative, and her serum IgE levels are
normal. She has a history of rhinorrhea and wheezing with exposure to aspirin.
Which one of the following additional conditions is most likely to be present?
A. Allergic rhinitis
B. Aspergillosis
C. Eczema
D. Nasal polyps
E. Obesity
ANSWER: D
This patient presents with adult-onset, severe and persistent, non-atopic asthma and aspirin
sensitivity. This type of asthma is defined as intrinsic asthma and affects approximately 10%
of patients with asthma. Patients with intrinsic asthma commonly also have nasal polyps.
Allergic rhinitis and eczema are both associated with atopy and patients with these conditions
would also have positive skin tests for inhalant allergens and elevated IgE levels. While
aspergillosis is associated with chronic sinusitis and nasal polyps, skin tests and IgE levels
would also be positive. Obesity is an independent risk factor for asthma but is not specifically
related to intrinsic asthma.
3- A 9-year-old male with a history of moderate persistent asthma is brought to the emergency
department with an acute exacerbation. His symptoms began with a runny nose and nasal
congestion 2 days ago. His parents state that he has not had any fevers or chills and he was
eating and drinking well until a few hours ago when his breathing started to appear more
labored. After multiple treatments with inhaled albuterol (Proventil, Ventolin) and oral
prednisolone he remains tachypneic and wheezy.
Which one of the following intravenous medications should be added to the patient’s current
treatment to reduce the likelihood of hospital admission?
A. Ketorolac
B. Magnesium sulfate
C. Methylprednisolone
D. Omalizumab (Xolair)
E. Theophylline

ANSWER: B
Children who present to the emergency department with an asthma exacerbation and fail to
improve adequately with inhaled short-acting bronchodilators and corticosteroids may benefit
from treatment with intravenous (IV) magnesium sulfate. A 2016 Cochrane review of three
randomized, controlled trials found that this reduced hospital admissions by 68%. Ketorolac is
not known to have any benefit in the treatment of asthma. Oral administration of
corticosteroids is as effective as IV administration, so there is no reason to give IV
methylprednisolone. Omalizumab may be used to prevent exacerbations in patients with
severe asthma who do not achieve adequate control with high-dose inhaled corticosteroids, but
it has no role in the management of acute exacerbations. IV theophylline is not recommended
for asthma exacerbations given its safety profile and poor efficacy compared to short-acting
bronchodilators.

ABFM2019
Q1-A 38-year-old female with a 6-month history of mild shortness of breath associated with
some intermittent wheezing during upper respiratory infections presents for follow-up. You
previously prescribed albuterol (Proventil, Ventolin) via metered-dose inhaler, which she says
helps her symptoms. You suspect asthma. Pulmonary function testing reveals a normal
FEV1/FVC ratio for her age. Which one of the following would be the most appropriate next
step?
A. Consider an alternative diagnosis
B. Assess her bronchodilator response
C. Perform a methacholine challenge
D. Prescribe an inhaled corticosteroid
E. Proceed with treatment for COPD
ANSWER: C
Spirometry is central to confirming the diagnosis of asthma, which is characterized by a
reversible obstructive pattern of pulmonary function. In this case the patient’s FEV1/FVC
ratio is normal, which neither confirms nor rules out asthma. A methacholine challenge is
recommended in this scenario to assess for the airway hyperresponsiveness that is the hallmark
of asthma. Methacholine is a cholinergic agonist. Bronchoconstriction (defined as a reduction
in FEV1 20%) observed at low levels of methacholine administration (<4 mg/mL) is
consistent with asthma. If the FEV1/FVC ratio is reduced on initial spirometry, a
bronchodilator response should be tested. A fixed or partially reversible obstructive pattern
suggests an alternative diagnosis such as COPD, and full reversal after bronchodilator use is
consistent with asthma. Inhaled corticosteroids are not appropriate for intermittent asthma.

Q2- A 2-year-old female is brought to your office for a well child check. She had an episode of
coughing and mild bronchospasm 3 months ago that was successfully treated with albuterol
(Proventil, Ventolin). The mother asks you if there are any factors that would increase the
patient’s risk of asthma.
Which one of the following factors would increase this patient’s risk of asthma?
A. Living in a high microbial environment
B. Exposure to respiratory syncytial virus
C. Recurrent otitis media
D. Persistent lactose intolerance

ANSWER: B
Immunologic profiles of patients with asthma are influenced by environmental exposures.
Those who are exposed to respiratory syncytial virus as an infant have an increased risk,
whereas those who are exposed to a high microbial environment have a lower risk than those
without such exposure. Otitis media and lactose intolerance are not known to be associated
with asthma risk.
ABFM2018
Q3-A 7-year-old female with asthma is brought to your office because of her fourth episode of
wheezing in the last 3 months. She has also had to use her short-acting -agonist rescue inhaler
more frequently. Which one of the following should be added to reduce the frequency of
asthma exacerbations?
A) A leukotriene receptor antagonist
B) A long-acting -agonist
C) An inhaled corticosteroid
D) Inhaled cromolyn via nebulizer

ANSWER: C
Pediatric asthma is the most commonly encountered chronic illness, occurring in nearly one
out of seven individuals. Short-acting -agonists in the form of metered- dose inhalers are
clearly favored for acute exacerbations, as well as for intermittent asthma. Treatment for
persistent asthma requires the use of inhaled corticosteroids, with short-acting -agonists used
for exacerbations. For patients not well controlled with those options, either a long-acting -
agonist or a leukotriene receptor antagonist may be added. While both cromolyn and
nedocromil are fairly devoid of adverse effects, their use is limited because of a lack of
efficacy in the prevention of acute asthma exacerbations.

Q4-A 24-year-old female seeks your advice regarding the recent onset of a cough when
running. She moved to the United States from Mexico last year and her symptoms first
became apparent during her first winter in the Midwest. The cough starts after she has been
running approximately 1 mile but no sputum is produced and no other symptoms occur. She
has no other health concerns.
A physical examination and office spirometry are consistent with a healthy young adult. You
ask her to run around the outside of the clinic several times and then you reexamine her. The
only change noted is an increase in her pulse rate and a 10% drop in her FEV1.
Which one of the following would be the most appropriate initial treatment for this patient?

A) An endurance conditioning program


B) An over-the-counter antihistamine as needed
C) An inhaled corticosteroid 2 hours before running
D) An inhaled short-acting 2-agonist 15 minutes before running E) Daily use of an inhaled
long-acting 2-agonist
ANSWER: D
This patient’s history and examination findings are typical for exercise-induced asthma. The
most appropriate initial treatment for this condition is an inhaled short-acting 2-agonist
(SABA) 15 minutes before exercise (SOR A). Daily use of an inhaled long-acting 2-agonist as
a single agent is not recommended even for those who continue to experience symptoms when
using an inhaled SABA (SOR B). The addition of a daily inhaled corticosteroid is an
appropriate consideration for patients who require more than a SABA to control symptoms but
these should not be used on an as-needed basis before exercise (SOR B). Use of an
antihistamine in an individual with exercise-induced asthma but no known allergies is not
recommended (SOR B). Other treatment considerations with weak recommendations include a
low-sodium diet, air humidification, and supplemental dietary fish oils.

Q5-A 45-year-old female has a history of intermittent asthma and her only medication is an
albuterol (Proventil, Ventolin) inhaler. Over the past 2 months her asthma has limited her
activities. She is using her inhaler daily and waking up at night once or twice a week with a
cough.
Which one of the following would be the preferred medication to control her asthma?
A) Fluticasone (Flovent)
B) Salmeterol (Serevent Diskus)
C) Fluticasone/salmeterol (Advair)
D) Montelukast (Singulair)

ANSWER: C

This patient has intermittent asthma that has become at least moderate persistent as defined by
the frequency of her symptoms. The National Asthma Education and Prevention guidelines
recommend a moderate-dose inhaled corticosteroid (ICS) with a long-acting bronchodilator as
the preferred treatment in moderate persistent asthma. Fluticasone/salmeterol at a dosage of
250/50 g is the only option that fits this category. Montelukast alone is an alternative treatment
for mild.
ABFM2017
Q6-11-An 8-year-old female with a history of persistent asthma is interested in participating in
a mile-long race for a school fundraiser. Her mother is very concerned and thinks she should
not run, and she brings in a form to fill out to excuse the child from participation. The child
states that she wants to participate. Her medications include fluticasone (Flovent HFA), 44 g
twice daily; loratadine (Claritin), 5 mg daily; and albuterol (Proventil, Ventolin) as needed.
She has no nighttime symptoms, has not used her inhaler at all in the past week, and can keep
up with other children during recess.
The best course of action is to
A) fill out the form as requested with no change in medication
B) recommend that she take 2 puffs of albuterol 30 minutes prior to the event and let her run
C) increase the fluticasone dosage to 110 g twice daily and let her run
D) add montelukast (Singulair) to her regimen and let her run
E) add a long-acting -agonist to her regimen and let her run

ANSWER: B
This child has well controlled asthma, as evidenced by her normal daily activities, lack of
nighttime symptoms, and limited use of her rescue inhaler. There is no reason that the
diagnosis of asthma should limit her activities. Because her asthma is well controlled there is
no need to add additional medications or increase the dosage of her current medications.
Long-acting -agonists are not recommended before the age of 12. Prophylactic pretreatment
with a short- acting -agonist has very little harm associated with it and may prevent the need
for a rescue inhaler during an athletic event. Children should have ready access to their rescue
inhalers at school and in other settings; this has been shown to reduce emergency department
visits.

Q7- A 13-year-old female with asthma sees you for the first time in a year. The patient has a
cough and you discover that she has not been taking her medications since you last saw her.
She has been having symptoms 3–4 days a week and has been awakening at night about 3
times a month. Her FEV1 in the office is 90% of predicted. This patient’s asthma should be
classified as
A) intermittent
B) mild persistent
C) moderate persistent
D) severe persistent.
ANSWER: B
FEV1 is >80% of predicted. Intermittent asthma does not interfere with normal activities and
inhaled short-acting 2-agonists during symptomatic periods are usually sufficient treatment.
Mild persistent asthma is defined as symptoms present >2 days per week but not daily,
nighttime awakenings 3–4 times per month, and inhaler use >2 days per week but not daily
and not more than once on any day. The FEV1 is >80% of predicted. Mild persistent asthma
can cause minor limitations during normal activities and should be treated with low-dose
inhaled corticosteroids (ICs). With moderate persistent asthma, symptoms are present daily,
nighttime awakenings occur >1 time per week but not nightly, and an inhaler is required daily.
A patient with moderate persistent asthma has an FEV1 that is 60%–80% of predicted and can
experience some limitations during normal activities.
Moderate persistent asthma is treated with a combination of low-dose ICs and long-acting 2-
agonists (LABA) or medium-dose ICs as monotherapy.
Severe asthma is defined as symptoms present throughout the day, nighttime awakenings up to
7 times per week, inhaler use several times per day, and an FEV1 <60% of predicted. Normal
activities are extremely limited by severe asthma. Treatment includes medium- to high-dose
ICs with a LABA, and consultation with an asthma specialist is recommended. Omalizumab is
also indicated for patients who have allergies.
Q8- A 60-year-old male returns for a reevaluation of his asthma that you have been working to
control. In the past he has been adequately maintained with daily use of inhaled
fluticasone/salmeterol (Advair), along with montelukast (Singulair). Last month he
experienced an exacerbation of his asthma that responded well to oral prednisone. However,
each time you have attempted to wean him off the prednisone his asthma symptoms have
returned.
Which one of the following would be most appropriate at this time?
a. Adding oral azithromycin (Zithromax)
b. Adding oral methotrexate
c. Adding oral theophylline
d. Adding long-term oral prednisone
e. Increasing the dosage of the corticosteroid

ANSWER: E
This patient has severe asthma that is not responding to a moderate dose of an inhaled
corticosteroid, a leukotriene inhibitor, and a long-acting -agonist. The next appropriate step is
to add a stronger dose of inhaled corticosteroid. Methotrexate and azithromycin are considered
inappropriate therapies. Theophylline and low- dose oral prednisone are considered
appropriate steps if the patient does not respond to high doses of an inhaled corticosteroid.
Other reasonable options for the treatment of severe asthma would be a muscarinic antagonist
such as tiotropium, or assessing for the presence of IgE-dependent allergic asthma that may
respond to omalizumab.

Q9-A 13-year-old female has been having difficulty breathing while playing soccer for the last
few weeks. She has no personal or family history of asthma and has never needed medical
treatments for breathing problems before. Her symptoms include coughing and wheezing only
when running intensely for long periods of time. These symptoms persist for up to an hour
after she stops exercising. She has no symptoms at other times during the day or at night. A
pulmonary function test shows better than average lung function with no change after albuterol
(Proventil, Ventolin) inhalation. Which one of the following treatments would be a good first
choice to help with this patient’s symptoms?
A) A 5-day burst of oral prednisone
B) Daily use of a cromolyn inhaler
C) Daily inhaled corticosteroids during soccer season
D) An albuterol inhaler 10–15 minutes prior to exercise
E) Inhaled salmeterol (Serevent Diskus) every morning during soccer season
ANSWER: D
Asthmatic symptoms during exercise are common. These can occur as exacerbations of
underlying airway inflammation or as bronchospasm in otherwise normal airways. This patient
seems to have bronchospasm that would be best managed by albuterol prior to exercise. If she
finds that she needs her inhaler frequently, the addition of an anti-inflammatory agent such as
inhaled corticosteroids or oral montelukast would be reasonable.
Ref: Krafczyk MA, Asplund CA: Exercise-induced bronchoconstriction: Diagnosis and
management.

Q10-A 13-year-old male is brought to the emergency department because of acute worsening
of his asthma. His routine medications are inhaled fluticasone (Flovent) and an albuterol
metered-dose inhaler (Proventil, Ventolin), which he has used every hour for the past 3 hours
without significant improvement. He is unable to talk in complete sentences and his peak
expiratory flow is 200 L/min, down from a baseline of 480 L/min. He has no previous history
of intubations andonly one short hospital admission.
Which one of the following should be given to reduce the likelihood of hospitalization?
A) Cromolyn
B) Epinephrine via nebulizer
C) Inhaled fluticasone
D) Montelukast (Singulair)
E) Oral prednisone

ANSWER: E
Rapid administration of systemic corticosteroids can reduce the frequency of hospitalization in
patients with severe asthma who present with an exacerbation (SOR A). There is no clear
benefit from parenteral versus oral administration (SOR B). The same benefit is not seen with
inhaled corticosteroids during an exacerbation. Montelukast and cromolyn are not appropriate
treatments for an asthma exacerbation. Nebulized epinephrine is used in the treatment of
croup.
ABFM2014
Q11-A 15-year-old male is brought to the office for a well child visit. His parents report that
he has had a nighttime cough and wheezing for the past several months. He is otherwise
healthy and up-to-date on all of his immunizations. Yoususpect that he has asthma.
Which one of the following would be most appropriate at this point?
A) Treat empirically with a short-acting $-agonist
B) Perform spirometry
C) Order radiologic testing
D) Start an inhaled corticosteroid
E) Start a leukotriene inhibitor

ANSWER: B
The American Academy of Asthma, Allergy, and Immunology recommends that asthma not
be diagnosed or treated without spirometry. Once the diagnosis is confirmed, treatment should
commence with a short-acting $-agonist as needed, followed by stepwise treatment based on
the severity of asthma.

Q12-A 69-year-old female with coronary artery disease, diabetes mellitus, and chronic asthma
presents to your office for follow-up. During the visit she expresses concerns about recent
reports of unsafe air quality in your region.
Which one of the following is reasonable counseling regarding the health risks of ambient air
pollution for this patient?
A)Ambient air pollution has not been clearly shown to increase complications of her chronic
diseases
B)She should take her normal vigorous daily walk outside despite air quality alerts because the
health benefits of exercise outweigh the risks
C)She should avoid areas near busy roads, which are major sources of multiple outdoor air
pollutants
D)Use of air conditioning will likely make her indoor air quality worse
E)Ozone pollution usually peaks in the winter months.
ANSWER: C
It is estimated that particulate air pollution caused 100,000 deaths in the United States in 2010.
The United States Environmental Protection Agency (EPA) publishes air quality data in the
form of the Air Quality Index (AQI), which is calculated based on measured levels of the five
major regulated air pollutants (surface ozone, particulate matter, sulfur dioxide, carbon
monoxide, and nitrogen dioxide). AQI is reported as a number from 1 to 300, with
corresponding colors ranging from green (1–50 or “good”) to maroon (200–300 or “very
unhealthy”).
Levels of particulate air pollution are consistently associated with exacerbations of
cardiovascular and pulmonary disease, as well as other illnesses. Physicians should counsel
people with chronic heart and lung disease like this patient to avoid heavy or prolonged
exertion when AFI levels are >100 (SOR C.)
Ozone production is increased by heat, sunlight, and humidity and is generally worse in the
warmer months. Motorized vehicles are known to produce carbon monoxide, nitrogen dioxide,
and particulate matter, and these pollutants are known to be present at higher concentrations
near busy roads. California has responded to this concern by banning construction of schools
within 500 feet of major highways. Recirculating air conditioning is likely to reduce indoor
exposure to ambient air pollution when levels are unsafe outside.

Q13A 25-year-old female with asthma uses her albuterol (Proventil, Ventolin) inhaler only
before running, but reports waking up short of breath four times per month. She went to the
emergency department recently for increased dyspnea during peak ragweed season and
remained overnight until her symptoms improved.
Which one of the following is the best treatment option now?
A) Oral prednisone as needed
B) Inhaled albuterol daily
C) Inhaled cromolyn sodium daily
D) Inhaled salmeterol (Serevent Diskus) daily
E) Inhaled fluticasone (Flovent) daily.
ANSWER: E
Patients with mild asthma are often undertreated. Constant inhaled corticosteroids improve
both asthma control and quality of life. Inhaled albuterol is useful as a quick treatment for acute
symptoms in patients with mild asthma. Oral prednisone causes many side effects and is best
for chronic use in patients whose symptoms are not controlled by other means. Cromolyn
sodium has a good side-effect profile, but is not as effective as inhaled corticosteroids. Inhaled
salmeterol, when used chronically, increases the risk of asthma-related death

Q14-A 38-year-old female with a 6-month history of mild shortness of breath associated with
some intermittent wheezing during upper respiratory infections presents for follow-up. You
previously prescribed albuterol (Proventil, Ventolin) via metered-dose inhaler, which she says
helps her symptoms. You suspect asthma. Pulmonary function testing reveals a normal
FEV1/FVC ratio for her age. Which one of the following would be the most appropriate next
step?
A. Consider an alternative diagnosis
B. Assess her bronchodilator response
C. Perform a methacholine challenge
D. Prescribe an inhaled corticosteroid
E. Proceed with treatment for COPD

ANSWER: C
Spirometry is central to confirming the diagnosis of asthma, which is characterized by a
reversible obstructive pattern of pulmonary function. In this case the patient’s FEV1/FVC
ratio is normal, which neither confirms nor rules out asthma. A methacholine challenge is
recommended in this scenario to assess for the airway hyperresponsiveness that is the hallmark
of asthma. Methacholine is a cholinergic agonist. Bronchoconstriction (defined as a reduction
in FEV1 20%) observed at low levels of methacholine administration (<4 mg/mL) is
consistent with asthma. If the FEV1/FVC ratio is reduced on initial spirometry, a
bronchodilator response should be tested. A fixed or partially reversible obstructive pattern
suggests an alternative diagnosis such as COPD, and full reversal after bronchodilator use is
consistent with asthma. Inhaled corticosteroids are not appropriate for intermittent asthma.
COPD
PRETEST2019
For questions 1 to 3, use the following scenario:
You are assessing a 59-year-old patient with an 80-pack-year history of smoking cigarettes.
He stopped smoking 1 year ago. He reports a cough productive of white frothy sputum for the
past 4 months. Reviewing his chart, you discover that he had a similar presentation last winter,
with a cough that lasted more than 3 months.

Q1. Given this information, which of the following tests is necessary for him?
a. CBC
b. Arterial blood gas measurements
c. Office spirometry
d. Computerized tomographic scans of the chest
e. An electrocardiogram

The answer is c.
Chronic and progressive dyspnea is the cardinal symptom for patients
with COPD, but cough is often the presenting symptom. Upon clinical diagnosis, office
spirometry is necessary to make the diagnosis, assess the disease severity, and monitor
response to treatment. CBC may be indicated to screen for polycythemia or to assess acute
illness in the febrile patient with COPD, but is not necessary. Arterial blood gas measurements
should only be obtained if the FEV1 is found to be less than 50% predicted. A high-resolution
CT scan is not a routine part of care unless the diagnosis is in doubt, or a procedure is being
considered.
ECGs may show changes due to COPD, but are not routinely indicated in the evaluation.

Q2. Treatment modalities for chronic obstructive pulmonary disease (COPD) are based on a
patient’s category of disease. What combination of factors is used in the Global Initiative for
Chronic Obstructive Lung Disease (GOLD)?

a. Duration of symptoms and pack years smoking


b. FEV1 and nocturnal oxygen levels
c. Spirometry and symptom severity
d. Number of exacerbations and breathlessness scale
e. Pack years smoking and changes on chest x-ray (CXR)
The answer is c.
An accurate assessment of a patient’s airflow limitation using office spirometry plus a
measure of their symptoms is essential to categorizing a patient’s COPD and risk for future
exacerbations. The GOLD divides patients into GOLD stages 1, 2, 3, and 4 Patients with
GOLD 1 or 2 categorization are at low risk for future exacerbations while patients in GOLD 3
or 4 are at high risk for future exacerbations and treatment modalities are selected to reduce
this future risk and improve symptoms. Nocturnal oximetry, smoking pack-years, and CXR
changes are not part of the GOLD classification system.

Q3. Which of the following measurements is most sensitive to diagnose COPD inthis patient?
a.Total lung capacity (TLC)
b.FVC
c.FEV1
d.Forced expiratory flow rate over the interval from 25% to 75% of the total FVC (FEF25%-
75%)
e.FEV1:FVC

The answer is e.
Office spirometry is helpful to diagnose COPD and assess its severity. While all the answer
choices are common measurements of airflow, the more sensitive measure to diagnose COPD
is the FEV1 to FVC ratio. It is considered normal if it is 70% or more of the predicted value
based on the patient’s gender, age, and
height. When a patient’s FEV1/FVC is less than 70%, the FEV1 is useful to assist in the
classification of the patient’s COPD; FEV1 is more than or equal to 80% predicted in GOLD
1, FEV1 is between 50%and 80% predicted in GOLD 2, FEV1 is between 30% and 50% in
GOLD 3, and FEV1 is less than 30% in GOLD 4 disease.
The TLC is not often used in the routine management of COPD, but is an important marker for
restrictive disease.
Q4. You have diagnosed a 66-year-old female patient of yours with COPD. Which of the
following therapies has been shown to improve the natural history of COPD?
a. Bronchodilators
b. Inhaled steroids
c. A combination of bronchodilators and inhaled steroids
d. Antibiotics
e. Supplemental oxygen
The answer is e.
The single most important intervention in smokers with COPD is to encourage smoking
cessation. However, the only drug therapy that has been shown to improve the natural history
of COPD progression is supplemental oxygen in those patients that are hypoxemic. Benefits of
oxygen therapy include longer survival, reduced hospitalizations, and better quality of life.
Bronchodilators do not alter the course of the disease or decline in function, and COPD is
generally not a steroid-responsive disease. Antibiotics can be useful to treat infection and
exacerbation, but no convincing evidence exists to support their use chronically.

Q5. You are seeing a 59-year-old patient in the office for a chief complaint of “COPD
exacerbation.” GOLD recommends the use of antibiotics in patients with acute exacerbations
of COPD who have which of the following three symptoms?
a. Hypoxia, fever, and lethargy
b. Increase in dyspnea, cough, and fever
c. Increase in dyspnea, pleuritic pain, and sputum volume
d. Decrease in SpO2, energy level, and FEV1
e. Increase in dyspnea, sputum volume, and sputum purulence

The answer is e.
In COPD patients with increases in dyspnea, sputum volume, andsputum purulence, the use of
antibiotics is recommended. Antibiotics are also recommended in any patient with COPD
requiring mechanical ventilation but not necessarily for a drop in SpO2. The use of antibiotics
in the setting of these cardinal symptoms has been shown to reduce the risk of death.

Q6. The patient described in the question above required admission to the hospital for his
exacerbation. Which of the following treatment plans will be likely to result in the shortest
hospitalization and lowest rate of re exacerbation?
a.Treatment with antibiotics only
b.Treatment with antibiotics and β-agonists only
c.Treatment with antibiotics, β-agonists, and an antimuscarinic agent (like ipratropium
bromide) only
d.Treatment with antibiotics, β-agonists, an antimuscarinic agent, and a short course of steroids
(5 days)
e.Treatment with antibiotics, β-agonists, an antimuscarinic agent, and a long course of steroids
(14 days)
The answer is d.

The Reduction in the Use of Corticosteroids in Exacerbated COPD trial (REDUCE)


randomized patients with an acute COPD exacerbation and a history of smoking to a 5-day or
14-day course of glucocorticoids. All patients received antibiotics, inhaled β-agonists, and
inhaled antimuscarinics. The 5-day group received less than half the dose of glucocorticoids,
had a shorter hospitalization, and no difference in the rate of readmission.
ABFM 2020
1- A 55-year-old male with a 40-pack-year smoking history comes to your office with the
results of spirometry he had at a health fair. He quit smoking 1 year ago. He does
nothaveanycough, dyspnea, wheezing, orsputum production,but he is concerned that the
spirometry results show an FEV1/ FVC ratio of 0.65 and an FEV1 of 70% of predicted, which
indicates mild to moderate airflow obstruction.
Based on the best available evidence, which one of the following should you recommend in
order to
prevent the development of symptomatic airflow obstruction?
A. No treatment
B. A long-acting anticholinergic
C. A long-acting beta-agonist
D. An inhaled corticosteroid
E. Combination therapy with a corticosteroid and long-acting beta-agonist

ANSWER: A
There is no evidence from randomized, controlled trials to show that treating asymptomatic
individuals, with or without risk factors for airflow obstruction, prevents future respiratory
symptoms or reduces subsequent declines in lung function. Partly for this reason, the U.S.
Preventive Services Task Force and joint guidelines issued by the American College of
Physicians, American College of Chest Physicians, American Thoracic Society, and European
Respiratory Society recommend against screening for COPD in asymptomatic adults.
Regardless of the results of this patient’s spirometry testing, treatment should not be initiated
in the absence of symptoms. A long-acting anticholinergic, a long-acting beta-agonist, an
inhaled corticosteroid, and combination therapy with a corticosteroid and long acting beta-
agonist would not be recommended for this patient.

2- A 60-year-old male presents with dyspnea on exertion, occasional wheezing, and a chronic
cough that is productive. He has never been hospitalized. He has smoked one pack of
cigarettes per day since the age of 20. An examination reveals diminished breath sounds but
no crackles, jugular venous distention, gallop, or edema. Spirometry shows a
postbronchodilator FEV1 that is 45% of the predicted value, and the severity of his disease is
rated as Global Initiative for Chronic Obstructive Lung Disease (GOLD) group C.

In addition to albuterol as needed for symptomatic relief and smoking cessation, the initial
treatment should include:
A. Beclomethasone
B. Budesonide/formoterol (Symbicort)
C. Roflumilast (Daliresp)
D. Theophylline
E. Tiotropium (Spiriva)

ANSWER: E
The goals for treatment of this patient’s COPD should include prevention of or a reduction in
hospitalizations, a decrease in dyspnea, slowing progression of the disease, and a decrease in
mortality. Disease severity is categorized by spirometry results, the severity of symptoms such
as cough and dyspnea, and the number of exacerbations, including those requiring
hospitalization. Classifying patients into Global Initiative for Chronic Obstructive Lung
Disease (GOLD) groups A through D helps guide treatment initiation and modification over
time.
The initial treatment for patients in GOLD group A is a short- or long-acting bronchodilator.
Patients in GOLD group B should begin treatment with a single long-acting muscarinic
antagonist (LAMA) or a long-acting beta-agonist (LABA). A LAMA is the initial
recommendation for patients in GOLD group C, although a combination inhaled corticosteroid
plus a LABA can be considered for treating persistent exacerbations. Individuals classified in
GOLD group D can begin treatment with a LAMA or a combination of an inhaled
corticosteroid plus a LABA.

3- Long-term oxygen therapy is recommended for patients with COPD who have a resting
oxygen saturation at or below a threshold of:
A. 88%
B. 89%
C. 90%
D. 93%
E. 95%
ANSWER: A
Patients with moderate to severe COPD should be evaluated periodically for hypoxemia to
determine the need for long-term oxygen therapy. In a Cochrane review of six randomized,
controlled trials, oxygen therapy improved survival in select patients with COPD and severe
resting hypoxemia (a resting arterial partial pressure of oxygen <55 mm Hg or an oxygen
saturation 88%).
4- A 42-year-old male sees you for follow-up after his third episode of pneumonia. He has no
other significant medical history. He has never smoked, drinks alcohol occasionally, and has
no other drug use or known exposures. A physical examination is normal. Pulmonary function
tests demonstrate an FEV1 of 72% of predicted and an FEV1/FVC ratio of 0.68, which does
not normalize with bronchodilator administration. A chest radiograph shows hyperinflation
but no other significant findings. Laboratory Findings:
Platelets 102,000/mm3 (N 150,000–450,000)
Creatinine 0.7 mg/dL (N 0.6–1.2)
AST 56 U/L (N 8–48)
ALT 43 U/L (N 7–55)
Albumin 3.3 g/dL (N 3.5–5.0)

Which one of the following conditions best explains this patient’s abnormal findings?

A. Alpha1-Antitrypsin deficiency
B. Cystic fibrosis
C. Goodpasture syndrome
D. Hereditary hemochromatosis
E. Sarcoidosis

ANSWER: A
This patient is in his forties without clear risk factors and has both an irreversible obstructive
pulmonary defect consistent with COPD (FEV1 <80% of predicted and an FEV1/FVC ratio
<0.70) and liver abnormalities associated with advanced fibrosis. These combined findings are
the hallmark of alpha1-antitrypsin deficiency. This patient has a high likelihood of advanced
liver fibrosis based on the low albumin level and noninvasive scoring using the fibrosis-4
(FIB-4) index (age × AST/(platelets [in mm3] × ALT½ = 3.52 for this patient). Further
evaluation with transient elastography should be performed to confirm cirrhosis.
Cystic fibrosis is less likely to cause a typical obstructive picture on pulmonary function tests
(PFTs) and does not typically cause liver fibrosis. Goodpasture syndrome is a vasculitis that
classically involves the lungs and the kidneys and is more likely to cause a restrictive pattern
on PFTs. Hereditary hemochromatosis can cause early liver disease, including cirrhosis, but is
not a significant cause of respiratory disease. Sarcoidosis may involve both the lung and the
liver, and can cause obstructive or restrictive patterns on PFTs. However, this patient’s
radiograph did not show the characteristic hilar adenopathy and granulomatous disease of
sarcoidosis.
ABFM2019
Q1-A 58-year-old female with COPD asks what she can do to avoid hospitalization. She does
not have any other medical problems.
Which one of the following interventions has been shown to reduce respiratory- related
hospital admissions in patients such as this?
A. Written self-management plans that include smoking cessation plans
B. Regular physical activity
C. Regular assessment of FEV1
D. Nightly CPAP therapy
E. Daily oxygen therapy

ANSWER: A
Written self-management plans have been shown to decrease respiratory-related
hospitalizations in patients with COPD. Although regular physical activity has clear health
benefits, the methods are so varied in studies of physical activity that there is currently no
strong evidence to show it reduces hospitalizations in COPD patients. Although FEV1 is
important for predicting hospitalizations for a population, it is not accurate enough to be useful
in an individual patient. Daily oxygen therapy does not help to postpone the first
hospitalization. Nightly CPAP therapy reduces hospitalizations in patients with COPD and
sleep apnea, but not those with COPD alone.

Q2- .A 42-year-old female presents for follow-up after being treated for recurrent respiratory
problems at an urgent care facility. She is feeling a little better after a short course of oral
prednisone and use of an albuterol (Proventil, Ventolin) inhaler. She has had a gradual
increase in shortness of breath, a chronic cough, and a decrease in her usual activity level over
the past year. She has brought a copy of a recent chest radiograph report for your review that
describes panlobular basal emphysema. She does not have a history of smoking, second hand
smoke exposure, or occupational exposures. Spirometry in the office reveals an FEV1/FVC
ratio of 0.67 with no change after bronchodilator administration.
Which one of the following underlying conditions is the most likely cause for this clinical
presentation?
A- a1-Antitrypsin deficiency
B- Bronchiectasis
C-Diffuse pan-bronchiolitis
D- Interstitial lung disease
E-Left heart failure
ANSWER: A
This patient presents with symptoms of chronic obstructive lung disease, and spirometry
confirms airflow limitation or obstruction with an FEV1/FVC <0.7. Her age, the lack of
tobacco smoke or occupational exposures, and the chest radiograph findings are typical of 1-
antitrypsin deficiency. While left heart failure, interstitial lung disease, bronchiectasis, and
diffuse pan-bronchiolitis are all causes of chronic cough, they are not necessarily associated
with the development of COPD and these spirometry findings. Furthermore, the radiologic
findings in this patient are not consistent with these conditions. Left heart failure would present
with pulmonary edema on a chest radiograph and volume restriction on pulmonary function
testing. Bronchiectasis would present with bronchial dilation and bronchial wall thickening on
a chest radiograph. Interstitial lung disease would present with reticular or increased interstitial
markings. Diffuse pan-bronchiolitis would present with diffuse small centrilobular nodular
opacities along with hyperinflation.
ABFM2018
Q3-A 29-year-old male smoker presents with a 10-day history of a cough. He also had a low-
grade fever for 2 days that has resolved. He has had some mild rhinorrhea and has noted that
the cough has become productive of greenish sputum over the past 3–4 days. He has not tried
any medication. An examinationreveals some mild rhinorrhea but his lungs are clear .
Which one of the following would be most appropriate at this point ?
A) Supportive care only
B) A chest radiograph
C) Albuterol (Proventil, Ventolin )
D) Antibiotic therapy
E) An inhaled corticosteroid

ANSWER: A
The defining symptom of acute bronchitis is cough. Even in smokers the etiologic agent is
viral at least 90% of the time, so antibiotics are not indicated. Unless wheezing is noted,
albuterol is not helpful. Inhaled corticosteroids are used in maintenance therapy for asthma.
Indications for an adult patient with acute bronchitis to have a chest radiograph include:
bloody sputum, rusty-colored sputum, or dyspnea; a pulse rate >100 beats/min; a respiratory
rate >24/min; or a temperature >37.8°C (100.0°F). A chest radiograph is also indicated if there
are abnormal findings on a chest examination such as fremitus, egophony, or focal
consolidation. Supportive care is made easier by informing the patient that symptoms are
likely to last 2–3 weeks. Symptoms may be managed with measures such as
dextromethorphan, guaifenesin, or honey.

Q4- A 58-year-old male with a 30-pack-year smoking history comes to your office to discuss
screening for COPD. His older brother and sister have both recently been diagnosed with
COPD and he wants to be screened for this soon. He continues to smoke and does not express
a desire to quit. He does not have shortness of breath, cough, orthopnea, paroxysmal nocturnal
dyspnea, or dyspnea on exertion. His only medication is aspirin, 81 mg daily. He has never
used inhaled medications such as albuterol (Proventil, Ventolin). His family history is
otherwise negative. You counsel him on tobacco cessation today .
Which one of the following is recommended with regard to COPD screening for this patient ?
A) No screening
B) Spirometry with pre- and postbronchodilator testing
C) Posteroanterior and lateral chest radiographs
D) Noncontrast CT of the chest
E) 1-Antitrypsin deficiency gene testing
ANSWER: A
All patients with a smoking history and symptoms of COPD such as a chronic cough with
sputum production and/or chronic and progressive dyspnea should be screened for COPD with
spirometry. However, asymptomatic individuals such as this patient should not be screened
with spirometry regardless of risk factors.
Neither chest radiography nor chest CT has a role in screening for COPD. Screening for 1-
antitrypsin deficiency in the absence of a family history is not recommended.

Q5-A 62-year-old female has a history of COPD graded as moderate on pulmonary function
testing, with an FEV1 of 65% of predicted and a PaO2 of 57 mm Hg.
Because her symptoms of dyspnea on exertion and fatigue seem out of proportion to her
pulmonary function tests, you order echocardiography, which shows a pulmonary artery
systolic pressure of 50 mm Hg, indicating pulmonary hypertension. ?Which one of the
following would be most effective for decreasing mortality in this situation
A) Supplemental oxygen
B) An endothelin receptor antagonist such as bosentan (Tracleer )
C) A calcium channel blocker such as nifedipine (Procardia )
D) A phosphodiesterase 5 inhibitor such as sildenafil (Revatio )
E) Referral for pulmonary artery endarterectomy

ANSWER: A

The only proven therapy for pulmonary hypertension related to COPD is supplemental
oxygen. Supplemental oxygen should be recommended when the PaO2 is <60 mm Hg,
because it has been shown to improve mortality by lowering pulmonary arterial pressures.
Treatments effective for pulmonary artery hypertension should not be used. Pulmonary
vasodilators such as nifedipine, sildenafil, and bosentan may cause a ventilation-perfusion
mismatch. Pulmonary endarterectomy may be indicated for pulmonary hypertension caused by
chronic thromboembolic disease.
Q6-In patients with COPD, which one of the following inhaled medications has been shown to
reduce exacerbations and exacerbation-related hospitalizations ?
A) Albuterol (Proventil, Ventolin)
B) Fluticasone (Flovent)
C) Ipratropium (Atrovent)
D) Salmeterol (Serevent (
E) Tiotropium (Spiriva )

ANSWER: E
A Cochrane review found that the long-acting antimuscarinic agent tiotropium improved
quality of life and reduced exacerbations and exacerbation-related hospitalizations in patients
with underlying COPD. Tiotropium was noted to be superior to long-acting -agonists such as
salmeterol. Albuterol, fluticasone, and ipratropium have not been shown to have these effects
(SOR A).

Q7-A 48-year-old female presents with dyspnea with exertion. She has never smoked. A
physical examination is normal, including vital signs and pulse oximetry. A chest radiograph
reveals mild hyperexpansion of the chest, and pulmonary function testing reveals an
FEV1/FVC ratio of 0.67, unchanged after bronchodilator use. An EKG and stress
echocardiogram are normal. You suspectCOPD . Which one of the following is the most likely
underlying cause of this patient’s pulmonary
disease ?
A)Allergic bronchopulmonary aspergillosis
B)1-Antitrypsin deficiency
C)Hemochromatosis
D)Primary pulmonary hypertension
E)Hypertrophic obstructive cardiomyopathy

ANSWER: B
This patient is a nonsmoker but has typical symptoms and findings of COPD. alfa1-
Antitrypsin deficiency should be considered in patients with very premature COPD or in
patients without risk factors for COPD such as smoking, secondhand smoke exposure, or other
smoke exposure. Dyspnea would be present and lung function would be normal in patients
with primary pulmonary hypertension or hypertrophic obstructive cardiomyopathy.
Hemochromatosis may cause liver function abnormalities but not abnormal lung function.
Allergic bronchopulmonary aspergillosis is associated with asthma, not COPD
Q8-A 45-year-old male with a 30-pack-year smoking history reports a chronic cough with a
small amount of phlegm production and dyspnea with strenuous exercise. You order
spirometry, which shows a pre- and postbronchodilator FEV1/FVC ratio of 0.6 and an FEV1
of 85% of predicted .
Which one of the following agents would be the best initial pharmacologic management ?
A) An inhaled corticosteroid
B) A short-acting anticholinergic
C) A long-acting anticholinergic
D) A long-acting 2-agonist
E) Theophylline

ANSWER: B
This patient has COPD and is in a risk category of A (low risk, fewer symptoms) based on the
Global Initiative for Chronic Obstructive Lung Disease (GOLD) combined assessment of
COPD. As a result, either a short-acting anticholinergic or a short-acting 2-agonist should be
selected as the initial pharmacologic management. Long-acting 2-agonists or long-acting
anticholinergics are indicated for patients with a GOLD combined assessment category of B or
worse. Long- acting inhaled corticosteroids are indicated for patients with a GOLD combined
assessment category of C or worse. Due to its narrow therapeutic window, modest benefit, and
need for monitoring, theophylline is not recommended as an initial agent and should be
considered as an alternative only for patients with severe refractory symptoms.
ABFM2017
Q9- .A 65-year-old male with a 40-pack-year history of smoking presents with shortness of
breath on exertion. Spirometry reveals the following:
FEV1/FVC 65% of predicted FVC normal
Bronchodilator therapy no improvement in FEV1
Which one of the following is suggested by these results?
A) Normal spirometry
B) Reversible obstructive lung disease
C) Irreversible obstructive lung disease
D) Restrictive lung disease
E) Mixed obstructive and restrictive lung disease

ANSWER: C
These spirometry results indicate an irreversible obstructive pattern. Patients with a restrictive
component to their lung disease have a decreased FVC. Reversible obstruction improves with
bronchodilator therapy.

Q10- .A 60-year-old male with COPD returns for a follow-up visit. He has been treated twice
for pneumonia in the past year and is using his albuterol metered- dose inhaler (Proventil,
Ventolin) several times a day. His FEV1 is 60% of predicted. To reduce symptoms and
prevent hospitalization, which one of the following would be the most appropriate medication
to add to this patient’s treatment regimen at this time?
A) Roflumilast (Daliresp)
B) A long-acting β 2-agonist bronchodilator
C) A long-acting β 2-agonist and an inhaled corticosteroid
D) A long-acting anticholinergic inhaler

ANSWER: D
To decrease the patient’s frequent use of a short-acting bronchodilator, the addition of a long-
acting anticholinergic inhaler for maintenance is most reasonable. Such medications have been
demonstrated to improve quality of life and reduce hospitalizations. Evidence has
demonstrated that a -agonist combined with an inhaled corticosteroid would not be as
preferable in this case, as they can increase the incidence of pneumonia. Long-acting
theophylline and roflumilast may be added to the regimen of patients still symptomatic on
triple therapy with long-acting β 2-agonists, inhaled corticosteroids, and anticholinergic
bronchodilators.
Q11 .A 63-year-old male presents to your office because his COPD is worsening. He indicates
that his exercise tolerance is steadily decreasing but he can still walk approximately 100 yards
on flat ground. His medications include formoterol (Perforomist) twice daily and albuterol
(Proventil, Ventolin) as needed, which he requires only occasionally. He had an exacerbation
requiring prednisone 7 months ago. He quit smoking 6 years ago. His oxygen saturation is 93%
on room air and 89% after walking briskly for 8 minutes. His FEV1 is 1.91 L (62% of
predicted) and his FEV1/FVC ratio is 0.57.
Which one of the following is most likely to improve his progressive dyspnea?
A) Stop formoterol and start fluticasone/salmeterol (Advair)
B) Stop formoterol and start tiotropium
C) Add tiotropium
D) Start oxygen at 2 L/min with exertion
E) Refer for pulmonary rehabilitation

Answer :E
Pulmonary rehabilitation has multidimensional benefits in COPD management, including
improved exercise tolerance, quality of life, and mood. Aerobic exercise improves muscle
mass, with high-intensity exercise proving more efficacious than low-intensity exercise.
According to Global Initiative for Obstructive Lung Disease (GOLD) guidelines, an inhaled
corticosteroid should be added for frequent exacerbations and an FEV1 <50%. Although it is
possible that a combined long- acting -agonist (LABA)/corticosteroid would improve dyspnea
more than a LABA alone, the benefit would not likely be more than a pulmonary
rehabilitation program. Switching from formoterol to tiotropium would not be expected to
provide a significant benefit, and the benefits of adding a LABA to tiotropium have not been
studied. Third-party payers restrict payment for oxygen therapy to those with an SaO2 <89%, a
PaO2 <55 mm Hg, a hematocrit >55%, or documented cor pulmonale.
ABFM2015
Q12- :Which one of the following is associated with treatment of COPD with inhaled
corticosteroids?
A) An increased risk of monilial vaginitis
B) An increased risk of bruising
C) Consistent improvement in FEV1
D) A decreased risk of pneumonia
E) Decreased mortality

ANSWER: B
Inhaled corticosteroids increase the risk of bruising, candidal infection of the oropharynx, and
pneumonia. They also have the potential for increasing bone loss and fractures. They decrease
the risk of COPD exacerbations but have no benefit on mortality and do not improve FEV1 on
a consistent basis..

Q13- A 63-year-old female with corticosteroid-dependent COPD has developed pneumonia.


Which one of the following pathogens should the antibiotic regimen cover in this patient that
would be unlikely in someone with pneumonia and otherwise healthy lungs?
A) Streptococcus pneumoniae
B) Mycoplasma pneumoniae
C) Haemophilus influenzae
D) Staphylococcus aureus
E) Pseudomonas aeruginosa

ANSWER: E
All of the pathogens listed can cause pneumonia in any patient. However, in patients with
chronic lung disease who are taking corticosteroids, Pseudomonas is more common than in
those with otherwise healthy lungs. The antibiotics chosen empirically should cover this
pathogen.
Q14-A 58-year-old male with COPD presents with a 5-day history of increased dyspnea and
purulent sputum production. He is afebrile. His respiratory rate is 24/min, heart rate 90
beats/min, blood pressure 140/80 mm Hg, and oxygen saturation 90% on room air. Breath
sounds are equal, and diffuse bilateral rhonchi are noted. He is currently using
albuterol/ipratropium by metered-dose inhaler three times daily.
In addition to antibiotics, which one of the following would be most appropriate for treating
this exacerbation?

A) A single dose of intramuscular dexamethasone


B) Oral prednisone for 5 days
C) Daily inhaled fluticasone (Flovent)
D) Hospital admission for intravenous methylprednisolone sodium succinate(Solu-Medrol)
E) No corticosteroids at this time

ANSWER: B
This patient most likely has a mild to moderate COPD exacerbation. His vital signs do not
indicate a serious condition at this time, so he can be treated as an outpatient. Since he is
already on a reasonable dose of an inhaled bronchodilator/anticholinergic combination, he
should be treated with an oral antibiotic and an oral corticosteroid. Intravenous corticosteroids
offer no advantages over oral therapy, provided there are no gastrointestinal tract limitations
such as poor motility or absorption . Oral corticosteroid therapy initiated early in a COPD
exacerbation reduces the rate of treatment failure, decreases hospitalization rates, improves
hypoxia and pulmonary function, and shortens the length of stay for patients requiring
hospitalization. Short courses of oral corticosteroids (5–7 days) are as effective as
longer ones (SOR A). Inhaled corticosteroids are ineffective in the treatment of a COPD
exacerbation. Intramuscular dexamethasone has no role in treating COPD.
ABFM2014
Q15-A 67-year-old male is admitted to your inpatient service with a week-long acute
exacerbation of COPD. He also has hypertension and type 2 diabetes mellitus. After 24 hours
of intravenous fluids and intravenous methylprednisolone, he is now tolerating oral intake.
Which one of the following corticosteroid regimens is best for this patient at this time?
A)Continue intravenous methylprednisolone until his COPD is back to baseline, then switch to
oral methylprednisolone for a 14-day total course of treatment
B)Switch to oral prednisone for a 14-day total course of treatment, including the initial 24-hour
intravenous treatment
C)Switch to oral prednisone for 4 more days of treatment
D)Use only inhaled corticosteroids by nebulizer
E)Discontinue corticosteroid treatment altogether after 24 hours

ANSWER: C
Systemic corticosteroid therapy reduces the hospital length of stay in patients with acute
COPD exacerbations (SOR A). Oral therapy has been shown to be as effective as the
intravenous route in patients who can tolerate oral intake (SOR B). A randomized, controlled
trial has demonstrated that 5-day courses of systemic corticosteroid therapy are at least as
effective as 14-day courses (SOR A). Inhaled corticosteroids are beneficial in some COPD
patients but nebulizers generally do not offer significant advantages over metered-dose
inhalers in most patients

Q16-A 66-year-old male with known GOLD stage 3 COPD is admitted to the hospital with
pneumonia. His pneumonia improves and he is discharged with home oxygen because of
hypoxemia. He did not require home oxygen before this.
Which one of the following would be most appropriate regarding his future use of home
oxygen?
A) Reduce oxygen use to nighttime only
B) Stop oxygen when his course of antibiotics and corticosteroids is completed
C) Reassess the need for oxygen within 3 months
D) Stop oxygen within 6 months
E) Continue oxygen indefinitely
ANSWER: C
The American College of Chest Physicians and the American Thoracic Society recommend
that for patients discharged on supplemental home oxygen following hospitalization for an
acute illness, the prescription for home oxygen should not be renewed without assessing the
patient for ongoing hypoxemia (SOR C). The rationale for this recommendation is that
hypoxemia often resolves after recovery from an acute illness. The guidelines recommend that
a plan be established to reassess the patient no later than 90 days after discharge and that
Medicare guidelines and evidence-based criteria should be followed to determine whether the
patient meets the criteria for supplemental oxygen. Continuous oxygen therapy is indicated in
patients with COPD and severe hypoxemia. There is good evidence that the addition of home
long-term continuous oxygen therapy for COPD increases survival rates in patients with
severe hypoxemia, defined as an oxygen saturation <90% or a PaO2 <8 kPa (60 mm Hg), but
not in patients with moderate hypoxemia or nocturnal desaturation. Continuous supplemental
oxygen should be used to improve exercise performance and survival in patients with
moderate to severe COPD who have severe daytime hypoxemia. The Centers for Medicare
and Medicaid Services (CMS) provides guidelines for supplemental oxygen therapy and sets
the standard for nearly all adult oxygen prescriptions. According to these standards, oxygen
therapy is covered for patients with a documented PaO2 £55 mm Hg or an oxygen saturation
£88% on room air at rest

Q17- You see a 58-year-old female whom you suspect has COPD and you recommend formal
testing in order to confirm this diagnosis. The cutoff most often used for COPD diagnosis on a
spirometry test performed while the patient is stable (not experiencing an acute exacerbation of
symptoms) is a postbronchodilator FEV1/FVC ratio
A) <50% of predicted
B) <70% of predicted
C) <85% of predicted
D) >70% of predicted
E) >85% of predicted

ANSWER: B
Suspected COPD should be confirmed by spirometry in stable patients, based on a
postbronchodilator FEV1/FVC ratio <70% of predicted (SOR C). While guidelines for the
treatment of COPD differ slightly among the leading national and international organizations,
most have come to a consensus on using this criterion for the diagnosis (SOR C).
Q18-A 67-year-old male is admitted to your inpatient service with a week-long acute
exacerbation of COPD. He also has hypertension and type 2 diabetes mellitus. After 24 hours
of intravenous fluids and intravenous methylprednisolone, he is now tolerating oral intake.
Which one of the following corticosteroid regimens is best for this patient at this time?

A)Continue intravenous methylprednisolone until his COPD is back to baseline, then switch to
oral methylprednisolone for a 14-day total course of treatment
B)Switch to oral prednisone for a 14-day total course of treatment, including the initial 24-hour
intravenous treatment
C)Switch to oral prednisone for 4 more days of treatment
D)Use only inhaled corticosteroids by nebulizer
E)Discontinue corticosteroid treatment altogether after 24 hours

ANSWER: C
Systemic corticosteroid therapy reduces the hospital length of stay in patients with acute
COPD exacerbations (SOR A). Oral therapy has been shown to be as effective as the
intravenous route in patients who can tolerate oral intake (SOR B). A randomized, controlled
trial has demonstrated that 5-day courses of systemic corticosteroid therapy are at least as
effective as 14-day courses (SOR A). Inhaled corticosteroids are beneficial in some COPD
patients but nebulizers generally do not offer significant advantages over metered-dose
inhalers in most patients.
Depression

PRETEST2019
Q1-You diagnosed depression in a 35-year-old man, and began treatment with a selective
serotonin reuptake inhibitor (SSRI). His response has been excellent, and his depression has
been controlled for 2 months. At his follow-up visit, he lets you know that he has noted some
erectile dysfunction. You do not want to change or adjust his antidepressant, since his
response has been so good. Which of the following supplements has been used to treat the
sexual dysfunction associated with the use of SSRIs?
a. Fish oils
b. Ginseng root
c. Ginkgo biloba leaf
d. Kava kava
e. Valerian root

The answer is c.
Rates of complementary and alternative medicine use have increased significantly over the past
decade, and family physicians are increasingly asked to opine or even recommend
supplementation for patients. Fish oils (omega-3 fatty acids) are commonly used to treat
hypertriglyceridemia, prevent CHD and stroke, and have been used for many things, including
depression, diabetes, and to reduce the risk of developing age-related maculopathy among
other things. Ginseng root has been used to as a stress reduction agent and to improve physical
performance and stamina. Kava kava and valerian root have both been used to reduce stress
and as an anxiolytic or sedative. Ginkgo biloba leaf has been used to slow cognitive decline in
patients with dementia,but is also used to improve blood circulation in patients with
claudication. It has also been used to treat sexual dysfunction associated with the use of
SSRIs. The standard oral dosage is 60 to 120 mg of extract twice daily.
Q2-You are caring for a 54-year-old woman who is treating her mild depression with St.M
John’s wort. Which of the following is true regarding the mechanism of action for this
botanical medicine?

a. It selectively inhibits serotonin reuptake.


b. It selectively inhibits norepinephrine reuptake.
c. It selectively inhibits dopamine reuptake.
d. It inhibits serotonin, norepinephrine, and dopamine reuptake.
e. The mechanism of action is unknown.

The answer is d.
Throughout history, plants have been used for medicinal purposes. In
the United States, botanicals are used by one in five Americans at an annual total cost of more
than $4 billion. St. John’s wort is a perennial plant native to Europe that has been transferred
to and grown in North America. It is one of the most studied botanicals. There have been more
than 60 randomized controlled clinical trials, systematic reviews, and meta analyses that have
been done to establish its evidence base. A 2015 meta-analysis of 66 studies involving more
than 15,000 patients found that St. John’s wort is as effective for the treatment of depression as
other drug classes with significantly fewer dropouts due to side effects. The mechanism of
action for St. John’s wort is that it inhibits serotonin,
norepinephrine, and dopamine reuptake in the central nervous system (CNS), and may
modulate autonomic system reactivity. Overall, it is very well-tolerated.

Q3-You are treating a 60-year-old patient with end-stage ovarian cancer. You are concerned
that she may be developing depression. Which of the following would be the most reliable
symptom of depression in this patient?

a. Loss of appetite
b. Fatigue
c. Insomnia
d. Sadness
e. Anhedonia.
The answer is e.
It is commonly assumed that all patients with cancer are, and should be, depressed. Physicians
often do not recognize depression because they feel they would be depressed in the same
situation. While neurovegetative symptoms (loss of appetite, difficulty with concentration,
fatigue, or insomnia) are a compelling indication of depression in the physically healthy
patient, they may be less reliable for the diagnosis of depression in patients with advanced
cancer. Loss of appetite may be due to therapy, and fatigue may be due to sleep loss from
untreated pain. Sadness may be appropriate, given the diagnosis. Anhedonia is a useful, if not
the most useful symptom to monitor. Hopelessness, guilt, and a wish to die
are also predictive.
ABFM 2020
1- Your practice is starting a screening program for depression. Which one of the following is
a cardinal symptom of depression included in the Patient Health Questionnaire–2 (PHQ-2)
screening instrument?

A. Fatigue
B. Lack of appetite
C. Little interest or pleasure in doing things
D. Restlessness
E. Sleep disturbance

ANSWER: C
The two cardinal symptoms of depression are depressed mood and anhedonia. These are the
two criteria in the Patient Health Questionnaire–2 (PHQ-2) screening instrument for
depression. A positive result on the PHQ-2 should prompt further evaluation, including
questions about other symptoms such as energy level, appetite changes, sleep disturbance,
psychomotor changes, and suicidality.

2- Which one of the following is most likely to cause a false-positive urine drug screen for
amphetamines?
A. Amlodipine (Norvasc)
B. Bupropion (Wellbutrin)
C. Levofloxacin (Levaquin)
D. Pantoprazole (Protonix)
E. Sertraline (Zoloft)

ANSWER: B
False-positive results on drug testing can occur from cross-reactivity of commonly used
medications with the assay. Multiple commonly used medications are known to cause a false-
positive result for amphetamines, including bupropion, labetalol, ranitidine, and trazodone.
Amlodipine is not implicated in abnormal drug screen results. Levofloxacin can cause a false-
positive result for opioids. Proton pump inhibitors such as pantoprazole are known to cause a
false-positive result for cannabinoids. Sertraline can cause a false-positive result for
benzodiazepines.
3- The U.S. Preventive Services Task Force recommends screening for depression for:
A. All adults
B. All women but not men
C. Only adults with a family history of depression
D. Only adults with a known personal history of depression
E. Only adults with a history of disability, medical illness, complicated grief, chronic sleep
disturbance, and/or loneliness
ANSWER: A
Currently the U.S. Preventive Services Task Force recommends that all adults should be
screened for depression, as it is one of the leading causes of disability in persons older than 15
years of age (B recommendation). The optimal interval is yet to be defined. Depression is
more common in women, but screening is recommended for all adults. While a family history
of depression, a personal history of depression, disability, medical illness, grief, sleep
disturbance, and loneliness are all risk factors for depression, these factors do not need to be
present to screen.

ABFM2019
Q1-A 35-year-old gravida 2 para 2 reports diminished sexual arousal since initiating
antidepressant therapy with sertraline (Zoloft). She has normal menstrual cycles and does not
have any other symptoms.
Which one of the following would you recommend?

A. Black cohosh, 40 mg daily


B. Bupropion (Wellbutrin), 150 mg twice daily
C. Ethinyl estradiol, 0.5 mg daily
D. Ospemifene (Osphena), 60 mg daily
E. A testosterone patch, 300 g applied twice weekly.
ANSWER: B
Bupropion can improve antidepressant-related sexual arousal dysfunction (SOR B). Black
cohosh is considered a safe alternative for treating menopausal vasomotor symptoms, but not
for treating sexual arousal dysfunction in women who are premenopausal. Ethinyl estradiol
may be taken to improve sexual dysfunction related to menopausal symptoms. Vaginal
estrogen therapy is recommended over oral estrogen when vaginal dryness is the primary
symptom. Ospemifene is indicated for dyspareunia related to vulvar and vaginal atrophy due to
menopause. Testosterone has proven to be effective for treating menopause- related low
sexual desire but the evidence is limited due to the lack of long-term data. The Endocrine
Society recommends consideration of a 3- to 6-month course of testosterone specifically for
postmenopausal women with low sexual desire.
Q2- A 78-year-old male is brought to your office by his daughter. She is concerned that her
father is no longer attending his weekly cribbage and bingo games, has stopped bathing
regularly, and is eating much less.
Which one of the following would be most appropriate at this time?
A. Administering the CAGE screening questionnaire
B. Administering the PHQ-9 screening questionnaire
C. A trial of megestrol
D. A trial of nortriptyline (Pamelor)
E. MRI of the brain.

ANSWER: B

This elderly patient is exhibiting classic signs of depression. The PHQ-2 has a similar
sensitivity to the PHQ-9, but the PHQ-9 has a higher specificity in diagnosing depression
(91%–94% compared to 78%–92%) and can assist in diagnosing depression. In addition to the
PHQ-2 and PHQ-9 there are specific screening tools for use in the elderly population,
including the Geriatric Depression Scale and the Cornell Scale for Depression in Dementia.
Somatic issues and dementia can make it more difficult to screen for and diagnose depression
in this population. The CAGE questionnaire screens for substance abuse. Megestrol is used to
stimulate the appetite, but in this patient the appetite symptoms are likely secondary to
depression so treating the depression would be a more appropriate starting point. The tricyclic
nortriptyline is used to treat depression but is not first-line therapy, especially in the elderly. In
general, a more extensive medical history and a physical examination are indicated before
ordering MRI of the brain.

Q3-A 48-year-old male presents with psychogenic erectile dysfunction in conjunction with
depression. You decide that treatment of his underlying mood disorder is the best initial step.
Which one of the following antidepressants would be most appropriate?
A. Bupropion (Wellbutrin)
B. Citalopram (Celexa)
C. Nortriptyline (Pamelor)
D. Sertraline (Zoloft)
E. Venlafaxine (Effexor XR)
ANSWER: A
When psychogenic erectile dysfunction (ED) coexists with depression, treatment of the
underlying mood disorder is often an appropriate first step (SOR C).
An antidepressant that is less likely to worsen the ED, such as bupropion, mirtazapine, or
fluvoxamine, should be chosen. Antidepressants that are more likely to cause sexual side
effects should be avoided, including SSRIs, SNRIs, and tricyclic and tetracyclic
antidepressants. Phosphodiesterase-5 inhibitors are the first line of treatment for ED (SOR A)
and can be used effectively in men with depression, in combination with treatments for mood
disorder.

ABFM2017
Q4-A 70-year-old female presents with a complaint of recent fatigue and ―not feeling well.‖
She is unable to provide further details about her fatigue but states that she has not felt like
doing anything recently. She does not have shortness of
breath, chest pain, weight gain or loss, hair loss, or constipation. Her past medical history is
significant only for hypertension that is well controlled on hydrochlorothiazide. She is a
nonsmoker, does not drink alcohol, and has no history of drug use. She is retired and lives
alone. Her husband died 3 years ago and she has no family in the area but is involved in her
church, although she says she has not enjoyed her church activities over the past few months.
In addition to laboratory testing, which one of the following would be appropriate at this time?

A) Referral to Adult Protective Services


B) Evaluation with the Geriatric Depression Scale or the PHQ-9
C) Paroxetine (Paxil)
D) An EKG and stress testing
E) Pulmonary function testing
ANSWER: B
While some workup may be indicated, it is also important to realize that depression may
present with fatigue and vague symptoms, particularly in the geriatric population. The
Geriatric Depression Scale and PHQ-9 are good screening tests for depression and would help
to determine if more focus should be placed on a mood disorder. If further evaluation leads to
a diagnosis of depression an SSRI may be indicated but paroxetine is not the first choice in the
elderly because of its long half-life. In addition, its concentrations are increased 70%–80% in
the elderly compared to younger adults, its clearance is decreased, and it has multiple side
effects and drug interactions. An EKG and stress testing are unlikely to be beneficial in a
patient who has no significant symptoms and few risk factors for coronary artery disease.
Pulmonary function testing may be helpful later in the evaluation if the initial workup does not
reveal a diagnosis, but would not be indicated at this time given the lack of respiratory
symptoms. A referral to Adult Protective Services is not warranted for suspected depression
but would be an option if there were a reasonable suspicion of abuse in a patient and medical
causes have been ruled out.

Q5-While you are examining a 3-day-old for a routine newborn checkup, his mother, who is
also your patient, tells you that she has been crying very easily and feeling overwhelmed, and
that some of her friends have suggested she ask about an antidepressant medication. She has
no previous history of depression or other mood disorders. She does not have any thoughts of
self-harm or intent to harm the baby or anyone else. She is breastfeeding and is otherwise
healthy.
Which one of the following would be most appropriate at this point?
A) Reassurance that she likely has baby blues that will resolve in a few days
B) Encouraging her to make an appointment with a therapist if she does not feel better in 1
week
C) Referral to a counselor for psychotherapy
D) Use of a validated screening tool to evaluate for postpartum depression
E) Starting an SSRI for postpartum depression
ANSWER: D
While it is possible that this patient has baby blues, given the short duration of symptoms
proximate to delivery, more evaluation is needed to differentiate between baby blues, which
are very common and unlikely to contribute to the development of postpartum depression, and
an episode of postpartum depression that has much more significant implications. There is
more than one screening tool for postpartum depression, including the Edinburgh Postnatal
Depression Scale, the PHQ-9 questionnaire, and the Postpartum Depression Screening Scale.
An SSRI is the appropriate treatment for confirmed moderate to severe postpartum depression
along with psychotherapy, while mild to moderate postpartum depression can be treated with
psychotherapy or an SSRI. Referral to psychotherapy may be appropriate, but further
evaluation should be pursued in order to determine the best treatment plan. Due to the risks
associated with postpartum depression, an evaluation should be performed at this visit rather
than asking the patient to monitor symptoms.

Q6- A 75-year-old male presents with weight loss, fatigue, and sleep disturbance. He also has a
6-week history of feeling depressed and not enjoying life anymore. He is not suicidal. After an
appropriate evaluation to rule out other causes, you diagnose major depressive disorder.
Which one of the following agents is indicated as first-line pharmacotherapy for this patient?

A) Aripiprazole (Abilify)
B) Dextroamphetamine/amphetamine (Adderall)
C) Nortriptyline (Pamelor)
D) Sertraline (Zoloft)
E) Venlafaxine (Effexor XR)

ANSWER: D
Because of their favorable side-effect profile and low cost, SSRIs are the first-line agents for
late-life depression. SNRIs may be used as second-line agents when remission is not obtained
with an SSRI. Tricyclic antidepressants work as well as SSRIs and may be considered in
recalcitrant cases, but side effects may be troublesome, especially in this age group. The use of
stimulants in depressed older adults has not been well studied. Second-generation
antipsychotic agents may be used as an add-on to an SSRI or SNRI medication when the
depression is resistant. Side effects and long-term safety may be issues with this approach.
Q7-A 75-year-old male presents with weight loss, fatigue, and sleep disturbance. He also has a
6-week history of feeling depressed and not enjoying life anymore. He is not suicidal. After an
appropriate evaluation to rule out other causes, you diagnose major depressive disorder.
Which one of the following agents is indicated as first-line pharmacotherapy for this patient?

A) Aripiprazole (Abilify)
B) Dextroamphetamine/amphetamine (Adderall)
C) Nortriptyline (Pamelor)
D) Sertraline (Zoloft)
E) Venlafaxine (Effexor XR).

ANSWER: D
Because of their favorable side-effect profile and low cost, SSRIs are the first-line agents for
late-life depression. SNRIs may be used as second-line agents when remission is not obtained
with an SSRI. Tricyclic antidepressants work as well as SSRIs and may be considered in
recalcitrant cases, but side effects may be troublesome, especially in this age group. The use of
stimulants in depressed older adults has not been well studied. Second-generation
antipsychotic agents may be used as an add-on to an SSRI or SNRI medication when the
depression is resistant. Side effects and long-term safety may be issues with this approach.

Q8-A 14-year-old female with mild depression sees you for follow-up. After her last visit she
began a trial of psychotherapy but her mother asks about additional treatment options. The
daughter has no other significant past medical history.
Which one of the following is recommended as first-line pharmacotherapy for this patient?
a. Citalopram (Celexa)
b. Fluoxetine (Prozac)
c. Mirtazapine (Remeron)
d. Paroxetine (Paxil)
e. Venlafaxine (Effexor XR)

ANSWER: B
Expert panel guideline recommendations suggest fluoxetine should be the first- line
pharmacotherapy option for adolescents after a trial of psychotherapy. The patient should be
monitored weekly for side effects for a month after starting fluoxetine. If fluoxetine is
ineffective, sertraline and citalopram are recommended as alternatives. Venlafaxine should be
avoided in adolescents because it is associated with a statistically increased risk of suicidal
behavior or ideation.
ABFM2016
Q9- Which one of the following antidepressants should be avoided in the elderly?
A) Bupropion (Wellbutrin)
B) Sertraline (Zoloft)
C) Mirtazapine (Remeron)
D) Paroxetine (Paxil)
E) Venlafaxine.

ANSWER: D
Geriatric patients are at greater risk of adverse drug reactions compared to their younger
counterparts. Guidelines recommend several antidepressant agents as good first-line options,
including venlafaxine, bupropion, and mirtazapine. Among the SSRIs, citalopram,
escitalopram, and sertraline are all good choices, but paroxetine is associated with more
anticholinergic effects and should be avoided (SOR C).

Q10-A 45-year-old male reports being held up at gunpoint while on vacation 3 months ago.
Since that time he has had intrusive memories of the event, as well as nightmares. Further
questioning reveals that he has been having dissociative reaction flashbacks and meets the
criteria for posttraumatic stress disorder.
Which one of the following is the most appropriate pharmacotherapy for this patient?
A) Clonazepam (Klonopin)
B) Clonidine (Catapres)
C) Mirtazapine (Remeron)
D) Sertraline (Zoloft)
E) Risperidone (Risperdal)

ANSWER: D.
The dissociative reactions (flashbacks) in this patient are consistent with the diagnosis of
posttraumatic stress disorder (PTSD). The first-line medications for this disorder are SSRIs
and SNRIs. Paroxetine and sertraline have FDA approval for PTSD. Other antidepressants such
as mirtazapine would be second-line therapy.
The effectiveness of central "2-agonists such as clonidine are unknown, and even though
benzodiazepines might help with hyperarousal symptoms, they can worsen other symptoms.
Atypical antipsychotics such as risperidone are not recommended.
Q11- Which one of the following antidepressants can prolong the QT interval and should be
avoided with concomitant QT-prolonging agents such as atypical antipsychotics?
A) Bupropion (Wellbutrin)
B) Citalopram (Celexa)
C) Mirtazapine (Remeron)
D) Sertraline (Zoloft)
E) Venlafaxine.

ANSWER: B
Prolongation of the QT interval is an important medication adverse effect to consider. This is
particularly true in patients taking multiple medications, because this effect can be additive and
increases the risk of life-threatening arrhythmias such as torsades de pointes. Among
commonly used antidepressants, citalopram and escitalopram may prolong the QT interval.
Other SSRIs, as well as bupropion, venlafaxine, and mirtazapine, do not have this effect. Both
tricyclic antidepressants and antipsychotics, commonly used in patients also taking SSRIs, can
cause QT prolongation, making their combined use problematic.

Q12- In an adult patient with significant depression and no other health problems, which one of
the following is the best initial choice for pharmacotherapy?
A) Amitriptyline
B) Duloxetine (Cymbalta)
C) Fluoxetine (Prozac)
D) Mirtazapine (Remeron)
E) Trazodone (Oleptro)
ANSWER: C
Depression can be treated with counseling and/or medication. The choice of medication
depends on many factors including side effects, dosing schedule, cost, patient preference, and
comorbidities. In a patient with significant depression and no comorbidities, a second-
generation SSRI should be the initial medication choice. Fluoxetine is the only SSRI choice
listed. Duloxetine is an SNRI that may lead to sweating, tachycardia, urinary retention, and
elevation of blood pressure. It is indicated if the patient has fibromyalgia. Trazodone is
associated with somnolence, orthostatic hypotension, and priapism. It may be used in low
doses as an adjuvant in patients with insomnia. Mirtazapine can cause sedation, weight gain,
increased appetite, dry mouth, dizziness, and constipation. It is helpful for patients with
significant insomnia and loss of appetite. Amitriptyline is a tricyclic antidepressant and has
significant anticholinergic side effects. It is also associated with conduction abnormalities that
can lead to arrhythmias.
Q13- A 44-year-old male complains of feeling tired and sad for the last few months. He has a
past medical history of obesity, diabetes mellitus with painful peripheral neuropathy, and
seizure disorder. He has also noticed that he is not as interested in his usual hobbies and is
eating more than usual. You diagnose depression.
Which one of the following would be the most appropriate agent for this patient, considering
his comorbidities and symptoms?
A) Bupropion (Wellbutrin)
B) Citalopram (Celexa)
C) Duloxetine (Cymbalta)
D) Nortriptyline (Pamelor)
ANSWER: C
SSRIs and SNRIs are both effective in reducing depressive symptoms, but SNRIs have been
shown to be superior to SSRIs for management of neuropathic pain (SOR A). Bupropion
would effectively treat the patient’s depression and could cause weight loss, but it is
contraindicated in patients with seizure disorders (SOR A). Tricyclic antidepressants such as
nortriptyline could also help with the pain but might also worsen the patient’s obesity and
fatigue (SOR A).

Q14-During a routine health maintenance visit, a 24-year-old female admits that she is not
feeling well due to being overwhelmed with stress. She feels she has always worried more
than most people, but recent troubles at home and at work have made things much worse. She
says she is irritable with people around her, has trouble focusing at work, and feels fatigued
late in the day. Despite her fatigue, she has difficulty falling asleep at night. The patient denies
anhedonia, suicidal thoughts, or a persistently depressed mood. She limits her caffeine intake,
does not smoke or drink alcohol, and is not using any illicit drugs. In addition to
psychotherapy, which one of the following medications is recommended for this patient?
A) Alprazolam extended release (Xanax XR)
B) Clonazepam (Klonopin)
C) Gabapentin (Neurontin)
D) Quetiapine (Seroquel)
E) Sertraline (Zoloft)
ANSWER: E
This patient’s symptoms are consistent with the DSM-5 criteria for generalized anxiety
disorder. First-line treatments for this condition are SSRIs, SNRIs, and tricyclic
antidepressants. Quetiapine and gabapentin are considered second-line medications for anxiety
if control cannot be obtained with more traditional agents. Benzodiazepines such as
alprazolam and clonazepam are sometimes necessary for short-term control of anxiety
symptoms but are generally discouraged due to sedating side effects, the potential for abuse or
diversion, and gradual tolerance.
ABFM2014

Q15-A 21-year-old female sees you because of a depressed mood since the birth of her son 2
months ago. She is breastfeeding, and her baby is doing well. She reports no difficulties
sleeping, other than what is to be expected when caring for a newborn. She denies any suicidal
or homicidal ideation and has never had thoughts about hurting the baby. She has a history of
depression 2 years ago that was associated with starting college and feeling very isolated in the
dormitory.
She began taking sertraline (Zoloft), changed her schedule, and spent more time exercising.
Within 6 months her depression resolved and she stopped the medication. She reports this
current depression feels worse than her previous depression.
Which one of the following would be the most appropriate medication for this patient?
A) Amitriptyline
B) Diazepam (Valium)
C) Phenytoin
D) Sertraline
E) Zolpidem (Ambien).

ANSWER: D
SSRIs are the most commonly used medications for postpartum depression. They have fewer
side effects and are considered safer than tricyclic antidepressants, especially in depressed
women who may be at increased risk for medication overdose (SOR C). In one study, infant
serum levels of sertraline and paroxetine were undetectable. It is also recommended that a
woman with postpartum depression be started on a medication that she had taken previously
with a good response, unless there is evidence of potential harm to her infant (SOR C.)
Tricyclic antidepressants are excreted into breast milk and there is some concern regarding
potential toxicity to the newborn. Phenytoin, diazepam, and zolpidem are not antidepressants.
Phenytoin and diazepam are Category D for use in pregnant women. Diazepam is potentially
toxic to infants and can accumulate in breastfed infants, and it is not recommended for
lactating women (SOR C). Zolpidem is category B in pregnancy and probably acceptable for
use in lactatingwomen if clinically indicated
Q16-An 85-year-old male is brought to your office by his family because they are concerned
that he may be depressed. Which one of the following is most likely in a depressed patient in
this age group?
A) Suicidal ideation
B) Somatic symptoms
C) Depressed mood
D) Preoccupation with guilt.

ANSWER: B
Somatic complaints are seen in up to two-thirds of primary care patients with depression, and
are more likely in certain groups, including pregnant women, children, the elderly, and low-
income groups.

Q17- An obese 70-year-old male with chronic pain due to osteoarthritis complains of fatigue,
anhedonia, hypersomnolence, and increased appetite. Which one of the following would be
the best pharmacologic agent for this patient?
A) Duloxetine (Cymbalta)
B) Mirtazapine (Remeron)
C) Citalopram (Celexa)
D) Paroxetine (Paxil)
E) Nortriptyline (Pamelor).

ANSWER: A
The best pharmacologic agent for this patient is duloxetine, as it is indicated for both
depression and chronic pain and is unlikely to cause weight gain. The other agents listed can
cause weight gain to varying degrees, and the tricyclic antidepressant nortriptyline is on the
Beers list of drugs not recommended for elderly patients (SOR A
Dermatological problems
Pretest2019
Q1- You are seeing a 25-year-old male patient with a rash. It began as pink spots on his
extremities including on his palms and soles, but the lesions have begun to coalesce and
become purple in color. He recently returned from a hiking trip in the mountains. Which of the
following is the most likely cause?
a.Lyme disease
b.Rocky Mountain spotted fever
c.Tularemia
d. Brown recluse spider bite

The answer is b.
The skin lesions of Rocky Mountain spotted fever are typically red macules on peripheral
extremities that become purpuric and confluent. Lyme disease typically presents as a slowly
spreading annular lesion—erythema chronicum migrans. Tularemia is characterized by pain
and ulceration at the bite site. Brown recluse spider bites most often present as local pain and
itching, then a hemorrhagic bulla with surrounding erythema and induration. The black widow
bite is characterized by a mild prick followed by pain at the bite site. Left untreated, fatality
rates in Rocky Mountain spotted fever are more than 70%.

Q2-You are caring for a person who presents with severe symptoms. He started with fatigue,
myalgias, arthralgias, headache, and low-grade fever several weeks ago. He also noted a
“rash” on his upper back near the right scapula that looked “like a bull’s eye.” That rash has
since resolved. Currently, he complains of musculoskeletal pain and attacks of joint pain and
swelling for the past week, and today he reports pleuritic chest pain. On examination, he has
lymphadenopathy, tenderness in his joints, and right axillary adenopathy. You also notice a
friction rub. What is the best treatment for this condition?

a. Doxycycline for 10 to 14 days


b. Amoxicillin for 10 to 14 days
c. Tetracycline for 2 to 3 days after the patient becomes afebrile
d. Streptomycin intramuscularly for 1 week
e. Ceftriaxone intravenously for 2 to 3 weeks
The answer is e.
The patient in the question likely has Lyme disease based on his early constitutional
symptoms and rash consistent with erythema chronicum migrans. Based on his current
symptoms, he likely has early disseminated disease. This is characterized by multiple system
involvement, lymphadenopathy, musculoskeletal pain, arthritis, and pericarditis. Treatment of
Lyme disease is dependent on the stage of the disease. Early localized disease can be treated
with oral antibiotics (doxycycline for 10 to 14 days). Longer courses of therapy are indicated
in more complicated disease. Amoxicillin can be used in pregnancy and cefuroxime is
acceptable alternative but much more costly. Early disseminated disease is treated with oral or
IV therapy for 2 to 3 weeks. Rocky Mountain Spotted Fever is treated with doxycycline that
continues 2 to 3 days after the patient is afebrile, and tularemia is treated with streptomycin
intramuscularly.

Q3-You are seeing a patient who is complaining of an itching scalp. There are erythematous
papules on her scalp especially behind the ears, and you note small black bulbs at the bases of
several hair follicles. Which of the following is the preferred treatment option for this
condition?

a.Extermination of the home


b.Permethrin 1%
c.Permethrin 5%
d.Lindane 1%
E.Oral ivermectin (Stromectol)

The answer is b.
This case describes the typical presentation and physical examination findings of head lice,
including the typical erythematous papular rash and “nits” on the hair follicles. Treatment
options include permethrin and lindane. The preferred treatment is permethrin 1%. Permethrin
5% is a second option, and lindane 1% is a third option. If treatment failure occurs, a second-
line medication is 0.5% malathion lotion. Extermination would be appropriate for flea
infestation, not for head lice. Oral ivermectin is effective for scabies, not for head lice, but is
not approved by the US Food and Drug Administration (FDA).
Q4-A 16-year-old camp counselor sees you to evaluate a severely pruritic rash. You note
pruritic erythematous papules in between his fingers, on his wrists, and around his waist. For
which of the following is this distribution characteristic?

a. Flea bites
b. Bedbugs
c. Body lice
d. Scabies
e. Chigger bites

The answer is d.
This case describes the classic distribution of scabies. Sarcoptes scabiei burrow into
intertriginous areas, wrists, or areas where clothing is tight next to the skin. The lesions of
chigger bites are similar, but bites are typically found in a linear pattern over wrists, ankles,
and legs. Bedbugs typically infest unclothed areas— the neck, hands, and face. Fleas typically
bite the lower extremities, and lesions from body lice would not follow the pattern described.

Q5-A 34-year-old woman presents to your office with a complaint of 4 days of fever, itchy
rash, “pink eye,” and aching in her joints. She has no prior medical
history and has not traveled recently, but she does note that her husband has just returned home
from a business trip to Brazil. On examination, you note a temperature of 101.3°F and a
diffuse maculopapular rash. There is no erythema or swelling of the joints or soft tissues.
Which of the following conditions is most likely?

a. Disseminated gonococcal disease


b. Dengue fever
c. Zika virus
d. Chikungunya fever
e. Yellow fever

The answer is c.
Zika virus is a flavivirus transmitted by the Aedes mosquito. Most infections are
asymptomatic, but when they occur, symptoms can include acute fever, maculopapular rash,
nonpurulent conjunctivitis, and arthralgias. Sexual transmission and vertical transmission do
occur. Historically, the virus was isolated to Africa and Asia but recently has caused outbreaks
in the Western Hemisphere, including a large outbreak in Brazil. Disseminated gonococcal
disease causes arthralgias and tenosynovitis, but not conjunctivitis. Dengue fever and
Chikungunya fever share similar features of arthralgias, rash, and fever, but Chikungunya
fever is typically associated with GI and neurologic symptoms.
Dengue fever is a biphasic illness with the rash and conjunctivitis appearing after resolution of
the fever. Yellow fever does not cause a rash in mild cases and is characterized by jaundice
and hemorrhage in more severe cases.

Q6-. You are seeing a 21-year-old patient with pruritic, erythematous papules in clusters on his
ankles and legs. He noticed the rash the day after he stayed in a hotel on his way back from a
Spring Break vacation in Florida. Based on this history and description, which is the most
likely culprit?

a. Flea bites

b. “Hot tub” folliculitis

c. Spider bites

d. Scabies

e. Lice

The answer is a.
Insect bites are typically pruritic erythematous papules or vesicles and are sometimes difficult
to differentiate. Location and distribution are helpful differentiators. Flea bites often occur in
clusters, and are typically on the lower extremities, as described in this question. Bedbug bites
are typically on the hands, face, and neck. Spider bites are generally not in clusters. Scabies are
generally found in warm areas of the body where clothing is tight against skin (belt line,
wrists) or where skin touches skin (in between fingers). The itching from lice generally begins
approximately 2 to 3 weeks after infestation, and may not limit distribution to the lower
extremities. Hot tub folliculitis is not clustered, and is associated with hair follicles.
Q7-. A 20-year-old man presents to you 30 minutes after being stung by a bee on his right
thigh. He was stung by a bee twice last year. The first sting caused a 3-cm
× 3-cm area of erythema, induration, and pain around the sting site. The second sting caused a
similar 5-cm × 7-cm area. When you examine him, he has an expanding 2-cm × 2-cm area of
erythema, induration, and pain around the sting site on his thigh. He reports pruritis, fatigue,
and some nausea, but denies dyspnea. Which of the following is true?

a.This is a typical local reaction, and should spontaneously resolve within hours.
b.This is a large local reaction, and the patient has minimal risk for the development of
anaphylaxis upon subsequent exposure.
c.This is a large local reaction, and the patient is at significant risk for the development of
anaphylaxis upon subsequent exposure.
d.This is considered a toxic systemic reaction, and increases his risk for anaphylaxis if he is
exposed in the future.
e.This is considered a mild anaphylactic reaction.

The answer is b.
Typical local reactions to stings include swelling, erythema, and pain at and around the site of
the sting. In general, they resolve quickly and minimal analgesia is all that is necessary. Large
local reactions include extended areas of swelling that last several days. They are not allergic
in origin and carry a minimal risk of anaphylaxis upon reexposure. Toxic systemic reactions
are associated with nausea, vomiting, headache, vertigo, syncope, convulsions, and fever.
Pruritis, erythema, and urticaria are less common. Persons who have a toxic reaction are at risk
for anaphylaxis with subsequent stings. The reaction described in the above question is not
anaphylactic in nature.
Q8-. You are seeing a 14-year-old high school wrestler for a skin condition. About a week ago,
he noted a patch of erythematous skin on his right thigh. The patch has enlarged since he first
noted it, and the central part of the lesion seems to be clearing. He reports that it is mildly
pruritic. You scrape the lesion and evaluate the shavings under the microscope using
potassium hydroxide (KOH). The slide is shown below. Which of the following is the most
likely diagnosis?

(Reproduced, with permission, from Usatine RP, Smith MA, Chumley H, Mayeaux EJ Jr,
Tysinger J. The Color Atlas of Family Medicine. New York, NY: McGraw-Hill; 2009: 542.)

a. Tinea corporis
b. Tinea cruris
c. Pityriasis rosea
d. Nummular eczema
e. Impetigo

The answer is a.
Tinea infections are common, and may be spread by close person-toperson contact (as in
school wrestling). The classic tinea lesion is well-demarcated and annular with central
clearing, erythema, and scaling of the periphery. This can often be confused with eczema or a
bacterial skin infection, but by scraping the lesion and visualizing hyphae with microscopic
examination, the diagnosis of tinea can be confirmed. Tinea cruris occurs in the groin, not on
the thigh; pityriasis rosea has a different classic appearance. Most tinea infections, including
tinea corporis, are treated using topical antifungals.
Q9. You are caring for a teenager who complains of acne. It is most apparent on his forehead
and his cheeks. It is causing a great deal of stress in his life and impacting his selfconfidence.
Of the following, which is most likely a contributing factor to his condition?

a. Leaning his face on his hands while sitting at his desk at school
b. Eating fast food
c. Not washing his hair often enough
d. Eating chocolate
e. Not eating enough vegetables
The answer is a.
Acne is associated with many myths regarding its cause. The true cause is multifactorial, but
familial factors are involved. The key factors are follicular keratinization, androgens, and
Propionibacterium acnes. In acne, the keratinization pattern in the pilosebaceous unit changes,
and keratin becomes more dense, blocking the secretion of sebum. The keratin plugs are called
“comedones.” Contributory factors to acne include certain medications, emotional stress, and
occlusion and pressure on the skin, such as by leaning the
face on the hands (acne mechanica). Acne is not caused by dirt, chocolate, greasy foods, or the
presence or absence of any foods in the diet.

Q10. You are caring for a 20-year-old male patient who is concerned about facial acne. He has
had moderate symptoms since his teenage years, but has not ever tried a formalized treatment
regimen. He is otherwise healthy and developmentally normal. Which of the following tests or
set of tests is best to help guide treatment of his disorder?

a. Free testosterone.
b. Dehydroepiandrosterone sulfate (DHEAS).
c. Free testosterone and DHEAS.
d. Free testosterone, DHEAS, follicle-stimulating hormone, and luteinizing hormone.
e. No laboratory examinations are required.

. The answer is e.
In the evaluation of acne, laboratory examinations are generally not required, unless history
and physical examination indicates the need to exclude hyperandrogenism and/or polycystic
ovarian syndrome. In the vast majority of acne patients, the hormone levels are normal.
Q11- You are caring for a 13-year-old girl with acne. She is becoming increasingly concerned
about her appearance and is worried about getting teased at school because of her skin. After
assessment, you diagnose her with moderate acne. Of the following treatment regimens, which
would be best for her at this time?

a. Topical antibiotics
b. Benzoyl peroxide gel
c. Topical antibiotics and benzoyl peroxide gel
d. Topical antibiotics, benzoyl peroxide gel, and topical retinoids
e. Oral antibiotics, benzoyl peroxide gel, and topical retinoids

The answer is e.
For mild acne, combination therapy with topical antibiotics and benzoyl peroxide gels, with or
without topical retinoids work best. While any of the individual components will work on
their own, there is a synergistic effect when used in combination. It is important to let patients
know that improvement occurs over a period of 2 to 5 months, and may take even longer for
noninflamed comedones. Topical retinoids should be applied in the evening, and the benzoyl
peroxide and topical antibiotics should be applied during the day. For moderate acne, oral
antibiotics should be added to a topical regimen. Minocycline is most effective, but
doxycycline is also effective. Use of oral isotretinoin has become more common to prevent
scarring in moderate acne, and if listed may have also been an appropriate choice.

Q12-. You are caring for a 16-year-old girl with moderate acne. Her current regimen includes
topical retinoids, benzoyl peroxide gel, and oral minocycline, but after 6 months on this
regimen, she has not had improvement. You are considering treatment with oral isotretinoin
(Accutane). In addition to ensuring that pregnancy is prevented during her therapy, which of
the following must occur during her therapy?

a. She must avoid Tylenol use.


b. She must stop wearing her contacts.
c. She must have transaminases checked regularly.
d. She must be screened for depression every 3 months.
e. She must not use topical glucocorticoids.
The answer is c.
The indications for oral isotretinoin include nodular acne, severe acne, or moderate
recalcitrant acne. The patient must have demonstrated resistance to other acne therapies,
including systemic antibiotics. Isotretinoin is teratogenic, and therefore pregnancy must be
prevented during its use. In addition, since both tetracycline and isotretinoin cause
pseudotumor cerebri, the two medications should never be used together. Hepatotoxicity is a
rare side effect in people using isotretinoin, but patients can still use Tylenol while taking
isotretinoin.
Transaminases should be checked before and during therapy, as some patients develop
significant elevations that normalize with dosage reduction. Dry eyes are a side effect, and
patients may have more difficulty with contacts, but they can still be worn. There are some
reports of depression while on the medication, but there are no guidelines about screening
throughout therapy, as this is a rare occurrence. Topical glucocorticoids are safe for use during
therapy and are sometimes used if eczematous rashes occur during treatment.

Q13-. You are caring for a 45-year-old woman who reports a 2- to 3-year history of episodic
flushing of her cheeks, nose, and forehead. Over the last several months, this has been more
constant, and she has developed papules and some pustules on her cheeks. Her picture is shown
below:
Which of the following is the most effective treatment for
this condition?
a. Topical metronidazole cream
b. Topical sodium sulfacetamide
c. Topical antibiotics
d. Topical steroids
e. Oral antibiotics
The answer is e.
The patient in the picture has rosacea. Although it is often considered along with acne, rosacea
is a distinct entity. Comedo formation, the hallmark of acne vulgaris, is absent in rosacea. In
stage I, there is persistent erythema, generally with telangiectasia formation. Stage II is
characterized by the addition of papules and tiny pustules. In stage III, the erythema is deep
and persistent, the telangiectasias are dense, and there may be edema of the central part of the
face due to sebaceous hyperplasia and lymphedema (rhinophyma and metophyma).
Management may include topical or oral therapies. Topical metronidazole, antibiotics, and
sodium sulfacetamide can work, but oral antibiotics are more effective than topical treatments.
Minocycline or doxycycline are very effective first-line therapies. Topical steroids are not
generally effective.
Q14-. You are seeing a 17-year-old woman for a rash. She was helping her father rake leaves
in their yard this past weekend, and now, 4 days later, complains of a severely pruritic rash. It
is on her legs, arms, and face. A picture of her leg is below. She reports that she hasn’t slept in
the last couple of days because the itching is keeping her awake. Which of the following is the
most effective treatment?

a. Topical glucocorticoid ointment


b. Topical glucocorticoid gel
c. Low-dose oral prednisone for 1 to 2 weeks
d. High-dose oral prednisone tapered over 2 weeks
e. Oral tacrolimus
The answer is d.
The picture shown is classic for allergic phytodermatitis (APD) or allergic contact dermatitis
caused by plants. It is caused by plants from the anacardiaceae family (poison ivy, poison oak,
poison sumac) among others. When the plant brushes against exposed skin, it gives rise to
linear lesions as in the picture, but most all lesions start as erythematous patches, progressing
to papules or plaques, leading to vesicles and/or bullae, that may become eroded and crust.
Diagnosis is by history and physical findings only. Topical therapy is effective, but for severe
cases systemic therapy is more effective. Oral therapy should begin with high doses of
glucocorticoid and taper over a 1- to 2-week period. Tacrolimus can be effective, but is not as
effective as oral steroids.

Q15. A 55-year-old male patient comes to you with concerns about a nodule underneath his
right eye. The nodule is shown below. He reports that it “popped up” a couple of weeks ago
and has been growing ever since. Which of the following is the most likely diagnosis?
a. Verruca vulgaris
b. Molluscum contagiosum
c. Keratoacanthoma
d. Nodular basal cell carcinoma (BCC)
e. Squamous cell carcinoma
The answer is c.
The patient shown has a keratoacanthoma. Often, these are difficult to distinguish from basal
cell cancers, nodular squamous cell cancers, or molluscum by appearance alone, but the
history is quite different. Keratoacanthoma are characterized by rapid growth. Basal cell
cancers and squamous cell cancers are slowly evolving. Verruca do not generally have the
depressed center or the pearly borders. Molluscum do have a central dimple, but do not have
such a significant keratotic plug. Keratoacanthoma were formerly considered a pseudocancer,
but are now regarded as a variant of squamous cell carcinoma. Most will spontaneously
regress in 6 to 12 months, but treatment by excision is often needed
Q16-You are caring for a 28-year-old man with a rash. The rash has been present for 3 to 4
years and has remained fairly stable. He describes the rash as a single patch that is mildly
itchy. On examination, you identify an erythematous scaly patch on his right elbow. You also
note pitting and subungual hyperkeratosis of the fingernails. Which of the following is the best
therapeutic choice for his rash?
a. Topical fluorinated glucocorticoids
b. Topical pimecrolimus (Elidel)
c. Oral penicillin
d. Oral retinoids (Accutane)
e. Oral methotrexate
.The answer is a.
The patient described has psoriasis. While the skin lesions could be confused with eczema,
fungal dermatitis, or other lesions, the history and the nail involvement should point to the
correct diagnosis. For localized skin rashes, topical corticosteroids are appropriate therapeutic
agent. Topical pimecrolimus is effective for inverse psoriasis (located on the perianal and
genital regions) or on the face and ear canals, but is generally not used for lesions on the trunk
or extremities. There is no place for antibiotics in treatment, except in the case of guttate
psoriasis, a form that follows streptococcal infection and appears as multiple teardrops that
erupt abruptly. Oral retinoids and methotrexate are both used to treat generalized psoriasis,
and help with nail involvement, but would notbe a first-line therapy for a localized rash.

Q17. You are talking with a 24-year-old man who reports an outbreak of a mildly pruritic rash.
The rash initially began with a large pink patch on his chest, to the right of his sternum. About
a week later, he noted a more generalized eruption. The rash is shown below. Which of the
following treatments is indicated?
a.Antihistamines
b.Antibiotics
c.Antivirals
d.Antifungals
e.Cyclosporine
The answer is a.
The rash shown is classic for pityriasis rosea, a self-limited papulosquamous eruption. The
most probable cause is a reactivation of human herpesvirus 7 or human herpesvirus 6. The
classic history includes a single herald patch (an oval, slightly raised plaque with scale)
followed in the next 1 to 2 weeks with a more generalized eruption. It will spontaneously
resolve in 6 to 12 weeks, and recurrences are uncommon. The treatment is symptomatic and
includes antihistamines or corticosteroids to relieve itch. There is no role for the other agents
listed.
Q18. You are seeing a young child whose mother brings him in with a rash. It developed on
his upper lip underneath his nose. He has recently had cold symptoms with a runny nose. On
examination, you note erythematous scattered discreet lesions on his upper lip, chin, and
around his nares. Many have a golden- yellow crust. Which of the following is the most likely
cause of this rash?
a. Contact dermatitis
b. Infection with S aureus
c. Infection with an Enterococcus species
d. Infection with H influenzae
e. Infection with a Pseudomonas species

The answer is b.
The rash described is consistent with impetigo. This diagnosis should be considered in the face
of well-demarcated erythematous lesions that, when disrupted, develop a secondary golden
crust. The lesions have a predilection for traumatized skin, in this case where nasal discharge
has disrupted the skin surface. Most cases are due to streptococci and S aureus. Impetigo
responds well to topical antibiotics like mupirocin applied to the lesion. Treatment of the nares
will help treat colonization with S aureus.

Q19. After returning from a ski trip in the mountains, your 35-year-old patient developed a
rash. He has multiple erythematous papules and pustules over his legs, arms, and chest. They
are not pruritic and do not seem to be spreading. He denies any new soaps, lotions, foods, or
medications. He did spend time in a hot tub on the trip. Which of the following treatments is
the best first-line therapy for this patient?
a.Reassurance and follow-up if no improvement
b.Topical steroid medication
c.Systemic steroid medication
d.Topical antibiotics with activity against Streptococcus and Staphylococcus species
e.Oral antibiotics with activity against Pseudomonas species
The answer is a.
The patient described has “hot tub folliculitis.” The infection is generally caused by exposure
to water that is contaminated by Pseudomonas aeruginosa or Pseudomonas cepacia. This
occurs when water is inadequately chlorinated. The condition is usually self-limited, and
therefore reassurance is all that is necessary. Antibiotic therapy is only indicated in recalcitrant
cases, or if patients are symptomatic. If patients are symptomatic, an oral fluoroquinolone with
activity against Pseudomonas is an appropriate treatment.
Q20- You are seeing a 26-year-old woman with a lesion on her lip. It began with a burning at
the site of the lesion, then an eruption of vesicles. She describes outbreaks of this rash many
times in the past. The rash is shown below. Which of the following is true of the treatment for
this infection?

a.Oral antiviral agents are more effective treating recurrences


than they are treating primary infections.
b.Oral therapy begun within 2 days of onset is the best
treatment for recurrent outbreaks.
c.Chronic suppression with daily therapy is beneficial for oral herpes.
d.Acyclovir resistance makes it a poor choice for therapy.
e.Docosanol cream (Abreva) is ineffective.

The answer is c.
The description and picture are consistent with a recurrence of HSV infection. Management
includes topical antiviral therapy or oral antiviral therapy. Antiviral agents are more effective
treating primary infections than recurrent infections. Pulse dosing (treating at the first sign of
an outbreak) may shorten or reduce the severity of an eruption, but are not otherwise
beneficial. Chronic suppression is best to decrease the frequency of symptomatic recurrences
and asymptomatic viral shedding. Acyclovir resistance is extremely rare. Docosanol cream
(Abreva) is available without a prescription. One randomized controlled trial showed a faster
healing time when compared with placebo cream, but in both arms of the trial, groups healed
within 10 days. If applied five times a day within 12 hours of onset, Abreva is safe and
somewhateffective.

Q21-. An otherwise healthy 61-year-old male patient complains of a burning sensation on the
back of his right shoulder for 24 hours, and subsequent development of a rash. On
examination, you note grouped vesicles on an erythematous base in a dermatomal pattern that
does not cross the midline.Which of the following is true about this condition?

a.The varicella (chicken pox) vaccine has led to an increase in cases of this rash in the general
population.
b.Compared with no treatment, antiviral therapy has been shown to decrease the incidence of
postherpetic neuralgia 6 months after treatment.
c.Treatment with corticosteroids will not reduce the prevalence of postherpeticneuralgia.
d.Narcotics are rarely necessary for this condition.
e.Antiviral resistance is common.
The answer is c.
The patient described has herpes zoster, or “shingles.” Antiviral therapy is the treatment of
choice, and can decrease the time for lesion healing and shorten the overall duration of pain if
initiated within 72 hours of onset of symptoms. The varicella (chicken pox) vaccine has not
led to an increase in zoster in immunized patients or in the general population. In fact,
vaccination has led to an overall decrease in zoster. Early antiviral treatment does not reduce
the pain of postherpetic neuralgia at 6 months, when compared with people that did not have
early antiviral treatment. While corticosteroids have not been shown to reduce the prevalence
of postherpetic neuralgia, they may accelerate times to crusting and healing, return to full
activity, and resumption of a good sleep pattern. Narcotic analgesics are appropriate, as the
pain can be severe in the acute phase. Antiviral resistance is uncommon in this setting.

Q22-. You are seeing a 5-year-old otherwise healthy girl whose mother brings her in for
evaluation. The mother reports that last week, she had 3 days of upper respiratory tract
symptoms and associated headache, sore throat, abdominal pain, and some loose bowel
movements. That resolved a few days ago, but today she developed a facial rash which was
concerning to the mother. A picture of therash is shown below:

Which of the following is the most likely cause of her


symptoms?

a.An enterovirus
b.A parvovirus
c.A parainfluenza virus
d.A varicella virus
e.Cytomegalovirus

The answer is b.
The clinical presentation and photograph are both classic for erythema infectious, otherwise
known as “Fifth Disease.” It is caused by parvovirus B19. The disease is common with most
individuals being infected during school years. It is contagious via the respiratory route, and
generally is most likely to occur in the late winter, spring, and early summer months.
Q23-The patient described in the question above is treated appropriately and recovers as
expected. Which of the following is true of the weeks following theacute symptoms?

a. The patient should be tested for anemia.


b. The patient is contagious as long as the rash persists.
c. The rash may recur in association with bathing.
d. The patient should continue antivirals as long as the rash persists.
e. Serum should be tested for immunoglobulin G (IgG) to ensure future immunity.

The answer is c.
Fifth disease is self-limited, and antiviral medications are not indicated. Patients with
preexisting anemia or symptoms of anemia should be tested for anemia, but in otherwise
healthy patients, there is no need for laboratory testing. After the viral prodrome, the patient is
no longer contagious. Since the illness is biphasic with the rash in phase 2, by the time the
rash appears, children are past the infectious state and can attend school and other activities.
Serum testing for immunity is not necessary. The rash associated with parvovirus B19 can
recur over several weeks in association with exercise, sun exposure, warm water (like in a
bathtub), or stress.

A24-. You are seeing a young man who is complaining of patchy hair loss. He denies pulling
the hair and complains that his scalp is itchy and flakey. His scalp is shown below. Which of
the following is the treatment of choice for this condition?

a.Selenium sulfide lotion, applied daily for 4 to 8 weeks


b.Ketoconazole (Nizoral) shampoo, applied daily for 4 to 8 weeks
c.Clotrimazole (Lotrimin) cream, twice daily for 4 to 8 weeks
d.Griseofulvin tablets, daily for 4 to 8 weeks
e.Fluconazole (Diflucan) tablets, daily for 4 to 8 weeks
f.
The answer is d.
The picture shows tinea capitis. Systemic therapy is necessary for a cure, but concurrent use of
topical ketoconazole shampoo or selenium sulfide lotion may be used concurrently.
Griseofulvin is considered the treatment of choice in the United States, and should be used for
8 to 16 weeks in children. Terbinafine can also be used, with no difference in efficacy.
Fluconazole is less effective, but is the only treatment currently FDA approved for children less
than 2 years old.
Q25-. You are caring for a 35-year-old woman who works in a veterinary hospital. She noted
an itchy rash on her calf approximately 3 to 4 weeks ago. It started as a small pink circular
lesion, but is spreading, and has not responded to over-the- counter antifungal ointments. The
rash is shown below.

Which of the following therapies is likely to be most effective?

a.Topical clotrimazole (Lotrimin)


b.Topical ketoconazole (Nizoral)
c.Topical terbinafine (Lamisil)
d.Oral griseofulvin
e.Oral griseofulvin with oral steroids

The answer is d.
Tinea corporis, otherwise called “ringworm” is most commonly caused by Trichophyton
rubrum. It appears as a well-demarcated plaque with central scaling. It is usually pruritic.
Tinea infections can also be caused by T tonsurans (tinea capitis), T mentagrophytes (tinea
cruris), and M canis (inflammatory tinea infections). Candida species are more commonly
seen as a superinfection of tinea pedis. Treating tinea corporis with any of the many topical
antifungal preparations will likely lead to cure. However, in the case described above, the
patient had been using topical antifungals without success. In this case, an oral antifungal is
necessary. Itraconazole (Sporanox), fluconazole (Diflucan), griseofulvin, ketoconazole
(Nizoral), and terbinafine (Lamisil) are all reasonably effective. There is no need to use oral
steroids in addition to an oral antifungal.

Q26- You are caring for a 5-month-old infant whose mother complains that he has a diaper
rash. Examination of his diaper area reveals erythematous patches with skin erosion and
satellite lesions. He also has a white coating on his buccal mucosa. Which of the following
interventions is most likely to cure his rash?

a.Change to cloth diapers


b.Frequent gentle cleaning with commercially available wipes
c.Application of a barrier preparation (such as zinc oxide, petroleum jelly, or vitamin A & D
ointment)
d.Topical antifungal creams such as clotrimazole
e.Combination antifungal cream and oral nystatin
The answer is e.
The patient described appears to have diaper dermatitis, likely due to candida. Satellite lesions
can often differentiate candida infection from a contact dermatitis, and the concomitant thrush
increases the likelihood of the diagnosis. Diaper dermatitis is the most common dermatitis of
infancy. It is present in around 25% of children presenting for an outpatient visit. Treatment
should include frequent diaper changes, using disposable diapers, and frequent gentle cleaning
with lukewarm tap water rather than commercial wipes containing alcohol. Barrier
preparations are important, with pastes better then ointments, and ointments better than
creams. When candida is suspected, treatment should include a topical antifungal cream
applied before the barrier preparation, as barrier preparations alone will not be enough. If there
is concomitant thrush, oral nystatin should be added. If the inflammation is severe, a
combination antifungal steroid agent can be used, but the steroid should not be stronger than
1% hydrocortisone cream.
Q27- A 7-year-old girl’s mother brought her in to have you evaluate her rash. The child
describes significant itching for 3 days, and it seems to be worsening. She shares these
symptoms with several friends, all of whom are now complaining of an itchy rash after having
had a sleep over 3 weeks ago. On examination, your patient has slightly erythematous papules
in her interdigital spaces, wrists, ankles, and waist. You diagnose scabies. Which of the
following treatments is most effective?
a. Permethrin 5% cream (Elimite)
b. Oral ivermectin
c. Benzyl benzoate
d. Lindane cream
e. 10% sulfur
The answer is a.
Scabies is very common, with an estimated 300 million cases per year worldwide. Human scabies
is caused by the mite Sarcoptes scabiei, with mites spending their entire life cycle (around 30
days) within the epidermis. Male mites die after copulation, with female mites burrowing under
the skin to lay eggs. It is spread by direct skin-to-skin contact, and may be transmitted from
animals to humans. The normal incubation period is around 3 to 4 weeks after an initial
infestation. High- risk populations include young children, health care workers, homeless
persons, and institutionalized individuals living in crowded conditions. Pruritis is a hallmark of
the disease, and skin findings include papules, burrows, nodules, and vesiculopustules. Classic
distribution includes the interdigital spaces, wrists, ankles, waist, axillae, and groin. Palms and
soles may also be included. The most effective treatment is permethrin 5% cream, though
scabies resistance to permethrin is increasing. Oral ivermectin is reserved for resistant cases.
Other less-effective treatments include topical benzyl benzoate, lindane (no longer available in
the United States), and 5% to 10% sulfur in paraffin. Tea tree oil contains oxygenic terpenoids,
and has been found to have scabicidal activity. Treatment includes environmental
decontamination and all household family members and contacts should be treated.
Q28-. You are caring for a 25-year-old man who presents to you for evaluation of a new lesion
found on his groin. On examination, you find a single small umbilicated flesh-colored papule,
4 mm in size, in his pubic region. Which of the following is true?

a. A family history of skin cancer is likely in this person.


b. The patient is probably immunocompromised.
c. This is likely to be a sexually transmitted infection.
d. This can be spread through aerosolized droplets.
e. Treatment for this condition must be surgical.
The answer is c.
The lesion described is molluscum contagiosum. It can appear in individuals of all ages and all
races. The lesion is due to an infection with a poxvirus transmitted through direct skin-to-skin
contact. Lesions are common in children in a daycare or nursery school setting. However, in
adults and in the pubic region, they are likely sexually transmitted. Although the lesions can be
similar to those seen with basal cell cancers, the lack of telangiectasia is a diagnostic clue. The
lesions can occur in immunocompetent persons, but in patients who are immunocompromised,
they are generally more numerous and larger.Most lesions will resolve spontaneously within
months of appearance, but they can be treated with cryotherapy, cautery, or curettage.

Q29-. You are caring for a 72-year-old woman who reports a mole on the right side of her
nose that has been increasing in size. She says it has been there for at least 4 years, but is
enlarging and more noticeable. On examination, you see a nodule with raised lighter edges,
telangiectasias, and a slightly ulcerated center. Given this history and these clinical features,
what is the most likely diagnosis?

a.Intradermal nevus
b.Molluscum contagiosum
c.Basal cell carcinoma
d.Squamous cell carcinoma
e.Seborrheic keratosis

The answer is c
BCC is the most common cancer in humans. It is usually found on the head and neck, and is
generally slow growing with rare metastasis. The most common type is a nodular BCC, as
described in this question. Risk factors include advanced age, cumulative sun exposure,
radiation exposure, family history, and genetic predisposition. The differential diagnosis is
broad, and often biopsy is needed to confirm the diagnosis based on appearance, but certain
historical characteristics make BCC more likely. Intradermal nevi may be nodular, dome-
shaped, pearly, and have telangiectasias.
However, they will present with stable size and will lackulceration. Molluscum may have
pearly edges and a central umbilication, but will be less likely to have telangiectasias and will
be less likely in this age group and in this location. In an immunocompetent adult, they are
more likely to be sexually transmitted, and not on the nose. Keratoacanthoma (a subtype of
squamous cell carcinoma) may look very much like a nodular BCC, complete with pearly
edges and telangiectasias. However, it is usually very fast growing and will change and grow
more rapidly. Seborrheic keratosis is more likely to be a well-demarcated darker rough patch,
rather than the type of lesion described in this question.
ABFM 2020

172. A3-year-oldfemaleisbroughttoyourofficebyhermotherwitha3-dayhistoryofa rash.The


patient’smotherreportsthatthechildiseatinglessthanusualbuttolerating fluids. The mother also
reports that other children in the patient’s day care have a similar rash.
On examination the patient’s temperature is 38.2°C (100.8°F). The patient appears alert and
active. Examination of the pharynx reveals ulcerations in the posterior oral cavity and soft
palate. There is a maculopapular rash on the fingers, soles of the feet, and genitals. The
remainder of the examination is normal.
Which one of the following would be the most appropriate next step?
1. Symptomatic treatment only, and increased oral hydration
2. Amoxicillin twice daily for 7 days
3. Valacyclovir (Valtrex) twice daily for 7 days
4. Antinuclear antibody testing
5. Rapid plasma reagin testing

ANSWER: A
This patient has hand-foot-and-mouth disease frequently caused by enterovirus 71 or
coxsackievirus A16. It is most common in children under 5 years of age and occurs most often
in the fall and spring. It is characterized by painful maculopapular or papulovesicular lesions
on the hands and feet, and in the oral cavity. Lesions can also appear on the genitals, trunk, or
cheek. Management includes symptomatic treatment of pain and oral hydration. Antibiotics
and antiviral treatment are not recommended. Laboratory testing is not appropriate for this
condition.

198. A 27-year-old male has noticed some small bumps around the tip of his penis and is
concerned that he might have a sexually transmitted infection. An examination reveals a row
of approximately 2-mm, dome-shaped, skin-colored papules in a ring-like distribution around
the corona of the glans penis.
Which one of the following is the most likely diagnosis?
1. Angiokeratomas
2. Genital warts
3. Lichen nitidus
4. Pearly papules
5. Squamous cell carcinoma
ANSWER: D
Pearly papules are a benign, normal anatomic variant and are not sexually transmitted. They
are dome-shaped, skin-colored papules 1–4 mm in size with a ring-like distribution around the
corona of the glans penis. Angiokeratomas are red or blue papules that are well circumscribed
and 1–6 mm in size. Genital warts are soft, raised masses that can be pearly and smooth or
have a rough, cauliflower-like appearance, and are not confined to the penile corona. Lichen
nitidus consists of discrete, hypopigmented, 1-mm papules that are not confined to the corona
and can also occur on the upper extremities and abdomen. Squamous cell carcinoma may be
endophytic (ulcerated) or exophytic (thickened skin or wart-like growths that can progress to a
large, irregularly shaped, fungating mass).

108. A 25-year-old female is concerned about skin lesions that have repeatedly appeared and
resolved, sometimes lasting for months. She has tried treatment with topical over-the-counter
antibiotic ointments with no success. She states that her mother has similar lesions. On
examination her vital signs are unremarkable except for a BMI of 32 kg/m2. The examination
also reveals multiple deep-seated inflammatory nodules in both axillae that are up to 1.5 cm in
size and painful to touch. You also note some malodorous drainage and scars in her axillae
that she says are from previous lesions. No other areas are affected. She is afebrile and has no
other symptoms.
Which one of the following is the most likely diagnosis?
A. Acne conglobata
B. Cutaneous Crohn’s disease
C. Hidradenitis suppurativa
D. Pilonidal cyst
E. Recurrent primary bacterial abscess

ANSWER: C
Hidradenitis suppurativa, or acne inversa, is a chronic folliculitis affecting intertriginous areas,
causing deep scarring and affecting quality of life. Patients should also be screened for
depression. Hidradenitis suppurativa usually occurs between 18 and 39 years of age and
affects females and African-Americans more often. Risk factors include family history,
smoking, and obesity. Hidradenitis suppurativa is associated with several other comorbidities,
including Crohn’s disease and diabetes mellitus. Clinical features range from mild
inflammatory nodules to widespread abscesses, sinus tracts, and scarring, affecting different
parts of the body. Lesions occur most commonly in the axillae, but gluteal folds, the groin, the
perianal area, and the perineum can also be affected. The nodules are painful and may vary in
size (0.5–2 cm) and last days to months. The abscesses are typically sterile with purulent,
malodorous drainage.
The Hurley classification system defines the different stages (stage I–III) and treatment is
based on the current stage, which ranges from topical treatment with clindamycin to
combinations of topical and oral antibiotics and use of biologics. This patient presents with the
typical features of hidradenitis suppurativa, including its depth and a chronic and recurrent
course. Acne conglobata affects mainly the back, face, chest, and neck in men. Cutaneous
Crohn’s disease involves perianal lesions with fistulous tracts past the dentate line, which this
patient does not have. A pilonidal cyst usually contains hair and skin debris and is typically
located near the tailbone. A primary bacterial abscess does not feature accompanying nodules
and is superficial rather than deep, and is nonchronic in nature.

144. A36-year-old male presents with a6-week history of a mildly pruritic rash in his groin.
An examination reveals small red-brown macules and larger patches with a sharp border .A
Wood’s lamp examination reveals coral-red fluorescence. Which one of the following would
be the most appropriate treatment for this condition?
A. 0.1% triamcinolone cream
B. 2.5% hydrocortisone cream
C. Nystatin cream
D. Erythromycin gel
E. Fluconazole (Diflucan) orally

ANSWER: D
Erythrasma is a superficial infection caused by Corynebacterium minutissimum. It presents as
small, red-brown macules that may coalesce into larger patches with sharp borders. It
fluoresces coral red on Wood’s lamp examination. Cutaneous erythrasma is treated with
erythromycin (topical, twice daily until the rash resolves, or oral, 250 mg four times daily for
2 weeks). Topical clindamycin, Whitfield ointment, and antibiotic soaps may also be
beneficial. Triamcinolone cream, hydrocortisone cream, nystatin cream, and oral antifungals
are not effective treatments for this bacterial infection.

139. A 45-year-old female sees you for the first time for a health maintenance visit. She has a
history of chronic low and midback pain for several years, which she treats with ibuprofen,
muscle relaxers, and a heating pad. She takes a combined oral contraceptive pill to control
heavy menstrualcycles.Sherecentlywenttoanurgent care clinic for treatment of an upper
respiratory tract infection. She also mentions the gradual appearance of a rash on her back but
says that it does not bother her. A skin examination is not able for reticular brown
hyperpigmentation ofher entire back (shown below).
Which one of the following is the most likely cause of the
skin changes?
A. Erythema ab igne
B.Henoch-Schönleinpurpura
C. Idiopathic guttate hypomelanosis
D. Tinea versicolor

ANSWER: A
Erythema ab igne is characterized by reticular brown hyperpigmented skin changes at the site
of exposure to heat, which this patient has developed due to regular use of a heating pad.
Exposure to excess heat and humidity can predispose an individual to this condition. Henoch-
Schönlein purpura is a leukocytoclastic vasculitis and is characterized by palpable purpuric
lesions on the lower extremities and buttocks, usually sparing the trunk. It may be preceded by
a streptococcal or viral upper respiratory infection. Idiopathic guttate hypomelanosis consists
of 2- to 5-mm white spots with sharply demarcated borders on sun-exposed areas of the arms
and legs. Tinea versicolor is a fungal infection that is characterized by white, pink, or brown
circular macules most commonly located on the upper trunk.

A previously healthy 18-year-old female presents with finger pain. About 5 days ago she
started to have mild burning of her left distal index finger. Two days later she developed
worsening pain and redness of her fingertip. She does not remember injuring her finger or
having a similar problem previously. On examination you note erythema of the medial palmar
tip of her affected finger, with several vesicles that have opaque fluid in them. The distal
digital pulp is soft but tender.
Which one of the following is the most appropriate treatment for this condition?

A. Warm water soaks


B. Pain control and dressings
C. Antibiotics
D. Antifungals
E. Incision and drainage

ANSWER: B
This patient has herpetic whitlow, which is a viral infection of the distal finger caused by
herpes simplex. Primary herpetic whitlow is generally a self-limited infection. The
recommended treatment is pain management and keeping it covered with a dressing to prevent
transmission. Warm water soaks are useful to manage superficial hand infections but are not
indicated to treat herpetic whitlow. Herpetic whitlow is a viral infection, so antibiotics and
antifungals would not be beneficial, although antibiotics would be appropriate if a secondary
bacterial infection is suspected or if an abscess is confirmed by ultrasonography. Off-label use
of antiviral medications should be considered only for patients with recurrent lesions, those
with symptoms for less than 48 hours, and those who are immunocompromised. Incision and
drainage should not be performed because it increases the risk of bacterial superinfection.

A 34-year-old female has a “bump” on her middle finger (shown below) She thinks it has been
there approximately a month but says that it only recently began to bother her. She has not
tried to treat it. This lesion is most likely a:

A. Basal cell carcinoma


B. Dermatofibroma
C. Keratoacanthoma
D. Mucous cyst
E. Wart

ANSWER: D
This patient’s lesion is a digital mucous cyst, also known as a cutaneous myxoid cyst. Mucous
cysts most commonly occur on the dorsal surface of the distal phalanx. Toe lesions are less
common. The etiology is controversial. Treatment options include intralesional corticosteroid
injections, repeated puncture and drainage, or surgical excision (SOR A). The other options
listed are unlikely to be found on the finger with the exception of a wart, which would have a
verrucous texture and appearance. While basal cell carcinomas and dermatofibromas are also
smooth and nodular, neither are common on the fingers. Keratoacanthomas are smooth, dome-
shaped, red papules that often expand rapidly over a few weeks’ time on sun-damaged skin
and may have a central keratin plug. They are more common in older patients.
A 16-year-old female presents with chronic acne on her nose, forehead, and chin consisting of
a few comedones and a few mildly inflamed papules and pustules. She says it is minimally
improved after 12 weeks of daily adapalene 0.1% gel. There are no scars or cysts. The patient
would like to try to achieve better control.
Which one of the following would you recommend at this time?

A. Continue adapalene 0.1% gel for 12 more weeks


B. Add clindamycin (Cleocin T) 1% gel for up to 12 weeks
C. Add clindamycin 1% gel for maintenance
D. Stop adapalene 0.1% gel and start clindamycin 1% gel for maintenance
E. Stop adapalene 0.1% gel and start erythromycin 2% gel for maintenance

ANSWER: B
Family physicians are often asked to manage mild to moderate acne vulgaris. Topical retinoids
such as adapalene and benzoyl peroxide are first-line therapy and a trial of therapy is typically
8–12 weeks. Topical antibiotics may be added to topical retinoids or benzoyl peroxide to
achieve better symptom control. To decrease emerging antibiotic resistance, studies support
limiting antibiotic use to 12 weeks except in severe cases, not using antibiotics as
monotherapy, and using clindamycin rather than erythromycin. Adding clindamycin gel rather
than erythromycin gel for up to 12 weeks is recommended for this patient at this time.

ABFM2019
Q1-A 57-year-old female who spends a lot of time working in her yard has pigmented areas on
sun-exposed skin. She has several of these and would like to address them for cosmetic
reasons. An examination is consistent with solar lentigines. One of these on her posterior leg
has increased in size more rapidly.
Which one of the following would be the most appropriate next step for the lesion on the
posterior leg?
A. Topical hydroquinone (Lustra)
B. A topical retinoid
C. Cryotherapy
D. Laser therapy
E. A biopsy
F.
ANSWER: E
Solar lentigines occur on sun-exposed skin and are known commonly as liver spots. A biopsy
should be performed if they grow rapidly, change rapidly, are painful, itch, bleed easily, heal
poorly, or have an atypical or suspicious appearance. If no suspicious changes or symptoms
are present there are various options for treatment, including topical therapy with
hydroquinone or retinoids, or ablative therapy with chemical peels, cryotherapy, intense pulsed
light, or laser therapies.

Q2-During an outbreak of head lice at a local school the principal asks you for advice to send
home to the parents. In addition to treatment with topical permethrin (Nix), which one of the
following would you recommend?
A. Using a hairbrush to remove any lice or eggs
B. Applying petroleum jelly to the hair and scalp
C. Using a conditioner or a combined shampoo and conditioner when applying treatment
D. Washing clothing and bedding in hot water and drying with hot air
E. Treating household pets such as cats and dogs

ANSWER: D
The appropriate recommendation for head lice is to wash any recently used bedding and
clothing with hot water or expose them for 5 minutes to a temperature >130°F to kill lice and
eggs. Items that cannot be washed or dried in this manner or dry-cleaned should be sealed in a
plastic bag for 2 weeks.
Additional or alternative treatments include topical ivermectin, benzoyl alcohol, malathion,
and spinosad. Other recommended measures include removal of any visible nits (eggs) with a
nit comb, not a brush. Topical petroleum jelly is not an effective treatment. Conditioners can
interfere with the action of permethrin, decreasing its effectiveness. Human head lice are
specific to humans, so pets are not affected.

Q3-Immunotherapy can prevent recurrence of severe allergic reactions triggered by which one
of the following common allergens?

A.Penicillin
B.Poison ivy
C.Shellfish
D.Tree nuts
E.Wasp stings
ANSWER: E
Wasp stings, penicillin, shellfish, and tree nuts are all relatively common causes of severe
allergic reactions, including anaphylaxis. Immunotherapy is available and recommended for
the stings of insects, including wasps and bees, to prevent recurrent anaphylaxis (SOR B). It is
not available for penicillin, poison ivy, shellfish, or tree nut allergens. Poison ivy typically
causes a type 4 hypersensitivity contact dermatitis rather than anaphylaxis.
Q4-A 3-month-old white male is brought to your office for evaluation of three lesions on his
buttocks. The lesions began as vesicles that quickly progressed to flaccid bullae with sharp
margins and no surrounding erythema. The lesions ruptured and formed yellow crusts, which
oozed yellowish liquid.
Which one of the following would be the most appropriate treatment?
A. Oral azithromycin (Zithromax)
B. Oral penicillin V potassium
C. Topical bacitracin/neomycin/polymyxin B (Neosporin)
D. Topical mupirocin (Bactroban)
E. Topical nystatin
ANSWER: D
Bullous impetigo is caused by Staphylococcus aureus, which produces a toxin responsible for
flaccid bullae and is more likely to affect the intertriginous areas. This usually resolves within
3 weeks without scarring. Impetigo, either bullous or nonbullous, may be treated with topical
antibiotics such as mupirocin. Because of emerging drug resistance, oral azithromycin and
other macrolides should not
be used. Oral penicillin is no longer recommended because it is less effective than other
antibiotics such as amoxicillin/clavulanate, cephalexin, clindamycin, or dicloxacillin.
Bacitracin/neomycin/polymyxin B is not indicated for either form of impetigo. Nystatin is
recommended for Candida infections.
Q5-A 5-year-old female is brought to your office with a progressive rash on her legs (shown
below) and buttocks. No rash is noted above the level of the mid- torso. Her mother also
reports that the child had two episodes of bloody diarrhea 3 days ago. She also has abdominal
pain and on examination she has abdominal tenderness with no rigidity but some voluntary
guarding. You also note swelling and tenderness in her left wrist and right knee. A CBC,
platelets, prothrombin time, and partial thromboplastin time are normal. A urinalysis reveals
mild proteinuria and 5–10 RBCs/hpf.

Which one of the following is the most likely diagnosis?


A.Erythema infectiosum (fifth disease)
B.Gianotti-Crosti syndrome
C.Hemolytic uremic syndrome
D.Henoch-Schönlein purpura
E.Thrombotic thrombocytopenic purpura

ANSWER: D
Henoch-Schönlein purpura is an IgA vasculitis that is usually diagnosed clinically. It presents
as palpable purpura of the lower extremities without thrombocytopenia or coagulopathy. It is
often associated with arthralgias and arthritis, abdominal pain, and renal dysfunction. It is self-
limited and treatment is supportive only.
Erythema infectiosum (fifth disease) can be identified by an erythematous rash on the cheeks
and a lacy reticular rash on the extremities. Gianotti-Crosti syndrome is a sudden papular or
papulovesicular eruption on the extensor surfaces of the arms, legs, buttocks, and face, and it
is not purpuric. Hemolytic uremic syndrome presents with the classic triad of hemolytic
anemia, thrombocytopenia, and kidney injury. Thrombotic thrombocytopenic purpura is rare
in the pediatric age group .
Q6- A gravida 2 para 0 at 34 weeks gestation presents to your office because of diffuse itching.
She does not have any known allergies other than seasonal allergies, and she does not have
any new contacts. An examination is normal other than some scattered excoriations, and there
is no other distinct rash. Shehas tried moisturizers but her symptoms have not improved.
Which one of the following would be most appropriate at this point?

A.Monitoring for the development of a rash


B.Liver function tests and serum bile acid levels
C.Topical corticosteroids
D.Oral antihistamines
E.Varicella-zoster immune globulin
ANSWER: B
Whenever a pregnant woman presents with pruritus without a primary rash, it is important to
evaluate her for intrahepatic cholestasis of pregnancy. This diagnosis is associated with
increased fetal mortality and warrants increased antenatal surveillance as well as possible
induction by 35–37 weeks gestation. It is most appropriate to check for elevation of liver
function tests and serum bile acids.
Emollients, topical corticosteroids, and oral antihistamines can all be helpful for pruritus and
certain rashes, but in this patient it is most important to promptly look for the cause of the
pruritus. Varicella-zoster immune globulin would be indicated if she had no immunity to
varicella and had been exposed to varicella or if she had a rash that was suspected to be
chickenpox.

Q7- A 35-year-old female is admitted to the hospital with fever, tachycardia, and hypotension.
Her medical history is significant for current injection drug use.
Which one of the following is the most common infection in patients such as this?
A.Cellulitis
B.Endocarditis
C.Osteomyelitis
D.Pneumonia
E.Pyelonephritis
ANSWER: A
The most common complications for patients who inject drugs are skin and soft- tissue
infections. These are the most frequent infections leading to hospitalization. This patient is
also at risk for endocarditis, osteomyelitis, pneumonia, and pyelonephritis, but these infections
are less common.
Q8- A 52-year-old female sees you because of hair loss. An examination reveals diffuse
thinning of the hair along the vertex, with sparing of the frontal hairline. Follicular orifices are
still visible.
Which one of the following is the most likely explanation for these findings?
A.Alopecia areata
B.Anagen effluvium
C.Androgenetic alopecia
D.Tinea capitis
E.Trichorrhexis nodosa

ANSWER: C
Androgenetic alopecia is associated with bitemporal thinning of the frontal and vertex scalp in
men, but in women the frontal hairline is spared and hair thinning is most apparent at the
vertex. There is often a family history of hair loss in patients with androgenetic alopecia.
Alopecia areata results in acute, patchy hair loss. Anagen effluvium results in diffuse hair loss
days to weeks after exposure to chemotherapeutic agents. The incidence of anagen effluvium
after chemotherapy is estimated at 65%. Tinea capitis is a dermatophyte infection of the hair
shaft and follicles that results in patchy hair loss and requires systemic antifungal treatment.
Trichorrhexis nodosa is characterized by breaks in the hair secondary to trauma or because of
fragile hair, often due to excessive brushing, heat application, or hairstyles that lead to pulling
on hairs.

Q9-. A 17-year-old female presents to your office with open and closed comedones on her
nose, forehead, and chin. No cystic lesions are noted and the acne does not extend to her back
or chest.
Of the following, which one would be most effective?

A. Topical adapalene (Differin) and benzoyl peroxide


B. Topical clindamycin (Cleocin T)
C. Topical salicylic acid
D. Topical tazarotene (Tazorac)
E. Oral norethindrone/ethinyl estradiol

ANSWER: A
Acne affects 85% of 12- to 25-year-olds in the United States. This patient has mild acne as
evidenced by the absence of cystic lesions and localization to the facial T- zone. Topical
retinoids are first-line treatment for any level of severity of acne (SOR A). Adapalene is an
effective retinoid and is available over-the-counter.
Benzoyl peroxide is also very effective in the control of acne because it reduces the
concentration of cystic acne with no risk of bacterial resistance. The combination of a topical
retinoid and benzoyl peroxide is more effective than either agent alone.
Topical antibiotics such as clindamycin and erythromycin should not be used as monotherapy
due to high rates of microbial resistance. There is little evidence that salicylic acid is effective
in combating acne despite its widespread use.
Tazarotene is effective in the treatment of acne but is teratogenic (pregnancy category X) and
should be avoided in women of reproductive age. Combined oral contraceptives can be
effective, but norethindrone and other first-generation progestins are androgenic and can
worsen acne.

18. A 30-year-old gravida 1 para 0 develops erythematous patches with slightly elevated scaly
borders during her first trimester. There was a 2-cm herald patch 2 weeks before multiple
smaller patches appeared. The rash on the back has a “Christmas tree” pattern. She has not had
any prenatal laboratory work.
This condition is associated with

A. no additional pregnancy risk


B. a small-for-gestational-age newborn
C. congenital cataracts
D. multiple birth defects
E. spontaneous abortion

ANSWER: E
This patient has classic pityriasis rosea. This is generally a benign disease except in
pregnancy. The epidemiology and clinical course suggest an infectious etiology. Pregnant
women are more susceptible to pityriasis rosea because of decreased immunity. Pityriasis
rosea is associated with an increased rate of spontaneous abortion in the first 15 weeks of
gestation. It is not associated with an increased risk for a small-for-gestational-age newborn,
congenital cataracts, or multiple birth defects.
39. A gravida 2 para 0 at 34 weeks gestation presents to your office because of diffuse itching.
She does not have any known allergies other than seasonal allergies, and she does not have
any new contacts. An examination is normal other than some scattered excoriations, and there
is no other distinct rash. She has tried moisturizers but her symptoms have not improved.
Which one of the following would be most appropriate at this point?
A. Monitoring for the development of a rash
B. Liver function tests and serum bile acid levels
C. Topical corticosteroids
D. Oral antihistamines
E. Varicella-zoster immune globulin
ANSWER: B
Whenever a pregnant woman presents with pruritus without a primary rash, it is important to
evaluate her for intrahepatic cholestasis of pregnancy. This diagnosis is associated with
increased fetal mortality and warrants increased antenatal surveillance as well as possible
induction by 35–37 weeks gestation. It is most appropriate to check for elevation of liver
function tests and serum bile acids. Emollients, topical corticosteroids, and oral antihistamines
can all be helpful for pruritus and certain rashes, but in this patient it is most important to
promptly look for the cause of the pruritus. Varicella-zoster immune globulin would be
indicated if she had no immunity to varicella and had been exposed to varicella or if she had a
rash that was suspected to be chickenpox.

ABFM2018
Q10-A 32-year-old female sees you for evaluation of hair loss. On examination she has a
smooth, circular area of complete hair loss on her scalp with no other skin changes.
Which one of the following would you recommend?
A) An oral antifungal agent
B) Topical minoxidil (Rogaine)
C) Topical immunotherapy
D) Topical corticosteroids
E) Intralesional corticosteroids
ANSWER: E
This patient has alopecia areata, which is a chronic, relapsing, immune-mediated inflammatory
disorder affecting hair follicles that results in patchy hair loss. The treatment of choice is
intralesional corticosteroid injections. Topical immunotherapy is reserved for patients with
extensive disease, such as >50% scalp involvement. Topical corticosteroids are less effective
and are usually reserved for children and adults who cannot tolerate intralesional injections.
Minoxidil is used for androgenetic alopecia and is less effective for alopecia areata. Oral
antifungal drugs are used to treat tinea capitis.
Q11-You see an adult patient who has chronic urticaria and no other known chronic
conditions. He continues to experience hives after a 3-month course of daily loratadine
(Claritin). Which one of the following would be the most appropriate addition to his treatment
regimen at this time?

A) A short course of oral corticosteroids


B) Cyclosporine
C) Ranitidine (Zantac)
D) Narrow-band UV light treatment
ANSWER: C
First- and second-generation H1 antihistamine receptor antagonists are generally considered
first-line treatment for chronic urticaria, and approximately 60% of patients experience a
satisfactory result. Second-generation options such as loratadine have the added benefit of a
lower likelihood of side effects such as drowsiness. For those who fail to achieve the desired
result with monotherapy using an H1 antihistamine receptor antagonist, the addition of an H2
antihistamine receptor antagonist such as cimetidine or ranitidine is often beneficial. The
tricyclic antidepressant doxepin has strong H1 and H2 antihistamine receptor antagonist
effects and has been used as an off-label treatment option in some studies. A short course of
oral corticosteroids, narrow- band UV light treatment, or cyclosporine can be used in the
management of recalcitrant chronic urticaria, but these are considered second- or third-line
adjunctive options.

Q12-A 69-year-old female presents with scaling, redness, and irritation under her breasts for
the past several months. She has tried several over-the-counter antifungal creams without any
improvement. On examination you note erythematous, well demarcated patches with some
scale under both breasts. You examine the rash with a Wood's lamp to confirm your suspected
diagnosis. ?This rash is most likely to fluoresce
A) bright yellow
B) coral pink
C) lime green
D) pale blue
E) totally white
ANSWER: B
A Wood's lamp may assist with the diagnosis of certain skin conditions. This patient's
presentation is consistent with erythrasma caused by a Corynebacterium minutissimum
infection, and use of an ultraviolet light would reveal a coral pink color. Pale blue
fluorescence occurs with Pseudomonas infections, yellow with tinea infections, and totally
white with vitiligo. A lime green fluorescence is not characteristic of a particular skin
condition.
Q13-A 60-year-old male presents with the lesion shown below. It has grown over the last few
months. His past medical history includes well controlled hypertension. He takes lisinopril
(Prinivil, Zestril), 10 mg daily, and aspirin, 81 mgdaily.
After the diagnosis is established with a biopsy, which one of the following has the highest
cure rate for this problem?
A) Standard wide excision
B) Electrodesiccation and curettage
C) Mohs surgery
D) Photodynamic therapy
E) Radiation therapy

ANSWER: C
This patient most likely has a basal cell carcinoma, which can be proven by a shave biopsy.
Given its size and location, Mohs surgery would be the preferred treatment. It also has the
highest cure rate of any of the options listed, including a standard wide excision,
electrodesiccation and curettage, photodynamic therapy, and radiation therapy. It has a 99%
cure rate for primary basal cell cancers, compared with just over 91% for other methods.
Photodynamic therapy and radiation therapy should be used for lesions such as this only if
surgery is not an option due to medical comorbidities and/or patient preference.

Q14-52-year-old male presents for evaluation of a long-standing facial rash. He reports that the
rash is itchy, with flaking and scaling around his mustache and nasolabial folds. Which one of
the following is most likely to be beneficial?
A) Topical antibacterial agents
B) Topical antifungal agents
C) Topical vitamin D analogues
D) Oral zinc supplementation

ANSWER: B
Seborrheic dermatitis is commonly seen in the office setting and affects the scalp, eyebrows,
nasolabial folds, and anterior chest. The affected skin appears as erythematous patches with
white to yellow greasy scales. The etiology is not exactly known, but it is likely that the yeast
Malassezia plays a role. Topical antifungals are effective and recommended as first-line
agents. Topical low- potency corticosteroids are also effective alone or when used in
combination with topical antifungals, but they should be used sparingly due to their adverse
effects. The other agents listed have no role in the management of seborrheic dermatitis (SOR
A).
Q15-A 26-year-old male presents with a rash on his anterior neck in the area of his beard that
has been present for over a year. On examination he has dark, curly facial hair, and you find
slightly tender, red, hyperpigmented papules on the superior anterior neck. Which one of the
following would you recommend to improve this patient’s rash?
A) Shaving with a multi-blade razor
B) Shaving with electric clippers
C) Pulling the skin taut while shaving
D) Plucking hairs rather than shaving
E) Oral cephalexin (Keflex)

ANSWER: B
This patient has pseudofolliculitis barbae, which is a common condition affecting the face and
neck in people with tightly curled hair. The condition occurs when hairs are cut at an angle
and curl in on themselves, creating a foreign body reaction. The condition may progress to
scarring and keloid formation. Cessation of hair removal improves the condition. If this is not
desired, less aggressive hair trimming is recommended. Clippers generally result in a less
close shave and contribute less to pseudofolliculitis barbae. Multi-blade razors, pulling the
skin taut, and plucking hairs all result in shorter hair and are likely to exacerbate the problem.
The description of the rash is not consistent with secondary infection, so oral cephalexin would
not be indicated at this time. Treatment is similar to the treatment of acne, with benzoyl
peroxide, topical retinoids, and topical antibiotics having a role, along with topical
corticosteroids.

Q16-A 46-year-old female with a past medical history of polycystic ovary syndrome and
migraine headaches presents with bilateral, hyperpigmented patches along her mandible. The
patches are asymptomatic but bother her cosmetically and seem to be darkening. Which one of
her medications would be most likely to contribute to her melasma?
A) B-complex vitamins
B) Metformin (Glucophage)
C) Oral contraceptives
D) Spironolactone (Aldactone)
E) Sumatriptan (Imitrex)
ANSWER: C
Melasma is a progressive, macular, nonscaling hypermelanosis of skin exposed to the sun,
typically involving the face and dorsal forearms. It is often associated with pregnancy and the
use of oral contraceptives or anticonvulsants (SOR C).
Some melasma is idiopathic. Women are nine times more likely to be affected than men, and
darker-skinned individuals are also at greater risk. There are three common patterns of
melasma: centrofacial, malar, and mandibular.
Q17-A 58-year-old male with a history of tobacco and alcohol abuse presents with the sudden
onset of many well circumscribed brown, oval, rough papules with a “stuck-on” appearance
on his trunk and proximal extremities. On examination you also note an unintentional 6-kg
(13-lb) weight loss over the last 3 months and conjunctival pallor. A review of systems is
positive for more frequent stomachaches, decreased appetite, and mild fatigue. You order a
laboratory workup.
Which one of the following would be most appropriate at this point?
A) Reassurance that the skin lesions are benign
B) A skin biopsy
C) Referral to a dermatologist
D) CT of the abdomen and pelvis
E) Upper and lower endoscopy

ANSWER: E
This patient’s age, risk factors, red-flag symptoms, and other clinical findings indicate the
need for endoscopy. The Leser-Trélat sign may be defined as the abrupt onset of multiple
seborrheic keratoses, which is an unusual finding that often indicates an underlying
malignancy, most commonly an adenocarcinoma of the stomach. CT is not an initial approach
for diagnosing a suspected malignancy of the stomach or colon. Further skin evaluation and
lifestyle changes, which are indicated, will not address the need for evaluation of weight loss
and other abnormal symptoms and findings.

Q19-A 49-year-old male is concerned about lesions on his penis that he has noticed over the
past 6 months. He was circumcised as a child and has had the same female sexual partner for 5
years. He does not have any pain, itching, or dysuria. On examination you note multiple
reddish-blue papules on the scrotum and a few similar lesions on the shaft of the penis.
The most likely diagnosis is
A) pearly penile papules
B) lichen nitidus
C) lichen sclerosus
D) angiokeratomas
E) squamous cell carcinoma in situ (Bowen’s disease)
ANSWER: D
Penile lesions are usually easily diagnosed from clinical findings. Pearly penile papules are
common and benign, and present as small, skin-colored, dome- shaped papules in a circular
pattern around the coronal sulcus. Lichen nitidus is benign but uncommon. It presents as
discrete, pinhead-sized hypopigmented papules that are asymptomatic. Papules are often
found scattered all over the penis, as well as on the abdomen and upper extremities.
Lichen sclerosus is more common and appears as hypopigmented lesions with the texture of
cellophane. The lesions are usually located on the glans or prepuce.
Atrophy, erosions, and bullae are common, and patients often present with itching, pain,
bleeding, and possibly phimosis or obstructed voiding. Lichen sclerosus is associated with
squamous cell cancer in a small percentage of cases. Carcinoma in situ is a premalignant
condition that is more common in uncircumcised males over age 60. Lesions are typically
beefy red, raised, irregular plaques and can be found on the glans, meatus, frenulum, coronal
sulcus, and prepuce. Lesions can be ulcerated or crusted. Pruritus and pain are common. A
biopsy is important for making the diagnosis.
Angiokeratomas are lesions that are usually asymptomatic, circumscribed, red or bluish
papules. They may appear solely on the glans of the penis, but are also found on the scrotum,
abdomen, thighs, groin, and extremities. They may be misdiagnosed as pearly papules or
carcinoma. Treatment is not necessary unless the lesions are bleeding or extensive. It is
important to realize that angiokeratomas on the shaft of the penis, the suprapubic region, or the
sacral region can be associated with Fabry disease. Patients with this finding should be
promptly referred.

Q20-A 26-year-old female presents with a skin rash and chronic diarrhea. She reports being
previously diagnosed with eczema, and while the rash has responded well to topical
corticosteroids it flares when they are stopped. The skin rash is very itchy and appears as
mildly erythematous papules and vesicles clustered on the elbows and knees, as well as the
posterior neck and scalp. A comprehensive metabolic panel is normal, and a CBC reveals a
mild microcytic, hypochromic anemia. Antinuclear antibodies are negative, a TSH level is
normal, and a tissue transglutaminase antibody test is positive.
Which one of the following is the most likely diagnosis?
A) Cutaneous lupus erythematosus
B) Dermatitis herpetiformis
C) Eczema
D) Eczema herpeticum
E) Lichen simplex chronicus
ANSWER: B
This patient has celiac disease with both intestinal and extraintestinal manifestations (diarrhea
and dermatitis herpetiformis, respectively). Iron deficiency anemia due to chronic blood loss is
a common finding in patients with celiac disease. Eczema herpeticum is the appearance of a
herpetic infection complicating eczema. This is a serious acute problem that can be life-
threatening in severe cases. Lichen simplex chronicus is a chronic skin condition that is
perpetuated by scratching. Lesions are commonly thickened and excoriated.
Diarrhea and anemia are not associated with this disorder. While eczema is possible based on
the description of the rash, the intestinal manifestations and positive tissue transglutaminase
antibody in this patient point to celiac disease. This patient’s rash does not suggest cutaneous
lupus, which is also unlikely given the negative antinuclear antibody test.

Q21-A 4-year-old male is brought to your office for a well child examination. The patient has
no significant medical history. The mother has noted new skin lesions first appearing on the
back, with a new lesion behind the right knee. She has not used any new detergents or skin or
hair care products. She has not made any changes in the patient’s diet. The child does not have
pruritus.
The examination reveals a temperature of 37.2°C (99.0°F), a pulse rate of 80 beats/min,and a
blood pressure within normal limits. The examination is unremarkable except for Non
erythematous flesh-colored, dome-shaped papules with a central indentation, on the lower back
and popliteal fossa.
Which one of the following would be most appropriate for the initial management of this
condition?
A) Observation only
B) Consistent use of emollients and avoiding frequent hot baths
C) Use of a topical low-dose corticosteroid cream once daily until resolved
D) Use of a topical antifungal cream until resolved
E) Paring, followed by topical salicylic acid or cryotherapy

ANSWER: A
Molluscum contagiosum is a common disease during childhood, but can also occur in
adolescents and adults. It is caused by a poxvirus and is characterized by flesh-colored, dome-
shaped papules with central umbilication, most commonly on the trunk, axilla, popliteal or
antecubital fossae, and crural folds. If lesions are asymptomatic and not inflamed, the initial
treatment is observation, with most lesions resolving spontaneously within 2–12 months. If the
lesions are inflamed or pruritic, then topical corticosteroid treatment, chemical treatment with
cantharidin, podofilox 0.5% solution, curettage, or cryotherapy may be indicated. Atopic
dermatitis (eczema) is initially treated with emollients and by avoiding frequent hot baths.
Verruca (warts) are commonly treated with paring, followed by topical salicylic acid or
cryotherapy. Antifungal cream would not be appropriate.

Q22-You see a 47-year-old female for follow-up of a rash. She is a carpenter and was seen 4
days ago for increasing redness and tenderness of her anterior shin after hitting the area with a
board 3 days earlier. She was afebrile during that visit and the area was red but not fluctuant.
She chose observation rather than treatment at that time. The patient smokes 10 cigarettes
daily. Past medical, surgical, and family histories are otherwise negative.
Screening for diabetes mellitus was normal last year. Today the patient’s anterior shin is still
tender. She is afebrile and other vital signs are unremarkable. The extent of the infection was
drawn 4 days ago with an indelible marker by your partner. Currently the area of redness
extends beyond this border. There is no fluctuance or drainage of the wound. The skin appears
mildly indurated.
Which one of the following would be best to provide coverage against Streptococcus
pyogenes or methicillin-resistant Staphylococcus aureus (MRSA) inthis patient?

A) Amoxicillin/clavulanate (Augmentin) and ciprofloxacin (Cipro)


B) Cephalexin and dicloxacillin
C) Dicloxacillin and fosfomycin (Monurol)
D) Doxycycline and trimethoprim/sulfamethoxazole (Bactrim)
E) Trimethoprim/sulfamethoxazole and cephalexin
ANSWER: E
Clindamycin or a combination of trimethoprim/sulfamethoxazole (or doxycycline or
minocycline) plus cephalexin (or dicloxacillin or amoxicillin/clavulanate) should provide
adequate coverage for Streptococcus and methicillin-resistant Staphylococcus aureus (MRSA)
for mild to moderate cellulitis. Doxycycline plus trimethoprim/sulfamethoxazole would
provide inadequate coverage for streptococcal bacteria. Cephalexin plus dicloxacillin would
provide inadequate coverage for MRSA. The primary indication for ciprofloxacin is treatment
of infections with gram-negative rods. Fosfomycin is indicated only for urinary tract
infections. Neither is typically used in the treatment of cellulitis.

Q24-A 30-year-old male is treated with topical medications for his papulopustular rosacea with
only partial improvement. The preferred antibiotic is
A) amoxicillin
B) cephalexin (Keflex)
C) doxycycline
D) erythromycin
E) trimethoprim/sulfamethoxazole (Bactrim)

ANSWER: C
Tetracycline and its derivatives have historically been used for the treatment of papulopustular
rosacea and there is data to support their use. A modified-release doxycycline is FDA-approved
for this indication. Amoxicillin, cephalexin, erythromycin, and trimethoprim/sulfamethoxazole
lack evidence to support their use in the treatment of papulopustular rosacea.

Q25- A 52-year-old male presents for evaluation of a long-standing facial rash. He reports that
the rash is itchy, with flaking and scaling around his mustache and nasolabial folds. Which
one of the following is most likely to be beneficial?
A) Topical antibacterial agents
B) Topical antifungal agents
C) Topical vitamin D analogues
D) Oral zinc supplementation
ANSWER: B
Seborrheic dermatitis is commonly seen in the office setting and affects the scalp, eyebrows,
nasolabial folds, and anterior chest. The affected skin appears as erythematous patches with
white to yellow greasy scales. The etiology is not exactly known, but it is likely that the yeast
Malassezia plays a role. Topical antifungals are effective and recommended as first-line
agents. Topical low- potency corticosteroids are also effective alone or when used in
combination with topical antifungals, but they should be used sparingly due to their adverse
effects. The other agents listed have no role in the management of seborrheic dermatitis (SOR
A).

Q26-A 26-year-old male presents with a rash on his anterior neck in the area of his beard that
has been present for over a year. On examination he has dark, curly facial hair, and you find
slightly tender, red, hyperpigmented papules on the superior anterior neck. Which one of the
following would you recommend to improve this patient’s rash?
A) Shaving with a multi-blade razor
B) Shaving with electric clippers
C) Pulling the skin taut while shaving
D) Plucking hairs rather than shaving
E) Oral cephalexin (Keflex)

ANSWER: B
This patient has pseudofolliculitis barbae, which is a common condition affecting the face and
neck in people with tightly curled hair. The condition occurs when hairs are cut at an angle
and curl in on themselves, creating a foreign body reaction. The condition may progress to
scarring and keloid formation. Cessation of hair removal improves the condition. If this is not
desired, less aggressive hair trimming is recommended. Clippers generally result in a less close
shave and contribute less to pseudofolliculitis barbae. Multi-blade razors, pulling the skin taut,
and plucking hairs all result in shorter hair and are likely to exacerbate the problem. The
description of the rash is not consistent with secondary infection, so oral cephalexin would not
be indicated at this time. Treatment is similar to the treatment of acne, with benzoyl peroxide,
topical retinoids, and topical antibiotics having a role, along with topical corticosteroids.
ABFM2107
Q27-A 15-year-old female comes to your office for treatment of acne vulgaris. Her complete
history and physical examination are unremarkable other than a moderate amount of closed
comedones and inflamed papules on her nose, forehead, and upper back. She has not
previously tried any
topical or oral therapies, including over-the-counter medications.
Which one of the following would be indicated for this patient as monotherapy?
A) Topical benzoyl peroxide
B) Topical clindamycin (Cleocin T)
C) Oral isotretinoin (Absorica)
D) Oral minocycline (Minocin)
E) Oral spironolactone (Aldactone)
ANSWER: A
Oral antibiotics are recommended for acne that is resistant to topical treatments.
Oral isotretinoin is indicated for severe nodular acne or moderate acne resistant to other
treatments. Since this patient has not tried any therapies, these two options would not yet be
appropriate. Topical antibiotics are recommended only in combination with benzoyl peroxide.
Appropriate treatments would be topical benzoyl peroxide, a topical retinoid, and oral
contraceptives. Antiandrogen therapies such as spironolactone are not indicated solely for
acne vulgaris, although they may be appropriate for concomitant conditions such as polycystic
ovary syndrome.

Q28- A 65-year-old male presents for follow-up for his chronic hypertension. He mentions that
he has a rough spot on his arm that he has scratched off a few times, but it keeps returning. He
asks if this spot should be removed. On examination you find several scaly, slightly raised
lesions on his forearm as shown on the following page. Which one of the following would be
the most appropriate management?
A) Observation only
B) Application of moisturizers twice daily
C) Topical corticosteroids
D) Destruction or removal
ANSWER: D
This lesion is consistent with actinic keratosis, which is considered to be a premalignant lesion
with the potential to progress to squamous cell carcinoma, so treatment is generally indicated.
This can be accomplished with destruction or removal, including with topical medication
therapies.

Q29- A 45-year-old female visits your office for her annual health maintenance visit and
mentions that her hair has been thinning over the past few years. She is now concerned that it
may be noticeable. She takes no medications and is otherwise healthy with normal menstrual
cycles. On examination she has a negative pull- away test. You note diffuse thinning in the
parietal regions, with sparing of the frontal hairline. She has no scalp scarring, scale, or
erythema.
Which one of the following would be the most appropriate pharmacotherapy?

A) Finasteride (Propecia), 1 mg daily


B) Griseofulvin, 500 mg daily
C) Hydroxychloroquine (Plaquenil), 200 mg twice daily
D) Minoxidil 2% (Rogaine), applied to the scalp twice daily
E) Triamcinolone 0.05% (Trianex), applied to the scalp twice daily

ANSWER: D
Female pattern hair loss is categorized as diffuse and nonscarring. It presents with parietal hair
thinning with preservation of the frontal hairline. Minoxidil 2% produces regrowth of hair in
female pattern hair loss (SOR B). Oral finasteride is appropriate only for men with male
pattern hair loss (SOR A). Hydroxychloroquine is used for inflammatory hair loss associated
with discoid lupus erythematosus, which is focal and scarring. Topical corticosteroids are
appropriate for alopecia areata (SOR B) but not for female pattern hair loss. Griseofulvin is
used to treat tinea capitis, which presents as focal scale with erythema.
Q30-A 43-year-old female presents with marked proximal muscle weakness, dysphagia, and
pain in the shoulders and hips, all beginning within the past 5 weeks. She reports difficulty
getting out of a chair. On examination she has a violaceous rash involving the periorbital skin,
and macular erythematous lesions over the anterior chest and upper lateral thighs. Which one
of the following additional findings would you expect?
A) Hyperkeratotic plaques in intertriginous areas
B) Macules over the extensor surfaces of her joints
C) Polygonal papules on the flexor surface of her wrists
D) Distal onycholysis
E) Atrophic cuticles with contracted nail-fold capillaries on dermoscopy.
F)
Answer : B
This patient’s symptoms and findings suggest dermatomyositis. This disease is distinguished
from autoimmune myopathies and polymyositis by distinct dermatologic findings, including
Gottron’s sign(nonpalpable macules over the extensor surface of joints). Patients may also
have dilated nail-fold capillaries and ragged, thickened cuticles. Distal onycholysis is most
commonly associated with onychomycosis, while hyperkeratotic plaques are not a feature of
dermatomyositis. Polygonal papules on the wrist flexor surfaces are seen in lichenplanus.

Q31-A 62-year-old male with a history of COPD sees you because of fever, chills, and redness
and swelling in his right lower extremity that has been progressing. He has a temperature of
38.0°C (100.4°F), a blood pressure of 112/72 mm Hg, and a pulse rate of 94 beats/min. The
physical examination is remarkable for an erythematous area with increased warmth that
extends approximately 18 cm (7 in) between the knee and ankle. You note no apparent abscess
formation. The patient does not recall any specific injury that could have caused this problem.

According to the guidelines of the Infectious Diseases Society of America, which one of the
following would be the most appropriate antibiotic regimen for this patient?

A) Ceftriaxone (Rocephin)
B) Piperacillin/tazobactam (Zosyn)
C) Vancomycin (Vancocin)
D) Vancomycin plus ceftriaxone
E) Vancomycin plus piperacillin/tazobactam
ANSWER: A
The most common pathogen for nonpurulent cellulitis is -hemolytic streptococci. Guidelines
from the Infectious Diseases Society of America recommend treating moderate nonpurulent
cellulitis with penicillin, ceftriaxone, cefazolin, or clindamycin alone. Vancomycin would be
indicated if the patient had a history of illicit drug use, purulent drainage, concurrent evidence
of MRSA infection elsewhere, nasal colonization with MRSA, or severe cellulitis.

Q32-A 13-year-old male is brought to your office because of pain in his foot. Two days ago he
stepped on a nail that went through his sneaker and caused a puncture wound to the base of his
foot. On examination today he has tenderness and erythema surrounding the wound, and you
can express pus from the wound. He is afebrile.
Which one of the following would be best to treat this patient’s cellulitis?
a. Amoxicillin/clavulanate (Augmentin)
b. Cephalexin (Keflex)
c. Ciprofloxacin (Cipro)
d. Doxycycline
e. Trimethoprim/sulfamethoxazole (Bactrim)

ANSWER: C
Puncture wounds to the foot commonly get infected. Most soft-tissue infections from puncture
wounds are caused by gram-positive organisms. Staphylococcus aureus is the most common,
followed by other staphylococcal and streptococcal species. When the puncture wound is
through the rubber sole of an athletic shoe, Pseudomonas is the most frequent pathogen.
Ciprofloxacin is the only oral antibiotic that has antipseudomonal activity, and would be the
most appropriate choice.
Q33-A previously healthy 29-year-old pediatric nurse has a 3-day history of malaise,
arthralgias, and a nonpruritic rash. The rash is a faint, maculopapular, irregular, reticulate
exanthem that covers her thighs and the inner aspects of her upper arms. Symmetric synovitis
is present in several distal and proximal interphalangeal joints and in her metacarpophalangeal
joints. Small effusions, warmth, and tenderness are noted in her left wrist and right elbow. No
other joints are affected.
The most likely cause of this problem is
A) varicella-zoster virus
B) measles (rubeola) virus
C) parvovirus B19
D) adenovirus
E) HIV

ANSWER: C
Also known as erythema infectiosum or fifth disease, parvovirus B19 infection is a fairly
common cause of an exanthematous rash and arthritis in younger women. This infection
should be particularly suspected in health care workers who have frequent contact with
children. By the age of 15 approximately 50% of children have detectable IgG antibodies to
the virus, and this figure rises to 90% in the elderly. Within households and caregivers the
secondary infection rate, especially among nonimmune children and young adults, approaches
50%. The specific characteristics of the rash, the pattern of joint involvement, and the place of
employment in an otherwise healthy person all offer clues suggesting parvovirus B19 as the
infecting agent. Measles virus, adenovirus, and HIV rarely cause arthritis, although HIV
infection can cause a musculoskeletal syndrome later in the disease. Varicella-zoster virus
may cause large-joint arthritis, but the rash is distinctively vesicular and pruritic
Q34-A 54-year-old female presents with painful sores in her mouth that appeared a few days
ago. She has had some trouble eating due to the pain, but she is able to swallow without
difficulty. She also began to have some pain around her right ear today. She has no fever,
chills, nasal congestion, cough, or difficulty hearing.
Her medical history is significant only for an anxiety disorder treated with sertraline (Zoloft).
On examination her vital signs are all normal. You see vesicles on the right side of the hard
palate and she has a swollen, red right pinna, with vesicles in the external auditory canal.
The organism responsible for this condition is
A) coxsackievirus
B) Epstein-Barr virus
C) group A Streptococcus
D) herpes simplex virus
E) varicella zoster virus

ANSWER: E
This patient has herpes zoster oticus, which is also known as Ramsay Hunt syndrome when
associated with a facial nerve palsy. It is caused by reactivation of the varicella-zoster virus
(VZV) in the geniculate ganglion of the facial nerve.
Typical symptoms include painful vesicles on one side of the palate and the ipsilateral ear.
When the reactivation involves other branches of the facial nerve it can result in a unilateral
facial herpetiform rash that may also involve the anterior two-thirds of the tongue, taste
disturbance, and reduced lacrimation. If the nearby cochlear and vestibular nerves become
involved, patients may also experience hearing loss, tinnitus, nausea, vomiting, and vertigo.
The diagnosis is usually made clinically, but if confirmation is needed polymerase chain
reaction testing of vesicular fluid or of a swab of the base of an ulcer may be done.
Treatment includes antivirals (acyclovir, valacyclovir) and prednisone, and is more effective
when started sooner in the course of illness. Herpes simplex virus (HSV) can cause oral
vesicles and ulcers, but the distribution of vesicles in the ear and the mouth of this patient is
not typical for HSV. Epstein-Barr virus can cause leukoplakia of the mouth but not vesicles
and is typically associated with systemic signs of illness. Group A Streptococcus causes throat
pain and fever, not vesicles. Coxsackievirus causes oral vesicles and ulcers but is usually
associated with fever and does not typically involve the ear.
A)
Q35-A 2-year-old female is brought to your office because of a round lesion on her lip that
appeared 2 days ago. Her temperature and all vital signs are normal. She has no past medical
history and takes no medications. Further history reveals that she was playing with a toy
trumpet in a busy store a few days before the lesion appeared. A physical examination reveals
a 1- cm round lesion with crusting, and no other skin abnormalities.
Which one of the following would be the best treatment at this time?
A) Bacitracin
B) Mupirocin (Bactroban)
C) Neomycin
D) Cephalexin (Keflex)
E) Clindamycin (Cleocin)

ANSWER: B
This patient has physical findings and a history consistent with impetigo, a skin infection
caused by
Staphylococcus aureus and/or Streptococcus pyogenes. Since she has only one lesion, systemic
antibiotics are not required as they would be for a patient with extensive disease or multiple
lesions. Although bacitracin and neomycin are commonly used, they are much less effective
for impetigo than mupirocin, despite some reports of resistance to mupirocin (level A-1
evidence).

Q36-An 8-year-old male is brought to your office because of a rash on his arms and legs that
has been present for the last several weeks. It seems to be spreading gradually, according to
his parents. No other family members have similar symptoms. A physical examination reveals
excoriated erythematous papules on both forearms and both lower legs that drain a small
amount of serous fluid. The child says that these are itchy. There are no signs of deeper
inflammation and no lesions are present on the scalp, hands, thorax, or groin. Which one of the
following is the most likely cause of this problem?
A) Bedbugs
B) Brown recluse spiders
C) Mites/chiggers
D) Scabies
E) Ticks
ANSWER: A
This child’s presentation appears most consistent with bites from an insect. Having multiple
exposures on skin often not covered by clothing would be typical of household fleas or
bedbugs. Tick bites are typically identified by the presence
of an actively feeding insect or a single papular lesion. Similarly, brown recluse spider bites
would not be expected to be multiple or recurrent. A chigger is the larval form of a mite,
which is an eight-legged arthropod. The larval form has only six legs, and tends to crawl into
spaces near constricted clothing and cause welts from their bites along the neckline, waistline,
sock line, or more rarely on the genitals. A scabies infestation often presents as an eczematous
rash in semiprotected folds of skin such as the web spaces of fingers, the umbilicus, the
axillae, or the genital region.

Q37-A 17-year-old seasonal farm worker presents with a 7-day history of left- sided facial
weakness that he first noted upon awakening. He has no facial pain. Approximately 2 weeks
prior to the onset of this problem he removed a tick from the left side of his neck, but he is
uncertain how long it had been present. The redness at the site resolved spontaneously and he
had no additional symptoms until the onset of facial weakness. He has not had any rash, fever,
swollen glands, or neck stiffness. On examination he has weakness of the muscles on the left
side of his face, including the forehead, and he can only partially close his left eye. The
remainder of the physical examination is normal, including the absence of rashes and
lymphadenopathy.
In addition to corticosteroids, which one of the following would be indicated atthis time?
A) Amoxicillin as a single dose
B) Ceftriaxone (Rocephin) for 7 days
C) Doxycycline for 14 days
D) Trimethoprim/sulfamethoxazole (Bactrim) for 10 days
E) No antibiotics

ANSWER: C
Early systemic Lyme disease may manifest with facial nerve palsy, and treatment should
includecorticosteroids. The evidence for efficacy of antivirals for facial nerve palsy is lacking,
especially beyond 3–4 days after onset. But in this case, specific treatment to eradicate the
Lyme disease is also indicated,in order to prevent later, more severe systemic
complications.For facial nerve palsy, treatment with doxycycline or amoxicillin for 14 days is
effective. Patients withmore severe neurologic manifestations of Lyme disease, such as altered
mental status,meningoencephalitis, or other cranial nerve palsies, require longer courses of
antibiotics, usually intravenously. A single-dose treatment with doxycycline or amoxicillin is
recommended as prophylaxis in asymptomaticpatients after a known tick bite. This is only
recommended for tick attachment longer than 36 hours, or of unknown duration.
Q38-A 78-year-old male presents for a routine health maintenance examination and is
concerned about a gradual loss in his vision during the past year. He has smoked 1 pack of
cigarettes per day for the past 60 years. He has no other medical problems. On Amsler grid
testing he notes distorted grid lines.
Which one of the following would you recommend for this patient?
A) Watchful waiting
B) Avoiding all vitamin supplements
C) Treatment to reverse his visual changes
D) Smoking cessation to prevent further vision loss
ANSWER: D
This patient presents with signs and symptoms that suggest age-related macular degeneration.
Smoking is a modifiable risk factor and smokers should be counseled to quit (SOR C). The
patient should be referred to an ophthalmologist for further evaluation and management.
Watchful waiting would not be appropriate. Vitamin supplements with Age-Related Eye
Disease (AREDS) and AREDS2 formulations have been shown to delay visual loss in patients
with age- related macular degeneration (SOR A). Age-related macular degeneration is not
reversible but treatment can delay progression or stabilize the changes (SOR A).

Q39- While percussing the chest of a 38-year-old male during his annual health maintenance
examination, you notice the lesion shown on the following page on the upper left side of his
back. The lesion is approximately 1.2 cm across the long axis. Which one of the following
would be the most appropriate initial intervention?
A) A superficial shave biopsy
B) A punch biopsy of the peripheral margin
C) A punch biopsy at the center of the lesion
D) An excisional biopsy with a 1- to 3-mm border around the lesion
E) Wide excision with a 2-cm border around the lesion
ANSWER: D
The appearance of this lesion strongly suggests malignant melanoma. Although any of the
options listed can provide tissue adequate for pathologic confirmation, the American Academy
of Dermatology recommends an excision with narrow margins (1–3 mm) that includes the
entire gross lesion with clinically negative margins to a depth that ensures the lesion is not
transected (SOR B, level of evidence 2). For smaller lesions punch excision with sutures or
shave removal to an adequate depth are acceptable alternatives if the recommended excision
objectives can be met. Once the tissue diagnosis is obtained, appropriate staging and definitive
surgery should be undertaken.
Q40-A 27-year-old male has been treating his plaque psoriasis with high-potencytopical
corticosteroids for several years. He comes to your office to discuss other options since the
lesions on his trunk and extremities are becoming resistant to this therapy.
Which one of the following treatment strategies would be most appropriate?
A) Switch to topical tazarotene (Avage, Tazorac)
B) Add topical calcipotriene (Dovonex, Sorilux)
C) Begin oral acitretin (Soriatane)
D) Begin an oral corticosteroid
E) Begin etanercept (Enbrel) injections

ANSWER: B
Oral corticosteroids are not indicated in the treatment of plaque psoriasis. All of the other
options are indicated only if topical treatments fail. Of the options listed, the combination of a
topical corticosteroid and topical calcipotriene is considered the most appropriate for this
patient. Another option would be to add topical tazarotene to the topical corticosteroid.
However, when tazarotene is used as monotherapy it often fails to clear plaques and increases
the incidence of skin irritation.

Q41-A 2-year-old male has a 3-day history of a runny nose and cough, and a 2-day history of
fever reaching 40.0°C (104.0°F). He woke up with a rash this morning. His appetite is good
and his activity level is normal. On examination the child is afebrile with normal vital signs,
and has a fine, maculopapular, erythematous rash on the trunk and extremities. The remainder
of the examination is normal.
Which one of the following is the most likely cause of this patient’s rash?
A) Atopic dermatitis
B) Erythema infectiosum
C) Molluscum contagiosum
D) Pityriasis rosea
E) Roseola infantum
Answer : E
This patient has the classic presentation for roseola infantum, which is caused by human
herpesvirus 6. The typical history includes a high fever in a child with either mild upper
respiratory symptoms or no other symptoms. After the fever subsides, a rash will appear. The
rash is self-limited and no treatment is required. Pityriasis rosea typically presents with a single
herald patch that is oval-shaped and scaly with central clearing, followed by a symmetric rash
on the trunk in a typical distribution along the Langer lines. The rash may last up to 12 weeks
and no treatment is required.
Erythema infectiosum is caused by parvovirus B19 and is also known as fifth disease. The
child will typically have mild symptoms then an erythematous facial rash that has a ―slapped
cheek‖ appearance. This is sometimes followed by pink patches and macules in a reticular
pattern. Once the rash appears the child is no longer contagious.
Molluscum contagiosum is caused by a poxvirus and is characterized by scattered flesh-colored
papules with umbilicated centers. Atopic dermatitis typically presents as scaly, erythematous
plaques, commonly on the flexor surfaces of theextremities.

Q42-A 56-year-old male presents to your office with the rash shown on the following page. It
started under his left arm 2 days ago and has spread slightly. It is itchy and burns a little. He
has been treating it with cortisone 1% cream twice daily with some relief. He has no
significant past
medical history.
Which one of the following would be the most appropriate treatment?
A) Topical clobetasol
B) Topical mupirocin (Bactroban)
C) Oral cephalexin (Keflex)
D) Oral prednisone
E) Oral valacyclovir (Valtrex)

ANSWER: E
This patient presents with the classic rash associated with varicella zoster (shingles). The rash
is unilateral along a dermatome. Treatment of varicella zoster with antiviral therapy should be
initiated if the patient can be treated within 72 hours of the onset of rash. This will decrease
the severity of the outbreak and may decrease the risk of persistent neuropathic pain.
Treatment can include topical corticosteroids and capsaicin cream, as well as oral pain
medications and a corticosteroid. This rash is not a contact dermatitis or a heat reaction, and
topical corticosteroids are not indicated as the sole treatment. Oral corticosteroids alone should
not be used to treat varicella zoster. Varicella zoster can be complicated by a bacterial
superinfection, in which case oral antistaphylococcal antibiotics are indicated. This rash is not
impetigo and there is no role for topical antibiotics.
ABFM2016
Q44- Which one of the following topical antibiotics used in the treatment of acne vulgaris is
not known to increase bacterial resistance?
A) Benzoyl peroxide
B) Clindamycin (Cleocin)
C) Dapsone (Aczone)
D) Erythromycin
E) Metronidazole

ANSWER: A
Benzoyl peroxide is the only topical antibiotic without evidence to suggest that its use
contributes to bacterial antimicrobial resistance. For this reason it is recommended as first-
line therapy in mild to moderate inflammatory acne and as an adjunctive agent with other
topical antibiotics that can induce bacterial resistance, such as clindamycin and erythromycin.
Dapsone is an antibiotic but is thought to primarily improve acne via its anti-inflammatory
effect. Metronidazole is used chiefly in acne rosacea rather than acne vulgaris and it may
contribute to antimicrobial resistance.

Q45-The rash associated with erythema infectiosum (fifth disease) is characterized by which
one of the following?
A) Small red papules with central umbilication
B) Annular patches with raised borders and central clearing
C) Pink pustules that form a thick yellow crust
D) An erythematous “slapped cheek” facial rash
E) A symmetrical rash in a Christmas tree pattern

ANSWER: D
Fifth disease, caused by parvovirus B19, is a common childhood infection that typically
presents with a prodrome of low-grade fever, malaise, sore throat, nausea, and headache,
followed a few days later by an erythematous facial rash on the cheeks. The rash fades in a few
days but a lacy, reticular rash develops on the extremities. Once the rash appears, patients are
no longer considered contagious. NSAIDs and antihistamines are often used for symptom
relief (SOR A).
Q46- A 40-year-old male develops a keloid 6 months after a laceration repair.
Which one of the
following is the most appropriate initial treatment to decrease the size of thekeloid?
A) Topical retinoids
B) Topical corticosteroids
C) Intralesional corticosteroid injections
D) Surgical excision
E) Mohs surgery

ANSWER: C
Keloids are overgrowths of scar tissue seen more commonly in individuals with dark skin. The
best initial treatment is intralesional corticosteroid injections. If this does not produce
acceptable results, other treatment modalities include surgery, laser therapy, and bleomycin
injection.

Q47 -A 50-year-old male presents to your office with a 4-day history of the rash. It spread
from the lower trunk to the lower extremities, including the genital area. He also complains of
pain and swelling of the testes. He considers himself to be in good health and takes no
medications. He is afebrile with a normal examination except for the pink-purple
maculopapular eruption and bilateral swollen testes. A CBC, urinalysis, and comprehensive
metabolic panel are normal.

Which one of the following is the most likely diagnosis?

A) Henoch-Schönlein purpura
B) Kawasaki disease
C) Polyarteritis nodosa
D) Rocky Mountain spotted fever
E) Thrombocytopenic purpura
ANSWER: A
Henoch-Schönlein purpura (HSP) presents most often in children but not infrequently in
adults. The purpuric rash is classically seen on the waist and extends to the legs, sparing the
proximal trunk and arms. Orchitis with testicular swelling occurs in 35% of men with HSP and
is often complicated by abdominal pain, arthritis, and renal insufficiency. Kawasaki disease is
a pediatric disease presenting with fever, conjunctivitis, and lesions of the lips. Polyarteritis
nodosa often presents with fever and multisystem symptoms and findings. There are most
often abnormalities on the CBC and chemistry profile. Rocky Mountain spotted fever is
associated with a petechial rash that involves the proximal trunk and extremities, including the
palms and soles. Thrombocytopenic purpura, by definition, is associated with a low platelet
count.

Q48. A 35-year-old female presents to your office with a 3-month history of dyspnea. She
does not smoke and has not had a productive cough. She has no other significant past medical
history and takes no medications. A chest radiograph reveals significant hilar adenopathy with
bilateral infiltrates. Which one of the following physical examination findings would be
consistent with the most likely diagnosis in this patient?

A) Bilateral conjunctivitis
B) Alopecia
C) Erythema nodosum
D) A malar rash
E) Xerostomia

ANSWER: C
This patient has sarcoidosis. Extrapulmonary manifestations are common in patients with
sarcoidosis and erythema nodosum is a common cutaneous sign. Ocular symptoms usually
include uveitis, not conjunctivitis. Xerostomia is associated with Sjögren’s syndrome. A malar
rash and alopecia are dermatologic findings associated with lupus erythematosus.
Q48-A 35-year-old female is concerned about a persistent rash on her face. It waxes and
wanes in severity but she has persistent redness over her forehead, nose, cheeks, and chin. She
also reports an occasional flush that usually lasts less than 5 minutes, sometimes spreads to her
neck and chest, and is accompanied by a sensation of warmth. In addition to redness and
flushing, the affected areas often contain papules, pustules, and small red streaks. Your
examination confirms erythema over her centrofacial features with accompanying
papulopustular lesions on her nose and telangiectasia on her chin and nose.
Which one of the following agents is an FDA-approved first-line topical therapy for this
patient’s condition?
A) Benzoyl peroxide
B) Clindamycin (Cleocin)
C) Metronidazole
D) Selenium sulfide
E) Tretinoin.

ANSWER: C
This patient presents with the classic symptoms and signs of rosacea. FDA- approved first-line
agents for treating rosacea include metronidazole 0.75% lotion, 0.75% cream, and 1% gel;
azelaic acid 15% gel; sulfacetamide 10%/sulfur 5% cream, foam, lotion, and suspension;
brimonidine 0.33% gel; and ivermectin 1% cream (SOR A).

Q49-A 61-year-old truck driver is admitted to your service through the emergency department
in the evening because of possible cellulitis. When you examine him in the morning he has
blistering along his left forearm and dorsal hand that has not receded from the lines drawn in
the emergency department. It is not erythematous or tender. He does not recall any injuries,
insect bites, infections, or fever, and has been afebrile since admission. He denies pain in his
arm but does complain of itching. Laboratory studies reveal a normal CBC and comprehensive
metabolic panel,with the exception of
an ALT (SGPT) level of 100 U/L (N 7–35)
and an AST (SGOT) level of 75 U/L (N 10–34).

The patient reports no significant past medical problems but has had similar blistering
intermittently on the back of both hands that is worse in the summer. He admits to drinking
alcohol heavily on the weekends when he is not driving and has smoked a pack of cigarettes
per day for the last 45 years.
Which one of the following would be most appropriate at this point?
A) Intravenous antibiotics until clinical improvement is noted
B) Discharge on oral antibiotics
C) Total serum, plasma, or spot urine porphyrins
D) Ultrasonography of the arm to rule out an underlying abscess
E) Surgical consultation for possible debridement

ANSWER: C
Porphyria cutanea tarda (PCT) should be suspected in patients who present with blistering
lesions on sun-exposed skin. Factors associated with increased susceptibility to PCT include
those that cause liver damage, such as alcohol use and hepatitis C, as well as smoking,
estrogen use, HIV infection, and HFE mutations. The initial test in suspected PCT should be
total serum, plasma, or spot urine porphyrins. The clinical presentation of this patient is less
consistent with cellulitis, which usually presents with erythema, pain, and warmth, and when
severe may lead to fever and leukocytosis.

Q50-A 4-year-old female is brought to your office after a recent camping excursion and
presents with a large number of mosquito bites. She has developed a number of areas of honey-
colored crusting lesions you diagnose as nonbullous impetigo.
Which one of the following oral medications is preferred for this patient?
A) Cephalexin (Keflex)
B) Doxycycline
C) Erythromycin
D) Penicillin VK
E) Trimethoprim/sulfamethoxazole (Bactrim).

ANSWER: A
Nonbullous impetigo is most often caused by Streptococcus pyogenes and methicillin-
sensitive Staphylococcus aureus. Cephalexin is the most appropriate option, with good
coverage for both of these bacteria. Penicillin VK has been found to be no more effective than
placebo in the treatment of impetigo.
Macrolide resistance limits the use of erythromycin. Tetracycline should not be used in
children <8 years old as it may cause staining of permanent teeth.
Trimethoprim/sulfamethoxazole has coverage against both methicillin-sensitive and
methicillin-resistant Staphylococcus aureus, but may have inadequate coverage for
Streptococcus.

Q51- While working yesterday, a 53-year-old roofer stepped on a nail that went through his
shoe and penetrated the sole of his foot. He irrigated his foot thoroughly with water when he
got home. He comes to your office today because he has developed some erythema and pain in
his foot, with a small amount of drainage. While Staphylococcus aureus and Streptococcus
species are the most common causes of skin and soft-tissue infection, which organism should
also be considered in this situation?
A) Corynebacterium
B) Klebsiella
C) Pasteurella multocida
D) Pseudomonas aeruginosa
E) Vibrio vulnificus

ANSWER: D
Streptococcus species and Staphylococcus aureus are the most common causes of skin and
soft-tissue infections. With a puncture wound caused by an object penetrating the sole of the
shoe, another organism that should be considered as a possible cause of the infection is
Pseudomonas aeruginosa. Klebsiella is not a common cause of these infections. Pasteurella
multocida is a common cause of infection from a cat bite, and can also be seen in dog bites.
Corynebacterium is also associated with infections from animal bites. Vibrio vulnificus should
be considered when there is a skin infection after exposure to salt water.
Q52-A 29-year-old male presents with a 3-week history of multiple small, brownish-red
macules in his left inguinal fold that are now coalescing into larger macules. The rash does not
itch, and he has not used any new bath or laundry products. Wood’s lamp illumination of the
area reveals a reddish fluorescence. Which one of the following topical treatments would be
most appropriate for this patient’s rash?
A) Erythromycin
B) Hydrocortisone
C) Nystatin
D) Selenium sulfide
E) Zinc oxide

ANSWER: A
An intertriginous rash of brownish-red macules that often coalesce is characteristic of a
bacterial infection caused by Corynebacterium minutissimum known as cutaneous erythrasma.
Coral-red fluorescence under a Wood’s lamp confirms the diagnosis. The best treatment for
erythrasma is oral or topical erythromycin, with the oral form being most effective. Topical
clindamycin and antibacterial soaps may also be of some benefit.
ABFM2015

Q53-Examination of a 2-day-old infant reveals flesh-colored papules with an erythematous


baselocated on the face and trunk, containing eosinophils. Which one of the following would
be most appropriate at this time?
A) An allergy evaluation
B) Low-dose antihistamines
C) Hydrocortisone cream 0.5%
D) A sepsis workup
E) Observation only

ANSWER: E
This infant has findings consistent with erythema toxicum neonatorum, which usually resolves
in the first week or two of life (SOR A). No testing is usually necessary because of the distinct
appearance of thelesions. The cause is unknown.

Q54-A 7-year-old male is brought to your office with a 2-day history of rash. He developed
two itchy spots on his legs yesterday and today he has multiple purple, slightly painful lesions
on his legs.A few days ago he was ill with cold-like symptoms, stomach pain, and a fever up
to 101.2°F. He complained of leg pain at the time and his left ankle is now swollen. His fever
resolved 2 days ago and he now feels fine but limps when he walks.
On examination he is afebrile with a normal blood pressure and pulse rate. He is active in the
examination room. His physical examination is normal except for purpuric lesions on his legs
and buttocks and edema and mild pain of the left ankle. A urinalysis is negative.

Which one of the following would be most appropriate in the management of this patient?

A) Acetaminophen
B) Amlodipine (Norvasc)
C) Amoxicillin
D) Cyclophosphamide
E) Prednisone

ANSWER: A
This patient meets the clinical criteria for Henoch-Schönlein purpura (HSP), an immune-
mediated vasculitis found commonly in children under the age of 10. The clinical triad of
purpura, abdominal pain, and arthritis is classic. Almost 95% of children with HSP
spontaneously improve, so supportive therapy is the main intervention.
Acetaminophen or ibuprofen can be used for the arthritic pain. However, ibuprofen should be
avoided in those with abdominal pain or known renal involvement. Prednisone has been found
to help in those with renal involvement or other complications of the disease such as
significant abdominal pain, scrotal swelling, or severe joint pains (SOR B). However, it is not
effective for preventing renal disease or reducing the severity of renal involvement, as was
once thought (SOR A).
Immunosuppressants such as cyclophosphamide and cyclosporine have been suggested for
treating patients with severe renal involvement, but there is insufficient evidence to support
their use. Amoxicillin is appropriate for patients with a bacterial infection, such as
streptococcal pharyngitis, which has led to HSP. In this patient, however, there is no indication
of pharyngitis or another bacterial focus. Patients with renal involvement and resultant
hypertension with HSP should be treated with calcium channel blockers such as amlodipine.
This patient exhibits neither renal involvement nor hypertension.

Q56-The intensely pruritic rash shown below is typical of ( no pic )


A) contact dermatitis
B) herpes simplex
C) pityriasis rosea
D) tinea corporis
ANSWER: A
Allergic contact dermatitis is secondary to a trigger that incites a delayed (type IV)
hypersensitivity reaction. The most common sensitizers include plants (poison ivy, poison oak,
and poison sumac), metals (nickel found in jewelry or belt buckles), and fragrances. Patch
testing data has shown that out of 3700
known contact allergens, nickel caused contact dermatitis in 14.3% of patients, fragrance mix
in 14%, neomycin in 11.6%, balsam of Peru in 10.4%, and thimerosal in 10.4%. The rash is
limited to the area of exposure and is characterized by an intensely pruritic papular eruption
with erythema. Herpes simplex is characterized by a vesicular eruption surrounded by
erythema and associated with localized burning and tingling. Tinea corporis presents as a
pruritic circular or oval erythematous lesion with superficial scaling
and erythema. Multiple oval or circular pruritic salmon-colored scaly lesions preceded by a
herald patch are typical of pityriasis rosea (SOR C).
Q57-A 45-year-old male is seen for a well-demarcated, nonpruritic rash in the right axilla. It is
fine-scaled with a cigarette-paper appearance. The rash has a coral-red fluorescence under a
Wood’s light.
Which one of the following is the most likely diagnosis?
A) Candidiasis
B) Tinea cruris
C) Erythrasma
D) Inverse psoriasis
ANSWER: C
All of the diagnoses listed are intertriginous rashes but only erythrasma fluoresces with Wood’s
light. Erythrasma is a superficial gram-positive bacterial infection caused by Corynebacterium
minutissimum. The fluorescence is caused by porphyrins. Erythrasma is most often seen
between the toe web spaces, followed by the groin and axillae. There are multiple treatments,
including topical and oral erythromycins and clindamycins (level of evidence 3, strength of
evidence 1).

Q58-A 31-year-old male has experienced multiple outbreaks of the rash shownbelow. He was
initially told that the rash was due to an allergy to an antibiotic prescribed for a suspected
dental abscess, but avoiding all medications has not prevented the recurrences. Which one of
the following oral medications has been shown to reduce the severity, duration, and recurrences
of this type of rash?
A) Acyclovir
B) Cetirizine (Zyrtec)
C) Prednisone
D) Ranitidine (Zantac)
E) Terbinafine (Lamisil)
ANSWER: A
Sharply demarcated lesions with raised borders surrounding a paler region containing a darker
center (target or iris lesions) are characteristic of erythema multiforme. The lesions of
erythema multiforme usually appear on the distal extremities, are often accompanied by
burning and pruritus, and may progress centrally. Usually the rash resolves spontaneously
within 4–6 weeks but some patients experience frequent recurrences. Erythema multiforme
results from a hypersensitivity reaction to any number of medications, vaccine preparations, or
infections, the most commonly identified being herpes simplex virus (HSV) infection. In a
minority of those harboring HSV infection, recurrent outbreaks of erythema multiforme are
often associated with HSV reactivations, even those that may occur unnoticed. Continuous
antiviral treatment using acyclovir, valacyclovir, or famciclovir has been shown to be effective
in reducing or eliminating the frequency of recurrent outbreaks in these patients (SOR A).
In patients not helped by daily antiviral suppressive therapy, treatment with dapsone,
azathioprine, cyclosporine, and thalidomide have been used with some success, but evidence-
based data supporting the use of these drugs is limited.

Q59-While performing a digital rectal examination of the prostate on a 67-year- old patient
with diabetes mellitus, you note the findings shown below. The patient confirms that the area
has been itchy for some time but he has been reluctant to seek care. He has tried a variety of
over-the-counter moisturizing lotions with limited success. Of the following topical
treatments, which one is most likely to provide significant improvement?

A) Antibacterial ointment
B) Antifungal cream
C) Antiviral ointment
D) Corticosteroid cream
E) Rubbing alcohol.

ANSWER: D
Plaque psoriasis is characterized by silvery-white scales adhered to well demarcated
erythematous papules and/or plaques, typically on the scalp, extensor surfaces of the elbows
and knees, or buttocks, and often extending to other exposed areas of the body. When limited
to skin folds or the genital region, psoriasis can easily be confused with other conditions such
as bacterial or fungal intertrigo. The lesions in this variant, known as flexural or inverse
psoriasis, usually appear smooth and moist to the point of maceration, often with minimal to
no scaling. Affected patients may report significant pruritus and an unpleasant odor in the
involved area. Evidence-based data for treatment options is limited but supports topical
application of mild corticosteroid creams, vitamin D preparations, or coal tar products.
Medium- or higher-potency corticosteroid creams are best avoided, as the affected areas are
either delicate, occlusive, or both, and susceptible to corticosteroid-induced atrophy.

Q60-A school nurse discovers head lice on a fourth-grade student. When should the student be
permitted to return to class?
A) Immediately
B) When there are no visible nits
C) After a single treatment with a topical agent
D) After two treatments with a topical agent, 7 days apart

ANSWER: A
Head lice are a common and easily treated inconvenience in school-aged children that, unlike
body lice, are not associated with significant illnesses. Transmission generally requires head-
to-head contact, as lice cannot survive when separated from their host for more than 24 hours
and do not fly or hop. Visible nits are generally present at the time of diagnosis, confirming
that the infestation has been present for some time, so immediate isolation from other children
would not be expected to change the natural course of events. The American Academy of
Pediatrics (AAP) recommends that children found to be infested with lice remain in class but
be discouraged from close contact with others until treated appropriately with a pediculicide.
The AAP position also recommends abandonment of “no nits” school policies, which prohibit
attendance until no visible nits are identified. Nits can be found long after their deposition at
and generally have already hatched by the time they are easily noted at some distance from the
scalp.

Q61-Which one of the following, especially in homeless people, is a vector for Bartonella
quintana, which causes trench fever, an influenza-like syndrome withrelapsing fever?
A) Fleas
B) Maggots
C) Bedbugs
D) Scabies
E) Lice

ANSWER: E
Lice, scabies, and secondary bacterial infections are endemic in the homeless. Body lice
transmit Bartonella quintana, which causes trench fever. This disease got its name in World
War I, when soldiers in the trenches were often infested with body lice. This is a serious
disease that can be treated with antibiotics.
Q62-Which one of the following is more typical of a keloid rather than a hypertrophic scar?
A) Location on an extensor surface
B) Expansion beyond the margins of the inciting injury
C) Development soon after the inciting trauma
D) Regression over time

ANSWER: B
Keloids may arise from scars that result from any cause of skin trauma or infection. Keloids,
unlike hypertrophic scars, are not confined to the margins of the primary injury. Hypertrophic
scars are more likely on extensor surfaces of the body, typically develop soon after the inciting
injury, and are more likely to regress with time.

Q63-Which one of the following has been shown to be effective for Lyme disease prophylaxis
after removal of an engorged deer tick?
A)Amoxicillin
B)Ceftriaxone (Rocephin)
C)Cefuroxime axetil (Ceftin)
D)Doxycycline
E)Clarithromycin (Biaxin)

ANSWER: D
While all of the antibiotics listed have been used to treat Lyme disease, the only antibiotic that
has been shown to be effective for chemoprophylaxis is doxycycline. A randomized,
controlled trial showed that a single 200-mg dose of doxycycline was 87% effective for
preventing Lyme disease if given within 72 hours after removal of a deer tick. Nevertheless, a
meta-analysis showed that the number needed to treat to prevent one case of erythema migrans
was 50, and routine prophylaxis is not recommended. It may be indicated, however, after
removal of an engorged nymphal deer tick.
ABFM2014
64-A 44-year-old male with papulopustular rosacea sees you for follow-up. You have been
treating his condition with topical azelaic acid (Finacea), and although his condition is
improved he is not satisfied with the results.
You suggest adding which one of the following oral medications?
A) Clarithromycin (Biaxin)
B) Clindamycin (Cleocin)
C) Doxycycline
D) Erythromycin
E) Metronidazole (Flagyl).

ANSWER: C
The only FDA-approved oral treatment for acne rosacea is doxycycline at a subantimicrobial
dosage (40 mg daily). This does not contribute to antibiotic resistance, even when used over
several months, and is better tolerated than higher dosages. Other antibiotics have limited and
low-quality supporting evidence of efficacy and may lead to antibiotic resistance

Q65- A 56-year-old male with diabetes mellitus and hypertension presents with a 6-month
history of generalized pruritus. He reports that he scratches frequently. On examination his
skin is dry and scaly. He has multiple linear excoriations and thickened skin on his forearms,
legs, and neck.
Which one of the following is the most likely cause of his pruritus?
A) Contact dermatitis
B) Chronic urticarial
C) Lichen simplex chronicus
D) Scabies

ANSWER: C
This patient has lichen simplex chronicus, consisting of lichenified plaques and excoriations
that result from excessive scratching. Treatment focuses on stopping the itch-scratch cycle.
Topical corticosteroids under an occlusive dressing or intralesional corticosteroids can be
helpful. Scabies lesions are small, erythematous papules that are frequently excoriated.
Contact dermatitis is usually associated with direct skin exposure to an allergen or irritant and
is typically localized to the area of exposure. Chronic urticaria causes a typical circumscribed,
raised, erythematous lesion with central pallor.
Q66-A 53-year-old white female with chronic hepatitis C is concerned about ulcers in her
mouth. She is not currently receiving therapy. Your examination reveals several ulcers
involving the buccal mucosa. The patient also points out a number of pruritic, reddish-purple
plaques on her wrists, ankles, and back.
Laboratory studies are within normal limits except for mildly elevated transaminases.
Which one of the following is the most likely diagnosis?
A) Behçet’s syndrome
B) Lichen planus
C) Aphthous stomatitis
D) Herpetic stomatitis
E) HIV infection.

ANSWER: B
Lichen planus is an idiopathic inflammatory disease affecting the skin and oral mucosa. The
characteristic violaceous, polygonal papules may be intensely itchy. There is a significant
association between lichen planus and hepatitis C virus infection 3-A 4-year-old female is
treated at a local urgent care center with amoxicillin for acute pharyngitis. Several days after
starting treatment her initial symptoms resolve. When she is 8 days into the 10-day course of
her antibiotic treatment she returns to your office because she has developed a diffuse
erythematous maculopapular rash starting on her torso and extending to her proximal
extremities.

Q67-Which one of the following is the best course of action at this time?

A)Continue the amoxicillin and begin prednisone and diphenhydramine (Benadryl)


B)Continue the amoxicillin and change the diagnosis to scarlet fever
C)Discontinue the amoxicillin and change the diagnosis to viral exanthema
D)Discontinue the amoxicillin and note amoxicillin as a potential allergy in her record

ANSWER: D
The cause of this patient’s rash is difficult to determine. There are many infections that could
result in a cutaneous reaction similar to what she is experiencing.
Scarlet fever is caused by a systemic reaction to Streptococcus. In this case, however, the
patient is already taking an antibiotic for streptococcal disease so the emergence of new
symptoms over a week after starting therapy is highly unlikely. A viral exanthem could also
cause a skin rash similar to the one described here.
Unfortunately, differentiating between a drug-induced rash and a viral exanthem is not
clinically possible. If this differentiation is necessary, the patient should undergo a skin biopsy
and allergy testing to determine the offending agent. However, since this approach is
impractical in the ambulatory setting, it is most straightforward to discontinue the agent she is
on and list it as a potential allergy. An alternative antibiotic such as erythromycin could be
used to complete the course of treatment at the discretion of the physician

Q67-A 24-year-old male who just moved to town for a new job presents to your office with a
2-week history of a rash. His previous medical records are not available. The physical
examination reveals pink, scaling papules and plaques on the trunk and proximal aspect of the
arms and legs. You suspect pityriasis rosea. To complete the diagnostic evaluation you should
order
A) a fungal culture
B) heterophile antibody testing
C) a platelet count
D) a rapid plasma reagin (RPR) test
E) a TSH level.
ANSWER: D
The differential diagnosis of multiple small scaling plaques includes drug eruptions, secondary
syphilis, guttate psoriasis, and erythema migrans. If the diagnosis cannot be made conclusively
by clinical examination, a test for syphilis should be ordered. The rash of secondary syphilis
may be indistinguishable from pityriasis rosea on initial examination, particularly when no
herald patch is noted. The rashes associated with hyperthyroidism, infectious mononucleosis,
idiopathic thrombocytopenic purpura, and fungal infections are not in the differential diagnosis
for this patient

Q68- A 70-year-old female presents with a blotchy red rash on both of her legs and feet. The
rash started 2 days ago and is associated with fatigue. Her past medical history is
unremarkable except for acute cystitis treated with a 3-day course of an antibiotic last week.
On examination her vital signs are normal, as is the remainder of her physical examination,
with the exception of a palpable purpuric rash on her lower extremities, shown below. The
patient’s CBC, INR, and partial thromboplastin time are all normal.
Which one of the following is now indicated?
A) Supportive care only
B) Antihistamines
C) Broad-spectrum antibiotics
D) Plasmapheresis
E) Platelet transfusion.
ANSWER: A
The presence of palpable purpura along with a history of recent or current medication use is
highly suggestive of drug-induced vasculitis. Skin is the organ most often affected by drug-
induced vasculitis, with lesions predominantly affecting the lower extremities. Although
clinical findings may be limited to the skin, systemic manifestations such as fever, fatigue, and
joint pains can be present. Offending drugs include sulfonamides, allopurinol, thiazides,
phenytoin, and penicillins. A skin biopsy will reveal the presence of eosinophils. The first step
in management includes discontinuation of the offending agent. Patients with mild and non–
life-threatening small-vessel vasculitis should be treated with supportive care, while those
with more severe features should receive
corticosteroids. There is no role for antihistamines, plasmapheresis, or platelet transfusion in
drug-induced vasculitis (SOR B.
Q69-A 20-year-old African-American female asks if you can help eradicate an unsightly
hypertrophic growth of skin that has developed in an area where she had a mole removed. She
reports that this tissue has grown to become at least three times larger than the original lesion
and that it is darkly pigmented, firm, and pruritic. On examination you note a firm, smooth,
shiny, raised 1×4-cm plaque on the patient’s chest that is darker than the surrounding skin.
Which one of the following is first-line therapy for this lesion?
A) Silicone gel sheeting
B) Topical imiquimod (Aldara)
C) Intralesional corticosteroid injection
D) Surgical excision
E) Laser destruction.

ANSWER: C
The patient’s history and physical findings are all consistent with a keloid, which is a benign
overgrowth of scar tissue at sites of trauma to the skin, such as acne, burns, surgery, ear
piercing, tattoos, and infections. Common locations include the earlobes, jawline, nape of the
neck, scalp, chest, and back. Lesions are sometimes asymptomatic, but often are associated
with hypersensitivity, pain, and pruritus. The incidence is higher in blacks, Hispanics, and
Asians. Intralesional corticosteroid injections are first-line therapy. Silicone gel sheeting,
topical imiquimod, and intralesional fluorouracil can be used when first-line therapy fails, but
these methods are more often associated with recurrence. Laser therapy and surgical excision
are associated with a high rate of recurrence when used as monotherapy
Q70-A 38-year-old female presents with an itchy rash she says has been present for the past
several months. She has been using over-the-counter hydrocortisone cream with no
improvement. On examination she has an oval 12-cm erythematous plaque on her buttocks.
The plaque is covered with silvery scales.
Which one of the following would be the best initial treatment for her rash?
A) Clobetasol 0.05% cream (Temovate) applied twice daily
B) Clotrimazole 1% cream applied twice daily
C) Diclofenac 3% gel applied twice daily
D) Mupirocin 2% ointment (Bactroban) applied twice daily
E) Salicylic acid 2% gel applied twice daily
ANSWER: A
This patient has mild plaque psoriasis. Topical corticosteroids, vitamin D analogs, and
tazarotene are effective treatments for mild psoriasis (SOR A). Antimicrobials, anitfungals,
diclofenac, and salicylic acid are not effective treatments

Q71-A 65-year-old female presents with an 11-mm lesion on her nasolabial fold.
You perform a shave biopsy that confirms basal cell carcinoma.
Which one of the following would be the most appropriate treatment of this lesion?
A) Excision with wide margins
B) Electrodesiccation and curettage
C) Mohs micrographic surgery
D) Cryotherapy
E) Imiquimod (Aldara) cream
ANSWER: C
Basal cell carcinoma is the most common invasive malignant cutaneous neoplasm in humans.
The tumor rarely metastasizes but it can advance by direct extension and can destroy normal
tissue. Approximately 85% of all basal cell carcinomas occur on the head and neck, with
25%–30% on the nose. Lesions on the nose, eyelid, chin, jaw, and ear have higher recurrence
rates than lesions in other locations. A biopsy is necessary to make a definitive diagnosis prior
to treatment. Excision is preferred for larger tumors with well-defined borders, but wide
margins are not necessary. It is very difficult to perform this surgery with a primary closure
around the nose. For lesions around the nose, especially those >1 cm, Mohs micrographic
surgery is the preferred treatment. This is a microscopically controlled technique that
facilitates removal of the entire lesion with the least amount of tissue removed.
Imiquimod is an immune response modifier that can be used on superficial basal cell
carcinomas but should not be used for a site with a high risk of recurrence.
Electrodesiccation and curettage is effective for smaller nodular basal cell carcinomas.
Cryotherapy is not recommended
Q72--You are treating an 18-year-old college freshman for allergic rhinitis. It is September,
and he tells you that he has severe symptoms every autumn that impair his academic
performance. He has a strongly positive family history of atopic dermatitis.
Which one of the following intranasal medications is considered optimal treatment for this
condition?
A) Glucocorticoids
B) Cromolyn sodium
C) Decongestants
D) Antihistamines

ANSWER: A
Topical intranasal glucocorticoids are currently believed to be the most efficacious medications
for the treatment of allergic rhinitis. They are far superior to oral preparations in terms of
safety. Cromolyn sodium is also an effective topical agent for allergic rhinitis; however, it is
more effective if started prior to the season of peak symptoms. Because of the high risk of
rhinitis medicamentosa with chronic use of topical decongestants, these agents have limited
usefulness in the treatment of allergic rhinitis.
Azelastine, an intranasal antihistamine, is effective for controlling symptoms but can cause
somnolence and a bitter taste. Oral antihistamines are not as useful for congestion as for
sneezing, pruritus, and rhinorrhea. Overall, they are not as effective as topical glucocorticoids.

Q73- An 18-month-old female with atopic dermatitis is brought to your office. She has recently
had a flare-up of her condition that has been slow to resolve, and the mother says the child
scratches “constantly” despite daily use of emollients.

Which one of the following would be the best treatment?

A) A topical mild-potency corticosteroid


B) Topical pimecrolimus (Elidel)
C) Oral diphenhydramine (Benadryl)
D) Oral cetirizine (Zyrtec)
E) Probiotics
ANSWER: A
Topical corticosteroids are the first-line treatment for atopic dermatitis flare-ups. Topical
calcineuron inhibitors such as pimecrolimus are a second-line therapy, but carry a warning of a
possible link to lymphomas and skin malignancies and are not recommended for children under
2 years of age. Oral antihistamines are not effective for the pruritus associated with atopic
dermatitis. Probiotic use is not supported by available evidence.

Q74- A 20-year-old college student who has been working in the woods on a forestry project
presents with a 3- to 4-day history of a severely pruritic rash on his arms, hands, and face.
There is erythema with multiple bullae and vesicles, some of which are in a streaked linear
distribution on the arms. There are patches of erythema on his face with some vesicles. The
itching is intense and he sleeps fitfully. In addition to cool compresses and antihistamines for
the itching, which one of the following is the best treatment option for this patient?
A) Triamcinolone, 20 mg intramuscularly as a single dose
B) A 6-day oral methylprednisolone (Medrol) dose pack, starting at 24 mg
C) A 7- to 10-day course of topical halobetasol propionate (Ultravate), 0.05% ointment
D) A 7- to 10-day course of topical mupirocin (Bactroban) 2%, after decompression of
vesicles and bullae
E) A 10- to 14-day tapering course of oral prednisone, starting at 60 mg.

ANSWER: E
Poison ivy dermatitis is caused by urushiol, a resin found in poison ivy, poison oak, and poison
sumac plants. Direct contact with the leaves or vines will result in an acute dermatitis
manifested initially by erythema, and later in more severe cases by vesicles and bullae. This is
a type IV T cell–mediated allergic reaction, so it typically takes at least 12 hours and often 2–3
days before the reaction is fully manifested. Depending on the degree of contact (i.e., the
amount of resin on the skin), the rash often progresses over a couple of days, giving the
impression that it is spreading. Also, delayed contact with resin from contaminated clothing,
gloves, or pets may result in new lesions appearing over several days. Brushing against the
leaves of the plant causes the linear streaking pattern characteristic of poison ivy dermatitis. It
has been demonstrated that the resin can be inactivated with any type of soap, thereby
preventing the reaction, but the sooner the better.
Approximately 50% of the resin can be removed by soap and water within 10 minutes of
contact, but after 30 minutes only about 10% can still be removed. Therapy depends on the
severity of the reaction. Group I–V topical corticosteroids are effective for limited eruptions
(less than 3%–5% body surface area) but are ineffective in areas with vesicles or bullae.
Group I–II fluorinated agents are at the strongest end of the spectrum and are not
recommended for use on the face or intertriginous areas. Short bursts of low-potency oral
corticosteroids such as a methylprednisolone dose pack have a high rate of relapse as the taper
finishes, so the expert consensus is to use a higher dosage tapered over a longer period,
generally 10–14 days, in order to prevent a relapse. Most experts recommend oral
corticosteroids over intramuscular corticosteroid suspensions, which may not provide high
enough concentrations in the skin (SOR C). However, 40–80 mg of intramuscular
triamcinolone (or an equivalent) is an alternative to oral treatment, especially if adherence is an
issue. Pruritus can be treated
with oral antihistamines. Secondary infection, which is common with vesiculobullous
involvement, is treated with appropriate oral antibiotics.

Q74-A 21-year-old male comes to your office for a follow-up visit to discuss pharmacologic
treatment for his acne. He has moderate inflammatory acne lesions with comedones and
several papules and pustules, but few nodules.
Multiple topical antibiotic therapies, in combination with benzoyl peroxide, have been
minimally effective. He is currently using just topical benzoyl peroxide. You would like to
prescribe an oral agent to add to his regimen.
Which one of the following would be the most effective oral medication to start at this time?
A) Amoxicillin
B) Ciprofloxacin (Cipro)
C) Minocycline (Minocin)
D) Prednisone

ANSWER: C
Oral antibiotics are effective for the treatment of moderate to severe acne (SOR A). Combined
treatment with benzoyl peroxide is recommended to reduce the risk of bacterial resistance
(SOR C). Amoxicillin and ciprofloxacin are not recommended for acne treatment.
Intralesional corticosteroid therapies have been tried for acne treatment, but long-term use of
oral corticosteroids is not recommended.
DM
RETEST2019
Q1. You are interested in providing nutrition classes to your patients at risk for developing
type 2 diabetes (T2DM). According to the American Diabetes Association (ADA), which of
the following patients is at risk for developing T2DM and should be screened for diabetes?

a.A 46-year-old male patient who has obesity, hypertension, and dyslipidemia who had an A1C
of 5.5% last year
b.A 23-year-old female with a BMI of 24 kg/m2 and whose grandfather was atype 1 diabetic
c.A 22-year-old sedentary male patient with a BMI of 29 kg/m2 and no past medical history
d.A 33-year-old morbidly obese female who is 3 months postpartum and had gestational
diabetes during her pregnancy
e.A 42-year-old Asian American male marathon runner with a BMI of 22 kg/m2

The answer is d.
The ADA recommends screening all persons older than 45 years for diabetes every 3 years.
Screens should start earlier in people with risk factors including a sedentary lifestyle, family
history of diabetes in a first-degree relative, hypertension, overweight or obesity (BMI ≥ 25
kg/m2 or ≥ 23 kg/m2 in Asian Americans), high-risk ethnic groups (African American,
Hispanic, Native American), a previous history of impaired glucose tolerance, abnormal lipids
(especially elevated triglycerides and low HDL), a history of CVD, and women with polycystic
ovary syndrome, a history of gestational diabetes, or a birth of a child greater than 9 lb.

Q2. You are evaluating a 36-year-old obese woman who complains of fatigue. She denies
polydipsia, polyuria, polyphagia, or weight loss. Which of the following laboratory
reports\confirms the diagnosis of diabetes?

a. A random glucose reading of 221 mg/dL


b. A random glucose reading of 221 mg/dL, and another, on a later date, of 208mg/dL
c. A fasting glucose measurement of 128 mg/dL
d. A glucose reading, taken 2 hours after a 75-g glucose load, of 163 mg/dL
e. A hemoglobin A1C of 6.3%

The answer is c.
The diagnosis of diabetes may be made by random glucose measurements more than 200
mg/dL with classic signs of diabetes (polydipsia, polyuria, polyphagia, weight loss), a fasting
glucose greater than 126 mg/dL, a hemoglobin A1C level of more than or equal to 6.5%, or a
glucose reading greater than 200 mg/dL 2 hours after a 75-g glucose load. All results should be
confirmed with repeat testing.
Q3. An 18-year-old morbidly obese patient in your office is found to have a fasting glucose of
314 mg/dL. Which of the following test results would indicate that he is a type 1 diabetic?
a. Low levels of C-peptide
b. Markedly elevated levels of C-peptide
c. Elevated levels of microalbumin in the urine
d. A markedly elevated hemoglobin A1C
e. The presence of parietal cell antibodies

The answer is a.
In the past, young adults diagnosed with diabetes were primarily type 1 (T1DM). However, the
epidemic of obesity in the United States has increased the rate of T2DMin people less than 20
years old from 5% to 30% over the last decade. C- terminal peptide is cleaved from natively
produced insulin. Fasting C-peptide levels are markedly elevated in patients with T2DM, but
in people with T1DM, C- peptide levels should be low
due to the absence of native insulin. Microalbuminuria, markedly elevated hemoglobin A1C,
and peripheral neuropathy can all occur in type 1 or 2 diabetes. T1DM is caused by
autoimmune destruction of the pancreatic islet cells and the antibody glutamic acid
decarboxylase (GAD 65) is most commonly used to diagnose T1DM. Parietal cell antibodies
are not associated with T1DM, but with pernicious anemia.

For questions 4 to 6, use the following clinical scenario:


You are managing a 36-year-old woman with a new diagnosis of T2DM. Past medical history
includes depression, venous insufficiency, and gestational diabetes with each of her
pregnancies. Her fasting plasma glucose is 287 mg/dL, the remainder of her metabolic panel is
normal, and her hemoglobin A1C is 9.2%. Urinalysis is normal, but her albumin-to-creatinine
ratio is elevated. She has no previous evidence of kidney disease. Her BMI is 33 kg/m2 and
her blood pressure is 128/76. She denies any symptoms of hyperglycemia, including
polydipsia, polyphagia, or blurred vision.
Q4. What is the most appropriate first course of action for this patient?
a.Admit her to the hospital and start an insulin drip and replace electrolytes as needed
b.Start metformin 1000 mg twice daily
c.Start insulin glargine 0.2 U/kg/d
d.Start metformin and a glucagon-like-peptide 1 (GLP-1) receptor agonist
e.Start metformin and a thiazolidinedione (TZD)

The answer is d.
The ADA recommends dietary modifications and pharmacologic therapy at diagnosis of
T2DM based on A1C and glucose levels. For patients with an A1C less than 9%, consider
monotherapy; for patients with an A1C more than or equal to 9%, consider dual therapy, and
for patients with an A1C more than or equal to 10%, blood glucose more than or equal to 300
mg/dL, or marked symptoms, consider combination injectable therapy. Patients with diabetic
ketoacidosis or hyperosmolar hyperglycemia should be treated inpatient with an initial insulin
drip and close monitoring. This patient has a normal urinalysis/electrolytes and is
asymptomatic, so it is unlikely she has either condition. Dual therapy should be considered for
this patient. In the absence of CVD, the ADA recommends metformin plus a second agent
selected based on consideration of drug-specific effects and patient factors. A GLP-1 receptor
agonist would be an appropriate choice for this patient due to their neutral or negative effects
on bodyweight. TZDs tend to cause fluid retention and should not be used in patients with
congestive heart failure. In her case, it would likely negatively affect her venous insufficiency.
Insulin alone is not an appropriate initial therapy, but may be appropriate in combination with
metformin.

Q5. At this time, how would you address this patient’s positive microalbumin screen?
a. Initiate therapy with an ACE-inhibitor
b. Initiate therapy with an ARB
c. Prescribe a low-protein diet
d. Optimize glycemic control and repeat screening
e. Refer to endocrinology

The answer is d.
All patients with T2DM should be screened annually for albuminuria
with a urinary albumin-to-creatinine ratio through a random urine collection. However, two of
three specimens in a 3- to 6-month period of time should be abnormal before considering a
patient to have albuminuria. When albuminuria is confirmed, patients with T2DM should be
treated with an ACE-inhibitor (or an ARB if they are intolerant of ACEs) to slow the
progression toward ESRD. Referral to nephrology is indicated when the GFR falls below 60,
or when the albumin-to- creatinine ratio exceeds 300 mg/g, or if the cause of the kidney
disease is unclear. Patients with albuminuria should limit dietary protein intake to 0.8g/kg
bodyweight/day. However, this patient has only had one positive albumin-to- creatinine ratio
and the most appropriate therapy at this time would be to optimize her glycemic control and
repeat the screening in 3 months.

Q6. Considering this patient’s diagnosis and history, which of the following is true regarding
retinopathy prevention and surveillance?
a.A daily aspirin will reduce her retinopathy risk.
b.She should have a dilated eye examination now and quarterly until her A1C is at goal.
c.Lowering her A1C does not reduce retinopathy risk.
d.She should have an annual dilated eye examination beginning at diagnosis.
e.Retinopathy occurs at lower levels of hemoglobin A1C for Caucasian patients.

The answer is d.
Diabetic retinopathy is the leading cause of blindness in the United States. The risk increases
with the length of time that the patient has had diabetes, and the condition worsens with
increasing hemoglobin A1C levels. However, African- American patients develop retinopathy
at lower levels of A1C. In type 2 diabetics it can be seen at diagnosis, and yearly dilated eye
examinations are recommended beginning at the time of diagnosis. In patients with T1DM,
dilated eye examinations should begin 5 years after diagnosis. More frequent examinations
may be indicated in patients when eye pathology is present. Aspirin has no effect on eye
complications.

Q7. You are seeing an African-American man with newly diagnosed diabetes. His blood
pressure at the last visit was 148/76 mm Hg, and at this visit it is 152/82 mm Hg. He has no
evidence of CVD or CKD and is not on any antihypertensive agents. Which of the following
statements is true regarding the use of an ACE- inhibitor in this patient?
a.An ACE-inhibitor should be added to his regimen because he is diabetic, regardless of his
blood pressure.
b.An ACE-inhibitor is the preferred agent to be added to his regimen based on his blood
pressure readings.
c.An ACE-inhibitor should not be added to his regimen unless his blood pressure goes above
160 systolic.
d.An ACE-inhibitor should not be added to his regimen unless he has microalbuminuria.
e.An ACE-inhibitor should not be given to this patient if his creatinine is elevated.

The answer is d.
ACE-inhibitors are clearly the first choice for blood pressure control in diabetic patients with
CKD. They control blood pressure effectively, help prevent progression of renal disease, and
are indicated in the presence of coronary disease and CHF. Although compelling, there is
insufficient evidence to recommend ACE-inhibitors in all diabetic patients. According to the
Eighth Joint National Committee (JNC-8) guidelines, African- American patients with T2DM
without albuminuria and blood pressure readings between 140/90 and 160/100 mm Hg should
receive either a thiazide-type diuretic or a calcium channel blocker alone or in combination as
initial treatment. If albuminuria is present, an ACE or an ARB should be considered first.
Among these four classes of medication, the ADA guidelines don’t specify a preferred initial
agent unless albuminuria is present, then they recommend an ACE or ARB preferentially.
ACE/ARBs can be used irrespective of creatinine levels, though potassium should be
monitored as creatinine rises.

Q8. You are seeing a 75-year-old male patient with a 10-year history of T2DM for his 3-
month diabetes follow-up. He takes 20 units of insulin glargine at bedtime and 1000 mg of
metformin twice daily. His home glucose readings show fasting sugars in the range of 60 to
130 and post-prandial readings in the range of 145 to
210. His A1C today is 8.2%. He has several co-morbidities, including CHF, hypertension,
CKD, and a history of a myocardial infarction (MI) 3 years ago. He has had several falls and
difficulty with ambulation because of diabetic neuropathy. His daughter is concerned about his
episodes of low blood sugar, but he states they don’t bother him. What A1C goal would you
recommend for this patient?
a.Around 6.5%
b.Around 7.0%
c.Around 7.5%
d.Around 8.0%
e.Around 8.5%
The answer is e.
The ADA recommends an A1C glycemic target of less than 7% for nonpregnant adults;
however, providers may suggest less stringent goals for individual patients if this cannot be
achieved without significant hypoglycemia. Less stringent goals may also be appropriate for
patients with a history of severe hypoglycemia, limited life expectancy, advanced
microvascular or macrovascular complications, extensive comorbid conditions, or long-
standing diabetes that is difficult to control. This patient is elderly, with multiple
comorbidities and evidence of hypoglycemia. The presence of hypoglycemia, especially
severe or asymptomatic hypoglycemia, should prompt the clinician to raise this patient’s
glycemic targets to avoid hypoglycemia, because it is associated with increased morbidity and
mortality. His current A1C is 8.2% and the only answer choice that is less stringent is 8.5%.

Q9. A 44-year-old African American with T2DM transfers care to you. Reviewing her
records, you find she is on the maximum dose of metformin and her hemoglobin A1C is 7.0%.
Review of her baseline laboratory tests reveals normal liver enzymes and a creatinine of 2.3
mg/dL (GFR 35 mL/min/1.73 m2). Her BMI is 26 kg/m2. She is hypertensive and is
well controlled on a thiazide diuretic and an ACE-inhibitor. Which of the following
management options would be the most beneficial?
a. Continue the current regimen
b. Add glyburide
c. Discontinue the metformin and add linagliptin
d. Discontinue the metformin and add dapagliflozin
e. Add sitagliptin

The answer is c.
The decline of renal function can make maintaining glycemic control a challenging endeavor.
Metformin, due to the risk of lactic acidosis, should be avoided in patients with a GFR less
than 45 mL/min/1.73 m2. This patient has good glycemic control, but her declining renal
function demands a change in her regimen.
Sulfonylureas (glyburide, glipizide, and glimepiride) should also be avoided in patients with
significant renal impairment, due to their renal clearance and the increasing risk of
hypoglycemia. Sodium glucose cotransporter 2 (SGLT-2) inhibitors (dapagliflozin,
canagliflozin, and empagliflozin) should be avoided or discontinued in patients with a GFR of
30 to 60 mL/min/1.73 m2. DPP-4 inhibitors have varying degrees of utility and dosing
adjustment in patients with renal impairment, but linagliptin is not cleared by the kidneys and,
therefore, requires no dosing adjustment in CKD.
Q10. You are seeing a 29-year-old woman for a T2DM follow-up visit. She has been taking a
GLP-1 receptor agonist for the last 6 months and complains that the side effects are too
bothersome and she would like to switch medications. You recommend a dipeptidyl peptidase
4 (DPP-4) inhibitor. Which of the following statements is true about the GLP-1 receptor
agonists compared to the DPP-4 inhibitors?
a.Both agents result in a supraphysiologic increase in GLP-1 in the body
b.Both agents have similar effects on bodyweight
c.Both GLP-1 receptor agonists and DPP-4 inhibitors increase glucose-dependent insulin
secretion from the pancreas
d.Both GLP-1 receptor agonists and DPP-4 inhibitors increase gluconeogenesis in the liver
e.Both GLP-1 receptor agonists and DPP-4 inhibitors slow gastric emptying

The answer is c.
GLP-1 is a gut hormone that, when stimulated by an oral glucose load, stimulates insulin
secretion. This “incretin” effect is diminished in patients with T2DM. Native GLP-1 has a
half-life of only a few minutes due to breakdown by the enzyme DPP-4, while GLP-1 receptor
agonists are resistant to the effects of DPP-4 and result in supraphysiologic levels of GLP-1 in
the body. DPP-4 inhibitors, in contrast, prolong the action of physiologic GLP-1, restoring
levels comparable to healthy controls. This results in drug classes with varying efficacy and
side-effect profiles. Both DPP-4 inhibitors and GLP-1 receptor agonists increase glucose-
dependent insulin secretion from the pancreas and decrease glucagon production in the liver.
However, the GLP-1 receptor agonists slow gastric emptying, increase satiety, and have a
weight loss effect. This slowed gastric emptying is also responsible for the most frequent
adverse reactions of nausea and vomiting. The DPP-4 inhibitors have a weight-neutral profile
with an absence of GI side effects.
Q11. You have been treating a 46-year-old woman for T2DM for 2 years with metformin and
recently started insulin glargine at night. She is compliant with her diet and medications, and
exercises regularly. She is 65 inches tall and weighs 200 lb. Her most recent HbA1C is 9.0%
which is elevated from 8.8% 3 months ago. When you added the insulin regimen, she had
hypoglycemic episodes, so at her visit 2 weeks ago, you added the sodium-glucose
cotransporter 2 (SGLT-2) inhibitor, dapagliflozin to her regimen and stopped her insulin. The
patient presents today complaining of a problem that she attributes to the new medication.
Which of the following is the most likely complaint?
a. Symptomatic hypoglycemia
b. Edema and weight gain
c. Cough
d. Urinary tract infection
e. GI intolerance

The answer is d.
SGLT-2 inhibitors block renal reabsorption of glucose, causing urinary excretion of excess
glucose. The most common side effects of SGLT-2 inhibitors are orthostatic hypotension,
urinary tract infections, and vaginal yeast infections. Hypoglycemia can occur with any
antihyperglycemic agent, but is not a prominent side effect of SGLT-2s. Edema and weight
gain are common side effects of TZDs, while cough occurs in some patients who take ACE-
inhibitors. GI intolerance, including nausea and vomiting or diarrhea, is common with
metformin and GLP-1 receptor agonists.

Q12. You are caring for a patient with T2DM whose measures of control have been worsening
despite maximal doses of oral medications. You are considering adding insulin therapy to help
with her post-prandial hyperglycemia and want to start a preparation with a rapid onset of
action. Which of the following insulin types has the most rapid onset of action?

a.Aspart (NovoLog)
b.Regular
c.Neutral protamine Hagedorn (NPH)
d.Detemir (Levemir)
e.Glargine (Lantus)

The answer is a.
It is important to thoroughly understand the action of the different types of insulin
preparations in order to make therapeutic decisions about diabetic patients and their control.
Aspart, lispro (Humalog), and glulisine (Apidra) have the most rapid onset of action, between
15 and 30 minutes. Regular insulin has an onset between 30 and 60 minutes. NPH has an onset
between 1 and 2 hours, as does glargine (Lantus) and detemir (Levemir).

Q13. You are thinking about starting a patient with T2DM on insulin therapy to improve her
glucose control. You would like to provide her with steady insulin action without much of a
peak time. Which of the following insulin preparations provides the most stable insulin
coverage without a peak time of maximum activity?
a.Aspart (Novolog)
b.Lispro (Humalog)
c.Regular
d.NPH
e.Glargine (Lantus)

The answer is e.
It is important to thoroughly understand the action of the different types of insulin
preparations in order to make therapeutic decisions about diabetic patients and their control.
Lispro’s activity peaks early, between 30 and 90 minutes after injection. Aspart’s activity
peaks around 1 and 3 hours after injection. Regular insulin peaks between 2 and 4 hours after
injection. NPH peaks 5 to 7 hours after injection. Glargine (Lantus) and Detemir (Levemir) do
not have a predictable peak, and last for around 20 and 24 hours. Insulin degludec (Tresiba)
has an effective duration of 42 hours.
Q14. You have maximized oral anti-hyperglycemic therapy for a 56-year-old female patient
with T2DM, hypertension, dyslipidemia, and obesity. She works hard at diet and exercise, but
her hemoglobin A1C is still elevated at 9.6%. You decide to add insulin to her regimen. She is
currently 67 in tall and weighs 100 kg. You decide to start her on 10 units of glargine (Lantus)
at night in addition to her current regimen. Which of the following recommendations would
you make for this patient regarding self-monitoring of blood glucose (SMBG)?

a.She should test her blood glucose levels any time of day.
b.She should not check her blood glucose levels, as there is no evidence to suggest that it is
helpful for patients on only basal insulin.
c.She should check her blood glucose levels, but she doesn’t need to bring her glucose log with
her to appointments.
d.She should check blood glucose levels once a day, fasting, and receive education about how
to make adjustments to her meals and insulin based on these readings.
e.She should check her blood glucose levels fasting, after meals, and at bedtime.

The answer is d.
Patients with T2DM may require insulin therapy if diet, exercise, and oral hypoglycemic agent
do not provide appropriate control. A low initiating dose of insulin glargine is commonly
used, estimating 0.1 U/kg of body weight, as an addition to the current regimen. The dose can
be titrated over time to meet fasting blood glucose targets. The utility and frequency of SMBG
is dependent on the type of regimen the patient is prescribed and what they do with the
information. For T2DM patients on intensive basal-bolus insulin regimens, clinical trials show
testing up to 6 to 10 times/day is associated with lower A1C levels. For T2DM patients on
basal insulin and/or oral agents, the evidence is not as clear.
Generally, patients on basal insulin should check at least fasting blood glucose levels and use
those levels to make adjustments to their basal insulin dose; this is associated with lower A1Cs.
Several studies have not shown much benefit to SMBG in noninsulin treated patients.The
most important factors of whether SMBG lowers blood glucose are educating about and acting
on the information. Therefore, patients should receive education about how to manage their
blood glucose levels and clinicians should review and make recommendations on their
patients’ levels.
ABFM 2020
An obese 45-year-old female returns for follow-up to discuss recent laboratory results. She has
known hypertension and takes amlodipine (Norvasc), 5 mg daily. She has bilateral knee
osteoarthritis and uses ibuprofen as needed for pain. Recent laboratory studies show a fasting
glucose level of 120 mg/dL, normal electrolytes, and normal kidney and liver function. She
has a total cholesterol level of 203 mg/dL, an LDL- cholesterol level of 134 mg/dL, an HDL-
cholesterol level of 46 mg/dL, and a triglyceride level of 260 mg/dL. She has a strong family
history of diabetes mellitus and cardiovascular disease.
Which one of the following has been shown to be most effective in preventing diabetes in
patients such as this?
A. Participation in the National Diabetes Prevention Program
B. Acarbose (Precose)
C. Liraglutide (Victoza)
D. Metformin (Glucophage)
ANSWER: A
Patients who participate in long-term lifestyle intervention programs such as the CDC’s
National DiabetesPrevention Program have an approximately 30% reduction in progression to
type 2 diabetes. These programs promote weight loss of 7% of body weight and encourage
physical activity with a weekly goal of 150 minutes of moderate-intensity exercise. Because
an estimated 1 in 3 adults in the United States has prediabetes (diagnosed by a fasting glucose
level of 100–125 mg/dL, a hemoglobin A1c of 5.7%– 6.4%, or a 2-hour plasma glucose level
of 140–199 mg/dL), interventions to reduce progression to full diabetes are vital. Weight loss
of 5% of body weight alone is not as effective as a comprehensive prevention program.
Metformin is also effective in reducing the progression to diabetes, but it is not as effective as
lifestyle intervention programs for most patients. While -glucosidase inhibitors have been
shown to reduce the incidence of diabetes in patients with prediabetes, they are not as
effective as metformin, which can reduce the progression by 18%. There is good evidence that
liraglutide leads to weight loss and the lowering of blood glucose, but it has not been found to
be as effective as lifestyle intervention programs or metformin in reducing progression to
diabetes.
87. A 34-year-old female with type 2 diabetes that had previously been managed with
metformin (Glucophage) and liraglutide (Victoza) was switched to insulin for glycemic
management during her pregnancy. In the week before delivery she was achieving fasting and
postprandial glucose targets with an insulin regimen of insulin detemir (Levemir), 16 U daily,
and insulin lispro (Humalog), 4 U with meals. Her labor was induced at 37 weeks gestation
due to elevated blood pressures and she had an uncomplicated vaginal delivery. She is
planning to breastfeed.
Which one of the following glucose management regimens would you recommend for her in
the immediate postpartum period?

A. Continue the current dosages of insulin


B. Continue insulin detemir and stop insulin lispro
C. Continue insulin lispro and stop insulin detemir
D. Discontinue insulin and reinitiate metformin
E. Discontinue insulin and reinitiate liraglutide

ANSWER: D
The incidence of type 2 diabetes continues to rise in the United States, including in women of
childbearing age. Type 2 diabetes complicates 1%–2% of pregnancies in the United States.
Tight glycemic control (hemoglobin A1c <6.5%) is recommended during pregnancy,
especially in the preconception period and the first trimester if possible. Risks of uncontrolled
diabetes in pregnancy include preeclampsia, congenital defects, preterm delivery, macrosomia,
and stillbirth. Insulin is the safest medication for glycemic control in pregnant patients, with
requirements to achieve glycemic targets typically varying over the course of a pregnancy.
After an initial increase in insulin requirements through week 9 of pregnancy, they often
decline in weeks 9–16 before rising again in weeks 16–37. Beyond week 37 insulin
requirements may again start to decline. Immediately after delivery of the placenta women
become exquisitely sensitive to insulin, and insulin requirements may drop as low as 50% of
pre-pregnancy needs. Patients with type 2 diabetes who were not taking insulin prior to
pregnancy typically no longer require insulin.
Metformin is safe in breastfeeding women and should be restarted in this patient who did well
on it prior to pregnancy. No information is available regarding the safety of liraglutide while
breastfeeding. Until more data becomes available it should be used with caution and is not the
preferred agent in this patient

102. A 60-year-old male with a long-standing history of type 2 diabetes is admitted to the
hospital. He takes four oral medications for the treatment of diabetes at home. You decide to
switch him to insulin instead of continuing oral medications while he is hospitalized. He is
eating his meals well.

After calculating the total daily insulin dose, which one of the following would be most
appropriate?
A. Administer the total daily dose as long-acting insulin in equal doses every 12 hours
B. Administer half of the total daily dose of insulin as long-acting insulin and the other half as
short-acting insulin in three divided doses, given with each meal
C. Administer the total daily dose as short-acting insulin in three divided doses, given with
each meal
D. Administer the total daily dose as short-acting insulin in four divided doses, given with
each meal and at bedtime
E. Administer the total daily dose as a short-acting sliding scale regimen based on bedside
glucose readings, in four divided doses

ANSWER: B

Frequently patients taking oral medications for the treatment of diabetes mellitus need to be
switched to insulin while hospitalized. There are formulas to calculate the total daily dose
based on weight, renal function, insulin resistance, and other factors. The recommended
regimen is half of the calculated total daily dose given as long-acting insulin such as glargine
to provide basal insulin and half given as short-acting insulin such as lispro to provide prandial
insulin. The short-acting insulin is divided into thirds to be given with each meal.
The American Diabetes Association (ADA) recommends an insulin regimen with a basal and
a prandial component for non-critically ill patients in the hospital with good nutritional intake.
A correction component can be added to this regimen. The ADA strongly discourages the use
of only a sliding scale insulin regimen. The reactive nature of sliding scale does not control
glucose levels well and does not address the basal insulin needs of patients.

155. A16-year-old Muslim male with type 1 diabetes would like to start fasting during
Ramadan. He currently takes insulin glargine (Lantus) every morning and regular insulin with
meals. Which one of the following is the most appropriate recommendation for this patient
during Ramadan?
A. Avoid fasting because it is unsafe in patients with type 1 diabetes
B. Decrease insulin glargine by 50%
C. Decrease insulin glargine by 50% and take it at night
D. Decrease insulin glargine by 25% and use sliding scale regular insulin with meals
E. Stop insulin glargine and use sliding scale regular insulin four times a day.

ANSWER: A
Ramadan is a holy month during which Muslims fast from dawn until sunset. It is an
obligation for all healthy adult Muslims. The Quran does exempt the sick from fasting, but
many ill people will still fast. Recommendations in the literature include risk stratification
criteria to determine whether fasting is safe for patients with diabetes mellitus. Generally,
patients with well controlled type 2 diabetes can safely fast with adjustments in their
medications; however, it is thought that fasting is unsafe for patients with type 1 diabetes.
Fasting can increase the risk of severe hypoglycemia as well as hyperglycemia and
ketoacidosis if medications are withheld.

Which one of the following U-100 insulin products has the longest duration of action?
A. Degludec (Tresiba)
B. Glargine (Lantus)
C. Isophane NPH (Humulin N)
D. Lispro (Humalog)
E. Regular (Humulin R)
ANSWER: A
Among the available U-100 insulin products, the one with the longest duration of action is
ultralong-acting degludec, which lasts 42 hours. The duration of action of rapid-acting lispro is
3–6.5 hours, short-acting regular is 5–8 hours, intermediate-acting isophane is 12–16 hours,
and long-acting glargine is 11–24 hours.

-A 55-year-old male with a BMI of 32 kg/m2 presents to your office to discuss weight
management. He has moderately well controlled type 2 diabetes and hypertension. He prefers
not to modify his diet and would like to know if he can expect significant weight loss from
exercising. He plans to walk briskly for 45 minutes daily.
Which one of the following would be the best advice for this patient?
A. Moderate exercise alone is ineffective for weight loss
B. Moderate exercise alone is superior to dietary changes for weight loss
C. Moderate exercise alone is moderately beneficial for weight loss
D. Adding moderate exercise to dietary changes substantially increases weight loss

ANSWER: C
Exercise alone does have some substantial benefits, including improved insulin and glycemic
control in diabetes, a beneficial effect on blood pressure, a reduction of cardiovascular risks,
and a maintenance of weight loss. However, it is only moderately beneficial for promoting
weight loss, including when exercise is added to diet changes.
ABFM2019
Q1-A 26-year-old G2P1001 at 30 weeks gestation was recently diagnosed with gestational
diabetes and is ready to start testing her blood glucose at home.
Which one of the following is the recommended goal for fasting blood glucose in this patient?
A. <75 mg/dL
B. <95 mg/dL
C. <120 mg/dL
D. <150 mg/dL
E. <180 mg/dL

ANSWER: B
The goal fasting blood glucose level in patients with gestational diabetes is <95 mg/dL. A
fasting glucose level <80 mg/dL is associated with increased maternal and fetal complications.
The goal 2-hour postprandial glucose level is <120 mg/dL and the goal 1-hour postprandial
glucose level is <140 mg/dL.

Q2-year-old white female with a history of mania sees you because of polyuria and increased
thirst over the past month. She has taken lithium, 1800 mg daily, for 3 years and her mania is
well controlled. She has not lost weight and there is no family history of her current problem.
There are no orthostatic blood pressure changes.
Laboratory Findings
Serum sodium 145 mEq/L (N 135–145)
Serum potassium 4.5 mEq/L (N 3.5–5.0) Serum glucose 92 mg/dL
Serum creatinine 0.9 mg/dL (N 0.6–1.5)
Serum lithium 1.38 mEq/L (therapeutic range 0.5–1.5) Urine volume 6.85 L/24 hr
Urine osmolality 161 mOsm/kg H2O
There is no significant change in urine osmolality in response to the administration of
vasopressin. Which one of the following is the most likely cause of this patient’s problem?
A.Drug-induced nephrogenic diabetes insipidus
B.Borderline diabetes mellitus
C.Panhypopituitarism
D.Psychogenic water drinking
ANSWER: A
Polyuria occurs in 20%–70% of patients on long-term lithium therapy, even when plasma
lithium levels are in the therapeutic range. This is a result of impaired renal concentrating
ability that is resistant to vasopressin (nephrogenic diabetes insipidus). Inappropriate
antidiuretic hormone secretion causes hyponatremia and fluid retention. The diuresis
associated with diabetes mellitus is a result of the osmotic effect of increased serum glucose,
which is not present in this case.
Patients with hypothalamic or pituitary injuries may develop central diabetes insipidus, which
responds to exogenous vasopressin. Psychogenic water drinking occurs in psychiatric patients,
but would not be expected to cause impairment of renal concentration or hypernatremia .

Q3-A 67-year-old male who recently had an ST-elevation myocardial infarction (STEMI) and
underwent placement of a circumflex artery stent presents for hospital follow-up. His past
medical history also includes hypertension, obesity, stage 3a chronic kidney disease, and
coronary artery disease. He is feeling well today but his cardiologist told him he probably has
diabetes mellitus and should see you to discuss treatment options. The cardiologist told the
patient that there are new medications for diabetes that will help lower his risk for future
myocardial infarctions and he is interested in starting one of these .
A physical examination is unremarkable. The patient weighs 145 kg (320 lb) and his BMI is
44 kg/m2. His blood pressure is 132/78 mm Hg and his heart rate is 65 beats/min . Laboratory
studies from the hospital include the following :
Hemoglobin A1c 8.2%
TSH normal
Sodium 141 mEq/L (N 136–142)
Potassium 4.5 mEq/L (N 3.5–5.0)
Chloride 100 mEq/L (N 98–107)
Bicarbonate 30 mEq/L (N 22–29)
BUN 26 mg/dL (N 8–23)
Creatinine 1.34 mg/dL (N 0.6–1.2)
Glomerular filtration rate 51 mL/min/1.73 m2

Based on American Diabetes Association guidelines, in addition to lifestyle modifications,


which one of the following would be most appropriate at this time?
A. No pharmacologic therapy
B. Empagliflozin (Jardiance)
C. Insulin glargine (Lantus)
D. Liraglutide (Victoza)
E. Metformin (Glucophage)
ANSWER: E
With the advent of new medications to treat diabetes mellitus, including many medications
that help lower cardiovascular risk, it is tempting to begin treatment with these medications.
Unless there are contraindications, however, metformin is still the initial medication of choice
for patients with newly diagnosed diabetes. If this patient’s hemoglobin A1c were 10%, it
would be reasonable to start insulin therapy. If it becomes necessary to add an additional
antidiabetic agent it would be reasonable to consider a GLP-1 receptor agonist or a DPP-4
inhibitor in addition to metformin at that time, given the patient’s history of coronary artery
disease.

Q4-A 45-year-old African-American male returns to your clinic to evaluate his progress after
6 months of dedicated adherence to a diet and exercise plan you prescribed to manage his
blood pressure. His blood pressure today is 148/96 mm Hg. He is not overweight and he does
not have other known medical conditions or drug allergies. Which one of the following would
be the most appropriate initial antihypertensive treatment option for this patient ?

A) Chlorthalidone
B) Hydralazine
C) Lisinopril (Prinivil, Zestril )
D) Losartan (Cozaar )
E) Metoprolol
ANSWER: A
Lifestyle modifications addressing diet, physical activity, and weight are important in the
treatment of hypertension, particularly for African-American and Hispanic patients. When
antihypertensive drugs are also required, the best options may vary according to the racial and
ethnic background of the patient. The presence or absence of comorbid conditions is also
important to consider. For African- Americans, thiazide diuretics and calcium channel
blockers, both as monotherapy and as a component in multidrug regimens, have been shown
to be more effective in lowering blood pressure than ACE inhibitors, angiotensin II receptor
blockers, or -blockers, and should be considered as first-line options over the other classes of
antihypertensive drugs unless a comorbid condition is present that would be better addressed
with a different class of drugs. Racial or ethnic background should not be the basis for the
exclusion of any drug class when multidrug regimens are required to reach treatment goals.
Q5-A 69-year-old male with type 2 diabetes mellitus, obesity, and a history of coronary artery
disease sees you for follow-up of his diabetes. His hemoglobin A1c has increased to 8.7%
despite therapy with metformin (Glucophage), 1000 mg twice daily, and insulin glargine
(Lantus). Which one of the following additional medications would be most effective for
reducing his blood glucose level and lowering his risk of cardiovascular events ?
A) Exenatide (Byetta )
B) Glipizide (Glucotrol )
C) Liraglutide (Victoza )
D) Rosiglitazone (Avandia)
E) Sitagliptin (Januvia )

ANSWER: C
Liraglutide, exenatide, and dulaglutide are all GLP-1 receptor agonists. Of these, only
liraglutide has been shown to lower the risk of recurrent cardiovascular events and has
received FDA approval for this indication. Glipizide (a sulfonylurea), rosiglitazone, and
sitagliptin have not been associated with improved cardiovascular outcomes. Empagliflozin,
an SGLT2 inhibitor, has also been associated with secondary prevention of cardiovascular
disease.

Q6-A 42-year-old male with hypertension and hyperlipidemia sees you for a routine health
maintenance examination. His blood pressure is 185/105 mm Hg. He does not have any
current symptoms, including headache, chest pain, edema, or shortness of breath. He is
adherent to his current medication regimen, which includes lisinopril (Prinivil, Zestril), 10 mg
daily, and simvastatin (Zocor), 20 mg at night. A thorough history and physical examination
are both unremarkable. Which one of the following would be the most appropriate nextstep

A) A 30-minute rest period followed by a repeat blood pressure reading


B) Clonidine (Catapres), 0.2 mg given in the office
C) A comprehensive metabolic panel, fasting lipid profile, and TSH level
D) A stress test
E) Hospital admission for blood pressure reduction
ANSWER: A
The first step in the management of severe hypertension is determining whether a hypertensive
emergency is present. A thorough history and physical examination are crucial (SOR C).
Severe hypertension (blood pressure >180 mm Hg systolic or >110 mm Hg diastolic) with
end-organ damage constitutes a hypertensive emergency. A physical examination should
center on evaluating for papilledema, neurologic deficits, respiratory compromise, and chest
pain. If end- organ damage is present the patient should be hospitalized for monitored blood
pressure reduction and further diagnostic workup. If end-organ damage is not present and the
physical examination is otherwise normal, a 30-minute rest with reevaluation is indicated.
Approximately 30% of patients will improve to an acceptable blood pressure without
treatment (SOR C). Home medications should then be adjusted with outpatient follow-up and
home blood pressure monitoring (SOR A). Short-acting antihypertensives are indicated if mild
symptoms are noted such as headache, lightheadedness, nausea, shortness of breath,
palpitations, anxiety, or epistaxis. Diagnostic testing is not immediately indicated for
asymptomatic patients (SOR C). A basic metabolic panel or other testing should be considered
if mild symptoms are present. Aggressive lowering of blood pressure can be detrimental and a
gradual reduction over days to weeks is preferred (SOR C).

Q7-A 72-year-old male with type 2 diabetes mellitus sees you for routine follow- up. He takes
metformin (Glucophage), 1000 mg twice daily. He is sedentary and does not adhere to his diet.
His BMI is 32.0 kg/m2. The examination is otherwise within normal limits. His hemoglobin
A1c is 9.5%. ?Which one of the following is recommended by the American Diabetes
Association to better control his blood glucose

A) Start an intensive diet and exercise program for weight loss


B) Start home monitoring of blood glucose with close follow-up
C) Start basal insulin at 10 units/day
D) Stop metformin and start a sulfonylurea
E) Stop metformin and start a basal and bolus insulin regimen
ANSWER: C
According to the American Diabetes Association's 2018 guidelines for the management of
diabetes, a healthy person with a reasonable life expectancy should have a hemoglobin A1c
goal of <7%. Metformin is recommended as first- line therapy as long as there are no
contraindications. If the hemoglobin A1c is not at the goal or is ?9%, then adding another
agent to metformin is recommended. Basal insulin at 10 units/day is an acceptable choice for
additional therapy to improve blood glucose control. Diet, exercise, and home monitoring of
blood glucose are recommended in addition to starting another agent for blood glucose
control.

Q8-Which one of the following antihypertensive drugs may reduce the severity of sleep apnea
A) Amlodipine (Norvasc )
B) Hydralazine
C) Lisinopril (Prinivil, Zestril )
D) Metoprolol
E) Spironolactone (Aldactone)

ANSWER: E
Diuretics lessen the severity of obstructive sleep apnea and reduce blood pressure.
Aldosterone antagonists offer further benefit beyond that of traditional diuretics. Resistant
hypertension is common in patients with obstructive sleep apnea. Resistant hypertension is
also associated with higher levels of aldosterone, which can lead to secondary pharyngeal
edema, increasing upper airway obstruction.

Q9-A 61-year-old white male with type 2 diabetes mellitus sees you for a follow- up visit. His
blood pressure is 156/94 mm Hg. At a visit 1 week ago his blood pressure was 150/92 mm Hg.
Laboratory studies obtained prior to this visit show a BUN of 16 mg/dL (N 6–20), a serum
creatinine level of 0.9 mg/dL (N 0.7–1.3), and microalbuminuria on a urinalysis. His diabetes
is well controlled with metformin (Glucophage) and he is taking aspirin.
Which one of the following would you recommend?
A) Observation only
B) An ACE inhibitor
C) A -blocker
D) A calcium channel blocker
E) E) A diuretic
ANSWER: B
The panel members of the Eighth Joint National Committee for the management of blood
pressure recommended that ACE inhibitors should be initiated for renal protection in adults
with diabetes mellitus, hypertension, and microalbuminuria. This patient appears to be in an
early stage of nephropathy, and ACE inhibitors will reduce the decline in renal function. -
Blockers are no longer recommended for first-line treatment. In white patients who do not
have diabetes, therapy may be started with ACE inhibitors, thiazide diuretics, or calcium
channel blockers.

Q11-The U.S. Preventive Services Task Force recommends routine screening for gestational
diabetes mellitus no sooner than
A) 16 weeks gestation
B) 20 weeks gestation
C) 24 weeks gestation
D) 32 weeks gestation

ANSWER: C
The U.S. Preventive Services Task Force recommends screening for gestational diabetes
mellitus after 24 weeks gestation with a fasting blood glucose level, a 50- g oral glucose
challenge, or an assessment of risk factors (A recommendation).
Screening at an earlier date receives a rating of insufficient evidence, and screening at later
dates is not recommended (SOR C).

Q12-Which one of the following diabetes mellitus medications is MOST likely to cause weight
gain ?
A) Empagliflozin (Jardiance)
B) Glimepiride (Amaryl (
C) Liraglutide (Victoza (
D) Metformin (Glucophage)
E) Sitagliptin (Januvia(
ANSWER: B
Since many patients with diabetes mellitus are obese, the impact of medications on the
patient’s weight is important to consider. Treatment with sulfonylureas, including glimepiride,
is associated with weight gain .
Empagliflozin, liraglutide, metformin, and sitagliptin are not associated with weight gain. In
particular,the SGLT2 inhibitors such as empagliflozin and the GLP1 agonists such as
liraglutide are associated with clinically significant weight loss.
Q13-A 48-year-old female with type 2 diabetes mellitus has been unable to achieve optimal
glycemic control with lifestyle modifications alone. You recommend that she start medication.
Which one of the following medications is generally recommended as the first-line medication
for initiating treatment for type 2 diabetes mellitus ?
A) Alogliptin (Nesina )
B) Empagliflozin (Jardiance )
C) Glipizide (Glucotrol )
D) Metformin (Glucophage )
E) Pioglitazone (Actos )

ANSWER: D
Metformin should be the first medication prescribed for diabetes mellitus when an oral agent
is required (SOR A). Metformin can efficiently lower glycemic levels and is linked to weight
loss and fewer occurrences of hypoglycemia. It is also less expensive than most other options.
If more than one agent is required, continuing metformin is recommended along with the
addition of one or more of the following :a sulfonylurea such as glipizide, a thiazolidinedione
such as pioglitazone, an SGLT2 inhibitor such as empagliflozin, or a DPP-4 inhibitor such as
alogliptin.

Q14-A 62-year-old white male with a 3-month history of diabetes mellitus has a hemoglobin
A1c of 7.8%. Which one of the following is the best parameter for determining if he can safely
take metformin (Glucophage) ?
A) 24-hour urine for creatinine clearance
B) BUN/creatinine ratio
C) Estimated glomerular filtration rate
D) Serum creatinine
E) Urine microalbumin

ANSWER: C
Until recently metformin was contraindicated for patients with renal dysfunction suggested by
a creatinine level of 1.5 mg/dL for men and 1.4 mg/dL for women. However, available
evidence now supports the use of metformin in individuals with mild to moderate chronic
renal disease, defined by the estimated glomerular filtration rate (eGFR). Patients with an
eGFR between 45 and 60 mL/min/1.73 m2 (chronic
mild kidney disease) are now permitted to take metformin. Metformin should not be used in
patients with an eGFR <45 mL/min/1.73 m2 (moderate kidney disease), as lactic acidosis is
more likely to occur. The eGFR is used instead of the serum creatinine level because the
equation includes age, sex, race, and other parameters.
Q15-A 54-year-old male comes to your office to establish care. He has a past history of
hypertension treated with lisinopril (Prinivil, Zestril) and hydrochlorothiazide but has not
taken his medications for over a year. He does not have any symptoms, including chest pain,
shortness of breath, or headache. On examination his blood pressure is 200/115 mm Hg on two
separate readings taken 5 minutes apart. The remainder of the physical examination is normal.
Which one of the following management options would be most appropriate ?

A)Institute out-of-office monitoring with an ambulatory device and follow up in 2 weeks


B)Restart the patient’s previous antihypertensive medications and follow upwithin 1 week
C)Administer a short-acting antihypertensive medication in the office to lower his blood
pressure to <160/100 mm Hg
D)Hospitalize for hypertensive emergency

ANSWER: B
This patient has severe asymptomatic hypertension (systolic blood pressure 180 mm Hg or
diastolic blood pressure 110 mm Hg). If there were signs or symptoms of acute target organ
injury, such as neurologic deficits, altered mental status, chest pain, shortness of breath, or
oliguria, hospitalization for a hypertensive emergency would be indicated. Because this patient
was asymptomatic and has a known history of hypertension, restarting his prior
antihypertensive regimen and following up in 2 weeks would be the most appropriate
management option. If he had no past history of hypertension it would be reasonable to
consider out-of- office monitoring with an ambulatory device for 2 weeks before initiating
treatment. In the absence of acute target organ injury, blood pressure should be gradually
lowered to less than 160/100 mmHg over several days to weeks.
Aggressively lowering blood pressure can lead to adverse events such as myocardial
infarction, cerebrovascular accident, or syncope, so administering a short-acting
antihypertensive medication in the office should be reserved for the management of
hypertensive emergencies.
Q16-A 73-year-old female is brought to your office by her daughter, who is concerned that
there may be some memory changes in her mother. She has noticed that her mother frequently
repeats herself and has made several medication errors lately .
The patient has type 2 diabetes mellitus, hypertension, depression, and hypothyroidism. Her
current medications include glyburide (DiaBeta), aspirin, lisinopril (Prinivil, Zestril) ,
hydrochlorothiazide, atorvastatin (Lipitor), and sertraline (Zoloft) . A physical examination
reveals a blood pressure of 136/72 mm Hg. She scores 26/30 on a Saint Louis University
Mental Status (SLUMS) examination, which suggests mild cognitive impairment. A cardiac
examination and a foot examination are normal .
Laboratory studies reveal a hemoglobin A1c of 7.0% and a TSH level of 3.8 U/mL (N 0.4
.)4.2–A basic metabolic panel is normal and her glucose level is 93 mg/dL. A CBC is normal .
Which one of the following medications should be stopped in this patient ?
A) Atorvastatin
B) Glyburide
C) Hydrochlorothiazide
D) Lisinopril
E) Sertraline

ANSWER: B
This patient has signs of mild neurocognitive impairment. In this case one possible contributor
to this condition is hypoglycemia. While it is unknown whether minor hypoglycemic events
can contribute to dementia, major events have been associated with a greater risk of dementia.
The sulfonylurea glyburide carries a risk of significant hypoglycemia, especially in elderly
patients. Her hemoglobin A1c of 6.1% correlates with an estimated average glucose of 128
mg/dL, corroborating this concern. Glyburide in particular is listed on the Beers Criteria
because of its potential to cause prolonged hypoglycemia.
Q17-A 38-year-old female presents for ongoing management of type 2 diabetes mellitus,
obesity, and chronic abdominal pain related to her history of recurrent pancreatitis. She says
that her self-monitored blood glucose has been running in the range of 200–300 mg/dL on
most occasions. She is not currently taking any medications but has tried metformin
(Glucophage) and extended-release metformin (Glucophage XR) unsuccessfully in the past.
On both occasions she experienced worsening abdominal pain and diarrhea. She does not feel
she can manage insulin and requests an oral medication. Her hemoglobin A1c in your office
today is 9.0%. In addition to lifestyle and nutrition counseling, which one of the following
would be the best treatment at this time ?
A) Restart metformin
B) Start empagliflozin (Jardiance )
C) Start liraglutide (Victoza )
D) Start sitagliptin (Januvia )

ANSWER: B
Metformin should be used as first-line therapy in type 2 diabetes to reduce microvascular
complications, assist in weight management, reduce the risk of cardiovascular events, and
reduce the risk of mortality in patients (SOR A).
Patients who are intolerant of metformin are unlikely to be successful with a third trial of that
agent. Empagliflozin, an SGLT2 inhibitor, is considered a second-line choice for patients who
are intolerant of metformin. Both sitagliptin, a DPP-4 inhibitor, and liraglutide, a GLP-1
receptor agonist, should be avoided or used with caution in patients with a history of
pancreatitis.

Q18-Which one of the following comorbidities would falsely lower the hemoglobin A1c level
in a patient with type 2 diabetes mellitus ?
A) Vitamin B12 deficiency
B) Iron deficiency anemia
C) Hemolytic anemia
D) Chronic kidney disease
E) A history of splenectomy
ANSWER: C
Several factors can alter the hemoglobin A1c value, including variability and erythrocyte
lifespan. When the mean erythrocyte lifespan is increased by a condition such as asplenia,
hemoglobin A1c increases because of increased RBC exposure time for glycation. Conversely,
when the mean erythrocyte lifespan is decreased by conditions such as hemolytic anemia,
hemoglobin A1c is decreased because of reduced RBC exposure time for glycation. Conditions
that decrease erythropoiesis, such as iron deficiency anemia, increase the mean age of the
RBC, thereby increasing hemoglobin A1c. Severe chronic kidney disease may increase RBC
glycation through lipid peroxidase of hemoglobin and by extending the erythrocyte lifespan
due to decreased erythropoietin levels, causing a false elevation of hemoglobin A1c. Vitamin
B12 deficiency also decreases erythropoiesis and leads to falsely elevated hemoglobin A1c.

Q19-A 38-year-old female with a 5-year history of diabetes mellitus has


developed a “pins and needles” sensation in her feet. Which one of the following is considered
first-line therapy for her condition ?
A) Acupuncture
B) Lidocaine 5% spray
C) Oxycodone (Roxicodone )
D) Pregabalin (Lyrica )
E) Venlafaxine (Effexor XR)

ANSWER: D
Pregabalin is considered first-line therapy for painful diabetic peripheral neuropathy (SOR A).
Based on a meta-analysis, the American Academy of Neurology recommends pregabalin as
first-line medication and gabapentin as a first-line alternative. While opioids such as
oxycodone may provide a possible benefit in the treatment of neuropathy, the risk of
dependency and adverse effects limits their use to patients with pain not relieved by first-line
therapies. Acupuncture is not recommended as a first-line therapy due to the lack of high-
quality, randomized, controlled trials. Venlafaxine and lidocaine 5% spray are considered
second-line therapies.
Q20-A 57-year-old female is admitted to the hospital with lower lobe pneumonia. She has no
history of diabetes mellitus. She has not met sepsis criteria but had a blood glucose level of
172 mg/dL in the emergency department .
Insulin should be started if this patient has a persistent blood glucose level greater than or equal
to
A) 120 mg/dL
B) 140 mg/dL
C) 160 mg/dL
D) 180 mg/dL

ANSWER: D
Insulin therapy should be initiated in hospitalized patients with persistent hyperglycemia,
starting at a threshold of 180 mg/dL. Once insulin therapy is started, a target glucose range of
140–180 mg/dL is recommended for the majority of hospitalized patients, regardless of
whether they have a critical illness.

Q21-A 50-year-old female sees you for follow-up of her hypertension. At her last visit 4 weeks
ago you started her on lisinopril (Prinivil, Zestril), 10 mg daily, because of a blood pressure of
158/92 mm Hg and confirmed hypertension on ambulatory blood pressure monitoring. She is
tolerating the medication well and has no side effects. She does not take any other medications.
Today her blood pressure is 149/90 mm Hg, which you confirm on repeat measurement. This
is also consistent with her home measurements. At her last visit a basic metabolic panel was
normal .
You repeat a basic metabolic panel today and the results are normal except for a BUN of 25
mg/dL (N 8–23) and a creatinine level of 1.5 mg/dL (N 0.6–1.1). At her last visit her BUN was
12 mg/dL and her creatinine level was 0.7 mg/dL .
Which one of the following would be most appropriate at this time ?
A)Continue her current treatment regimen
B)Increase lisinopril to 20 mg daily
C)Continue lisinopril at the current dosage and add amlodipine (Norvasc), 5 mg daily
D)Discontinue lisinopril and begin amlodipine, 5 mg daily
E)Discontinue lisinopril and begin losartan (Cozaar), 25 mg daily
ANSWER: D
This patient has essential hypertension and her goal blood pressure is <140/90 mm Hg based
on JNC 8 guidelines, or 130/80 mm Hg based on the more recent recommendations of the
American College of Cardiology/American Heart Association Task Force on Clinical Practice
Guidelines. Until recently, it was recommended that physicians should tolerate a rise of <30%
in serum creatinine after ACE inhibitor or angiotensin receptor blocker (ARB) initiation. Rises
in serum creatinine of >30% from baseline increase the risk of renal failure, adverse cardiac
outcomes, and death. A recent study suggests that rises in serum creatinine of <30% also put
patients at risk for these outcomes, with a dose-response relationship between the magnitude
of creatinine change and the risk of adverse outcomes. This patient has more than a 30% rise
in creatinine and has no other factors, such as diabetes mellitus, heart failure, or chronic
kidney disease, that would indicate a need for ACE or ARB therapy for her hypertension.
Discontinuing her ACE inhibitor and starting a medication from a different class is the most
appropriate treatment at this time. Based on JNC 8 guidelines, additional options for blood
pressure medications include thiazide diuretics and calcium channel blockers.

Q22-A 14-year-old female is brought to your office for an annual well child check and sports
preparticipation physical examination. She says she does a lot of running during basketball
practices and games but has trouble controlling her weight. Most of her family is overweight.
She does not have any difficulty participating in sports, and has no symptoms such as chest
pain, shortness of breath, or headaches. She has no significant past medical history . On
examination the patient’s height is 154 cm (61 in) and she weighs 63 kg (139 lb). Her BMI is
26.4 kg/m2, which places her in the 90th percentile for her age. Her blood pressure is 130/85
mm Hg, which places her between the 95th and 99th percentile for her age, height, and sex.
Her chart reveals that her blood pressure was at this level at the last two visits. The physical
examination is otherwise normal .In addition to counseling and support for weight loss, which
one of the following would be most appropriate at this point ?

A)Informing the patient and her parents that she is prehypertensive and having her return for a
blood pressure check in 3 months
B)Plasma renin and catecholamine levels
C)An imaging study of the renal arteries
D)A fasting basic metabolic panel, a lipid profile, and a urinalysis
E)Antihypertensive drug therapy
ANSWER: D
In a pediatric patient, blood pressure should be evaluated using comparisons based on age,
sex, and height. Although this adolescent’s blood pressure is prehypertensive for an adult
according to JNC 8 guidelines, it is stage 1 hypertension (between 95% and 99%) for her age,
sex, and height. All pediatric patients with confirmed hypertension should have further
evaluation to check for renal dysfunction as well as other cardiac risk factors. Additionally,
renal ultrasonography is recommended to evaluate for renal disease and echocardiography to
evaluate for end-organ damage that would affect treatment goals. Additional studies, such as
plasma renin and catecholamine levels or renovascular imaging, may be indicated in children
with abnormalities on initial evaluation that suggest secondary causes of hypertension .
Pharmacologic therapy is usually recommended for pediatric patients with symptomatic
hypertension, secondary hypertension, target organ damage, diabetes mellitus, or persistent
hypertension in spite of nonpharmacologic treatment. A low-sodium diet may be helpful for
decreasing blood pressure, and given this patient’s obesity, intensive counseling about lifestyle
changes is appropriate.

ABFM2017
Q23-A 75-year-old male is admitted to the intensive-care unit with sepsis. His past medical
history is significant for diabetes mellitus and coronary artery disease.
Which one of the following would be the most appropriate maximum bloodglucose goal?
A) 100 mg/dL
B) 140 mg/dL
C) 180 mg/dL
D) 220 mg/dL
ANSWER: C
Sepsis is a severe life-threatening disorder that has a 25%–30% mortality rate. Early
aggressive management has been shown to decrease the mortality rate. The initial step in the
management of sepsis is respiratory stabilization. Fluid resuscitation should be started and
followed by vasopressor therapy if there is an inadequate blood pressure response. Antibiotics
should be initiated within 1 hour of presentation. Other interventions in early goal-directed
therapy that have been shown to improve mortality rates include blood transfusions, low-dose
corticosteroid therapy, and conventional (not intensive) glycemic control with a target glucose
level of <180 mg/dL. Intensive management of glucose in critically ill adult patients (a target
glucose level of 80–110 mg/dL) has been shown to increase mortality.-
Q24-GLP-1 agonists such as exenatide (Byetta) can be used as second-line agents to help
improve glycemic control in patients with type 2 diabetes mellitus. Which one of the following
is a CONTRAINDICATION to their use?
A) Hypothyroidism
B) Thyroid cancer
C) Coronary artery disease
D) Heart failure

ANSWER: B
GLP-1 agonists are contraindicated in patients with medullary thyroid cancer or multiple
endocrine neoplasm syndrome, or with a family history of these conditions. They are not
associated with heart failure, coronary artery disease, or hypothyroidism. They have been
associated with pancreatitis in rare cases, but this is not a contraindication to prescribing them.

Q25-A 50-year-old male sees you for a health maintenance visit. He has a 20-year history of
smoking 1 pack of cigarettes per day but his history is otherwise unremarkable. He is not
aware of any disease that runs in his family, including diabetes mellitus. He has not visited a
physician for the past 5 years. A physical examination reveals a BMI of 28.2 kg/m2, normal
blood pressure, and no other significant findings. Laboratory testing reveals a fasting plasma
glucose level of 107 mg/dL. According to the U.S. Preventive Services Task Force, which one
of thefollowing would be most appropriate at this point?
A) No further diabetes screening unless his BMI increases to equal or more 30kg/m2
B) A repeat fasting plasma glucose level in 1–2 weeks
C) A repeat fasting plasma glucose level in 1 year
D) A repeat fasting plasma glucose level in 3 years
E) Treatment with metformin (Glucophage) 74
ANSWER: B
The U.S. Preventive Services Task Force (USPSTF) recommends screening for abnormal
blood glucose levels as part of a cardiovascular disease risk assessment for adults who are 40–
70 years of age and who are overweight (BMI 25.0–29.9 kg/m2) or obese (BMI equal or
above 30.0 kg/m2). Since his fasting blood glucose result was in the range consistent with
impaired fasting glucose (100–125 mg/dL), the USPSTF recommends confirming the
diagnosis of the abnormal glucose level soon by performing the same test on a different day.
Appropriate treatment should begin once the diagnosis is confirmed.

Q26-Which one of the following is diagnostic for type 2 diabetes mellitus?


A) A fasting plasma glucose level ≥126 mg/dL on two separate occasions
B) An oral glucose tolerance test (75-g load) with a 2-hour glucose level ≥160 mg/dL
C) A random blood glucose level ≥200 mg/dL in an asymptomatic person
D) A hemoglobin A1c ≥6.0% on two separate occasions

ANSWER: A
The American Diabetes Association recommends screening for all asymptomatic adults with a
BMI >25.0 kg/m2 who have one or more additional risk factors for diabetes mellitus, and
screening for all adults with no risk factors every 3 years beginning at age 45. Current criteria
for the diagnosis of diabetes mellitus include a hemoglobin A1c ≥6.5%, a fasting plasma
glucose level ≥126 mg/dL, a 2-hour plasma glucose level ≥200 mg/dL, or, in a symptomatic
patient, a random blood glucose level ≥200 mg/dL. In the absence of unequivocal
hyperglycemia, results require confirmation by repeat testing.

Q27- A 70-year-old male is being treated with medication for type 2 diabetes mellitus. Which
one of the following hemoglobin A1c values is associated with the lowest mortality in this
situation?
A) 5.0%–5.9%
B) 6.0%–6.9%
C) 7.0%–7.9%
D) 9.0%–9.9%
Answer : C

values of 7%–8% have shown the greatest reduction in mortality in multiple studies. It is
suggested that frequent hypoglycemia is associated with lower hemoglobin A1c values, and
that presents a greater risk. Values over 9% are associated with greater mortality (SOR B).
Thus, while the risk of complications increases linearly with hemoglobin A1c, mortality has a
V-shaped curve.

Q28-A 36-year-old female sees you for a 6-week postpartum visit. Her pregnancy was
complicated by gestational diabetes mellitus. Her BMI at this visit is 33.0 kg/m2 and she has a
family history of diabetes. Which one of the following is this patient’s greatest risk factor for
developing type 2 diabetes in the future?
A) Her age
B) Obesity
C) The history of a completed pregnancy
D) The history of gestational diabetes
E) The family history of diabetes

ANSWER: D
A history of gestational diabetes mellitus (GDM) is the greatest risk factor for future
development of diabetes mellitus. It is thought that GDM unmasks an underlying propensity to
diabetes. While a healthy pregnancy is a diabetogenic state, it is not thought to lead to future
diabetes. This patient’s age is not a risk factor. Obesity and family history are risk factors for
the development of diabetes, but having GDM leads to a fourfold greater risk of developing
diabetes, independent of other risk factors (SOR C). It is thought that 5%–10% of women who
have GDM will be diagnosed with type 2 diabetes within 6 months of delivery. About 50% of
women with a history of GDM will develop type 2 diabetes within 10 years of the affected
pregnancy.
Q29-You recently diagnosed diabetes mellitus in a 49-year-old male who also has chronic
kidney disease and New York Heart Association class III heart failure.
Laboratory studies are remarkable for a serum creatinine level of 2.0 mg/dL (N 0.6–1.2) and an
estimated glomerular filtration rate of 40 mL/min/1.73 m2.
Which one of the following classes of agents would be most appropriate for this patient?
A) A biguanide such as metformin (Glucophage)
B) A GLP-1 agonist such as liraglutide (Victoza)
C) An SGLT2 inhibitor such as canagliflozin (Invokana)
D) A thiazolidinedione such as rosiglitazone (Avandia)

ANSWER: B
Of the options given, only a GLP-1 agonist such as liraglutide could be used for this patient
because of his comorbidities of chronic kidney disease and heart failure. Metformin is
contraindicated in males with a creatinine level >1.5 mg/dL and in females with a creatinine
level >1.4 mg/dL. SGLT2 inhibitors are not as safe or effective if the patient’s estimated
glomerular filtration rate (eGFR) is <50 mL/min/1.73m2, and it is not recommended in
patients with an eGFR <30 mL/min/1.73 m2. The initiation of rosiglitazone is contraindicated
in patients with established New York Heart Association class III or classIV heart failure.

ABFP2015

Q30-A 43-year-old female smoker has type 2 diabetes mellitus, morbid obesity, and a recent
diagnosis of symptomatic peripheral arterial disease. You have started her on atorvastatin
(Lipitor), offered a supervised exercise program, and discussed smoking cessation and
interventions.
Which one of the following should be recommended to prevent cardiovascular events in this
patient?
A) Aspirin
B) Cilostazol (Pletal)
C) Enoxaparin (Lovenox)
D) Pentoxifylline
E) Warfarin (Coumadin)
ANSWER: A
Patients with symptomatic peripheral arterial disease should be started on a daily dose of either
aspirin or clopidogrel to prevent cardiovascular events such as acute myocardial infarction or
stroke (SOR B). Cilostazol is a phosphodiesterase inhibitor with both antiplatelet and arterial
vasodilatory activity. It has been shown to improve claudication symptoms by 50% compared
to placebo. Likewise, pentoxifylline is also used in the treatment of claudication symptoms but
is less effective than cilostazol and is reserved as a second-line agent. Neither agent has been
shown to decrease cardiovascular events in patients with symptomatic peripheral artery
disease. Neither enoxaparin nor warfarin is indicated for symptomatic peripheral artery
disease

Q31-A 65-year-old male presents to an urgent care center with a foot ulcer. His past medical
history is significant for hypertension, COPD, and diabetes mellitus. He has been hospitalized
several times in the past year for COPD exacerbations and a hip fracture. He does not have any
other current problems.

On examination he has a temperature of 37.3°C (99.1°F), a pulse rate of 105 beats/min, a


respiratory rate of 16/min, and a blood pressure of 142/83 mm Hg. His examination is
unremarkable except for a 2-cm ulcer on the ball of his left foot that has 3 cm of surrounding
erythema and some purulent drainage. His CBC is normal except for a WBC count of
14,300/mm3 (N 4300–10,800). Which one of the following would be the most appropriate
choice for initial treatment?

A) Amoxicillin/clavulanate (Augmentin)
B) Linezolid (Zyvox)
C) Ciprofloxacin (Cipro)
D) Ceftriaxone (Rocephin) and levofloxacin (Levaquin)
E) Piperacillin/tazobactam (Zosyn) and vancomycin (Vancocin).
ANSWER: E
This patient has a severe diabetic foot ulcer. It appears to be infected and there are signs of a
systemic inflammatory response. This is an indication for intravenous antibiotics.
Piperacillin/tazobactam and vancomycin would be the most appropriate choice of antibiotics
because together they cover the most common pathogens in diabetic foot ulcers, as well as
MRSA, which is present in 10%–32% of diabetic foot ulcers. This patient has recently been
hospitalized and would thus be at high risk for a MRSA infection. Moderate to severe diabetic
foot ulcers are often polymicrobial and can include gram-positive cocci, gram-negative bacilli,
and anaerobic pathogens.

Q32-A 68-year-old female with diabetes mellitus, coronary artery disease, fibromyalgia, and
dyspepsia presents for follow-up. She has been taking omeprazole (Prilosec) for 10 years. It
was started during a hospitalization, and her symptoms have returned with previous trials of
discontinuation.
Which one of the following adverse events is this patient at risk for as a result of her
omeprazole use?

A) Hypermagnesemia
B) Urinary tract infections
C) Nephrolithiasis
D) Hip fractures

ANSWER: D
Proton pump inhibitors (PPIs), including omeprazole, are generally safe and effective for
peptic ulcer disease, gastroesophageal reflux disease, and stress ulcer prevention in critically ill
patients. As use has
increased, however, risks of long-term use of PPIs have emerged. Currently known risks
include increased fractures of the hip, wrist, and spine (SOR B), community-acquired
pneumonia (SOR B), Clostridium
difficile and other enteric infections (SOR C), hypomagnesemia (SOR B), and cardiac events
when coadministered with clopidogrel (SOR B). PPIs may also affect the absorption of
vitamins and minerals, including iron, vitamin B12, and folate (SOR C). There is no known
association of PPIs with nephrolithiasis or urinary tract infections.
Q33-Which one of the following conditions can affect hemoglobin A1c levels?
A) Heart failure
B) Chronic hemolytic anemia
C) COPD
D) Hypothyroidism

ANSWER: B
The hemoglobin A1c (HbA1c) blood test provides information regarding average glucose
levels over the past 3 months. Any condition that shortens erythrocyte survival or decreases
mean erythrocyte age, such as recent acute blood loss or hemolytic anemia, will falsely lower
HbA1c levels. Hemoglobin variants and iron deficiency, kidney failure, and liver disease can
also affect HbA1c results. Heart failure, COPD, and hypothyroidism do not influence HbA1c
values.

Q34-A 31-year-old gravida 1 para 0 presents for a routine visit at 32 weeks gestation. She has
gestational diabetes mellitus (GDM) and has been following the dietary guidelines from her
dietitian. However, her blood glucose is still elevated and you discuss starting medications for
management of her GDM. She is adamant about not starting insulin but is willing to consider
taking metformin (Glucophage). Before making a decision she would like to know the specific
benefits to her and her baby. You would tell her that one benefit of treatment of GDM is a
decreased risk for
A) maternal type 2 diabetes mellitus after delivery
B) maternal preeclampsia
C) perinatal death
D) a small-for-gestational-age infant.

ANSWER: B
Although there is no specific recommendation about when to initiate pharmacotherapy for the
treatment of gestational diabetes mellitus (GDM), many women do require specific treatment
beyond diet and exercise. Insulin has traditionally been used but oral medications are
becoming increasingly common despite the lack of long-term safety data. Many outcomes for
both the mother and infant are improved with pharmacologic management of GDM. These
include a decreased risk for operative delivery, large-for-gestational-age infants, shoulder
dystocia, and maternal preeclampsia. Although a significant percentage of women with GDM
subsequently develop type 2 diabetes mellitus after delivery, pharmacologic treatment of
GDM has not been shown to decrease that risk. In addition, neither perinatal death nor the
likelihood of small-for-gestational-age infants is significantly affected. The risk of neonatal
hypoglycemia has also not consistently been shown to be affected by treatment.
Q35-A 58-year-old male delivery truck driver is diagnosed with type 2 diabetes mellitus and
after several months of working on lifestyle modification his hemoglobin A1c is 8.0%. You
suggest it is time to start a medication to help control his condition but he is very worried
about having a “low sugar reaction” that would prevent him from driving. He is on no other
medications at this time. His only other health problem is long-standing controlled
hypertension. His BMI is 33.1 kg/m2 and his serum creatinine level is 1.2 mg/dL (N 0.6–1.5).
Which one of the following medications would be least likely to cause hypoglycemia in this
patient?
A) Canagliflozin (Invokana)
B) Glimepiride (Amaryl)
C) Glipizide (Glucotrol)
D) Insulin glargine (Lantus)
E) Metformin (Glucophage)

ANSWER: E
Metformin is an inexpensive first-line oral agent for type 2 diabetes mellitus. Its mechanism of
action is to increase the sensitivity of the liver and peripheral tissues to insulin. This assists the
patient with weight loss efforts and, unlike insulin secretagogues, has been proven to reduce
mortality with long-term use. When metformin is used as monotherapy it is not associated with
episodes of hypoglycemia. For many years there has been a concern that metformin can
increase the risk for lactic acidosis. This risk has been assumed to be greater in conditions that
can lead to tissue hypoperfusion, such as heart failure or hypovolemia, or with renal
impairment. The FDA has historically recommended against the use of metformin for any
patient with even mild renal impairment (creatinine >1.4 mg/dL for women and >1.5 mg/dL
for men). However, a recent meta-analysis did not find supportive evidence for such
restrictions. Newer evidence suggests that the use of metformin is safe even with mild to
moderate renal impairment (eGFR >30 mL/min) (SOR A)
Q36-Which one of the following classes of diabetes medications increases the risk of
genitourinary infections by blocking glucose reabsorption by the kidneys?
A) SGLT2 inhibitors such as canagliflozin (Invokana)
B) GLP-1 receptor agonists such as exenatide (Byetta)
C) DPP-4 inhibitors such as sitagliptin (Januvia)
D) Meglitinides such as repaglinide (Prandin)
E) "-Glucosidase inhibitors such as acarbose (Precose)
ANSWER: A
SGLT2 inhibitors inhibit SGLT2 in the proximal nephron. This blocks glucose reabsorption by
the kidney,increasing glucosuria. The advantages of this medication include no hypoglycemia,
decreased weight, decreased blood pressure, and effectiveness at all stages of type 2 diabetes
mellitus. Disadvantages are that it increases the risk of genitourinary infections, polyuria, and
volume depletion and increases LDL-cholesterol and creatinine levels. GLP-1 receptor
agonists work by activating the GLP-1 receptors, causing an increase in insulin secretion, a
decrease in glucagon secretion, slowing of gastric emptying, and increasing satiety. DPP-4
inhibitors inhibit DPP-4 activity, which increases postprandial active incretin concentration.
This increases insulin secretion and decreases glucagon secretion. Meglitinides act by closing
the ATP-sensitive K+ channels on the B-cell plasma membranes, which increases insulin
secretion. "- Glucosidase inhibitors inhibit intestinal "-glucosidase, which slows intestinal
carbohydrate digestion and absorption.

Q37-A 57-year-old female with a past medical history significant for well- controlled type 2
diabetes mellitus, hypertension, and hyperthyroidism presents to your office with a chief
complaint of a sore throat and a fever to 101.5°F at home. She has had chills and night sweats
but has not had a cough, chest pain, or abdominal pain.
Physical Findings
General ill appearing
HEENT diffuse tender anterior cervical adenopathy; thyroid nontender; oropharynx
erythematous with some purulence on her tonsils
Cardiovascular tachycardia without murmur
Lungs clear to auscultation bilaterally
Skin mild jaundice
Laboratory Findings
Rapid strep test negative
Total WBC count 3000/mm3 (N 4500–11,000) and
absolute neutrophil count 0
Total bilirubin 5 mg/dL (N 0–1.0)
Alkaline phosphatase 151 U/L (N 38–126)
Which one of the following medications is most likely to cause these laboratory abnormalities?
A) Amlodipine (Norvasc)
B) Aspirin
C) Metformin (Glucophage)
D) Methimazole (Tapazole)
ANSWER: D
Approximately 0.3% of patients taking methimazole develop agranulocytosis, usually within
the first 60 days of starting therapy. Other rare complications of methimazole include serum
sickness, cholestatic jaundice, alopecia, nephrotic syndrome, hypoglycemia, and loss of taste.
It is associated with an increased risk of fetal anomalies, so propylthiouracil (PTU) is preferred
in pregnancy. The other medications listed are not known to cause the combination of
agranulocytosis and cholestatic jaundice that this patient has.

Q38-A 45-year-old male with diabetes mellitus returns to your office for follow- up. He is on
metformin (Glucophage), 1000 mg/day, as well as atorvastatin (Lipitor), 40 mg daily for
hyperlipidemia. There is no diagnosis of hypertension, and his blood pressure at today’s visit
is 120/70 mm Hg. Laboratory results include a hemoglobin A1c of 6.4% and an LDL-
cholesterol level of 105 mg/dL. His urine albumin/creatinine ratio is in the microalbuminuric
range for the first time.
Which one of the following would be most appropriate at this point?
A) Renal ultrasonography
B) A repeat urine albumin/creatinine ratio
A) 24-hour urine for microalbumin
B) Increasing the atorvastatin dosage
C) Stopping metformin

ANSWER: B
This normotensive diabetic patient, appropriately screened for microalbuminuria, should have
this finding confirmed on at least one of two additional spot tests, since temporary factors
other than nephropathy can also result in microalbuminuria. Once a diagnosis of chronic
kidney disease is confirmed, renal ultrasonography should be ordered to detect potentially
reversible causes. A 24- hour urine is not necessary since the urine microalbumin/creatinine
ratio correlates well with a 24-hour urine for albumin. Metformin is not contraindicated in the
presence of microalbuminuria alone without a decline in the glomerular filtration rate. The
patient is already on high-intensity statin therapy and there is no specific indication to increase
the statin dosage based on his current LDL-cholesterol level since treatment to the target LDL-
cholesterol goal has fallen out of favor.
ABFM2014
Q39-A 62-year-old female with type 2 diabetes mellitus routinely has fasting blood glucose
levels in the 80–100 mg/dL range and her hemoglobin A1c level is 7.8%. She has been
diligently monitoring her blood glucose levels and all are acceptable with the exception of
elevated bedtime readings. She currently is oninsulin glargine (Lantus), 18 U at night.
Which one of the following changes would be most appropriate for this patient ?
A) Adding rapid-acting insulin at breakfast
B) Adding rapid-acting insulin at lunch
C) Adding rapid-acting insulin at dinner
D) Increasing the nightly insulin glargine dose
E) Increasing the insulin glargine dosage and giving two-thirds in the morning and one-third
at night
ANSWER: C
This patient continues to have an elevated hemoglobin A1c and bedtime hyperglycemia. The
addition of a rapid-acting insulin at dinner would be the next step in management. For patients
exhibiting blood glucose elevations before dinner, the addition of rapid-acting insulin at lunch
is preferred. For patients with elevations before lunch, rapid-acting insulin with breakfast
would most likely improve glucose control. Increasing or splitting the insulin glargine would
be unlikely to improve management

Q40- A 48-year-old female sees you for routine follow-up. She was diagnosed with type 2
diabetes mellitus 2 years ago and has been treated with metformin (Glucophage), 850 mg
orally 3 times daily, and glipizide (Glucotrol XL), 20 mg orally daily, along with diet and
exercise. Her other medical problems include hypertension and obesity. She has no known
cardiovascular disease or microvascular complications. She came in for laboratory testing
yesterday, and her hemoglobin A1c is 8.0% (N <5.7%.)

Which one of the following medications would help with both glycemic control and weight
loss for this patient?

A) Exenatide (Byetta)
B) Pioglitazone (Actos)
C) Sitagliptin (Januvia)
D) Insulin
ANSWER: A
Given the information about this patient, such as her relatively recent diagnosis, her age, and
her lack of macro- or microvascular complications, a more strict hemoglobin A1c goal is
indicated. There are several oral and injectable medicines that are reasonable choices in this
case. Exenatide is an injectable GLP-1 agonist that is associated with weight loss. Pioglitazone
is also effective but is associated with fluid retention rather than weight loss. Sitagliptin is a
dipeptidyl peptidase IV (DPP-IV) inhibitor that may be a reasonable option in this case, but is
not associated with weight loss. Insulin, either basal only, mixed, or basal-bolus regimens,
may also be the best option for the patient described, but it does cause weight gain. Cost is
another major consideration in treatment decisions, but more information would be needed to
address this issue

Q42-An 86-year-old female nursing-home resident has type 2 diabetes mellitus, chronic
diastolic heart failure, chronic kidney disease, advanced osteoarthritis, hypertension,
Alzheimer’s disease, and other comorbidities. She requires assistance with dressing, bathing,
and feeding.
For this patient, the American Geriatrics Society recommends a hemoglobin A1c goal of
A) <7.0%
B) <8.0%
C) <8.5%
D) <9.0%
E) <9.5%

ANSWER: C
The American Geriatrics Society consensus report for considering treatment goals for glycemia
recommends a hemoglobin A1c goal of <8.5% for individuals with very complex health
problems or poor health. This includes individuals in long- term care and those with end-stage
chronic illnesses, moderate to severe cognitive impairment, or more than two activity of daily
living (ADL) dependencies. A hemoglobin A1c <8.5% equates to an estimated average
glucose level of approximately 200 mg/dL. Looser glycemic targets than this may expose
patients to acute risks from glycosuria, dehydration, hyperglycemic hyperosmolar syndrome,
and poor wound healing
Q43-Which one of the following is most likely to cause hypoglycemia in elderly patients?
A) Metformin (Glucophage)
B) Pioglitazone (Actos)
C) Glipizide (Glucotrol)
D) Sitagliptin (Januvia)
E) Glyburide (DiaBeta

ANSWER: E
The sulfonylureas are the oral hypoglycemic agents most likely to cause hypoglycemia, with
glyburide more likely to cause low glucose levels than glipizide, due to its longer half-life. The
use of these agents should be rare in elderly patients with diabetes mellitus

-Q44- A 32-year-old male presents with a 1-year history of increasing fatigue, polyuria, and a
gradual 30-lb weight loss. Serum chemistries reveal a bicarbonate level of 23 mEq/L (N 22–
28), a corrected anion gap of 8 mEq/L (N 3–11), and a glucose level of 658 mg/dL (N 60–
110). The patient is admitted to the hospital and his serum glucose drops to 174 mg/dL after he
is given 2 L of intravenous normal saline and 10 units of regular insulin subcutaneously. He is
observed overnight and further laboratory testing is done the next morning.
Which one of the following is more consistent with type 2 diabetes mellitus than with type 1
diabetes mellitus?
A) The patient’s history of weight loss
B) The patient’s response to the initial dose of insulin
C) The time course of symptom onset
D) Morning laboratory studies showing a C-peptide level of <1.1 ng/mL (N 1.1–4.4
ANSWER: C
This patient presents with marked hyperglycemia but no evidence of ketoacidosis or nonketotic
coma. Differentiating between type 1 and type 2 diabetes mellitus is important for guiding
therapy. The gradual onset of symptoms is more consistent with type 2 diabetes mellitus,
whereas type 1 diabetes typically has a more rapid onset. Patients with type 1 diabetes
typically need lower doses of insulin to correct hyperglycemia, as they lack the insulin
insensitivity that is the hallmark of type 2 diabetes. Positive anti-GAD antibodies and low C-
peptide at the time of the initial diagnosis are also consistent with type 1 diabetes, although C-
peptide levels can also be low in long-standing type 2 diabetes. Weight loss occurs in both
types of diabetes mellitus when glucose is profoundly elevated
Q45-3-A 49-year-old unemployed African-American male has multiple chronic conditions,
including type 2 diabetes mellitus, chronic kidney disease, hypertension, obstructive sleep
apnea, and lower extremity edema. He is on several medications, which he reports taking
sporadically due to a lack of health insurance and a limited income. He has several abnormal
laboratory values, including a serum creatinine level of 3.1 mg/dL, an increase from his usual
levels, which have ranged from 1.2–1.6 mg/dL over the past 5 years. You calculate his
glomerular filtration rate to be 27 mL/min/1.73 m2.
Which one of the following medications that he takes should be stopped at this time?
A) Amlodipine (Norvasc)
B) Aspirin
C) Insulin
D) Metformin (Glucophage)
E) Metoprolol (Lopressor, Toprol-XL

ANSWER: D
The National Kidney Foundation defines chronic kidney disease (CKD) as a glomerular
filtration rate (GFR) <60 mL/min/1.73 m2, or evidence of kidney damage with or without a
decreased GFR, for 3 or more months (SOR C). Individuals with CKD are at increased risk for
serious cardiovascular disease events, especially if they have other comorbid conditions such
as hypertension or diabetes mellitus. It is important to have a multifactorial strategy for this
patient that includes controlling his blood glucose and blood pressure in addition to continuing
his antiplatelet therapy (SOR A). According to the U.S. Food and Drug Administration,
metformin is contraindicated when a patient’s serum creatinine is >1.5 mg/dL in men or >1.4
mg/dL in women (SOR C). This recommendation is based on the fact that metformin is
cleared by the kidneys, and when the GFR is <60 mL/min/1.73 m2, the clearance of
metformin decreases by 75%. Because lactic acidosis caused by metformin is rare, some
experts advocate for continued metformin use with higher serum creatinine levels. However,
most recommendations agree it should be stopped when the GFR falls below 30 mL/min/1.73
m2 (SOR C). This patient should stop his metformin, and his insulin should be adjusted
appropriately to achieve optimal glycemic control. It is also important that he continue his
antihypertensive and antiplatelet medications.
Q45- Which one of the following tests is recommended for the detection and diagnosis of
gestational diabetes mellitus ?
A) Hemoglobin A1c
B) Fasting blood glucose
C) 2-hour postprandial glucose
D) An oral glucose tolerance test
E) A fasting insulin/glucagon ratio
ANSWER: D
In pregnant women not known to have diabetes mellitus, screening for gestational diabetes
mellitus should be done at 24–28 weeks gestation. There are two acceptable screening
strategies. The one-step 2-hour 75-g oral glucose tolerance test should be performed in the
morning after a minimum 8-hour fast. The diagnosis of gestational diabetes mellitus is made if
the fasting plasma glucose level is ³92 mg/dL, if the level at 1 hour is ³180 mg/dL, or if the
level at 2 hours is ³153 mg/dL.
A two-step approach may also be used, consisting of a nonfasting 1-hour 50-g oral glucose
tolerance test. If the 1-hour glucose level is ³140 mg/dL, a 3-hour 100-mg glucose tolerance
test should be performed. (ACOG recommends 135 mg/dL in ethnic minorities with a higher
risk of diabetes mellitus, and some experts recommend 130 mg/dL.) The 3-hour test should be
performed when the patient is fasting. A diagnosis of gestational diabetes is made when at
least two of the following four plasma glucose levels are met or exceeded:
Carpenter/Coustan National Diabetes Group Fasting 95 mg/dL 105 mg/dL 1h 180 mg/dL 190
mg/dL 2h 155 mg/dL 165 mg/dL 3h 140 mg/dL 145 mg/Dl Women with risk factors for
diabetes mellitus should also be screened for undiagnosed diabetes at their first prenatal visit
Q46-A 52-year-old male with diabetes mellitus reports that he ran out of insulin a week ago.
He is drowsy but responds to your verbal commands, and the remainder of his examination is
unremarkable.
Laboratory Findings
Blood glucose. 625 mg/dL S
erum sodium 128 mEq/L (N 135–145)
Serum potassium. . . . . . . . . . . . . . . . . . . . . . . 5.9 mEq/L (N 3.5–5.0)
Serum bicarbonate. 12 mEq/L (N 22–26)
BUN 52 mg/dL (N 8–25)
Which one of the laboratory abnormalities is an indication that he has severe diabetic
ketoacidosis?
A) Glucose
B) Sodium
C) Potassium
D) Bicarbonate
E) BUN
ANSWER: D
The diagnosis of diabetic ketoacidosis (DKA) is based on an elevated serum glucose level
(>250 mg/dL), an elevated serum ketone level, a pH <7.3, and a serum bicarbonate level <18
mEq/L. The severity of DKA is determined by the arterial pH, bicarbonate level, anion gap,
and mental status of the patient.
Elevation of BUN and serum creatinine levels reflects intravascular volume loss. The
measured serum sodium is reduced as a result of the hyperglycemia, as serum sodium is
reduced by 1.6 mEq/L for each 100 mg/dL rise in serum glucose. The degree of hyperglycemia
does not necessarily correlate closely with the degree of DKA since a variety of factors
determine the level of hyperglycemia, such as oral intake and urinary glucose loss.
Dyslipidemia
PRETEST2019
Q1. You are doing a screening physical examination for a 40-year-old female patient. She
does not have diabetes or known coronary artery disease. You calculate her 10-year
Atherosclerotic Cardiovascular Disease (ASCVD) risk to be 6.5%. Her low-density
lipoprotein (LDL) cholesterol is 165 mg/dL, high-density lipoprotein (HDL) cholesterol is 48
mg/dL, and triglycerides are 193 mg/dL. Given this information, what do you recommend as
treatment for her dyslipidemia?
a. Lifestyle modifications such as diet and physical activity
b. Treatment with a low-intensity statin
c. Treatment with a moderate-intensity statin
d. Treatment with a high-intensity statin
e. Treatment with a fenofibrate agent

The answer is a.
Recent guidelines describe four groups of patients who benefit from
statin therapy for ASCVD prevention: (1) patients with clinical ASCVD, such as coronary
artery disease, stroke, transient ischemic attack (TIA), or peripheral artery disease (PAD); (2)
patients with primary elevations of LDL-C more than or equal to 190 md/dL; (3) patients aged
40 to 75 with diabetes and LDL-C between70 and 189 mg/dL and without clinical
ASCVD; or (4) patients without clinical ASCVD or diabetes with an LDL-C between 70 and
189 mg/dL AND an estimated 10-year ASCVD risk more than or equal to 7.5%.
Recommendations regarding intensity of statin level are based on these groups, but this patient
does not benefit from statin therapy at this time. Lifestyle modifications are recommended as
the mainstay of all prevention efforts for patients with dyslipidemias or risk factors for
ASCVD.

Q2. You are caring for a newly established 54-year-old white male patient with T2DM,
hypertension, and dyslipidemia. He does not have any clinical ASCVD, but his father died
from a MI at the age of 56. He admits he has not been great about following up with a
physician and would like to make a greater effort to control his cardiac risks. His blood
pressure is well controlled with lisinopril- hydrochlorothiazide. He has never smoked. His 10-
year ASCVD risk is calculated at 11.2%. His recent laboratory values are below.
What treatment recommendation do you suggest for his dyslipidemia?
a. Lifestyle modifications such as diet and physical activity
b. Treatment with a low-intensity statin
c. Treatment with a moderate-intensity statin
d. Treatment with a high-intensity statin
e. Treatment with a fenofibrate agent

The answer is d.
According to the ACC/AHA blood cholesterol guidelines, for the
prevention of ASCVD events this patient should receive a high-intensity statin based on his
risk factors. Patients with clinical ASCVD, patients with diabetes and 10-year ASCVD risk
more than or equal to 7.5%, and patients with an LDL-C more than or equal to 190 mg/Dl
benefit from a high-intensity statin. Patients 40 to 75 years old with diabetes and an LDL-C
between 70 and 189 mg/dL or patients 40 to 75 years old with a 10-year ASCVD risk more
than or equal to 7.5%, and those with clinical ASCVD who are more than 75 years old or
cannot tolerate a high- intensity statin receive benefit from a moderate-intensity statin. While
his triglycerides are elevated, treating them with a fenofibrate agent would not be expected to
improve his health outcome. Use of a high-intensity statin will be likely to lower this patient’s
LDL-C by approximately 50%.

Q3. You are caring for a 26-year-old man with dyslipidemia and a family history of early
coronary arterial disease. Laboratory analysis reveals a low HDL. Which of the following
interventions, if adopted by the patient, would raise his HDL levels to the greatest extent?
a. Eat oat bran
b. Lose weight
c. Start exercising
d. Quit smoking
e. Reduce life stress
The answer is c.
Several lifestyle modification efforts are known to increase HDL cholesterol. By losing
weight, a person can expect to raise HDL by 5 to 10 points. Smoking cessation has the same
approximate effect. Adopting an exercise program is even more effective, raising HDL by up
to 15 points. Eating oat bran and decreasing life stress can lower LDL, but is not likely to raise
HDL. Additionally, alcohol, in moderation, raises HDL cholesterol.

Q4- You have performed a screening lipid profile on an otherwise healthy man. His results
indicate elevated triglycerides, a low HDL, a high LDL, an elevated total cholesterol, and an
elevated very-low-density lipoprotein (VLDL). You would like to rescreen him in the fasting
state. Which of the following laboratory values is likely to decrease in the fasting state?
a. Serum triglycerides
b. HDL
c. LDL
d. Total cholesterol
e. VLDL
The answer is a.
Blood lipids change acutely in response to food intake. The triglyceride level is lowest in the
fasting state and rises by an average of 50 mg/dL postprandially. As the triglyceride level rises,
the total and LDL cholesterol each fall. Thus total and LDL cholesterol tend to be higher when
fasting. HDL varies little whether fasting or not.

Q5. You have been caring for a 36-year-old man, and identified that he meets criteria for
therapeutic lifestyle changes (TLC) to improve his lipid profile. After 4 months of adhering to
your recommendations, his LDL is still higher than goal.
Which of the following drug classes should be initiated?
a. A statin
b. A bile acid sequestrant
c. Nicotinic acid
d. A fibrate
e. A cholesterol absorption inhibitor

The answer is a. The National Cholesterol Education Program ATP III guidelines recommend
that medications should be added to TLC after 3 months if LDL goals are not achieved, even
in lower risk patients. Given their track record, statin agents are the drugs of first choice.
Q6. You started a 43-year-old female patient of yours on a statin for dyslipidemia. She has no
other medical conditions, but does have a family history of coronary heart disease. At a
follow-up visit for a different chief complaint, you order a laboratory panel that includes
serum transaminases. Her AST is found to be 56 U/L (normal is 10-40 U/L) and her ALT is
found to be 115 U/L (normal is 7-56 U/L). Which of the following is most appropriate given
these values?
a.Discontinue the statin.
b.Decrease the dose of the statin.
c.Test muscle enzymes (creatine phosphokinase [CPK]) and discontinue the statin if also
elevated.
d.Test muscle enzymes (CPK) and decrease the dose of the statin if also elevated.
e.No change is indicated.
. The answer is e.
The primary side effects seen with statin agents include myopathy and increased liver enzymes.
Rhabdomyolysis occurs in less than 0.1% of cases, but elevated transaminases occur more
commonly. Discontinuation of the agent is required only if liver enzymes increase to more
than 3 times normal (and that occurs in only 1% of patients on statins). Routine monitoring of
serum aminotransferase levels is no longer recommended by the FDA since hepatic side
effects will most often trigger an evaluation, and the effects are reversible. Routine monitoring
of muscle enzymes is not supported by evidence, but unexplained pain in large muscle groups
should prompt an investigation for myopathy.

Q7. You have prescribed niacin for a patient with VLDL cholesterol and elevated triglycerides.
He reports nonadherence to this regimen because of significant flushing that occurs when he
takes the medication. What would you recommend to avoid this side effect?
a. Take the niacin at night
b. Take the niacin with food
c. Take the niacin with milk
d. Take aspirin before taking the niacin
e. Take a proton-pump inhibitor before taking the niacin

The answer is d.
Niacin is associated with a prostaglandin-mediated flushing effect. Aspirin (81- 325 mg)
blocks much of the flushing that is associated with sustained-release niacin preparations.
Taking niacin at night, with food, on an empty stomach or with milk, or with a proton-pump
inhibitor will not impact the side effects. Initiating niacin at a low dose (100 mg daily) and
slowly titrating up can also decrease flushing. Only 50% to 60% of patients can tolerate the
full-dose of Niacin; extended-release dosing is better tolerated.
Q8. You are treating a patient who is interested in more “natural” methods to control his
cholesterol. He wants to use niacin. Which of the following statements regarding niacin is true?
a. It substantially decreases LDL.
b. It substantially raises HDL.
c. It has no effect on triglycerides.
d. Its side effects generally prevent it from being used.
e. It can’t be used in patients who have concurrent diabetes.
The answer is b.
Niacin was the first lipid-lowering agent associated with decreased total mortality. It
moderately decreases LDL, can increase HDL by 20% to 25%, and moderately decreases
triglycerides. It causes a prostaglandin-mediated flushing that patients often describe as “hot
flashes.” This side effect can be easily moderated by having the patient take an NSAID or
aspirin at least an hour before taking the niacin. Although niacin can increase blood sugar, it is
safe for diabetics to use.

Q9. You are treating a patient with dyslipidemia and clinical ASCVD. She has elevated total
cholesterol, LDL-C, and triglycerides. Her HDL is low. Which of the following changes will
have the greatest effect on mortality?
a. Increasing HDL
b. Decreasing LDL-C
c. Decreasing triglycerides
d. Decreasing non-HDL cholesterol
e. Decreasing apolipoprotein-A

The answer is b.
Numerous studies have demonstrated the benefits of lowering LDL-C on all-cause and
cardiovascular mortality. Non-HDL cholesterol is the sum of HDL and\VLDL cholesterol;
while reductions in VLDL are considered beneficial, there are no studies evaluating its effect
on mortality. Niacin increases HDL and lowers VLDL and has been shown to reduce
cardiovascular events and mortality, but not to the extent that lowering LDL does.
Additionally, raising HDL doesn’t seem to have an added benefit to lowering LDL
cholesterol. Apolipoprotein A is protective against CVD and lowering it would negatively
affect outcomes. The effect of lowering triglycerides is not well understood; studies have
shown higher risks of CVD in patients with elevated triglycerides. However, the current data
do not support lowering triglycerides for CVD prevention.
ABFM 2020
154. A 38-year-old male comes to your office for follow-up of his diabetes mellitus. He takes
metformin (Glucophage) and dulaglutide (Trulicity) and his hemoglobinA1c is 6.5%. He has
an LDL-cholesterol level of 120 mg/dL, an HDL-cholesterol level of 55 mg/dL, and a
triglyceride level of 190mg/Dl .He asks your advice about statin therapy for cholesterol
management. You advise him to start:
A. A low-intensity statin now
B. A moderate-intensity statin now
C. Statin therapy at 40 years of age
D. Statin therapy when his 10-year atherosclerotic cardiovascular disease (ASCVD) risk is >5%
E. Statin therapy when his 10-year ASCVD risk is >7.5%

ANSWER: C
There is high-quality evidence from randomized, controlled trials that a moderate-intensity
statin should be initiated for all patients age 40–75 years with diabetes mellitus regardless of
their calculated 10-year atherosclerotic cardiovascular disease (ASCVD) risk. There is not
strong evidence supporting the use of statins before age 40 in patients with diabetes unless
their LDL-cholesterol level is very high. The ASCVD risk score is valid for patients over 40
years of age and cannot be calculated before then

A 73-year-old female sees you for the first time for a health maintenance visit. Her medical
history includes hyperlipidemia, GERD, insomnia, and osteoarthritis, but she is otherwise
healthy. Her estimated 10-year risk of atherosclerotic cardiovascular disease is 14%. She lives
independently. She has a past history of alcohol abuse but has not used alcohol in 20 years,
and is a lifetime nonsmoker. Her current medication regimen includes aspirin, 81 mg once
daily; melatonin, 3 mg at bedtime; acetaminophen, 500 mg three times daily; atorvastatin
(Lipitor), 20 mg once daily; and famotidine (Pepcid), 20 mg once daily. Today her blood
pressure is 130/70 mm Hg, pulse rate 72 beats/min, and oxygen saturation 95% on room air.
Which one of the following changes to her current medication regimen would improve the
likelihood of benefit and reduce the likelihood of harm?
A. Stopping aspirin
B. Stopping melatonin
C. Stopping acetaminophen and starting diclofenac, 50 mg twice daily
D. Stopping atorvastatin and starting rosuvastatin (Crestor), 40 mg at bedtime
E. Stopping famotidine and starting omeprazole (Prilosec), 20 mg once daily
ANSWER: A
This patient is generally healthy and highly functional at baseline. She has multiple medical
problems but they are unlikely to significantly reduce her longevity. She has no known history
of cardiovascular or cerebrovascular disease, so aspirin was being used for primary
prevention. The U.S. Preventive Services Task Force (USPSTF) has stated that there is
insufficient evidence to recommend for or against the use of aspirin for primary prevention of
cardiovascular disease in adults over the age of 70. Since the USPSTF guidelines were
published, evidence from two large randomized, controlled trials provided strong support for
the discontinuation of aspirin for most older adults without prior cardiovascular disease, with
an indication that it increases the risk of all-cause mortality.
In this patient at low to moderate risk who is already taking a statin, the risk of continuing
aspirin exceeds the potential benefit. There is no indication for changing this patient from a
moderate-intensity to a high-intensity statin. For uncomplicated gastric reflux in older adults,
H2-blockers are preferred over proton pump inhibitors, which are associated with an increased
risk of infections and fractures. This patient has no concerns about osteoarthritis, and although
she has a remote history of alcohol abuse, her dosing of daily acetaminophen is well below the
threshold of concern for liver injury. Switching to an NSAID such as diclofenac would place
her at risk for short-term and long-term renal complications. Melatonin is not known to have
long-term adverse effects in older adults.

ABFM2016
88. You see a 45-year-old white male smoker for follow-up of his hypertension. His past
medical history is otherwise unremarkable. At this visit he has a blood pressure of 136/84 mm
Hg. A lipid panel reveals the following:
Triglycerides ………………………… 226 mg/dL
Total cholesterol ……………………… 228 mg/dL
LDL-cholesterol …………………………159 mg/dL
HDL-cholesterol ………………………… 31 mg/dL
The ASCVD Risk Estimator yields a 10-year atherosclerotic cardiovascular disease risk of
15.4%. Based on the 2013 American College of Cardiology/American Heart Association
guideline for reducing atherosclerotic cardiovascular disease risk in adults, which one of the
following is the most appropriate medication regimen for this patient?
A) Moderate- or high-intensity statin therapy
B) Statin therapy titrated to reduce LDL-cholesterol by 20%–30%
C) Combined statin and fibrate therapy to reduce both LDL-cholesterol and triglycerides
D) Combined statin and niacin therapy to reduce LDL-cholesterol and raise HDL-cholesterol
E) Combined statin and PCSK9 inhibitor therapy to reduce the LDL-cholesterol level to <70
mg/dL
ANSWER: A
The 2013 American College of Cardiology/American Heart Association guideline for
reducing atherosclerotic cardiovascular disease (ASCVD) risk in adults recommended several
significant changes in the management of hyperlipidemia. This guideline recommends looking
at overall risk as estimated by a tool, the ASCVD Risk Estimator, which considers not only
lipid parameters but also age, sex, ethnicity, systolic blood pressure, and the presence or
absence of diabetes mellitus, treated hypertension, and smoking. Four major risk groups were
identified for treatment, one of which was adults ≥40 years of age with an estimated 10-year
risk of ASCVD ≥7.5%.
Therapy is graded by intensity (low-, moderate-, or high-intensity statin therapy), and
therapeutic targets for LDL-cholesterol were abandoned since there is no demonstrable benefit
from achieving a certain level of LDL-cholesterol in treated patients. This patient has an
estimated 10-year ASCVD risk well above 7.5% and is a candidate for moderate- or high-
intensity statin therapy under the guideline. Niacin and fibrate therapy do not have a
demonstrable impact on cardiovascular outcomes and are not recommended in the guideline.
PCSK9 inhibitors are also not yet recommended in any guideline.

201. A 45-year-old male with hyperlipidemia presents to your office for follow-up. Because of
his cardiovascular risk level he has been on moderate-intensity statin therapy with atorvastatin
(Lipitor) for the past 6 months. His HDL-cholesterol level at this visit is 29 mg/dL.
Which one of the following steps is most appropriate?
A) Continue the statin
B) Add fibrates
C) Add niacin
D) Supplement with fish oil
ANSWER: A
Drug therapy aimed at increasing HDL-cholesterol levels when added to a statin treatment
does not decrease a patient’s cardiovascular risk. Such agents have no effect on all-cause
mortality, cardiovascular mortality, or the risk of stroke (SOR B). Current guidelines for
treatment of hyperlipidemia are based on individual cardiovascular risk stratification rather
than LDL-cholesterol levels, and recommend low-, medium-, or high-intensity therapy with
statins, based on an individual patient’s risk.
ABFM2015
Q1-A 55-year-old nonsmoking African-American female with diabetes mellitus sees you for a
routine visit. She has no other cardiac risk factors. Her blood pressure is 120/74 mm Hg and
she has a fasting total cholesterol level of 180 mg/dL, an HDL-cholesterol level of 52 mg/dL,
and an LDL-cholesterol level of 100 mg/dL. Her calculated 10-year risk of atherosclerotic
cardiovascular disease is 5.8%.
According to the 2013 American College of Cardiology/American Heart Association
cholesterol guidelines, which one of the following is recommended for this patient?
A) No statin therapy
B) Low-intensity statin therapy
C) Moderate-intensity statin therapy
D) High-intensity statin therapy.

ANSWER: C
The 2013 ACC/AHA cholesterol guidelines outline four major groups in whom statin therapy
is beneficial:
(1) individuals with clinical atherosclerotic cardiovascular disease (ASCVD), (2) those with
primary elevations of LDL-C >190 mg/dL, (3) patients age 40–75 with diabetes mellitus, an
LDL-C level of 70–189 mg/dL, and no clinical ASCVD, (4) patients age 40–75 without
clinical ASCVD or diabetes, an LDL-C level of 70–189 mg/dL, and an estimated 10-year
ASCVD risk >7.5%. For patients age 40–75 with diabetes, an LDL-C level of 70–189 mg/dL,
and no clinical ASCVD, a moderate- intensity statin is recommended.

Q2-A 40-year-old female sees you for a health maintenance visit. She has no complaints and
other than being overweight she has an unremarkable examination. Laboratory results are also
unremarkable except for her lipid profile. She has a total cholesterol level of 251 mg/dL, an
HDL-cholesterol level of 31 mg/dL, and a triglyceride level of 1250 mg/dL. The LDL-
cholesterol level could not be calculated and measured 145 mg/dL.In addition to lifestyle
changes, this patient would most likely benefit from

A) niacin
B) omega-3 fatty acid supplementation
C) atorvastatin (Lipitor)
D) ezetimibe (Zetia)
E) fenofibrate (Tricor)
ANSWER: E
Treatment of hypertriglyceridemia depends on its severity. Contributing factors include a
sedentary lifestyle, being overweight, excessive alcohol intake, type 2 diabetes mellitus, and
genetic disorders. Triglyceride levels of 150–199 mg/dL are considered mild
hypertriglyceridemia, levels of 200–999 mg/Dl are moderate, 1000–1999 mg/dL are severe,
and levels >2000 mg/dL are considered very severe. Patients with hypertriglyceridemia in the
mild to moderate range may be at risk for cardiovascular disease, but those who have severe or
very severe hypertriglyceridemia have a significant risk of pancreatitis.
In addition to having the patient exercise, reduce intake of fat and carbohydrates, and lose
weight, she should also be counseled to avoid alcohol. For patients at risk for pancreatitis,
fibrates are recommended as the initial treatment for pancreatitis. It should be noted that
statins may have a modest triglyceride-lowering effect and may be helpful in decreasing
cardiovascular risk in those who have moderately elevated triglycerides. However, they should
not be used alone in patients who have severe hypertriglyceridemia. Studies have also shown
that while omega-3 fatty acids decrease triglycerides and very low density lipoprotein
cholesterol levels, they may increase LDL-cholesterol levels. Treatment with omega-3 fatty
acids does not decrease total mortality or cardiovascular events, and therefore is not
recommended.
Dyspepsia
ABFM 2020
145. A46-year-old female with symptoms of solid and liquid dysphagia undergoes esophageal
manometry for suspected achalasia. These symptoms of achalasia are caused by which one of
the following pathophysiologic mechanisms?
A. More than 50% of swallows being weak or failed
B. Aperistalsis in the distal two-thirds of the esophagus ,with incomplete lower esophageal
sphincter relaxation
C. High-pressure esophageal contractions, with normal relaxation of the esophagogastric
junction
D. Increased premature contractions of the distal esophagus ,with normal relaxation of the
esophagogastric junction
E. Mechanical obstruction due to an upper esophageal web

ANSWER: B
Achalasia is associated with the loss of ganglion cells in the esophageal wall, which leads to
the loss of normal esophageal peristalsis and failure of relaxation of the lower esophageal
sphincter (LES). Incomplete LES relaxation is highly specific for achalasia. Ineffective
esophageal motility is demonstrated by an inability to generate an effective swallow 50% of
the time during motility testing. Hypercontractile esophagus, also known as jackhammer
esophagus, leads to high-pressure contractions in the esophagus but normal LES functioning.
Diffuse esophageal spasm leads to increased premature contractions of the esophagus with
normal functioning of the LES. Esophageal webs are mechanical obstructions unrelated to
achalasia.

A 42-year-old male presents with a 10-day history of hoarseness. He also has a 2-month
history of reflux symptoms and has been taking antacids as needed. He does not take any other
medications. There is no history of fever, weight loss, night sweats, or appetite changes. You
note that the patient is hoarse, and a physical examination is normal, including HEENT,
cardiovascular, and pulmonary examinations. He is a member of a community choir that
rehearses twice a week. In addition to voice rest, which one of the following would be most
appropriate at this time?
A. Supportive care only
B. Azithromycin (Zithromax)
C. Omeprazole (Prilosec)
D. A course of prednisone
ANSWER: C
This patient has hoarseness that has been present for less than 2 weeks. In addition to voice
rest, treatment in patients with a history of GERD should include a 3- to 4-month trial of a
high-dose proton pump inhibitor (SOR C). In patients with hoarseness lasting longer than 2
weeks without an apparent benign etiology, the larynx should be examined by direct or
indirect laryngoscopy (SOR C). Antibiotics and oral corticosteroids should not be used for the
empiric treatment of hoarseness in the absence of signs and symptoms that suggest an
underlying cause.
-Long-term proton pump inhibitor use is associated with an increased risk for ?
A. Barrett’s esophagus
B. Gout
C. Hypertension
D. Pneumonia
E. Type 2 diabetes
ANSWER: D
Acid suppression therapy is associated with an increased risk of community-acquired and
health care–associated pneumonia, which is related to gastric overgrowth by gram-negative
bacteria. Long-term treatment of Barrett’s esophagus is an indication for chronic proton pump
inhibitor (PPI) use. PPI therapy does not increase the risk of gout, hypertension, or type 2
diabetes.

ABFM2019
Q1-A 4-month-old female is brought to your office by her parents because she spits up after
most feedings. She is their first child and was born at term with no prenatal or postnatal
complications. She is formula fed. A review of her growth chart shows normal growth. Her
physical examination and vital signs are normal as well.
Which one of the following would be most appropriate at this point?
A) Reassurance only
B) Advising the parents to place the infant in a prone position for sleeping
C) An upper GI series
D) Omeprazole (Prilosec)
E) Ranitidine (Zantac).
ANSWER: A
Gastroesophageal reflux is very common in infants. Reflux in infants peaks at 4 months of
age, and two-thirds of infants regurgitate at least once a day at this age. The incidence declines
dramatically in the next few months, and by 1 year of age less than 5% of infants regurgitate
on a daily basis. If the infant is healthy and growing normally, reassurance is appropriate (SOR
C).Conservative measures are recommended if the reflux causes distress (SOR C). Such
measures include placing the infant on her side or prone while awake to reduce reflux.
However, infants should not be placed prone while sleeping, to prevent SIDS. Appropriate
conservative measures also include smaller, more frequent feedings, the addition of thickening
agents such as rice cereal to formula, and changing to amino acid formulas if infants are
allergic to cow’s milk protein. If conservative measures and time do not alleviate the
symptoms, a 4-week trial of H2-blockers or proton pump inhibitors can be tried. The
American Academy of Pediatrics and the Society of Hospital Medicine (Pediatric) recommend
not routinely treating reflux with medication. Imaging, such as esophagogastroduodenoscopy,
is not routinely used in the initial workup of reflux in infants. It is reserved for recalcitrant
cases, atypical symptoms, or complications (SOR C).

Q2-. A 52-year-old male with a history of severe esophageal reflux and dysphagia undergoes
upper GI endoscopy that reveals a mid-esophageal stricture and severe erosive esophagitis.
The stricture is dilated by the gastroenterologist and he recommends long-term proton pump
inhibitor therapy.
While long-term proton pump inhibitor therapy should alleviate symptoms of reflux, it may be
associated with an increased risk of which one of the following?

A) Clostridium difficile infection


B) Helicobacter pylori infection
C) Type 2 diabetes mellitus
D) Iron deficiency anemia
E) Hypothyroidism.

ANSWER: A
Proton pump inhibitor use has been shown to increase the risk for Clostridium difficile and
other enteric infections, and elderly patients and those with significant comorbid conditions
may already be at increased risk. Studies have not shown an increased risk for iron deficiency.
There is no increased risk for hypothyroidism, Helicobacter pylori infection, or type 2 diabetes
mellitus.

Q3-A 75-year-old female presents with a complaint of paresthesias in her feet. On examination
she has mild erythema of her tongue and decreased vibratory sensation in her feet. A CBC
reveals a hemoglobin level of 11.1 g/dL (N 12.0–16.0) and a mean corpuscular volume of 105
:m3 (N 78–102).
The patient takes the following over-the-counter drugs: aspirin, 81 mg/day; ranitidine
(Zantac), 150 mg twice daily; and acetaminophen, 325 mg twice daily.

Which one of the following prescription medications the patient takes is most likely causing
her problem?
A) Hydrochlorothiazide
B) Lisinopril (Prinivil, Zestril)
C) Amlodipine (Norvasc)
D) Simvastatin (Zocor)
E) Omeprazole (Prilosec

ANSWER: E
The use of gastric acid inhibitors, particularly when a proton pump inhibitor and H2-receptor
antagonist are combined, is significantly associated with vitamin B12 deficiency. This is more
common when combined therapy has been used for 2 years or longer. Because gastric acid is
required for the liberation of vitamin B12 bound to food protein before it is bound to intrinsic
factor for absorption, suppression of gastric acid may lead to vitamin B12 deficiency.

Q4-A 69-year-old female is unable to obtain adequate calcium from dietary sources. She is on
long-term therapy with pantoprazole (Protonix) for peptic ulcer disease.
Which one of the following would be the most appropriate calcium supplementfor this patient?
A) Oyster shell calcium
B) Calcium carbonate
C) Calcium lactate
D) Calcium gluconate
E) Calcium citrate

ANSWER: E
Women over the age of 50 require at least 1200 mg of calcium a day. Inadequate calcium
intake is common, particularly in older women, and is associated with increased bone loss and
an increased fracture risk. Supplements should be considered when dietary intake is
inadequate. Calcium carbonate is the least expensive and most commonly used supplement,
but it is constipating and stomach acid improves its absorption. Calcium gluconate and
calcium lactate are rarely used for fracture prevention or calcium supplementation. Bone meal
and oyster shell calcium are primarily composed of calcium carbonate but may contain lead or
other contaminants since they are derived from natural sources. In addition, their absorption
without stomach acid is erratic. Calcium citrate is less dependent on stomach acidity for
absorption and it may be used with long-term gastric acid suppression agents. It may be taken
without regard to food or meals.
ABFM2017
Q5- A 43-year-old female is very distressed about symptoms of postprandial fullness and early
satiety. She has seen several physicians over the last 6 months but said she was always told
―nothing’s wrong.‖ You review her most recent workup, including normal blood tests,
normal gallbladder testing, and upper endoscopy that shows no abnormalities, including
negative testing for Helicobacter pylori. She has tried multiple antacid medications, including
omeprazole 63 (Prilosec), lansoprazole (Prevacid), and ranitidine (Zantac), with no success.
Which one of the following medications has the best chance of providing this patient with
symptom relief?
A) Clonazepam (Klonopin)
B) Escitalopram (Lexapro)
C) Metoclopramide (Reglan)
D) Ondansetron (Zofran)
E) Sucralfate (Carafate)

ANSWER: C
This patient’s history fits the diagnosis of functional dyspepsia. Two subtypes of this disorder
have been described. The first, epigastric pain syndrome, is described as intermittent pain and
burning in the epigastrium. The second, postprandial distress syndrome, is more typical of the
symptoms this patient describes: postprandial fullness and early satiety. Although there is
considerable benefit from reassurance and ―naming‖ a patient’s condition, empiric treatment is
also warranted. Patients with epigastric pain syndrome are more likely to respond to proton
pump inhibitors or H2-blockers. Patients with predominantly postprandial distress symptoms
are more likely to improve with a motility agent such as metoclopramide.
Sucralfate, antacids, and selective antidepressants have not been shown to be more effective
than placebo in functional dyspepsia, whereas tricyclic antidepressants and buspirone have
shown some benefit and are reasonable next steps for this patient.
GAD
ABFM 2020
1. A 48-year-old male presents with a 1-year history of feeling nervous. He feels well
otherwise except for mild discomfort from arthritis in both knees. A physical examination is
normal, and laboratory studies, including thyroid function, are also normal. You make a
diagnosis of generalized anxiety disorder. The patient declines psychotherapy and prefers
pharmacologic treatment.

Which one of the following is the first-line long-term treatment for this patient?
A. Alprazolam (Xanax)
B. Buspirone
C. Duloxetine (Cymbalta)
D. Imipramine (Tofranil)
E. Quetiapine (Seroquel)
ANSWER: C
Most adults with generalized anxiety disorder (GAD) should be offered drug therapy if
nondrug therapies are ineffective or if the patient is not interested in them. SSRIs and SNRIs
are recommended as first-line drug therapies because of their tolerability and efficacy
compared with other drug therapies. Of the options listed, the SNRI duloxetine would be most
appropriate for the treatment of this patient’s GAD. Alprazolam may be helpful for short-term
treatment of anxiety but not as a long-term treatment. Buspirone, imipramine, and quetiapine
are not indicated as initial therapy.

2. A 21-year-old female sees you for a medical evaluation prior to admission to a treatment
program for anorexia nervosa. The effects of anorexia on the hypothalamic-pituitary axis can
cause which one of the following?
A. Bone loss
B. Elevated testosterone
C. Hyperglycemia
D. Hypothyroidism
E. Menorrhagia
ANSWER: A
Anorexia has multiple effects on the hypothalamic-pituitary axis. Bone loss can be
significant.In a study of 130 women, bone mineral density was reduced by at least 1.0
standard deviation at one or more skeletal sites in 92% of patients. Testosterone levels are
often low, contributing to bone loss. Hypoglycemia, not hyperglycemia, can occur but this is
not common. Anorexia often results in amenorrhea and infertility, and TSH and T4 levels may
be normal or low.
3. You have assumed the care of a well-established patient in your practice whose medications
include chronic alprazolam (Xanax) treatment for anxiety and codeine for chronic back pain
following a work accident years earlier. His Prescription Drug Monitoring Program report
shows a consistent pattern of filling the medications as prescribed. You order a urine
immunoassay for opioids and benzodiazepines. The results are positive for opioids but
negative for benzodiazepines.
Which one of the following would be the most appropriate next step?
A. Perform confirmatory testing for alprazolam
B. Repeat the urine immunoassay for benzodiazepines
C. Investigate for possible diversion of alprazolam
D. Stop prescribing alprazolam
E. Stop prescribing controlled substances
ANSWER: A
Because of the importance of urine drug testing and the ramifications for patients, it is
essential that physicians understand and properly interpret these results. The most appropriate
next step in this case is to perform confirmatory testing for alprazolam. Immunoassays can
have false-positive and false-negative results, and unexpected negative results must have
confirmatory testing for verification (SOR C).
The immunoassay for benzodiazepines detects the metabolite nordiazepam only, which is a
metabolite of diazepam, oxazepam, and temazepam but not of alprazolam, lorazepam, or
clonazepam. This negative immunoassay screening test would require confirmatory testing for
alprazolam. A repeat immunoassay for benzodiazepines would likely show the same negative
result and would not change decision-making. The immunoassay for opioids detects only
nonsynthetic opioids such as morphine and codeine, and a positive immunoassay for codeine
would be expected in this case. It would be inappropriate to suspect drug diversion, or to stop
prescribing alprazolam or controlled substances, based upon the negative immunoassay for
benzodiazepines in this patient
ABFM2017
Q1-A 24-year-old male complains of feeling on edge all of the time. For the past 2 years he has
had difficulty controlling his worrying about work, school, and relationships. He has had more
difficulty concentrating at work and school, is more irritable, and has difficulty staying asleep
all night. He drinks alcohol moderately and does not use drugs. You recommend regular
exercise and refer him to a therapist for cognitive-behavioral therapy to help manage his
symptoms.
Which one of the following would be first-line medical therapy for this patient?
A) Bupropion (Wellbutrin)
B) Fluoxetine (Prozac)
C) Lorazepam (Ativan)
D) Methylphenidate (Ritalin, Concerta)
E) Quetiapine (Seroquel)
ANSWER: B
Though symptoms of generalized anxiety disorder (GAD) overlap with other psychiatric and
medical conditions, the case presented is most consistent with GAD. SSRIs are first-line
therapy for GAD (SOR B). Benzodiazepines such as lorazepam can improve anxiety-related
symptoms, but due to the side effects and addiction potential they are recommended for short-
term use (SOR B). Bupropion is approved for the treatment of depression but is not used to
treat GAD. Quetiapine may be considered as second-line therapy for GAD (SOR B).
Methylphenidate is first-line therapy for attention-deficit/hyperactivity disorder but is not
indicated to treat GAD. Psychotherapy, especially cognitive-behavioral therapy, is also first-
line treatment for GAD (SOR A), and exercise can also improve symptoms (SOR B).

ABFM2014
Q2-A 32-year-old female comes to your office because she has had increasing difficulty
performing her daily tasks for the past 6 months. She says she worries excessively about routine
events every day and constantly feels tense, restless, jittery, “on edge,” irritable, and unable to
relax. She also reports that sometimes her heart pounds and races, her hands feel sweaty and
clammy, and her mouth feels like it is “full of cotton.” She reports difficulty concentrating and
falling asleep at night. A recent TSH level was normal. During your examination you note
frequent sighing, a fine tremor in her hands, bitten nails, and clammy hands. Based on her
symptoms and examination, which one of the following pharmacologic agents for managing her
condition is supported by the best available evidence?
A) Alprazolam (Xanax)
B) Buspirone
C) Hydroxyzine
D) Escitalopram (Lexapro)
E) Quetiapine (Seroquel).
ANSWER: D
This patient’s symptoms and examination meet the diagnostic criteria for generalized anxiety
disorder. Based on the best evidence, pharmacologic agents that are beneficial for treating
generalized anxiety disorder in adults include antidepressants such as imipramine, duloxetine,
paroxetine, sertraline, escitalopram, and venlafaxine. Buspirone and hydroxyzine are likely to
be beneficial but are supported by less convincing evidence and can have unpleasant adverse
effects. The antipsychotics and benzodiazepines must be evaluated in terms of trade-offs
between benefits and harms. Quetiapine is approved for use in adults as add-on treatment in
major depressive disorder for patients who do not have an adequate response to antidepressant
therapy alone, for acute depressive episodes in bipolar disorder, for acute manic or mixed
episodes in bipolar disorder either alone or with lithium or divalproex, for long-term treatment
of bipolar disorder with lithium or divalproex, and for schizophrenia
Headache
ABFM 2020
193. A 39-year-old female presents to your office for evaluation of a left-sided headache. She
notes pain in the temporal region and inferior to the zygoma. The pain is constant and
dullbutworsens with chewing. There is no history of recent trauma. Her past medical history
includes fibromyalgia and her only medication is amitriptyline, 25 mg at bedtime. Her vital
signs are within normal limits and she is afebrile. On examination you note tenderness to
palpation over the temporalis region but no mass orcord.There is no swelling or edema of the
head,face,or neck.
Which one of the following is the most likely diagnosis?
1. Giant cell arteritis
2. A salivary stone
C. Sinusitis
4. Temporomandibular disorder
5. Trigeminal neuralgia
ANSWER: D
Approximately 10%–15% of adults will experience a temporomandibular disorder (TMD).
TMD represents a spectrum of illnesses frequently seen and readily treatable by family
physicians. This multifactorial disorder is consistently associated with other pain conditions
such as fibromyalgia, as in this case. Patients typically present with facial pain, jaw pain,
headache, or ear pain. The symptoms are generally associated with jaw movement such as
chewing. Physical examination findings can be broad but often include pain with palpation of
the temporomandibular joint, or pain and/or spasm of the muscles of mastication. Giant cell
arteritis should be included in the differential diagnosis but is typically seen in patients over
age 50 and is often associated with other findings such as jaw claudication, visual symptoms,
and palpable abnormalities over the temporal artery. Salivary stones can involve pain in a
similar region, such as the parotid gland, but this is often an intermittent pain triggered only by
eating. Tenderness or swelling over the gland itself may be seen. Sinusitis would usually
present with nasal congestion, maxillary sinus tenderness, mucus, and fever. While trigeminal
neuralgia may have a similar distribution of pain, this is typically reported as a brief attack of
intense, sharp pain often produced by specific stimuli.
A 34-year-old female at 32 weeks gestation presents with a right-sided, pounding headache
that began 8 hours ago and is similar to headaches she has had in the past. She is sensitive to
light and sound, and has vomited several times since the onset of pain. She has taken
acetaminophen without relief. She takes prenatal vitamins but no other routine medications.
On examination her blood pressure is normal.
Which one of the following would be the most appropriate treatment for this patient?
A. Dihydroergotamine
B. Metoclopramide (Reglan)
C. Naproxen
D. Oxycodone (OxyContin)
E. Sumatriptan (Imitrex)
ANSWER: B
Metoclopramide and acetaminophen are the only two medications considered safe for abortive
migraine treatment during pregnancy (SOR B). The dopamine antagonist antiemetics are
considered second-line abortive treatments in the general population. Dihydroergotamine
should not be used during pregnancy due to its oxytocic properties and the potential risk of
intrauterine growth restriction with its use. NSAIDs are not considered safe during pregnancy,
particularly in the first and third trimesters. Opioids are only moderately useful for migraine
treatment and should be avoided during pregnancy due to their abuse potential. Triptans are
generally considered safe during the first trimester but not in the second and third trimesters.
Their use has been associated with uterine atony, increased risk of bleeding during delivery,
and increased risk of preterm birth.

ABFM2019
Q1-A patient comes to your outpatient clinic with a persistent migraine that she has been
unable to treat effectively at home. The symptoms began several hours ago and are typical for
her. She has already tried her usual treatments of ibuprofen, 800 mg, and rizatriptan (Maxalt),
10 mg, but they have not provided any relief. She took a second dose of rizatriptan 2 hours later
without benefit. She is in significant pain, which is causing mild nausea, and she has
photophobia and phonophobia.
Which one of the following would be most appropriate at this point?
A. Oral butalbital/acetaminophen/caffeine (Fioricet)
B. Oral ergotamine/caffeine (Cafergot)
C. Subcutaneous sumatriptan (Imitrex)
D. Intramuscular morphine
E. Intramuscular prochlorperazine
ANSWER: E
Multiple studies have determined that parenteral antiemetics have benefits for the treatment of
acute migraine beyond their effect on nausea. Most outpatient clinics do not have the ability to
administer intravenous metoclopramide, which is the preferred treatment. However, most
clinics do have the ability to administer intramuscular prochlorperazine or promethazine. Due
to concerns about oversedation, misuse, and rebound, treatment with parenteral opiates is
discouraged but may be an option if other treatments fail. Oral
butalbital/acetaminophen/caffeine and oral ergotamine/caffeine have less evidence of success
in the treatment of acute migraine. Sumatriptan is contraindicated within 24 hours of the use
of rizatriptan.

ABFM2017
Q2-44-A 26-year-old female has a 4-month history of continuous right-sided headache. The
headache is associated with tearing and nasal congestion only on the right, and has not
responded to over-the-counter analgesics. The patient went to the emergency department a few
nights ago because of the pain, and CT of the head at that visit was normal. On examination
you note conjunctival injection on the right. Findings are otherwise normal.
Which one of the following would be the most appropriate treatment at this time?
A) Sumatriptan (Imitrex)
B) Amitriptyline
C) Indomethacin
D) Topiramate (Topamax)
ANSWER: C
There are several types of chronic headaches, and they often respond to different treatments.
Migraine is very prevalent and is characterized by headaches that are periodic, often unilateral,
and frequently pulsatile. Migraine is familial and typically starts in childhood, adolescence, or
young adulthood, and the headaches decrease in frequency over time. Some are associated with
aura, which causes visual disturbances. In mild cases, over-the-counter medications may
control symptoms. For most patients, however, treatment to control the attack can include
triptans such as sumatriptan, and/or ergot alkaloids such as ergotamine. Treatment to prevent
attacks may also be appropriate, and could include a -blocker, antiepileptic drugs, or
amitriptyline.
Tension headaches are usually bilateral and are typically described as dull or aching, but patients
often describe tightness or pressure. They are not associated with symptoms such as throbbing,
nausea, or photophobia. Tension headaches are more frequent than migraine but patients often
treat them at home without seeking medical treatment. Frequent or persistent tension
headaches can be treated with several drugs used for anxiety or depression, including
amitriptyline.
Stronger analgesics and ergotamine are not helpful. Cluster headache is another type of
chronic headache. This occurs most frequently in adult males, and often occurs over a period
which may extend over many weeks, with repeated episodes or clusters. It most often occurs at
night, and may recur several times during the night. The headache is unilateral and is
associated with orbital pain and vasomotor phenomenon such as blocked nasal passages,
rhinorrhea, conjunctival injection, and miosis. The headache can be treated with inhalation of
100% oxygen, and the headache cycle can be terminated with verapamil. Ergotamine or
sumatriptan can be used at night to prevent attacks. There are also variants of cluster
headaches, including chronic paroxysmal hemicrania, which resembles cluster headache but
has some important differences. Like cluster headaches, these headaches are unilateral and
accompanied by conjunctival hyperemia and rhinorrhea. However, these headaches are more
frequent in women, and the paroxysms occur many times each day. This type of headache falls
into a group of headaches that have been labeled indomethacin-responsive headaches because
they respond dramatically to indomethacin.

Q3-A 37-year-old male complains of severe headaches that typically involve his right eye, and
often cause the eye to tear. The headaches occur at about the same time each day and recur for
several days in a row before remitting. He reports that he is currently experiencing a third
episode of these headaches.
Which one of the following therapies will help prevent future recurrences of this patient’s
headache?
A) Oxygen
B) Sumatriptan (Imitrex)
C) Lithium
D) Verapamil (Calan, Verelan).

ANSWER: D
This patient suffers from cluster headaches. Both verapamil and lithium are the mainstays of
treatment for chronic cluster headaches, but of the options listed, only verapamil is indicated for
the prevention of cluster headaches, and it is actually the first-line prophylactic agent (SOR A).
Oxygen and sumatriptan are first-line abortive therapies for cluster headaches (SOR A).
Heart failure

Pretest 2019
Q1-You have diagnosed a 66-year-old woman with heart failure. She has a history of
hypertension, but has never had heart failure before. Which of the following tests is routinely
indicated in the initial evaluation of a person with a new diagnosis of heart failure?
a.Echocardiogram
b.Holter monitor
c.Left heart catheterization d. Treadmill stress test
d.Pharmacologic stress test

The answer is a.
Routine laboratory testing in a person with the new diagnosis of heart failure includes an
electrocardiogram, a CBC, a urinalysis, serum creatinine, potassium and albumin levels, and
thyroid function studies. An echocardiogram is imperative to help identify structural
abnormalities of the heart and to measure the ejection fraction (EF), thereby differentiating
between heart failure with reduced ejection fraction (HFrEF) and heart failure with preserved
ejection fraction (HFpEF) as well as between right and left ventricular heart failure. Holter
monitoring is not routinely warranted, as it would not identify a cause for heart failure, but
would be used to identify an arrhythmia. Catheterization or stress testing may be important if
ischemia or ischemic cardiomyopathy is identified as a cause, but is not a routine initial test.

Q2-According to the American College of Cardiology/American Heart Association/Heart


Failure Society of America (ACC/AHA/HFSA) heart failure guidelines, once this patient’s
symptoms have been controlled with diuresis, what is the most appropriate first-line therapy for
her HFrEF?
a.ACE-inhibitors
b.β-Blockers
c.Calcium channel blockers
d.Nitrates
e.Hydralazine
The answer is a.
The ACC/AHA/HFSA guidelines recommend therapy based on their four categories of heart
failure. Stage A includes patients at risk for heart failure but without structural heart disease or
symptoms. Stage B includes patients with structural heart disease (atherosclerosis, valvular
disease) without signs or symptoms of heart failure. Patients in stage C heart failure have
current or prior symptoms of heart failure and structural heart disease, while stage D patients
have refractory heart failure. In contrast, the New York Heart Association (NYHA) functional
classification categorizes patients based on the degree of symptoms they currently have. All
NYHA classes fall into the ACC/AHA/HFSA stages C or D. Class I patients have no
limitation of activity. Class II patients have slight limitations, are comfortable at rest, but have
fatigue, palpitations, dyspnea, or angina with ordinary activity. Class III patients are also
comfortable at rest, but less-than- ordinary activity causes symptoms. Class IV patients have
symptoms at rest and increased symptoms with even minor activity. For this question, many
clinical trials have shown that ACE-inhibitors decrease symptoms, improve quality of life,
decrease hospitalizations, and reduce mortality in patients with Stage C (NYHA class II to IV)
HFrEF. In addition, they slow the progression to heart failure among asymptomatic patients
with HFrEF. All patients with heart failure should be prescribed an ACE-inhibitor unless they
have a contraindication. β-Blockers are helpful, but not necessarily as a first-line agent, and
should be reserved for patients who are euvolemic. Nitrates and hydralazine can be used in
patients who do not tolerate ACE-inhibitors, although they lack evidence to show that they
improve prognosis. ARBs are generally utilized in patients who cannot tolerate ACE-
inhibitors. Some calcium channel blockers (nifedipine, diltiazem, and nicardipine) may
worsen systolic dysfunction.

Q3-This same patient returns to your office for a follow-up visit 3 months later. She is
following her low-sodium diet and taking her medications daily without side effects. She
appears euvolemic and vitals today are: heart rate 88 beats/min, blood pressure 130/83 mm
Hg, respiratory rate 14 breaths/min, and SpO2 92%.
However, she continues to have considerable dyspnea on exertion and her repeat
echocardiogram shows an unchanged EF. What is the next best course of action?

a. Admit her to the hospital for acute diuresis and oxygen


b. Increase the diuretic
c.Add an angiotensin II-receptor blocker (ARB)
d.Add a β-blocker
e.Add metolazone
The answer is d.
This patient is euvolemic and has normal oxygen saturation and does not have any indication
for acute, inpatient treatment. In patients who continue to be symptomatic with an EF less than
35% and are already euvolemic and on an ACE or ARB, the next step is the addition of a β-
blocker. Studies have shown that three β-blockers (bisoprolol, metoprolol, and carvedilol) can
reduce symptoms, improve quality of life, and reduce mortality. The concomitant use of ACEs
and ARBs is not recommended due to the increased risk of hyperkalemia. Metolazone is useful
in patients with refractory edema, but this is not the case with this patient.

Q4- A 62-year-old woman comes to your office complaining of dyspnea. She has a history of
COPD, hypertension, and diabetes. She also smokes and drinks heavily. Her evaluation
reveals that she has HFrEF. Which of the following interventions will lead to functional
improvement in this patient?

a. Optimizing the treatment of her COPD


b. Optimizing the treatment of her hypertension
c. Optimizing her glycemic control
d. Discontinuing cigarette smoking
e. Discontinuing alcohol use

The answer is e.
Many noncardiac comorbid conditions may affect the proper diagnosis and clinical course of
heart failure. All of the interventions in this question should be done, but only discontinuing
alcohol use has actually been shown to improve function significantly. Optimally treating
COPD is important, as exacerbations from heart failure are often difficult to distinguish from
COPD exacerbations.
Optimally treating diabetes and hypertension will minimize the negative effects of these
conditions on the heart, but will not improve damage already done.
Cigarette smoking should be discontinued, but generally does not lead to functional
improvement. Those with alcoholic cardiomyopathy actually see improvement of the left
ventricular function with abstinence.
Q5-You have been treating a 68-year-old man suffering from chronic left HFrEF with
furosemide (Lasix), a β-blocker, and an ACE-inhibitor. Despite this therapy, he continues with
refractory edema. In his baseline state, he is comfortable at rest, but experiences some
symptoms of heart failure with ordinary activity.
Which of the following would be the best diuretic to add?

a.Hydrochlorothiazide
b.Triamterene
c.Hydrochlorothiazide and triamterene combined (Dyazide, Maxzide)
d.Metolazone (Zaroxolyn)
e.Spironolactone (Aldactone)
The answer is d.
Some patients have difficulty maintaining optimal fluid balance, and a second diuretic is
needed. In this case, adding metolazone can significantly increase diuresis in the outpatient
treatment of heart failure with volume overload.
Prolonged therapy should be avoided; it is often only needed 1 or 2 times a week.
Hydrochlorothiazide would not enhance diuresis, nor would triamterene. Spironolactone can
be used, but is usually only considered for NYHA class III or IV patients or those with a serum
potassium level less than 5.0 mmol/L.
ABFM 2020
A 68-year-old male with a history of diabetes mellitus, hypertension, coronary artery disease,
and heart failure with reduced ejection fraction sees you for a follow-up visit 2 weeks after
cardiac catheterization and placement of two drug-eluting stents. He tells you that shortness of
breath slightly limits his physical activity. Before the procedure an echocardiogram revealed
apical hypokinesis and an estimated left ventricular ejection fraction of 25%. His current
medications include carvedilol (Coreg), atorvastatin (Lipitor), aspirin, clopidogrel (Plavix),
spironolactone (Aldactone), sacubitril/valsartan (Entresto), metformin (Glucophage), and
empagliflozin (Jardiance). On examination the patient is euvolemic, his pulse rate is 58
beats/min, his blood pressure is 112/60 mm Hg, and his hemoglobin A1c is 7.2%.

Which one of the following would be the most appropriate management of his heart disease?
A. No medication changes and repeat echocardiography in 2 months
B. Replacing empagliflozin with sitagliptin (Januvia)
C. Replacing sacubitril/valsartan with lisinopril (Prinivil, Zestril)
D. Immediate placement of an implantable cardiac defibrillator
E. Referral for left atrial appendage closure

ANSWER: A
This patient presents with ischemic cardiomyopathy associated with heart failure with reduced
ejection fraction and New York Heart Association (NYHA) class II symptoms. He is taking
appropriate medical therapy, including dual antiplatelet agents, a high-intensity statin, beta-
blocker, an aldosterone antagonist, and an angiotensin receptor–neprilysin inhibitor.
Sacubitril/valsartan is superior to an ACE inhibitor in patients such as this (SOR A). SGLT2
inhibitors such as empagliflozin, unlike DPP-4 inhibitors such as sitagliptin, have been
associated with improved symptoms and lower rates of cardiovascular death in patients with
heart failure (SOR A).
An implantable cardiac defibrillator (ICD) is indicated to decrease sudden cardiac death in a
patient with heart failure treated with guideline-directed medical therapy who has a left
ventricular ejection fraction (LVEF) 30% with NYHA class I symptoms or an LVEF 35%
with NYHA class II–III symptoms (SOR A). However, the patient must be at least 40 days out
from a myocardial infarction and at least 90 days out from revascularization. The patient in
this case should have repeat echocardiography to reassess his LVEF before he is referred for
ICD placement.

89. An EKG performed on a patient with a history of heart failure and an ejection fraction of
30% reveals sinus rhythm with a heart rate of 55 beats/min and new left bundle branch block
with a QRS interval of 160 msec. The patient is taking a beta-blocker but has a history of
acetylcholinesterase inhibitor–induced angioedema.
Which one of the following would be most appropriate at this point?
A. Amiodarone
B. Ivabradine (Corlanor)
C. Sacubitril/valsartan (Entresto)
D. Sinoatrial node ablation
E. Cardiac resynchronization therapy

ANSWER: E

Cardiac resynchronization therapy is strongly recommended for patients with symptomatic


heart failure, an ejection fraction <35%, and a left bundle branch block with a QRS interval
>150 msec. Amiodarone is an antiarrhythmic and would not be indicated for this patient.
Ivabradine, a sinoatrial node modulator, is used in patients with symptomatic heart failure as
an add-on therapy to decrease the heart rate. It is indicated in patients with a heart rate >70
beats/min despite -blockade. Sacubitril/valsartan is contraindicated in patients with a history
of angioedema. Sinoatrial node ablation is indicated for some patients with sinus node
dysfunction.
107. A 72-year-old female has a known history of aortic stenosis. Which one of the following
symptoms or physical findings would indicate a need for urgent repeat echocardiography and
cardiology referral?
A. Headache
B. Palpitations
C. Presyncope
D. Edema of both lower extremities
E. A grade 2/6 systolic ejection murmur radiating to the aorta

ANSWER: C
The three cardinal symptoms of aortic stenosis are angina, dyspnea, and presyncope or
syncope. Once these events occur, the natural history of the disease changes dramatically. The
risk for death increases from <1% per year to 2% per month, such that 75% of symptomatic
patients die within 3 years unless they receive a valve replacement. Headache, palpitations,
edema of the lower extremities, and a grade 2/6 ejection systolic murmur radiating to the aorta
are not indicative of severe aortic stenosis.
A 57-year-old male with a history of heart failure sees you for follow-up. He describes
symptoms of mild dyspnea on exertion with ordinary activities such as shopping or yard work.
An echocardiogram shows an ejection fraction of 37%.
According to the New York Heart Association criteria, this patient’s heart failure would be
classified as which one of the following?
A. Class I
B. Class II
C. Class III
D. Class IV

ANSWER: B
The appropriate classification of heart failure is important for monitoring the disease. The
most common currently used system is the New York Heart Association (NYHA) functional
classification. In this system, class I is defined as heart disease in a patient with no symptoms
and no limitations of physical activity. Patients with class II heart failure have mild symptoms
with normal physical activity. Class III heart failure refers to significant limitations of activity,
including symptoms with less than normal activities. Patients with class IV heart failure have
symptoms at rest and are unable to carry on activity without discomfort.

ABFM2019
Q1- A 67-year-old male presents with a 3-month history of a worsening cough, shortness of
breath, and paroxysmal nocturnal dyspnea. On examination he has a heart rate of 78
beats/min, a blood pressure of 138/88 mm Hg, a respiratory rate of 18/min, and an oxygen
saturation of 94% on room air. A physical examination reveals jugular venous distention,
bibasilar crackles, 2+ pitting ankle edema, and a displaced apical impulse. An echocardiogram
shows normal valvular structure and function with a left ventricular ejection fraction of 55%
and diastolic dysfunction.

Which one of the following would be most appropriate for this patient?
A. Furosemide (Lasix), 20 mg daily
B. Isosorbide mononitrate extended release, 60 mg daily
C. Lisinopril (Prinivil, Zestril), 20 mg daily
D. Losartan (Cozaar), 50 mg daily
E. Sacubitril/valsartan (Entresto), 24/26 mg twice daily.
ANSWER: A
This patient has diastolic heart failure, also called heart failure with preserved ejection fraction
(HFpEF). Patients who have HFpEF with active fluid overload should be treated with diuretics
such as furosemide (SOR B). If concomitant hypertension is present along with HFpEF, the
patient’s blood pressure should be treated according to accepted evidence-based hypertension
guidelines (SOR C). Although this patient’s blood pressure is elevated, a diagnosis of
hypertension has not been confirmed, so antihypertensives are not warranted at this time.

Q2- A 64-year-old female with a long-standing history of COPD presents with a 1-day history
of dizziness and worsening shortness of breath. An EKG is shown below.

Which one of the following is the most likely cause of her symptoms?
A. Atrial fibrillation
B. Multifocal atrial tachycardia
C. Myocardial infarction
D. Pulmonary embolism.

ANSWER: A
Atrial fibrillation is characterized by an irregularly irregular rhythm, without any discernible P
waves. In multifocal atrial tachycardia the P waves show varying morphology, and this rhythm
is typically seen in patients with underlying lung disease, especially COPD. Acute ST-
segment elevation myocardial infarction is characterized by the presence of hyperacute ST-
segment and T-wave changes in contiguous EKG leads, accompanied by reciprocal changes.
Pulmonary embolism is characterized by EKG findings of sinus tachycardia (a heart rate 100
beats/min), nonspecific ST-T changes, and new-onset right bundle branch block (SOR C).
ABFM2018
A 32-year-old female requests a physical examination prior to participating in an adult soccer
league. Her blood pressure is 118/70 mm Hg and her pulse rate is 68 beats/min. The
examination is otherwise normal except for a systolic murmur that intensifies with Valsalva
maneuvers. She says that she has recently been experiencing mild exertional dyspnea and
moderate chest pain. The chest pain has been atypical and is not necessarily related to
exertion. Echocardiography reveals hypertrophic cardiomyopathy. In addition to referring the
patient to a cardiologist, you recommended initiating therapy with
A) amiodarone (Cordarone)
B) amlodipine (Norvasc)
C) furosemide (Lasix)
D) lisinopril (Prinivil, Zestril)
E) metoprolol

ANSWER: E
Hypertrophic cardiomyopathy is the most common primary cardiomyopathy, with a
prevalence of 1:500 persons. Many patients with hypertrophic cardiomyopathy are
asymptomatic and are diagnosed during family screening, by auscultation of a heart murmur,
or incidentally after an abnormal result on electrocardiography. On examination physicians
may hear a systolic murmur that increases in intensity during Valsalva maneuvers. The main
goals of therapy are to decrease exertional dyspnea and chest pain and prevent sudden cardiac
death. -Blockers are the initial therapy for patients with symptomatic hypertrophic
cardiomyopathy. Nondihydropyridine calcium channel blockers such as verapamil can be used
if -blockers are not well tolerated.

A 75-year-old white male presents to your office following hospitalization for an episode of
heart failure. His edema has resolved but he still becomes symptomatic with minor exertion
such as walking less than a block. A recent chest radiograph shows cardiomegaly, and
echocardiography reveals an ejection fraction of 25%. He is currently taking furosemide
(Lasix), 20 mg daily; carvedilol (Coreg), 25 mg twice daily; and lisinopril (Prinivil, Zestril),
20 mg daily. His vital signs include a pulse rate of 60 beats/min, a blood pressure of 110/70
mm Hg, a respiratory rate of 18/min, and a temperature of 37.0°C (98.6°F). No crackles or
hepatojugular reflux are noted on auscultation.
Which one of the following would improve this patient’s symptoms and decrease his
mortality risk?

A) Digoxin
B) Hydralazine and isosorbide dinitrate (BiDil)
C) Hydrochlorothiazide
D) Spironolactone (Aldactone)
ANSWER: D
For patients with left ventricular systolic dysfunction, clinical trials have demonstrated that
ACE inhibitors,-blockers, angiotensin receptor blockers, and aldosterone antagonists decrease
hospitalizations and all-cause mortality. In African-American patients, all-cause mortality and
hospitalizations have been reduced by hydralazine and isosorbide dinitrate. Aldosterone
antagonists such as spironolactone, as well as -blockers, decrease mortality in patients with
symptomatic heart failure (SOR A). Digoxin improves symptoms of heart failure but does not
improve mortality.

An 89-year-old female with a history of hypertension and glaucoma is brought to the


emergency department by her family with shortness of breath. She has been trying to get her
home ready for sale prior to moving into an assisted living facility. She says that she has not
been sleeping well for weeks because she is worried about the move.
On admission the patient has a blood pressure of 140/92 mm Hg, a pulse rate of 86
beats/min, a respiratory rate of 26/min, a temperature of 36.6°C (97.9°F), and an oxygen
saturation of 95% on room air. A physical examination is normal other than faint basilar
crackles. A chest radiograph shows a slightly prominent cardiac silhouette, peribronchial
cuffing, and coarse perihilar lung markings. An EKG reveals a normal sinus rhythm with
global T-wave inversion of the precordial and limb leads. Her troponin I peaks at 0.953
ng/mL (N 0.000–0.780). Echocardiography reveals a normal size right ventricle with
moderate right ventricular hypokinesis, left ventricular apical ballooning, a left ventricular
ejection fraction estimated at 30%, and a moderately increased pulmonary artery pressure
estimated at 43 mm Hg. A radionuclide myocardial perfusion imaging study is normal.
Which one of the following is the most likely diagnosis?
A) Acute coronary syndrome
B) Acute pericarditis
C) Cardiac amyloidosis
D) Takotsubo cardiomyopathy
E) Viral myocarditis
ANSWER: D
Takotsubo cardiomyopathy (TTC) is also known as apical ballooning syndrome and stress-
induced cardiomyopathy. It generally occurs in postmenopausal women with a mean age of
62–76 years. The clinical presentation is similar to that of acute coronary syndrome.
Evaluation with an EKG, cardiac biomarkers, and imaging is needed to differentiate between
these two conditions. This patient presents with classic apical and midsegment left ventricular
hypokinesis, or apical ballooning, and a new T-wave inversion with modest elevations in
cardiac troponin. While she has an identifiable characteristic emotional stressor, up to one-
third of patients with TTC do not have an identifiable stressor.
In this scenario, a negative myocardial perfusion scan makes coronary artery disease or acute
coronary syndrome unlikely. Patients with viral myocarditis typically present with fever,
myalgia, and signs and symptoms of heart failure following a viral syndrome. Cardiac
amyloidosis is a restrictive cardiomyopathy that is typically associated with thickened walls of
both ventricles and markedly dilated atria. Patients with acute pericarditis present with chest
pain, a pericardial friction rub on examination, an ST-segment elevation on EKG, and a
pericardial effusion on echocardiography.

ABFM2016
156. An 84-year-old female sees you for a follow-up visit for cardiovascular issues, including
a previous myocardial infarction. Her current medication regimen consists of Lisinopril
(Prinivil, Zestril) and carvedilol (Coreg). While she is not dyspneic at rest, she now becomes
short of breath after walking half a block. On examination her blood pressure is 122/74 mm
Hg, pulse rate 72 beats/min, respirations 18/min, and oxygen saturation 97% on room air. She
has no jugular venous distention and her lungs are clear. No edema is noted. A recent
echocardiogram showed a left ventricular ejection fraction of 30%.
Adding which one of the following would help to decrease both mortality and the risk of
hospitalization?
A) Digoxin
B) Furosemide
C) Isosorbide dinitrate/hydralazine (BiDil)
D) Losartan (Cozaar)
E) Spironolactone (Aldactone)
ANSWER: E
Systolic heart failure, or heart failure with a reduced left ventricular ejection fraction (<40%),
should be managed with an ACE inhibitor and a β-blocker. If there is volume overload, a
diuretic should be added. For those who cannot tolerate an ACE inhibitor, an angiotensin
receptor blocker can be used. However, an ACE inhibitor should not be given with an
angiotensin receptor blocker, as this combination increases mortality.
Digoxin will decrease symptoms and lessen the chance of hospitalization, but mortality is not
reduced. Mortality and hospitalization can be reduced by the addition of an aldosterone
antagonist such as spironolactone. In African-American patients, using a combination of
hydralazine and isosorbide dinitrate improves both symptoms and mortality for those with
New York Heart Association class III or IV heart failure with a reduced ejection fraction.

ABFM2015

Q3-A 66-year-old male who was hospitalized 2 months ago for an episode of heart failure sees
you For follow-up. He complains of pain in his chest and on examination you note tenderness
and a slight fullness deep to his nipple bilaterally.
Which one of the following drugs on his medication reconciliation list is most likely to cause
this
type of discomfort?
A) Digoxin (Lanoxin)
B) Enalapril (Vasotec)
C) Eplerenone (Inspra)
D) Hydralazine
E) Spironolactone (Aldactone)
ANSWER: E
Spironolactone, an aldosterone antagonist, can bind to androgen and progesterone receptors, in
addition to the mineralocorticoid receptors, resulting in breast tenderness and gynecomastia.
Eplerenone, another aldosterone antagonist, has greater specificity for the mineralocorticoid
receptors and is therefore less likely to cause breast tenderness and gynecomastia than
spironolactone. While there have been case reports of gynecomastia with ACE inhibitors and
digoxin, it is noted to be rare. The side effect profile of hydralazine does not include
gynecomastia

ABFM2014
Q4-A 53-year-old male complains of fatigue, dyspnea, and orthopnea. Which one of the
following would have the highest specificity for heart failure?.
A) Ankle edema
B) A third heart sound (S3 gallop)
C) Crackles
D) Cardiomegaly on a chest radiograph
E) Elevated BNP
ANSWER: B
Among the constellation of history and physical findings that can be found in patients with
heart failure, none provides a proof-positive diagnosis alone, as most are found in other
disease states as well. Each of the options listed raises the possibility of heart failure but the
only one that has a specificity >90% is the third heart sound, which is 99% specific for the
diagnosis of heart failure. Other findings with >90% sensitivity include a displaced point of
maximal impulse, interstitial edema or venous congestion on a chest radiograph, jugular vein
distention, and hepatojugular reflux. The other options listed here have specificities for heart
failure that fall within the range of 65%–80%.

Q5-A 72-year-old white female is admitted to the hospital with her first episode of acute heart
failure. She has a history of hypertension treated with a thiazide diuretic. An echocardiogram
reveals no evidence of valvular disease and no segmental wall motion abnormalities. Left
ventricular hypertrophy is noted, and her ejection fraction is 55%. Her pulse rate is 72
beats/min.
The most likely cause of her heart failure is
A) systolic dysfunction
B) diastolic dysfunction
C) hypertrophic cardiomyopathy
D) high-output failure
ANSWER: B
Diastolic dysfunction is now recognized as an important cause of heart failure. It is due to left
ventricular hypertrophy as a response to chronic systolic hypertension. The ventricle becomes
stiff and unable to relax or fill adequately, thus limiting its forward output. The typical patient
is an elderly person who has systolic hypertension, left ventricular hypertrophy, and a normal
ejection fraction (50%–55 ).
Q6- An 84-year-old female sees you for a follow-up visit for cardiovascular issues, including a
previous myocardial infarction. Her current medication regimen consists of Lisinopril
(Prinivil, Zestril) and carvedilol (Coreg). While she is not dyspneic at rest, she now becomes
short of breath after walking half a block. On examination her blood pressure is 122/74 mm
Hg, pulse rate 72 beats/min, respirations 18/min, and oxygen saturation 97% on room air. She
has no jugular venous distention and her lungs are clear. No edema is noted. A recent
echocardiogram showed a left ventricular ejection fraction of 30%.
Adding which one of the following would help to decrease both mortality and the risk of
hospitalization?

A) Digoxin
B) Furosemide
C) Isosorbide dinitrate/hydralazine (BiDil)
D) Losartan (Cozaar)
E) Spironolactone (Aldactone)

ANSWER: E
Systolic heart failure, or heart failure with a reduced left ventricular ejection fraction (<40%),
should be managed with an ACE inhibitor and a β-blocker. If there is volume overload, a
diuretic should be added. For those who cannot tolerate an ACE inhibitor, an angiotensin
receptor blocker can be used. However, an ACE inhibitor should not be given with an
angiotensin receptor blocker, as this combination increases mortality.
Digoxin will decrease symptoms and lessen the chance of hospitalization, but mortality is not
reduced. Mortality and hospitalization can be reduced by the addition of an aldosterone
antagonist such as spironolactone. In African-American patients, using a combination of
hydralazine and isosorbide dinitrate improves both symptoms and mortality for those with
New York Heart Association class III or IV heart failure with a reduced ejection fraction.
Hypertension
PRETEST2019
Q1-You are evaluating a 35-year-old man whose systolic blood pressure (SBP) has been
between 140 and 160 mm Hg on three separate occasions in the office, despite lifestyle
modifications. He asks you why it is important to follow-up on these findings. According to
observational studies, which of the following is true?

a.The risk of CVD increases in a linear fashion between SBP between 140 and 180 mm Hg.
b.An SBP 20 mm Hg higher than normal is associated with a doubling in the risk of death from
stroke.
c.An increased risk of CVD is associated with higher SBP at all ages.
d.The relative risk of CVD is lower for this patient than it would be for an elderly person with
the same blood pressure.
e.This patient is at greater risk for MI and stroke, but not for heart failure or peripheral artery
disease (PAD).

The answer is b.
Many observational studies have demonstrated graded associations between higher SBP and
increased CVD risk. Risks for CVD increase in a log-linear fashion with SBP levels less than
115 mm Hg to more than 180 mm Hg and from diastolic blood pressure (DBP) levels less than
75 mm Hg to more than 105 mm Hg. In addition, blood
pressure 20 mm Hg higher than normal is associated with a doubling in the risk of death from
stroke, heart disease, or other vascular disease. Studies have also shown that higher SBP is
associated with increased risk of angina, MI, HF, stroke, PAD, and abdominal aortic
aneurysm. The increased risk of CVD associated with higher SBP has been reported across a
broad age spectrum, from 30 years to more than 80 years of age. Interestingly, the relative risk
of incident CVD associated with higher SBP is smaller at older ages.

For questions 2 to 6, assume you and your staff have appropriately measured blood pressures
of several people in your office.
Which of the following categories would these blood pressures fall into? Use the following
answer key:
a. Normal blood pressure
b. Elevated blood pressure
c. Stage I hypertension
d. Stage II hypertension
e. Hypertensive crisis

Q2-A 24-year-old woman with these three consecutive independent readings: 118/78, 126/84,
and 112/68.
The answer is a.

Q3-A 36-year-old African-American man with these three consecutive independent readings:
124/86, 130/90, and 122/84.
The answer is c.

Q4- A 52-year-old obese woman with these three consecutive independent readings: 132/92,
129/90, and 134/94.
The answer is d.

Q5-A 43-year-old man with these three consecutive independent readings: 122/82, 124/86, and
118/84.
The answer is c.

Q6- A 33-year-old man with a family history of hypertension with these three consecutive
independent readings: 116/72, 122/68, and 128/78.
The answer is b.

Categorizing blood pressure is important to help determine treatment options. The stage is
determined after appropriately taking two or more blood pressure readings on two or more
occasions and averaging those readings. A table outlining the classifications is below. If the
SBP and DBP are in two separate categories, the blood pressure should be designated in the
higher category. The newer categorization eliminates the category of “prehypertension
Q7- You have diagnosed a 42-year-old patient with hypertension. He is 5 ft 9 in tall, weighs
230 lb, and admits to poor eating habits, drinking four alcoholic beverages daily, and no
regular exercise. Which of the following lifestyle modifications, if instituted, will result in the
largest SBP reduction?

a.Moderate alcohol consumption to no more than two drinks daily


b.Engage in physical activity for 30 minutes per day, most days of the week
c.Reduce dietary sodium intake to no more than 100 mEq/L per day
d.Adopt a Dietary Approaches to Stop Hypertension (DASH) eating plan (a diet rich in fruits,
vegetables, and low-fat dairy products with a reduced saturated and total fat content)
e.Lose 8 to 10 lbs

The answer is d.
Lifestyle modifications can help to manage hypertension. Weight reduction is beneficial, and
SBP can fall up to 4 to 5 mm Hg for each 4 kg of weight lost. A DASH diet can lower blood
pressure between 8 and 14 mm Hg. Dietary sodium reduction, increased exercise, and
moderation of alcohol can be expected to lower SBP between 3 and 6 mm Hg.

Q8- You have seen a 36-year-old African-American man with elevated blood pressure. On
one occasion, his blood pressure was 128/84 mm Hg, and on a second occasion, his blood
pressure was 138/88 mm Hg. You have encouraged lifestyle modifications including weight
loss using exercise and dietary changes. Despite some modest weight loss, at his current visit,
his blood pressure is 128/89 mm Hg. Which of the following is the best treatment strategy at
this point?

a. Use a thiazide diuretic


b. Use an ACE-inhibitor
c. Use an angiotensin receptor blocker
d. Use a β-blocker
e. Use a two-drug combination of medications
The answer is a.
The patient in this question has stage I hypertension (SBP between 130and 139 mm Hg, or
diastolic blood pressure between 80 and 89 mm Hg). Since lifestyle modifications have not
helped, the next step is to institute drug therapy. The most effective first-line treatment for
preventing the occurrence of CVD is either an ACE-inhibitor, an ARB, a calcium channel
blocker, or a thiazide diuretic. In the African-American hypertensive population, a calcium
channel blocker or a thiazide diuretic is recommended.

Q9-You have just diagnosed a 35-year-old man with hypertension. He is otherwise healthy
and has no complaints. Which of the following is indicated in the initial evaluation?

a. Uric acid level


b. Resting electrocardiogram
c. Stress test
d. Echocardiogram
e. Renal ultrasound

The answer is b.
Baseline laboratory screening is important to assess for end-organ damage and
identify patients at high risk for cardiovascular complications. The routine tests for a newly
diagnosed hypertensive patient include hemoglobin and hematocrit, sodium, potassium,
creatinine, fasting glucose, calcium, a fasting lipid profile, urinalysis, a TSH, and a resting
electrocardiogram. An echocardiogram and a uric acid level are optional. Other tests are not
indicated unless physical examination or history makes them likely to be positive.

Q10-You are treating a 61-year-old man for hypertension. He is not responding well to
combination therapy with a thiazide diuretic and a β-blocker. On physical examination, you
note an abdominal bruit. Which of the following tests is most likely to help you evaluate him
further?

a. CXR
b. MRI or magnetic resonance arteriography (MRA)
c. Urinary metanephrines and vanillylmandelic acid levels
d. Aortic CT scan
e. Echocardiogram

The answer is b.
The prevalence of renovascular disease as the cause in a person with hypertension alone is
around 5%. The patient described in the question has physical examination findings consistent
with renal artery stenosis. Imaging options include MRI/MRA, CT scan, and ultrasound. The
preferred method depends on capabilities of the institution. Urinary metanephrines and
vanillylmandelic acid levels would help rule out pheochromocytoma. A CXR would be helpful
if coarctation of the aorta were suspected. An aortic CT would help to or quantify an aortic
aneurysm, and an echocardiogram would help to identify left ventricular hypertrophy or
systolic dysfunction.

Q11- Despite lifestyle changes, a 37-year-old patient of yours still has stage I hypertension
with SBP more than 10 mm Hg above goal. She has no other medical concerns and no
abnormalities on physical examination or initial laboratory evaluation. Which of the following
medications is best as an initial first-line monotherapy?

a. A thiazide diuretic
b. An ACE-inhibitor
c. An ARB
d. A calcium channel blocker
e. A β-blocker

The answer is a.
2017 ACC/AHA guidelines recommend that low-dose diuretics are the most
effective first-line treatment for preventing the occurrence of cardiovascular morbidity and
mortality. The use of ACE-inhibitors, ARB, calcium channel blockers, or thiazide diuretics can
also be used as first-line therapy. β-Blockers are not recommended as first-line therapy unless
the patient has ischemic heart disease or heart failure.
Q12-A 48-year-old male patient suffered from an ischemic stroke. Prior to his stroke, he was
untreated for hypertension. After full recovery, he follows up at your office. On examination,
his blood pressure is 132/82 mm Hg. Which of the following is true?

a.An ACE-inhibitor should be used to help him achieve a blood pressure less than 130/80.
b.Hydrochlorothiazide should be used to help him achieve a blood pressure less than 130/80.
c.A β-blocker should be used to help him achieve a blood pressure less than130/80.
d.A β-blocker and hydrochlorothiazide should be used to help him achieve a blood pressure
less than 130/80.
e.The usefulness of initiating antihypertensive treatment in this patient is notwell established.

The answer is e.
2017 ACC/AHA guidelines indicate that for secondary stroke prevention, the usefulness of
initiating antihypertensive treatment is not well established for blood pressures less than
140/90. For previously treated adults, a goal of 130/80is reasonable.

Q13- A 55-year-old man comes to your office after not being seen by a physician in more than
10 years. He is found to be hypertensive, and his creatinine is found to be 2.3 mg/dL (high).
Which medication is most likely to control his blood pressure and decrease the likelihood of
progression of his renal disease?

a. A thiazide diuretic
b. An ACE-inhibitor
c. A calcium channel blocker
d. A β-blocker
e. An aldosterone antagonist

The answer is b.
Several clinical trials have documented the benefit of ACE-inhibitors in patients with
hypertension and CKD. ARBs are also beneficial. His blood pressure goal should be less than
130/80.
Q14- You diagnosed a 47-year-old woman with hypertension around 6 months ago. Her home
blood pressure readings are not at goal, and her blood pressure readings in the office are also
not at goal. Of the following, which is the most likely cause of her uncontrolled hypertension?

a. Treatment nonadherence
b. Undiagnosed renal artery stenosis
c. Primary aldosteronism
d. Undiagnosed thyroid disorder
e. Concomitant use of stimulant medications

The answer is a.
The three most common causes of uncontrolled hypertension in patients who are
diagnosed with the disease are patient nonadherence, inadequate therapy, and inappropriate
therapy. Other secondary causes of hypertension are possible, but less likely than
nonadherence, and while use of some medications may make blood pressure control difficult,
nonadherence is more common.

Q15- You are monitoring the blood pressure in a 53-year-old patient with stage II
hypertension. The patient has no signs or symptoms indicative of secondary hypertension. The
patient is not at goal despite maximized dosage of a generic ACE-inhibitor and thiazide
diuretic. Which of the following is the appropriate next step?

a. Diagnose the patient with resistant hypertension and refer to specialist


b. Add a third agent from a different class to the regimen
c. Change the ACE-inhibitor to an ARB
d. Change the ACE-inhibitor to a different ACE-inhibitor
e. Change the thiazide diuretic to a loop diuretic

The answer is b.
Most patients with hypertension do not reach their blood pressure goal with a single
medication, and most need a second from a different class. If a patient is not at goal with two
medications at maximized dosage, the addition of a third agent from another class is the best
next step. Changing to a different brand within the same class may help in some cases, but
using a third class is likely to be more effective. Changing to a different class (rather than
adding a medication from a third class) is not likely to help the patient reach his/her blood
pressure goal.
ABFM 2020
A57-year old female with a history of diabetes mellitus,hypertension,and depression sees you
for a routine followup visit. Her vital signs include a heart rate of 88 beats/min, a blood
pressure of 162/84 mm Hg, and a BMI of 32 kg/m2.
The recommended antihypertensive regimen for reducing cardiovascular events in this patient
is an ACE inhibitor plus:
A. An alpha-blocker
B. An angiotensin receptor blocker
C. A calcium channel blocker
D. A loop diuretic
ANSWER: C
The ACCOMPLISH trial demonstrated that an ACE inhibitor (ACEI) in combination with a
calcium channel blocker (CCB) reduced both fatal and nonfatal cardiovascular events in
patients with diabetes mellitus and hypertension. The benefit of an ACEI and a CCB for
reducing cardiovascular events was greater than that of an ACEI and a thiazide diuretic.
Evidence has shown that combination therapy for most patients should include a CCB, an
ACEI or angiotensin receptor blocker (ARB), or a thiazide diuretic (SOR A). The American
College of Cardiology/American Heart Association guidelines recommend against centrally
acting medications such as alpha-blockers for first-line therapy. Combining ACEIs and ARBs
is not recommended, as the risk of side effects such as hyperkalemia outweighs the benefits.
Loop diuretics are not considered first-line antihypertensive agents

81. A 47-year-old male with obesity, hypertension, and type 2 diabetes sees you because of
growth of his breasts. An examination confirms gynecomastia.
Stopping which one of the following medications in this patient’s current regimen would most
likely result in regression of his gynecomastia?
A. Amlodipine (Norvasc)
B. Hydrochlorothiazide
C. Liraglutide (Victoza)
D. Pioglitazone (Actos)
E. Spironolactone (Aldactone)

ANSWER: E
Spironolactone has antiandrogenic properties and may cause gynecomastia in approximately
9% of male users. Stopping the medication typically leads to regression of the gynecomastia
within 3 months. Amlodipine, hydrochlorothiazide, liraglutide, and pioglitazone do not have
antiandrogenic properties and are not associated with gynecomastia.
90. Which one of the following antihypertensive medications is associated with reduced
progression of diabetic kidney disease?
A. Amlodipine (Norvasc)
B. Chlorthalidone
C. Labetalol (Trandate)
D. Spironolactone (Aldactone)
E. Valsartan (Diovan)

ANSWER: E
ACE inhibitors and angiotensin receptor blockers are associated with decreased progression of
diabetic kidney disease. Unless otherwise contraindicated, patients with diabetes should be
taking one of these agents. Amlodipine, chlorthalidone, labetalol, and spironolactone are not
directly associated with decreased progression of diabetic kidney disease, but they all may
contribute to a decrease in progression due to long-term control of blood pressure.

101. A 65-year-old female with hypertension and hyperlipidemia sees you for follow-up after
a visit 1 week ago because of uncontrolled hypertension and new-onset type 2 diabetes. She
has smoked one pack of cigarettes per day for the past 40 years. Her medications include
amlodipine (Norvasc), 10 mg daily; hydrochlorothiazide, 12.5 mg daily; and atorvastatin
(Lipitor), 20 mg daily. At last week’s visit you added metformin (Glucophage), 500 mg twice
daily, and lisinopril (Prinivil, Zestril), 5 mg daily. The patient is physically active and
asymptomatic. She is 165 cm (65 in) tall and weighs 59 kg (130 lb) with a BMI of 22 kg/m2.
A physical examination is normal. A summary of her recent vital signs and laboratory data is
listed below.

One week ago, Today


Blood pressure 165/92 mm Hg 162/88 mm Hg
Pulse 72 beats/min 76 beats/min
Hemoglobin A1c 7.2% not measured
Glucose 183 mg/dL 156 mg/dL
Sodium 140 mEq/L (N 135–145) 141 mEq/L
Potassium 4.2 mEq/L (N 3.5–5.0) 5.1 mEq/L
BUN 12 mg/dL (N 8–25) 20 mg/dL
Creatinine 0.8 mg/dL (N 0.6–1.2) 1.4 mg/Dl

You diagnose resistant hypertension and consider evaluating for an underlying cause of
secondary hypertension. Which one of the following tests would most likely yield a diagnosis
in this case?
A. Plasma free metanephrines
B. A low-dose dexamethasone suppression test
C. MR angiography of the renal arteries
D. Polysomnography

ANSWER: C
This patient has resistant hypertension as defined by persistent uncontrolled hypertension
despite the use of three adequate antihypertensives, including a diuretic. She has multiple risk
factors for atherosclerotic disease, and a rise in her creatinine level after the addition of an
ACE inhibitor suggests renovascular hypertension. An imaging procedure to evaluate for the
presence of renovascular hypertension, such as MR angiography of the renal arteries, is
indicated.

151. An obese 10-year-old female has been diagnosed with hypertension and is starting
treatment. Which one of the following should be the target for lowering the systolic and
diastolic blood pressure?
A. <130/80mmHg
B. <140/90mmHg
C. <90th percentile for age, sex, and height
D. <95th percentile for age, sex, and height
ANSWER: C
Hypertension in children is defined as a blood pressure 95th percentile for age, sex, and
height on three separate office visits. The goal for treatment should be to lower the systolic
and diastolic blood pressures below the 90th percentile for age, sex, and height. Once children
are over 13 years of age, the target should be a blood pressure <130/80 mm Hg.

113. Which one of the following patient factors would be most likely to contraindicate use of a
thiazide diuretic such as chlorthalidone?
A. Chronic gout
B. An ejection fraction of 45%
C. Hypothyroidism
D. Stage 3b chronic kidney disease
E. A history of a rash when taking trimethoprim/sulfamethoxazole (Bactrim) in childhood

ANSWER: A
According to American College of Cardiology/American Heart Association guidelines, first-
line agents for antihypertensive drug therapy include thiazide diuretics, calcium channel
blockers, ACE inhibitors, or angiotensin receptor blockers.
In the largest head-to-head comparison of first-step drug therapy for hypertension, the
thiazide-type diuretic chlorthalidone was superior to the calcium channel blocker amlodipine
and the ACE inhibitor lisinopril in preventing heart failure. Thiazides increase serum uric acid
and the risk of gouty attacks, so they should be used with caution in patients with a history of
gout. The presence of a reduced ejection fraction or chronic kidney disease does not preclude
the use of a thiazide diuretic, although progressive kidney disease or concomitant use of loop
diuretics does increase the risk of electrolyte abnormalities. Furthermore, thiazide diuretics are
ineffective in patients with severe renal disease (stage 4 or stage 5). Hypothyroidism is not a
contraindication to the use of thiazide diuretics. The potential for cross reactivity between
antibiotic sulfonamides and non-antibiotic sulfonamides is extremely low and may be
nonexistent.

123. You are working in an urgent care clinic when a 68-year-old male with chronic
hypertension sees you for refills of his medications. He has been out of his medication for the
past month and could not get an appointment with his primary care physician for refills. His
blood pressure is persistently 190/115 mm Hg, even after he has rested in a quiet room for 30
minutes. His blood pressure previously had been well controlled. He has a moderate headache
but otherwise feels well. An examination, including a funduscopic examination, is normal.
Which one of the following management options would be most appropriate at this time?
A. Refill his usual medications and arrange for follow-up in 1 week
B. Administer oral labetalol (Trandate) every 30 minutes until his blood pressure is < 180/110
mm Hg
C. Administer oral nifedipine (Procardia) every 30 minutes until his blood pressure is < 180/110
mm Hg
D. Administer sublingual nifedipine every 30 minutes until his blood pressure is <180/110mm
Hg
E. Refer for immediate hospitalization for intravenous antihypertensive treatment

ANSWER: B
This patient has a hypertensive urgency, defined as symptomatic acute severe hypertension
without evidence of acute end-organ injury. Hypertensive urgencies may be managed in the
ambulatory setting. Emergent intravenous treatment at the hospital is not indicated. This
patient should be treated with an oral agent with a fairly rapid onset of action, such as
clonidine, labetalol, captopril, or prazosin. Topical nitroglycerin is also an option. Nifedipine
may cause unpredictable blood pressure reduction and should be avoided. The patient may be
discharged to resume his usual medications after his symptoms have improved and his blood
pressure is below 160–180/110 mm Hg, with follow-up within a week.
125. You advise a 58-year-old male with chronic kidney disease to limit his salt intake. He
asks about the benefit of this change, since he thoroughly enjoys his high-salt diet.
For patients with chronic kidney disease, current evidence shows that a low-salt diet:
A. Decreases mortality
B. Lowers blood pressure
C. Increases proteinuria
D. Increases the time to end-stage renal disease and the need for dialysis
ANSWER: B
A recent Cochrane review revealed that a low-salt diet consistently lowers blood pressure in
patients with chronic kidney disease (CKD). Current evidence is lacking as to whether a low-
salt diet leads to decreased mortality or an increased time to dialysis. A low-salt diet has also
been shown to decrease proteinuria in patients with CKD.

A 42-year-old Asian male presents for follow-up of elevated blood pressure. He has
no additional chronic medical problems and is otherwise asymptomatic. An
examination is significant for a blood pressure of 162/95 mm Hg but is otherwise
unremarkable.
Laboratory Findings
Sodium ….. 138 mEq/L (N 135–145)
Potassium …. 3.9 mEq/L (N 3.5–5.5)
Fasting glucose .. 86 mg/dL
BUN …. 14 mg/dL (N 10–20)
Creatinine … 0.6 mg/dL (N 0.6–1.3)
Urine … negative
According to the American College of Cardiology/American Heart Association 2017
guidelines,
which one of the following would be the most appropriate medication to initiate at this time?
A. Clonidine (Catapres), 0.1 mg twice daily
B. Hydralazine, 25 mg three times daily
C. Lisinopril/hydrochlorothiazide (Zestoretic), 10/12.5 mg daily
D. Metoprolol tartrate (Lopressor), 25 mg twice daily
E. Triamterene (Dyrenium), 50 mg daily

ANSWER: C
This patient has hypertension and according to both JNC 8 and American College of
Cardiology/American Heart Association 2017 guidelines, antihypertensive treatment should
be initiated. For the general non–African-American population, monotherapy with an ACE
inhibitor, an angiotensin receptor blocker, a calcium channel blocker, or a thiazide diuretic
would be appropriate for initial management.
It is also appropriate to initiate combination antihypertensive therapy as an initial management
strategy, although patients should not take an ACE inhibitor and an angiotensin receptor
blocker simultaneously. Studies have shown that blood pressure control is achieved faster with
the initiation of combination therapy compared to monotherapy, without an increase in
morbidity. Lisinopril/hydrochlorothiazide would be an appropriate choice in this patient. -
Blockers, vasodilators, -blockers, and potassium-sparing diuretics are not recommended as
initial choices for the treatment of hypertension.

.ABFM2019
Q1-According to the most recent American College of Cardiology/American Heart Association
guidelines, hypertension is defined as a blood pressure reading greater than
A).120/80 mm Hg
B).130/80 mm Hg
C).135/85 mm Hg
D).140/90 mm Hg
E).150/90 mm Hg

ANSWER: B
The latest American College of Cardiology/American Heart Association guidelines promote a
radical change in the management of hypertension, which they now define as a blood pressure
130/80 mm Hg. Elevated blood pressure is defined as a systolic pressure of 120– 129 mm Hg
and a diastolic pressure <80

Q2- An otherwise asymptomatic 7-year-old male has a blood pressure above the 95th
percentile for gender, age, and height on serial measurements. Which one of the following
studies would be most appropriate at this time?

A).Renin and aldosterone levels


B).24-hour urinary fractionated metanephrines and normetanephrines
C).Renal ultrasonography
D).Doppler ultrasonography of the renal arteries E).A sleep study

ANSWER: C
Renal parenchymal diseases such as glomerulonephritis, congenital abnormalities, and reflux
nephropathy are the most common cause of hypertension in preadolescent children.
Preadolescent children with hypertension should be evaluated for possible secondary causes
and renal ultrasonography should be the first choice of imaging in this age group.
Renin and aldosterone levels are indicated if there is a reason to suspect primary
hyperaldosteronism, such as unexplained hypokalemia. Measurement of 24-hour urinary
fractionated metanephrines and normetanephrines is used to diagnose pheochromocytomas,
which are rare and usually present with a triad of symptoms including headache, palpitations,
and sweating. Doppler ultrasonography of the renal arteries is useful for diagnosing renal
artery stenosis, which should be suspected in patients with coronary or peripheral
atherosclerosis or young adults, especially women 19–39 years of age, who are more at risk for
renal artery stenosis due to fibromuscular dysplasia. Sleep studies are indicated in patients who
are obese or have signs or symptoms of obstructive sleep apnea.

Q3- A 65-year-old female with hypertension, osteoporosis, and GERD presents to your office
for a well woman visit. She reports no new symptoms or concerns. A review of laboratory
work performed prior to her visit reveals lipid levels at goal, normal glucose and sodium
levels, a calcium level of 10.6 mg/dL (N 8.6–10.3), an albumin level of 4.1 g/dL (N 3.6–5.1),
and a 25-hydroxyvitamin D level of 35 ng/mL (N 20–50). Her calcium level was 10.5 mg/dL
on a basic metabolic panel 6 months ago.
The patient’s medications include hydrochlorothiazide, 12.5 mg daily; lisinopril (Prinivil,
Zestril), 10 mg daily; alendronate (Fosamax), 70 mg weekly; omeprazole (Prilosec), 20 mg
daily as needed; and vitamin D, 2000 IU daily. The patient’s blood pressure is 110/60 mm Hg.
An examination is normal.

In addition to ordering follow-up laboratory studies and scheduling a follow-up visit in 1


month, which one of the following would be most appropriate?

a. Discontinue alendronate
b. Discontinuehydrochlorothiazide
c. Discontinue lisinopril
d. Discontinue omeprazole and begin ranitidine (Zantac)
e. Increase vitamin D to 5000 IU daily
ANSWER:B
This patient has hypercalcemia with a normal albumin level. Hydrochlorothiazide can cause
drug-induced hypercalcemia. Alendronate, lisinopril, and omeprazole do not cause
hypercalcemia. A high vitamin D level can cause hypercalcemia, so increasing vitamin D is
not appropriate at this point. A laboratory evaluation can help differentiate between PTH- and
non– PTH-mediated hypercalcemia.

Q4. An 85-year-old female presents to your office to establish care. She has a history of
essential hypertension, bilateral osteoarthritis of the knees, depression, and hypothyroidism.
The patient feels well and has only mild pain from her osteoarthritis. A recent basic metabolic
panel showed a sodium level of 132 mEq/L (N 135–145) but was otherwise unremarkable. A
TSH level was 1.07 U/mL (N 0.45–4.50).

Which one of the medications in this patient’s current regimen is most likely causing the
hyponatremia?

A.Acetaminophen
B.Hydrochlorothiazide
C.Levothyroxine (Synthroid)
D.Metoprolol succinate (Toprol-XL)
E.E. VitaminD .

ANSWER: B
Hyponatremia is an extremely common condition, affecting 4%–7% of the ambulatory
population. While a number of medications can cause hyponatremia, among the most common
offenders are SSRIs and thiazide diuretics.
Acetaminophen and metoprolol are not known to affect sodium levels. Vitamin D deficiency
has also been linked to hyponatremia so supplementation would likely improve rather than
worsen hyponatremia.
Q5. A 43-year-old female comes to your office to establish care. She reports a long history of
uncontrolled hypertension. She is taking several medications that have been prescribed over the
years. Which one of the following medications should be discontinued 4–6 weeks prior to
obtaining a plasma aldosterone/renin activity ratio?

a) Hydralazine
b) Potassiumchloride
c) Spironolactone (Aldactone)
d) Terazosin (Hytrin)
e) Verapamil(Calan)

ANSWER: C
Patients with secondary hypertension frequently take several medications and are classified as
having “resistant” hypertension. It is important to recognize that many antihypertensive
medications can affect the results of the aldosterone/renin ratio. An aldosterone antagonist,
such as spironolactone or eplerenone, can increase renin and aldosterone levels and should be
discontinued 4–6 weeks before obtaining a ratio. Angiotensin receptor blockers and ACE
inhibitors can increase renin levels. Oral potassium supplements should be continued or started
in patients with hypokalemia, since aldosterone biosynthesis is dependent on potassium.
Medications such as hydralazine, terazosin, and verapamil may be continued or started for
blood pressure control because theydo not affect aldosterone or renin levels.

Q6. A 15-year-old male presents for a sports pre-participation examination. His family history
includes hypertension in his mother. There are no other concerns. He does not take any
medications or supplements. An examination shows a BMI at the 75th percentile for his age.
His weight is 59 kg (130 lb) and his blood pressure is 130/88 mm Hg. On repeat
measurements 30 minutes apart, his blood pressure is 132/90 mm Hg and 132/88 mm Hg. The
remainder of the examination, including vital signs, is normal. Which one of the following
would you recommend at this time?
a) Therapeutic lifestyle changes with close follow-up in 2–3 weeks
b) Yearly blood pressure monitoring
c) Atenolol (Tenormin), 25 mg daily
d) Enalapril (Vasotec), 5 mg daily
e) Hydrochlorothiazide, 12.5 mg daily
ANSWER:A
This patient has stage 1 hypertension based on his average blood pressure of 131/88 mm Hg
after three measurements. He is asymptomatic so the initial recommendation is therapeutic
lifestyle changes and close follow-up in 2–3 weeks (SOR C). If his blood pressure remains
elevated, an evaluation for secondary hypertension would be appropriate. Medication would
be appropriate for stage 1 hypertension with either evidence of end-organ disease or persistent
hypertension after a trial of therapeutic lifestyle changes or progression to stage 2 hypertension
(SOR C). Thiazide diuretics or ACE inhibitors would be appropriate choices for initial
treatment. -Blockers are no longer considered first-line treatment for hypertension in
adolescents or children. This patient would need blood pressure monitoring at every visit due
to his elevated blood pressure.

ABFM2018
Q7- A 45-year-old African-American male returns to your clinic to evaluate his progress after
6 months of dedicated adherence to a diet and exercise plan you prescribed to manage his
blood pressure. His blood pressure today is 148/96 mm Hg. He is not overweight and he does
not have other known medical conditions or drug allergies.
Which one of the following would be the most appropriate initial antihypertensive
treatment option for this patient?
a. Chlorthalidone
b. Hydralazine
c. Lisinopril (Prinivil, Zestril)
d. Losartan (Cozaar)
e. Metoprolol
ANSWER: A
Lifestyle modifications addressing diet, physical activity, and weight are important in the
treatment of hypertension, particularly for African-American and Hispanic patients. When
antihypertensive drugs are also required, the best options may vary according to the racial and
ethnic background of the patient. The presence or absence of comorbid conditions is also
important to consider. For African- Americans, thiazide diuretics and calcium channel
blockers, both as monotherapy and as a component in multidrug regimens, have been shown
to be more effective in lowering blood pressure than ACE inhibitors, angiotensin II receptor
blockers, or -blockers, and should be considered as first-line options over the other classes of
antihypertensive drugs unless a comorbid condition is present that would be better addressed
with a different class of drugs. Racial or ethnic background should not be the basis for the
exclusion of any drug class when multidrug regimens are required to reach treatment goals.
Ref: James PA, Oparil S, Carter BL, et al: 2014 evidence-based guideline for the management
of high blood pressure in adults
Q8- A 42-year-old male with hypertension and hyperlipidemia sees you for a routine health
maintenance examination. His blood pressure is 185/105 mm Hg. He does not have any
current symptoms, including headache, chest pain, edema, or shortness of breath. He is
adherent to his current medication regimen, which includes lisinopril (Prinivil, Zestril), 10 mg
daily, and simvastatin (Zocor), 20 mg at night. A thorough history and physical examination
are both unremarkable.

Which one of the following would be the most appropriate next step?

A)A 30-minute rest period followed by a repeat blood pressure reading


B)Clonidine (Catapres), 0.2 mg given in the office
C)A comprehensive metabolic panel, fasting lipid profile, and TSH level
D)A stress test
E)Hospital admission for blood pressure reduction

ANSWER: A
The first step in the management of severe hypertension is determining whether
a hypertensive emergency is present. A thorough history and physical examination are crucial
(SOR C). Severe hypertension (blood pressure >180 mm Hg systolic or >110 mm Hg
diastolic) with end-organ damage constitutes a hypertensive emergency. A physical
examination should center on evaluating for papilledema, neurologic deficits, respiratory
compromise, and chest pain. If end- organ damage is present the patient should be hospitalized
for monitored blood pressure reduction and further diagnostic workup. If end- organ damage
is not present and the physical examination is otherwise normal, a 30-minute rest with
reevaluation is indicated. Approximately 30% of patients will improve to an acceptable blood
pressure without treatment (SOR C). Home medications should then be adjusted with
outpatient follow-up and home blood pressure monitoring (SOR A). Short- acting
antihypertensives are indicated if mild symptoms are noted such as headache, lightheadedness,
nausea, shortness of breath, palpitations, anxiety, or epistaxis. Diagnostic testing is not
immediately indicated for asymptomatic patients (SOR C). A basic metabolic panel or other
testing should be considered if mild symptoms are present. Aggressive lowering of blood
pressure can be detrimental and a gradual reduction over days to weeks .)is preferred (SOR C
Q9- Which one of the following antihypertensive drugs may reduce the severity of sleep apnea
A) Amlodipine (Norvasc
B) Hydralazine
C) Lisinopril (Prinivil, Zestril
D) Metoprolol
E) Spironolactone (Aldactone

ANSWER: E
Diuretics lessen the severity of obstructive sleep apnea and reduce blood pressure.
Aldosterone antagonists offer further benefit beyond that of traditional diuretics. Resistant
hypertension is common in patients with obstructive sleep apnea. Resistant hypertension is
also associated with higher levels of aldosterone, which can lead to secondary pharyngeal
.edema, increasing upper airway obstruction.

Q10-. A 21-year-old female is being evaluated for secondary causes of refractory hypertension.
Which one of the following would be most specific for fibromuscular dysplasia?

A)A serum creatinine level


B)An aldosterone:renin ratio
C)24-hour urine for metanephrines
D)Renal ultrasonography
E)Magnetic resonance angiography of the renal arteries

ANSWER: E
In young adults diagnosed with secondary hypertension, evaluation for fibromuscular
dysplasia of the renal arteries with MR angiography or CT angiography is indicated (SOR C).
The aldosterone/renin ratio is the most sensitive test to diagnose primary hyperaldosteronism.
Renal ultrasonography is an indirect test that is not as sensitive or specific for fibromuscular
dysplasia. Serum creatinine elevation shows renal involvement but does not identify the cause.
Testing for metanephrines is indicated only if a pheochromocytoma is suspected.
Q11- A 61-year-old white male with type 2 diabetes mellitus sees you for a follow- up visit.
His blood pressure is 156/94 mm Hg. At a visit 1 week ago his blood pressure was 150/92 mm
Hg. Laboratory studies obtained prior to this visit show a BUN of 16 mg/dL (N 6–20), a
serum creatinine level of 0.9 mg/dL (N 0.7–1.3), and microalbuminuria on a urinalysis. His
diabetes is well controlled with metformin (Glucophage) and he is taking aspirin.

Which one of the following would you recommend? A) Observation only


B) An ACE inhibitor
C) A -blocker
D) A calcium channel blocker
E) A diuretic

ANSWER: B
The panel members of the Eighth Joint National Committee for the management
of blood pressure recommended that ACE inhibitors should be initiated for renal protection in
adults with diabetes mellitus, hypertension, and microalbuminuria. This patient appears to be
in an early stage of nephropathy, and ACE inhibitors will reduce the decline in renal function.
- Blockers are no longer recommended for first-line treatment. In white patients who do not
have diabetes, therapy may be started with ACE inhibitors, thiazide diuretics, or calcium
channel blockers.

Q12- A 54-year-old male comes to your office to establish care. He has a past history of
hypertension treated with lisinopril (Prinivil, Zestril) and hydrochlorothiazide but has not
taken his medications for over a year. He does not have any symptoms, including chest pain,
shortness of breath, or headache. On examination his blood pressure is 200/115 mm Hg on two
separate readings taken 5 minutes apart. The remainder of the physical examination is normal.
Which one of the following management options would be most appropriate?

A) Institute out-of-office monitoring with an ambulatory device and follow up in 2 weeks


B) Restart the patient’s previous antihypertensive medications and follow up within 1 week
Administer a short-acting antihypertensive medication in the office to lower his blood pressure
to <160/100 mm Hg

C) Hospitalize for hypertensive emergency


ANSWER: B
This patient has severe asymptomatic hypertension (systolic blood pressure 180 mm Hg or
diastolic blood pressure 110 mm Hg). If there were signs or symptoms of acute target organ
injury, such as neurologic deficits, altered mental status, chest pain, shortness of breath, or
oliguria, hospitalization for a hypertensive emergency would be indicated. Because this patient
was asymptomatic and has a known history of hypertension, restarting his prior
antihypertensive regimen and following up in 2 weeks would be the most appropriate
management option. If he had no past history of hypertension it would be reasonable to
consider out-of- office monitoring with an ambulatory device for 2 weeks before initiating
treatment. In the absence of acute target organ injury, blood pressure should be gradually
lowered to less than 160/100 mmHg over several days to weeks.
Aggressively lowering blood pressure can lead to adverse events such as myocardial
infarction, cerebrovascular accident, or syncope, so administering a short-acting
antihypertensive medication in the office should be reserved for the management of
hypertensive emergencies.

Q13-A 50-year-old female sees you for follow-up of her hypertension. At her last visit 4
weeks ago you started her on lisinopril (Prinivil, Zestril), 10 mg daily, because of a blood
pressure of 158/92 mm Hg and confirmed hypertension on ambulatory blood pressure
monitoring. She is tolerating the medication well and has no side effects. She does not take
any other medications. Today her blood pressure is 149/90 mm Hg, which you confirm on
repeat measurement. This is also consistent with her home measurements. At her last visit a
basic metabolic panel was normal. You repeat a basic metabolic panel today and the results
are normal except for a BUN of 25 mg/dL (N 8–23) and a creatinine level of 1.5 mg/dL (N
0.6–1.1). At her last visit her BUN was 12 mg/dL and her creatinine level was
0.7 mg/dL.

Which one of the following would be most appropriate at this time?


A) Continue her current treatment regimen
B) Increase lisinopril to 20 mg daily
C) Continue lisinopril at the current dosage and add amlodipine (Norvasc), 5 mg daily
D) Discontinue lisinopril and begin amlodipine, 5 mg daily
E) Discontinue lisinopril and begin losartan (Cozaar), 25 mg daily
ANSWER: D
This patient has essential hypertension and her goal blood pressure is <140/90
mm Hg based on JNC 8 guidelines, or 130/80 mm Hg based on the more recent
recommendations of the American College of Cardiology/American Heart Association Task
Force on Clinical Practice Guidelines. Until recently, it was recommended that physicians
should tolerate a rise of <30% in serum creatinine after ACE inhibitor or angiotensin receptor
blocker (ARB) initiation. Rises in serum creatinine of >30% from baseline increase the risk of
renal failure, adverse cardiac outcomes, and death. A recent study suggests that rises in serum
creatinine of <30% also put patients at risk for these outcomes, with a dose-response
relationship between the magnitude of creatinine change and the risk of adverse outcomes.
This patient has more than a 30% rise in creatinine and has no other factors, such as diabetes
mellitus, heart failure, or chronic kidney disease, that would indicate a need for ACE or ARB
therapy for her hypertension. Discontinuing her ACE inhibitor and starting a medication from a
different class is the most appropriate treatment at this time. Based on JNC 8 guidelines,
additional options for blood pressure medications include thiazide diuretics and calcium
channel blockers.

Q14-A 14-year-old female is brought to your office for an annual well child check and sports
preparticipation physical examination. She says she does a lot of running during basketball
practices and games but has trouble controlling her weight. Most of her family is overweight.
She does not have any difficulty participating in sports, and has no symptoms such as chest
pain, shortness of breath, or headaches. She has no significant past medical history. On
examinationthe patient’s height is 154 cm (61 in) and she weighs 63 kg (139 lb). Her BMI is
26.4 kg/m2, which places her in the 90th percentile for her age. Her blood pressure is 130/85
mm Hg, which places her between the 95th and 99th percentile for her age, height, and sex.
Her chart reveals that her blood pressure was at this level at the last two visits. The physical
examination is otherwise normal.

In addition to counseling and support for weight loss, which one of the following would be
most appropriate at this point?
A)Informing the patient and her parents that she is prehypertensive and having her return for a
blood pressure check in 3 months
B)Plasma renin and catecholamine levels
C)An imaging study of the renal arteries
D)A fasting basic metabolic panel, a lipid profile, and a urinalysis
E)Antihypertensive drug therapy
ANSWER: D
In a pediatric patient, blood pressure should be evaluated using comparisons
based on age, sex, and height. Although this adolescent’s blood pressure is prehypertensive for
an adult according to JNC 8 guidelines, it is stage 1 hypertension (between 95% and 99%) for
her age, sex, and height. All pediatric patients with confirmed hypertension should have
further evaluation to check for renal dysfunction as well as other cardiac risk factors.
Additionally, renal ultrasonography is recommended to evaluate for renal disease and
echocardiography to evaluate for end-organ damage that would affect treatment goals.
Additional studies, such as plasma renin and catecholamine levels or renovascular imaging,
may be indicated in children with abnormalities on initial evaluation that suggest secondary
causes of hypertension. Pharmacologic therapy is usually recommended for pediatric patients
with symptomatic hypertension, secondary hypertension, target organ damage, diabetes
mellitus, or persistent hypertension in spite of nonpharmacologic treatment. A low-sodium
diet may be helpful for decreasing blood pressure, and given this patient’s obesity, intensive
counseling about lifestyle changes is appropriate.
ABFM2017
Q15-A 36-year-old female has had elevated blood pressure readings since establishing care
with you 6 months ago. You have increased her antihypertensive therapy monthly in an attempt
to treat her hypertension, and she is currently taking the maximum dosage of three
antihypertensive medications from different classes. She confirms that her blood pressure is
also elevated at home, typically ranging from 155/92 mm Hg to 165/98 mm Hg. She is
otherwise well. She does not have chest pain, shortness of breath, headaches, daytime
sleepiness, or lower extremity edema. Her family history is significant for hypertension
diagnosed in her father in his 50s.

On examination the patient has a blood pressure of 168/95 mm Hg. Her pulse rate is 78
beats/min and her BMI is 28.1 kg/m2. She has a normal cardiac examination and no peripheral
edema.
You order laboratory testing, with the following significant findings:
144 mEq/L (N 136–142)
3.0 mEq/L (N 3.5–5.0)
0.72 mg/dL (N 0.6–1.2
98 mg/dL
1.46 U/mL (N 0.4–4.2)
normal
Which one of the following additional tests is most likely to reveal the cause of her
hypertension?

A) A dexamethasone suppression test


B) Plasma aldosterone/renin activity
C) Plasma free metanephrines
D) A sleep study
E) Renal ultrasonography

ANSWER: B
In young patients with hypertension it is important to consider secondary causes in addition to
the more common essential hypertension. This patient’s relatively young age and elevated
home blood pressure readings despite drug therapy warrant further evaluation. The initial
evaluation showed hypokalemia, which suggests an endocrine cause of hypertension,
specifically hyperaldosteronism. Other potential causes of secondary hypertension include
coarctation of the aorta, renal artery stenosis, thyroid disorders, obstructive sleep apnea,
pheochromocytoma, and Cushing syndrome. Each of these presents with clinical findings that
help to distinguish them from other potential causes, and the laboratory evaluation would
depend on the suspected cause
Q16- A 65-year-old white female is diagnosed with hypertension. She has no significant past
medical history. According to the JNC 8 panel, which one of the following would be an
appropriate systolic blood pressure goal for this patient?

A)<120 mm Hg
B)<130 mm Hg
C)<140 mm Hg
D)<150 mm Hg
E)<160 mm Hg

ANSWER: D
In 2013 the members of the Eighth Joint National Committee (JNC 8) updated guidelines for
the management of hypertension, recommending treatment of systolic blood pressure to <150
mm Hg in patients >60 years of age without comorbid conditions, based on evidence that a
systolic blood pressure 150 mm Hg reduces rates of cardiovascular disease, stroke, and death
(SOR A). While this was a recent recommendation, the evidence supporting the beneficial
effects of lowering blood pressure is decades old .The JNC 8 panel concluded that there is
insufficient evidence to support a specific systolic blood pressure goal in adults <60 years of
age. A Cochrane review concluded that treatment of patients with mild hypertension (systolic
blood pressure of 140–159 mm Hg and/or diastolic blood pressure of 90– 99 mm Hg) did not
reduce morbidity or mortality compared with placebo (SOR A)
Q16- A 53-year-old male sees you for follow-up of his hypertension. His medical history
includes prediabetes and gout, and he is currently taking lisinopril (Prinivil, Zestril), 40 mg
daily, to control his blood pressure. His blood pressure after resting is 148/86 mm Hg.
Laboratory findings include a serum creatinine level of 0.8 mg/dL (N 0.6–1.2) and a serum
potassium level of 4.5 mEq/L (N 3.5–5.1). Which one of the following would be the most
appropriate management of this patient’s hypertension?

A) No change in medication
B) Add amlodipine (Norvasc)
C) Add hydrochlorothiazide
D) Add losartan (Cozaar)
E) Add metoprolol succinate (Toprol-XL)

ANSWER: B
First-line agents for hypertension include ACE inhibitors, and calcium ,angiotensin receptor
blockers, thiazide diuretics channel blockers. Hydrochlorothiazide would be relatively
contraindicated due to the patient’s gout. Losartan, an angiotensin receptor blocker, should not
be added because the patient is taking an ACE inhibitor. Metoprolol succinate, a -blocker, is
not a first- line agent for blood pressure unless there is another indication
.such as systolic heart failure or migraine prophylaxis

Q17- A 55-year-old male comes to your office for a routine visit due to an insurance change.
He has no health complaints and the physical examination is unremarkable except for some
seborrheic keratoses, onychomycosis, and a blood pressure of 171/90 mm Hg. On two
subsequent visits his blood pressure is 168/92 mm Hg and 171/91 mm Hg and you recommend
treatment. The patient states that he will not take any prescription medications, as he believes
they are harmful.
Which one of the following nonpharmacologic measures would be appropriate to recommend
for this patient?
A)Limiting sodium intake to 4 g per day
B)Limiting alcohol to no more than 3 drinks per day
C)Daily coenzyme Q10
D)Daily magnesium supplements
E)Moderate physical activity for 150 min or more per week
ANSWER: E
Although pharmacologic therapy is the mainstay of treatment for hypertension in adults, there
are several nonpharmacologic options that have been shown to lower blood pressure.
Moderate exercise 3–4 times per week for 40 minutes or more has been shown to lower high
blood pressure, with the greatest effect seen when patients exercise 150 minutes or more per
week. Limiting sodium intake to 2400 mg/day decreases blood pressure, and further effects
are seen when it is limited to 1500 mg/day. Alcohol should be limited to no more than two
drinks per day in men, and one drink per day in women. Magnesium and coenzyme Q10 do not
lower blood pressure.

Q18- At a routine visit, a 40-year-old female asks about beginning an exercise regimen. She
has a family history of heart disease and hypertension. She currently has no medical problems,
but she is sedentary. Which one of the following would be the most appropriate
recommendation for this patient?
A) A baseline EKG and rhythm strip
B) An exercise stress test prior to beginning exercise
C) Jogging for 30 minutes twice a week
D) Fast walking for 30 minutes 5 or more days per week

ANSWER: D
This patient would benefit from exercise to prevent or delay the onset of heart disease and
hypertension, and to manage her weight. Exercise stress testing is not specifically indicated for
this patient. Current recommendations are for healthy adults to engage in 30 minutes of
accumulated moderate-intensity physical activity on 5 or more days per week.
ABFM2016
Q19-A healthy 83-year-old female presents for an annual Medicare visit. Her blood pressure is
165/95 mm Hg on initial evaluation today and 160/92 mm Hg on repeat measurement. She has
no health complaints.
The JNC 8 panel recommends which one of the following as the goal blood pressure for this
patient?
A) <135/85 mm Hg
B) <140/90 mm Hg
C) <150/90 mm Hg
D) <160/90 mm Hg
E) <160/100 mm Hg
ANSWER: C
In general, all patients with hypertension, regardless of age, benefit from treatment. Patients
≥80 years of age who are healthy and functionally independent should be treated according to
current recommendations for patients >65 years old. Available guidelines recommend a target
blood pressure of <150/90 mm Hg unless the patient is frail or has numerous comorbidities. In
thesecases, the target blood pressure should be determined on a case-by- case basis (SOR B).

Q20-A 44-year-old male presents to the emergency department in the evening with facial, lip,
and tongue edema that has been gradually progressing over the last few hours. He was started
on lisinopril (Prinivil, Zestril) earlier today for mild hypertension. He does not have a rash or
pruritus. He reports tightness in his throat and, although he is moving air well at this time, you
do note some mild stridor.

Which one of the following is most likely to prevent the need for intubation?
A) Epinephrine
B) An antihistamine such as diphenhydramine (Benadryl)
C) An angiotensin receptor blocker such as losartan (Cozaar)
D) A bradykinin receptor antagonist such as icatibant (Firazyr)
E) A corticosteroid such as methylprednisolone (Medrol)
ANSWER: D
Less than 1% of patients started on an ACE inhibitor develop angioedema, but some studies
have reported that up to 10% of these patients require intubation. This type of angioedema is
due to increased bradykinin rather than histamine, and antihistamines, anticholinergics,
corticosteroids, and epinephrine would not be effective. Icatibant is a bradykinin receptor type
2 blocker and is recommended in patients with laryngeal angioedema compromising airway
function (level 2 evidence). Angiotensin receptor blockers, although probably not harmful,
would not be helpful.

Q21- According to the JNC 8 panel guidelines, which one of the following would be first-line
drug therapy for hypertension in an African-American male who has no other medical
problems?
A) An ACE inhibitor
B) An aldosterone antagonist
C) An "-blocker
D) Hydralazine
E) A thiazide-type diuretic
ANSWER: E
The 2014 evidence-based guideline from the JNC 8 panel recommends that in the general
African-American population, including those with diabetes mellitus, initial antihypertensive
treatment should include a thiazide-type diuretic or calcium channel blocker (for general
African-American population: SOR B; for African-American patients with diabetes: SOR C).

Q22-A 45-year-old Hispanic female presents to your office for follow-up of a blood pressure
measurement of 155/95 mm Hg at a health fair screening. She has no significant past medical
history. She reports that she exercises daily for 30 minutes, follows a low-salt diet, and rarely
drinks alcohol. Her blood pressure in the office today is 154/95 mm Hg.

Which one of the following medications would be most appropriate for this patient?

A) Chlorthalidone
B) Clonidine (Catapres)
C) Doxazosin (Cardura)
D) Metoprolol succinate (Toprol-XL)
E) E) Spironolactone (Aldactone)
ANSWER: A
Thiazide diuretics such as chlorthalidone are considered a first-line therapy for hypertension.
β-Blockers, aldosterone antagonists, and other antihypertensive medications may be used as
add-on therapy to reach blood pressure goals.

Q23- You see a 90-year-old patient with elevated blood pressure. Which one of the following is
an expected physiologic change in elderly patients?
A)Increased renal blood flow
B)Increased diastolic blood pressure
C)Decreased systolic blood pressure
D)Decreased peripheral resistance
E)Wider pulse pressure
ANSWER: E
Age-related physiologic differences such as wider pulse pressure should be a consideration
when treating hypertension in the elderly. These patients also have lower cardiac output, higher
peripheral resistance, lower intravascular volume, and lower renal blood flow compared with
younger patients. Pulse pressure (the difference between systolic blood pressure and diastolic
blood pressure) is a measure of the degree of age-related vascular stiffness and is a risk factor
for coronary artery disease events. With increasing age the strongest predictor of coronary
artery disease gradually shifts from diastolic blood pressure to systolic blood pressure, and then
to pulse pressure. Systolic blood pressure rises gradually throughout adult life, whereas
diastolic blood pressure peaks and plateaus in late middle age, and declines slightly thereafter.
Diastolic hypertension occurs in <10% of all patients with hypertension after age 70.

Q24-Which one of the following is the strongest risk factor for primary hypertension in
children and adolescents?
A)A family history of hypertension
B)Elevated BMI
C)Male sex
D)African American ethnicity
ANSWER: B
There is an increasing prevalence of hypertension in children and adolescents. BMI is the
strongest risk factor for hypertension in this age group. Other risk factors include a family
history of hypertension, low birth weight, male sex, and certain ethnic backgrounds. The
prevalence of hypertension in obese children is 11%, which is more than double that of the
general pediatric population (reported at 1%–5%).
Q25- A 52-year-old male runner with a long-standing history of essential hypertension
presents for a follow-up visit. A review of his medical record shows blood pressure readings
that are consistently 10–15 mm Hg above his goal. He is currently on hydrochlorothiazide, 25
mg/day, and is compliant with a low-sodium diet. An EKG is shown down.
Based on the EKG, which one of the following would be most appropriate?

A) A coronary calcium score


B) An exercise stress test
C) 24-hour Holter monitoring
D) Echocardiography
E) Coronary angiography

ANSWER: D
The diagnosis and treatment of left ventricular hypertrophy (LVH) in patients with
hypertension is important, as it is associated with increased morbidity and mortality from heart
failure, arrhythmias, and sudden cardiac death. LVH is often seen in patients with long-
standing uncontrolled hypertension. If the sum of the S wave in lead V1 and the R wave in lead
V5 or V6 is ≥35 mm or the R wave in aVL is ≥11 mm, it suggests the presence of LVH
(Sokolow-Lyon indices). The sensitivity of an EKG for LVH ranges from 7% to 50%,
however, so echocardiography is the test of choice to make the diagnosis. Echocardiography
helps to directly visualize and quantify left ventricle wall size, in addition to providing other
valuable information such as cardiac wall motion abnormalities, ejection fraction, and cardiac
chamber dimensions. There is no primary role for the other tests listed in this setting (SOR B).

Q26. An asymptomatic 46-year-old male with autosomal dominant polycystic kidney disease
sees you for a routine visit. His vital signs in the office include a blood pressure of 152/93 mm
Hg, a heart rate of 82 beats/min, a respiratory rate of 17/min, a temperature of 37.0°C (98.6°F),
and an oxygen saturation of 99% on room air. His glomerular filtration rate is 49 mL/min/1.73
m2. Which one of the following is the preferred initial therapy for controlling this patient’s
blood pressure?
A)Amlodipine (Norvasc)
B)Furosemide
C)Lisinopril (Prinivil, Zestril)
D)Metoprolol
E)Spironolactone (Aldactone
ANSWER: C
Autosomal dominant polycystic kidney disease (ADPCKD) is the most common genetic
kidney disease and accounts for 4.7% of end-stage kidney disease cases in America. Many
patients with ADPCKD are asymptomatic, but early symptoms can include flank pain, gross
hematuria, or recurrent urinary tract infections. The most common extrarenal manifestation of
ADPCKD is hypertension, which can precipitate cardiovascular dysfunction, including left
ventricular hypertrophy. Thus, early diagnosis and management of hypertension is crucial.
The goal blood pressure should be <140/90 mm Hg in patients under the age of 60. All
ADPCKD patients eventually develop a loss of renal function, and approximately 80%
develop end-stage renal disease by age 70. An ACE inhibitor is the recommended first-line
therapy (SOR C), so lisinopril is the best choice for this patient. Angiotensin receptor blockers
are acceptable in patients who cannot tolerate ACE inhibitors.

Q27-A 52-year-old male was recently diagnosed with mild essential hypertension. His average
blood pressure over several repeated weekly readings is 142/92 mm Hg. He now comes to
your office to discuss a management plan. He is particularly interested in trying to manage his
hypertension nonpharmacologically if possible.

Which one of the following nonpharmacologic interventions should you recommend for this
patient?
A)Limit table salt consumption to 500 mg/day
B)Consume 3 glasses of red wine daily
C)Consume a high-protein diet
D)Monitor home blood pressure readings
E) Start a yoga program

ANSWER: D
Self-measured blood pressure monitoring, with or without additional support (e.g., education,
counseling, telemedicine, home visits, Web-based logging), lowers blood pressure when
compared with usual care, although the benefits beyond 12 months are not clear (SOR A).
Limiting sodium intake to 2400 mg/day (approximately 1 teaspoon of table salt) is
recommended to lower blood pressure. Additional benefit occurs with a limit of 1500 mg/day
(SOR B). A diet that emphasizes vegetables, fruits, and whole grains is also recommended to
lower blood pressure (SOR A), as well as limiting alcohol consumption to no more than 2
drinks/day for men, and 1 drink/day for women (SOR C). Because of mixed results from
therapeutic trials and many limitations to the trials, the American Heart Association does not
recommend either yoga or acupuncture to lower blood pressure.
Q28- A 42-year-old female continues to have elevated blood pressure while on three
antihypertensive agents. You are concerned that she may have idiopathic adrenal hyperplasia
or an aldosterone-producing adenoma.
Which one of the following would be the most appropriate laboratory test?
A) Serum cortisol
B) An aldosterone:renin ratio
C) 17-Hydroxyprogesterone
D) A dexamethasone suppression test
E) Renal ultrasonography

ANSWER: B
Up to 28% of patients may be defined as having resistant hypertension (not controlled on three
drugs or controlled on four or more drugs). Primary aldosteronism is present in up to 5%–10%
of all hypertensive patients and 7%– 20% of those with resistant hypertension. This may be
due to bilateral adrenal hyperplasia or a unilateral aldosterone-secreting adenoma, which can
be diagnosed if there is elevated serum aldosterone in the presence of suppressed renin levels.
A cortisol level and a dexamethasone suppression test are appropriate tests for Cushing
syndrome. A 17-hydroxyprogesterone level tests for congenital adrenal hyperplasia. Renal
ultrasonography will not adequately screenfor any of these conditions.

ABFM 2015
Q29-According to the guidelines developed by the JNC 8 panel, which one of the following
should NOT be used as a first-line treatment for hypertension?

A) ACE inhibitors
B) Angiotensin receptor blockers
C) Calcium channel blockers
D) B-Blockers
E) Thiazide-type diuretics

Answer is D
Q30-A 71-year-old female comes in for follow-up of hypertension. She is worried about her
heart and says that some of her friends have had stress tests and she would like to get one as
well just to be on the safe side. She has no chest pain, shortness of breath, or exercise
intolerance, and a complete review of systems is negative.
The patient’s current medications include lisinopril (Prinivil, Zestril), 20 mg daily; metoprolol
succinate (Toprol-XL), 25 mg daily; and omeprazole (Prilosec), 20 mg daily. Her past medical
history includes hypertension, obesity, and gastroesophageal reflux disease. A physical
examination reveals a blood pressure of 130/70 mm Hg, a heart rate of 90/min, and a BMI of
31.2 kg/m2. An EKG 2 years ago was normal.

Which one of the following should be ordered to assess this patient’s cardiovascular risk?

A) A lipid profile
B) A coronary artery calcification score
C) A C-reactive protein level An EKG
D) An exercise stress test
Answer is A

Q31- A doctor and patient are discussing using a particular drug to treat the patient’s
uncontrolled hypertension. Which one of the following potential effects of the drug is a patient-
oriented outcome that should be discussed during shared decision-making?
A) A decrease in diastolic blood pressure
B) A decrease in hemoglobin A1c
E) A decrease in carotid intimal thickness
F) A decrease in all-cause mortality
G) Improvement in the Framingham cardiac risk score

Answer is D
Q32-A 70-year-old male who recently moved to your area sees you for the first time. He has a
previous history of myocardial infarction, has a pacemaker, and has hypertension that had been
well controlled on hydrochlorothiazide and atenolol (Tenormin) for several years. About 6
months ago his previous physician had to add amlodipine (Norvasc) to his regimen. On
examination he has mild arteriolar narrowing in his fundi and there is a systolic bruit just to
the right of his umbilicus. He has a log of home blood pressure readings that average 138/88
mm Hg for the past 2 months. His serum creatinine level has gone from 1.2 mg/dL to 1.4
mg/dL (N 0.6–1.2) in the past 2 months.

Which one of the following would be most appropriate at this time?

A) Referral for stent placement


B) Scheduling an arteriogram
C) A captopril renal scan
D) Adding losartan (Cozaar) to his regimen
E) Continued monitoring of serum creatinine

Answer is E

Q33-You see a 27-year-old male with autosomal dominant polycystic kidney disease. He has
no other medical problems and his renal function has always been normal on annual testing.
Today the patient reports his blood pressure at home has been 142–150/84–90 mm Hg. His
blood pressure at this visit is 145/88 mm Hg.

Which one of the following medications is preferred for the initial management of hypertension
in this patient?

A) Amlodipine (Norvasc)
B) Chlorthalidone
C) Furosemide (Lasix)
D) Lisinopril (Prinivil, Zestril)

Answer is D
Q34-A 50-year-old female sees you for follow-up of uncontrolled hypertension. Her recent
blood pressure measurements average >175/105 mm Hg. The patient has diabetes mellitus and
a BMI of 32.3 kg/m2. Physical findings are otherwise noncontributory. Recent laboratory
studies include three different potassium levels <3.5 mEq/L (N 3.5–5.0) despite increasing
dosages of oral potassium supplements, with the dosage now at 100 mEq daily.

Which one of the following would be most appropriate at this point?


A)Measurement of peripheral aldosterone concentration and peripheral renin activity
B)CT of the abdomen
C)Renal CT angiography
D)An aldosterone suppression test
Answer is A

Q35-Which one of the following is most likely to be associated with resistant hypertension in
adults?
A) Obstructive sleep apnea
B) Primary aldosteronism
C) Renal artery stenosis
D) Renal parenchymal disease
E) Thyroid disease
Answer is A

Q36-A 28-year-old gravida 1 para 0 at 39 weeks gestation presents for routine outpatient
obstetric care and is found to have a blood pressure of 145/95 mm Hg. A complete review of
systems is notable only for chronic low back pain causing poor sleep. The physical
examination is normal, including a nontender, gravid uterus and a fetal heart rate of 150
beats/min. The cervical examination reveals firm consistency, 1 cm dilation, 50% effacement,
and –3 station. The patient’s blood pressure is checked 5 hours later and is 142/94 mm Hg.
Based on the 2013 ACOG guidelines for management of hypertension in pregnancy, which
one following should be the next step in management?
A)Admit the patient for induction of labor
B)Measure 24-hour urine protein, with induction of labor if the level exceeds 300 mg
C)Begin oral nifedipine (Procardia) and recheck her blood pressure in 24–48hours
D)Place the patient on strict bed rest and check her blood pressure twice weekly
E)Begin twice-weekly office visits with assessment for preeclampsia
Answer is A
Coronary Artery Disease
Pretest 2019
Q1- You are evaluating a 40-year-old male patient in the office who is complaining of chest
pain. His father had a myocardial infarction at age 42, and the patient is quite concerned.
Which characteristic, if included in the history, decreases the likelihood that his chest pain is
cardiac in origin?

a. The pain is worse with inspiration.


b. The pain radiates to his right arm.
c. The pain radiates to his left arm.
d. The pain is associated with nausea.
e. The pain is associated with sweatiness.

The answer is a.
When someone presents to the office complaining of chest pain, the history is invaluable in
helping determine if the pain is due to a life-threatening cause (myocardial infarction, PE,
aortic dissection, and tension pneumothorax, to name a few). The likelihood ratios for the
clinical features associated with acute myocardial infarction follow:

• Chest pain radiating to the left arm: 2.3


• Chest pain radiating to the right arm: 2.9
• Chest pain associated with nausea or vomiting: 1.9
• Chest pain associated with diaphoresis: 2.0
• Pleuritic chest pain: 0.2
Sharp or stabbing pain (rather than dull, aching, a feeling of pressure, tightness, or squeezing)
and positional chest pain also decrease the likelihood that the pain is ischemic.

Q2-You are evaluating a 61-year-old man in the office who is complaining of chest pain. Given
his history and risk factors, you are concerned about myocardial ischemia, and order an
electrocardiogram (ECG). Which of the following ECG features, if present, would most
markedly increase the likelihood of an acutemyocardial infarction?

a. Any ST-segment elevation greater than or equal to 1 mm


b. Any ST-segment depression
C.Any Q wave
D.Any conduction defect
E.A new conduction defect
The answer is a.
Unless a competing diagnosis can be confirmed, an ECG is warranted in the initial evaluation
of most patients with acute chest pain. The likelihood ratios for ECG features associated with
acute myocardial infarction are listed below.
• Any ST-segment elevation: 11.2
• Any ST-segment depression: 3.2
• Any Q wave: 3.9
• Any conduction defect: 2.7
• New conduction defect: 6.3
ST-segment elevation is the ECG finding that is the strongest predictor of acute myocardial
infarction. However, it is important to remember that up to 20% of patients with acute
coronary syndrome can have a normal ECG.

Q3-A 43-year-old woman with a history of well-controlled hypertension and diabetes presents
to your office complaining of intermittent chest pain for the last 3 months. The most recent
episode was 1 week ago, after climbing four flights of stairs at work. The pain was relieved
with rest. An ECG in your office is shown below. She is currently asymptomatic. Which of
the following is the most appropriate next step?
a.Reassure the patient and have her return if symptoms continue.
b.Reassure the patient, but increase her medication to ensure tight control of her blood pressure
and glucose levels.
c.Admit the patient to the hospital for serial enzymes. d. Obtain a treadmill stress ECG.
d.Obtain a treadmill stress echocardiogram.
The answer is e.
The ECG shown has no acute changes, but is suggestive of LVH. Her symptoms are quite
suggestive of angina. Since she is currently asymptomatic and her ECG shows nothing acute,
transfer to the hospital is unwarranted. The best approach would include patient education for
warning signs, and some sort of stress testing. For women in her age group, stress ECGs are
often false positive, so a stress test with imaging is most appropriate.

Q3-You are caring for a 38-year-old male patient who reports episodic chest pain. He reports
that the pain feels like “tightness,” is located right behind his sternum, lasts less than 3
minutes, and is relieved with rest. He takes no medications, has no family history of coronary
disease, and has never smoked. His ECG in the office is normal. Which of the following tests
should be done to determine whether or not his chest pain is due to ischemia?

a. Exercise ECG
b. Resting echocardiogram
c. Stress echocardiography
d. Radionuclide angiography
e. Electron-beam computed tomography (CT)
The answer is a.
Based on the patient’s symptoms, angina seems to be a likely diagnosis. Exercise ECG testing
is the most commonly used noninvasive procedure for evaluating whether the chest pain is due
to angina. Stress testing is often combined with imaging studies, but in low-risk patients
without baseline ECG abnormalities, exercise ECG remains the recommended initial
procedure because of its low cost and convenience. Myocardial stress imaging (scintigraphy or
echocardiography) is indicated if the resting ECG makes an exercise ECG difficult to interpret,
for confirmation of the results of the exercise ECG, to localize the region of ischemia, to
distinguish ischemic from infarcted myocardium, or to assess the completeness of
revascularization following an intervention. The electron beam CT can quantify coronary artery
calcification, but is not helpful to evaluate angina.
Q5-You are evaluating a 33-year-old woman complaining of palpitations. Which of the
following characteristics, if present, increase the likelihood that the symptoms are cardiac in
etiology?

a.The fact that the patient is female


b.The fact that the patient has a sister with similar symptoms
c. Her description of the symptoms as an “irregular heartbeat”
d.The fact that her father has a history of heart disease
e.The fact that the episodes last less than 1 minute
The answer is c.
There are several characteristics of palpitations that can help the physician determine whether
or not the symptoms are from a cardiac cause. These include male sex, the description of the
symptom as an “irregular heartbeat,” a personal history of heart disease, and event duration
greater than 5 minutes. Family history of similar symptoms would not be a risk factor for
cardiac disease.

Q6- You are evaluating a 23-year-old swimmer who is complaining of episodes of


symptomatic rapid heart beating. Twice during swim practice, he has developed a sensation
that his heart is racing. When he measures his heart rate, he finds it to be between 160 and 220
beats/min. The first episode lasted approximately 4 minutes and the second lasted more than
10 minutes. He denies light-headedness or other symptoms during the events. Limited
laboratory evaluation is normal, and ECG shows a normal QRS duration. Which of the
following is the next step in the evaluation?

a. Reassure and continue observation


b. Ambulatory ECG monitoring
c. Consultation with an electrophysiologist d. Stress testing
d. Echocardiography
The answer is d.
Since this patient’s arrhythmia only seems to occur with exercise, stress testing would be
useful. Ambulatory ECG monitoring and echocardiography would not be useful. Consultation
with an electrophysiologist may be appropriate, depending on the results of the stress test.
ABFM 2020
183. A 36-year-old female with no significant past medical history presents to your office
following a syncopal episode. A thorough history and physical examination are normal.
Based on American College of Cardiology/American Heart Association/Heart Rhythm Society
guidelines, which one of the following would you recommend at this time?
1. Reassurance, and follow-up only if she has another syncopal episode
2. A troponin level
3. Electrolyte levels
4. A chest radiograph
5. An EKG
ANSWER: E
According to guidelines from the American College of Cardiology, American Heart
Association, and Heart Rhythm Society, the evaluation of patients with syncope should
include a thorough history and physical examination, as well as an EKG. Further studies
would be indicated if the cause of syncope is unclear.

96. A 72-year-old male comes to your office for an annual health maintenance visit. He
mentions that some of his friends recently underwent health screenings and he asks if there are
any cardiovascular screening tests recommended for him. His blood pressure is well controlled
and he does not have any shortness of breath or chest pain. He exercises regularly. He started
smoking cigarettes while he attended college but quit at age 25. He does not have a significant
family history of cardiovascular disease.

Which one of the following tests is recommended by the U.S. Preventive Services Task Force
for patients such as this?
A. An ankle-brachial index
B. A high-sensitivity C-reactive protein level
C. A coronary artery calcium score
D. Abdominal aortic aneurysm screening with ultrasonography
E. Echocardiography

ANSWER: D
This patient is male, over the age of 65, and smoked in the past, so he meets the criteria for
one-time screening for an abdominal aortic aneurysm (AAA) recommended by the U.S.
Preventive Services Task Force (USPSTF) (B recommendation). This screening is associated
with decreased AAA-related mortality. The USPSTF found insufficient evidence for screening
asymptomatic adults for cardiovascular disease with an ankle-brachial index, high-sensitivity
C-reactive protein level, or coronary artery calcium score. Screening asymptomatic
individuals with echocardiography is not recommended at this time.
147. A 42-year-old male comes to your office for a health maintenance evaluation. He has not
been to your office in the past 5 years and has no medical issues. He tells you that his father
was diagnosed with hyperlipidemia and hypertension in his sixties .He recently used an online
clinic where he could get free medical advice and testosterone testing. He was advised to see
you for a stress test .He has recently started an exercise
programandiswalking5daysaweekfor30 minutes a day with no significant chest pain or
shortness of breath .He wants to start running and work up to a 5K race. Based on U.S.
Preventive Services Task Force and other expert guidelines on stress testing, which one of the
following would you advise for this patient?
A. No stress testing
B. An exercise EKG
C. Exercise echocardiography
D. An exercise sestamibi stress test
E. Dobutamine stress echocardiography

ANSWER: A
The U.S. Preventive Services Task Force, the American College of Physicians, the American
College of Cardiology, and the American Academy of Family Physicians recommend against
cardiac screening with stress testing in low-risk asymptomatic individuals, so an exercise
EKG, exercise echocardiography, an exercise sestamibi stress test, and dobutamine stress
echocardiography would not be recommended for this patient. Cardiovascular screening tests
in asymptomatic patients have a low yield and may produce many false positives, leading to
costly and potentially harmful invasive procedures. Many patients ask for this type of
screening, but explaining current guidelines may help them understand why they should not be
screened.
153. Ahealthy 18-year-old African-American female omes to your office with a5-day history
of sharp chest pain that worsens with both inspiration and expiration .She has also noticed that
the pain worsens with laughing or coughing. She had symptoms of an upper respiratory
infection a week ago with no fever or shortness of breath. She has no history of trauma and no
significant past medical history, and takes no regular93 medications. Her vital signs are
normal, including oxygen saturation, and a physical examination is unremarkable. A chest
radiograph is also normal. Which one of the following would be the most appropriate next
step?
A. A D-dimer level
B. An EKG C. Spiral chest CT
D. Diclofenac, 75 mg twice daily
E. Prednisone, 50 mg daily for 5 days
ANSWER: D
Ruling out life-threatening causes of pleuritic chest pain is the most important consideration in
a pleurisy evaluation. A full history and complete physical examination with vital signs should
be performed (SOR C). Tachycardia, tachypnea, hypotension, fever, or respiratory distress
should raise concerns. Chest radiography should be performed if the cause of the pain is
unclear (SOR C). If no red flags are raised on examination and a chest radiograph is clear, a
trial of NSAIDs should be started (SOR B). NSAIDs are preferred to narcotic medications as
they do not suppress respiratory drive and do not have the risk of addiction and abuse. If a
lifethreatening cause is suspected from the history and physical examination, then further
diagnostic testing is indicated.

A 62-year-old male with hypertension presents to your office with substernal chest pain
radiating into his left arm for the past 20 minutes. He also has diaphoresis and nausea. He has
a blood pressure of 156/96 mm Hg, a pulse rate of 84 beats/min, and an oxygen saturation of
93% on room air. An EKG shows ST-segment elevations in leads V1 and V2. Your medical
assistant calls 911 for immediate transport to the local hospital’s emergency department.
While awaiting the ambulance’s arrival you give the patient low-dose aspirin and sublingual
nitroglycerin.
Which one of the following would be most appropriate regarding oxygen therapy at this time?
A. No oxygen therapy
B. Oxygen via nasal cannula at 2 L/min
C. Oxygen via nasal cannula at 6 L/min
D. 100% oxygen with a regular mask
E. 100% oxygen with a nonrebreathing mask
ANSWER: A
While oxygen supplementation is routinely initiated for patients who are suspected of having
acute coronary syndrome, evidence does not support a benefit from this unless the patient is
hypoxic. Oxygen supplementation is recommended if the patient has an oxygen saturation
<90%, if the patient is at risk for hypoxemia, or if the patient is in respiratory distress.

A 70-year-old female develops thrombocytopenia during a prolonged hospitalization for


endocarditis. Her current medications include scheduled unfractionated heparin injections for
venous thromboembolism prophylaxis. You suspect heparin-induced thrombocytopenia (HIT).
Assuming that her thrombocytopenia is caused by HIT, which one of the following is the most
likely complication?
A. Anaphylaxis
B. Disseminated intravascular coagulation
C. Hemorrhage
D. Sepsis
E. Thrombosis
ANSWER: E
Heparin-induced thrombocytopenia (HIT) is an immune-mediated process that occurs in
approximately 1 in 5000 hospitalized patients. Patients are at highest risk 7–10 days after
exposure to unfractionated heparin, and the risk is particularly high after cardiac surgery,
which is associated with an estimated rate of 1%–3%. In contrast to other causes of
thrombocytopenia, HIT places patients at a paradoxically increased risk of thrombotic
complications, with clotting events occurring in roughly 50% of confirmed cases of HIT.
Lower-extremity deep vein thrombosis and pulmonary embolism are the most common
thrombotic complications, followed by arterial thromboses, stroke, and myocardial infarction,
in descending order of frequency. Thromboses often occur concurrently with the development
of thrombocytopenia or shortly thereafter. The risk of HIT can be determined with the 4T
scoring system, which evaluates the acuity of thrombocytopenia, timing of onset, presence of
thrombosis, and alternative causes of thrombocytopenia. Patients with an intermediate or high
pretest probability should be managed with prompt discontinuation of heparin and initiation of
full-dose anticoagulation with a non-heparin anticoagulant, such as argatroban, danaparoid,
fondaparinux, or bivalirudin, pending results of further HIT diagnostic evaluation.
Anaphylaxis, disseminated intravascular coagulation, hemorrhage, and sepsis are all less
common complications of HIT compared to thrombotic events.

A 43-year-old male presents to the emergency department with the acute onset of sharp,
stabbing chest pain when inhaling and exhaling. The pain worsens with coughing and deep
breathing. He has no significant previous medical history but recently returned from a work
trip to Japan and has noted right leg swelling for the past week. He has no other symptoms. He
is a smoker and has a family history of coronary artery disease in his paternal grandfather. On
examination he appears uncomfortable, and his lungs are clear. A cardiac examination is
notable for tachycardia. He has a blood pressure of 110/70 mm Hg, a heart rate of 112
beats/min, a respiratory rate of 18/min, a temperature of 37.7°C (99.9°F), and an oxygen
saturation of 89% on room air.

Which one of the following test results is most likely to confirm the diagnosis?
A. Elevated troponin levels
B. Acid-fast bacilli on a Gram stain
C. A filling defect on CT angiography
D. Air in the pleural space on a chest radiograph
E. Diffuse ST elevation on an EKG

ANSWER: C
The differential diagnosis of pleuritic chest pain includes several serious causes that should be
considered in the evaluation of a patient with this type of pain. Pulmonary embolism is the
most common cause of pleuritic chest pain. This patient presents with the acute onset of
pleuritic chest pain associated with travel, a swollen leg, and smoking, which are common risk
factors for pulmonary embolism. A filling defect on CT angiography can confirm this
diagnosis. Elevated troponin levels would confirm a diagnosis of acute myocardial infarction,
which would be more likely if the patient were older, experienced pain with exertion, and had
other red-flag symptoms such as diaphoresis, nausea and vomiting, or radiating pain. Acid-fast
bacilli on a Gram stain would confirm a diagnosis of tuberculosis (TB), which is associated
with travel to or exposure to contacts from high-risk areas. TB would also present with other
red-flag symptoms such as hemoptysis, fever, night sweats, and weight loss. A chest
radiograph showing air in the pleural space would confirm a diagnosis of pneumothorax,
which is usually present with decreased breath sounds on physical examination. An EKG with
diffuse ST-segment elevation would confirm a diagnosis of pericarditis, which is usually
associated with a recent or current viral infection or prior history of pericarditis.

ABFM2019
12. A 70-year-old male presents to your office for follow-up after he was hospitalized for
acute coronary syndrome. He has not experienced any pain since discharge and is currently in
a supervised cardiac rehabilitation exercise program. His medications include aspirin,
lisinopril (Prinivil, Zestril), and metoprolol, but he was unable to tolerate atorvastatin
(Lipitor), 40 mg daily, because he developed muscle aches.
Which one of the following would you recommend?
A. Evolocumab (Repatha)
B. Ezetimibe/simvastatin (Vytorin)
C. Fenofibrate (Tricor)
D. Niacin
E. Omega-3 fatty acid supplements
ANSWER: B
High-intensity statin therapy is recommended for patients younger than 75 years of age with
known coronary artery disease. For those who are intolerant of high-intensity statins, a trial of
a moderate-intensity statin is appropriate. There is evidence to support ezetimibe plus a statin
in patients with acute coronary syndrome or chronic kidney disease. Omega-3 fatty acids,
fibrates, and niacin should not be prescribed for primary or secondary prevention of
atherosclerotic cardiovascular disease because they do not affect patient-oriented outcomes.
PCSK9 inhibitors such as evolocumab are injectable monoclonal antibodies that lower LDL-
cholesterol levels significantly and have produced some promising results, but more studies
are needed to determine when this would be cost effective.
11. A 49-year-old African-American male sees you for a routine health maintenance
examination. His past medical history is significant for sarcoidosis. He has noticed some
fatigue and shortness of breath over the last several months, but he is asymptomatic today. His
vital signs are normal except for an irregular pulse. An EKG performed in the office is shown
below.

Which one of the following would be most appropriate at this point?


A. Observation only
B. Amiodarone (Cordarone)
C. Apixaban (Eliquis)
D. Metoprolol succinate (Toprol-XL)
E. A cardiology assessment for placement of a pacemaker
ANSWER: E
This patient’s EKG shows type II second degree (Mobitz type II) atrioventricular (AV) block.
Conduction disturbances are one of the most common manifestations of cardiac sarcoidosis. In
addition to AV block, supraventricular and ventricular arrhythmias can be seen. Mobitz type II
AV block is treated with pacemaker placement. Metoprolol could be used for treatment of
nonsustained ventricular tachycardia, apixaban for anticoagulation in patients with atrial
fibrillation or atrial flutter, and amiodarone for either supraventricular or ventricular
tachycardias.
ABFM2018

Q1-An 85-year-old female with a previous history of diabetes mellitus, hypertension,


dementia, and peptic ulcer disease has been in a skilled nursing facility for 4 weeks for
rehabilitation after a hip fracture repair secondary to a fall during an ischemic stroke. She is
transported to the emergency department today when she develops confusion, shortness of
breath, and diaphoresis. Her blood pressure is 172/98 mm Hg, her heart rate is 122 beats/min
with an irregular rhythm, and her respiratory rate is 22/min. An EKG demonstrates atrial
fibrillation and 0.2 mV ST-segment elevation compared to previous EKGs. Her first troponin
level is elevated. Which one of the following conditions in this patient is considered an
ABSOLUTE contraindication to fibrinolytic therapy?

A) Poorly controlled hypertension


B) Peptic ulcer disease
C) Alzheimer’s dementia
D) Hip fracture repair
E) Ischemic stroke

ANSWER: E
A history of an ischemic stroke within the past 3 months is an absolute contraindication to
fibrinolytic therapy in patients with an ST-elevation myocardial infarction (STEMI), unless
the stroke is diagnosed within 41⁄2 hours. Poorly controlled hypertension, dementia, peptic
ulcer disease, and major surgery less than 3 weeks before the STEMI are relative
contraindications that should be considered on an individual basis.

Q2-A 65-year-old male with type 2 diabetes mellitus, hypertension, and obstructive sleep
apnea sees you for follow-up. He does not use tobacco or other drugs, and his alcohol
consumption consists of two drinks per day. His BMI is 31.0 kg/m2, and he just started a
fitness program. The patient tells you that his brother was recently diagnosed with atrial
fibrillation and he asks you if this increases his own risk. Which one of the following factors
would increase the risk of atrial fibrillation in this patient?
A)Alcohol use
B)Treatment with lisinopril (Prinivil, Zestril)
C)Treatment with pioglitazone (Actos)
D)Use of a continuous positive airway pressure (CPAP) device
E)Physical stress
ANSWER: A
Alcohol consumption greater than one drink/day has been associated with atrial fibrillation.
While not recommended to prevent atrial fibrillation, pioglitazone and lisinopril have both been
associated with lower rates of atrial fibrillation compared to alternative therapies. Treatment of
obstructive sleep apnea, along with a regular fitness regimen, has been associated with a
decrease in the recurrence
of atrial fibrillation.

Q3-A 32-year-old female requests a physical examination prior to participating in an adult


soccer league. Her blood pressure is 118/70 mm Hg and her pulse rate is 68 beats/min. The
examination is otherwise normal except for a systolic murmur that intensifies with Valsalva
maneuvers. She says that she has recently been experiencing mild exertional dyspnea and
moderate chest pain. The chest pain has been atypical and is not necessarily related to
exertion. Echocardiography reveals hypertrophic cardiomyopathy. In addition to referring the
patient to a cardiologist, you recommended initiating therapy with
A) amiodarone (Cordarone)
B) amlodipine (Norvasc)
C) FURSOMIDE
D) lisinopril (Prinivil, Zestril)
E) metoprolol

ANSWER: E
Hypertrophic cardiomyopathy is the most common primary cardiomyopathy, with a prevalence
of 1:500 persons. Many patients with hypertrophic cardiomyopathy are asymptomatic and are
diagnosed during family screening, by auscultation of a heart murmur, or incidentally after an
abnormal result on electrocardiography.
On examination physicians may hear a systolic murmur that increases in intensity during
Valsalva maneuvers. The main goals of therapy are to decrease exertional dyspnea and chest
pain and prevent sudden cardiac death. -Blockers are the initial therapy for patients with
symptomatic hypertrophic cardiomyopathy. Nondihydropyridine calcium channel blockers
such as verapamil can be used if - blockers are not well tolerated.
Q4-A 66-year-old male recently underwent percutaneous angioplasty for persistent angina
with exertion. He does not have any symptoms now. His LDL- cholesterol level is 90 mg/dL.
Which one of the following would be most appropriate for secondary preventionof this
patient’s coronary artery disease? A)No drug treatment
B) Evolocumab (Repatha), 140 mg subcutaneously every 2 weeks
C) Ezetimibe (Zetia), 10 mg daily
D) Rosuvastatin (Crestor), 20 mg daily
E) Simvastatin (Zocor), 40 mg daily

ANSWER: D
Patients <75 years of age with established coronary artery disease should be on high-intensity
statin regimens if tolerated. These regimens include atorvastatin, 40–80 mg/day, and
rosuvastatin, 20–40 mg/day. Moderate-intensity regimens include simvastatin, 40 mg/day.
Monotherapy with non-statin medications (bile acid sequestrants, niacin, ezetimibe, and
fibrates) does not reduce cardiovascular morbidity or mortality. The PCSK9 inhibitors
evolocumab and alirocumab are second-line or add-on therapies at this time.

Q5-A 55-year-old female presents with the new onset of palpitations. An underlying cardiac
cause should be suspected if the patient's palpitations
A) affect her sleep
B) are associated with dry mouth
C) are worse in public places
D) last less than 5 minutes

ANSWER: A
Palpitations are a common symptom in ambulatory care. Cardiac causes are the most
worrisome so it is important to distinguish cardiac from noncardiac causes. Patients with a
history of cardiovascular disease, palpitations that affect their sleep, or palpitations that occur
at work have an increased risk of an underlying cardiac cause (positive likelihood ratio 2.0-2.3)
(SOR C). Psychiatric illness, adverse effects of medications, and substance abuse are other
common causes .Palpitations that are worse in public places and those of very short duration
(<5 minutes), especially if there is a history of anxiety, are often related to panic disorder.
However, even a known behavioral issue should not be presumed to be the cause of
palpitations, as nonpsychiatric causes are found in up to 13% of such cases. The use of illicit
substances such as cocaine and methamphetamine can cause palpitations that are associated
with dry mouth, pupillary dilation, sweating, and aberrant behavior. Excessive caffeine can
also cause palpitations.
Q6-A 68-year-old female presents for evaluation of shortness of breath with activity for the
past several weeks. She used to walk 2 miles daily for exercise but can no longer do so
because of dyspnea and chest tightness. She also reports mild lower extremity edema. She has
a history of a bicuspid aortic valve and aortic stenosis. Echocardiography 1 year ago showed
moderately severe aortic stenosis with a mean valve area of 1.1 cm2. Echocardiography today
shows aortic stenosis with an aortic valve area of 0.9 cm2, a mean pressure gradient of 42 mm
Hg, and a transaortic velocity of 4.3 m/sec. The ejection fraction is estimated to be 50%.
Which one of the following is indicated at this time?
A) Atorvastatin (Lipitor)
B) Furosemide (Lasix)
C) Lisinopril (Prinivil, Zestril)
D) Metoprolol succinate (Toprol-XL)
E) Referral for aortic valve replacement

ANSWER: E
This patient has severe symptomatic aortic stenosis. The only therapy shown to improve
symptoms and mortality in such patients is an aortic valve replacement. In patients with
asymptomatic disease, watchful waiting is usually the recommended course of action. No
medications or other therapies have been shown to prevent disease progression or alleviate
symptoms. Patients with coexisting hypertension should be managed medically according to
accepted guidelines. Diuretics should be used with caution due to their potential to reduce left
ventricular filling and cardiac output, which leads to an increase in symptoms.

Q7-A 75-year-old white male presents to your office following hospitalization for an episode
of heart failure. His edema has resolved but he still becomes symptomatic with minor exertion
such as walking less than a block. A recent chest radiograph shows cardiomegaly, and
echocardiography reveals an ejection fraction of 25%. He is currently taking furosemide
(Lasix), 20 mg daily; carvedilol (Coreg), 25 mg twice daily; and lisinopril (Prinivil, Zestril),
20 mg daily. His vital signs include a pulse rate of 60 beats/min, a blood pressure of 110/70
mm Hg, a respiratory rate of 18/min, and a temperature of 37.0°C (98.6°F). No crackles or
hepatojugular reflux are noted on auscultation.
Which one of the following would improve this patient’s symptoms and decrease his mortality
risk?
A) Digoxin
B) Hydralazine and isosorbide dinitrate (BiDil)
C) Hydrochlorothiazide
D) Spironolactone (Aldactone)
ANSWER: D
For patients with left ventricular systolic dysfunction, clinical trials have demonstrated that
ACE inhibitors,-blockers, angiotensin receptor blockers, and aldosterone antagonists decrease
hospitalizations and all-cause mortality. In African-American patients, all-cause mortality and
hospitalizations have been reduced by hydralazine and isosorbide dinitrate. Aldosterone
antagonists such as spironolactone, as well as -blockers, decrease mortality in patients with
symptomatic heart failure (SOR A). Digoxin improves symptoms of heart failure but does not
improve mortality.

Q8-You see a 58-year-old female who received a drug-eluting stent 10 days ago during a
hospitalization for acute coronary syndrome and coronary artery disease.
She asks for
recommendations about anticoagulation. You determine that she is not at highrisk for
bleeding.
Which one of the following would you recommend?
A) Long-term aspirin use
B) Clopidogrel (Plavix) and aspirin for 30 days and then aspirin alone
C) Clopidogrel alone for 1 year and then aspirin alone
D) Clopidogrel and aspirin for 1 year and then aspirin alone
E) Prasugrel (Effient) for 1 year with no anticoagulation after that

ANSWER: D
Coronary artery stenting is a common procedure, and stent restenosis carries a high mortality
rate. Current American College of Cardiology guidelines recommend dual antiplatelet therapy
(aspirin with a second agent such as clopidogrel) for at least 12 months following the
placement of a drug-eluting stent. Dual antiplatelet therapy with aspirin plus clopidogrel for
more than 1 year gives no additional benefit and carries an additional risk of bleeding. Aspirin
has been shown to be effective for the secondary prevention of heart disease and should be
continued after 1 year.
Q9-A 74-year-old female with a long-standing history of coronary artery disease is hospitalized
for pneumonia. The patient improves with treatment and is hemodynamically stable. An EKG
performed on the third day of hospitalization is shown below. Which one of the following
would be the most appropriate next step?

A) Cardiac rhythm monitoring with no additional treatment


B) Atropine
C) Transcutaneous pacing
D) Transvenous pacing.

ANSWER: A
Second degree Mobitz type I (Wenckebach) heart block is characterized by an intermittent
blockade of electrical impulses from the atria to the ventricles at the level of the
atrioventricular node. This prevents generation of a QRS complex. It is characterized by
progressive prolongation of the PR interval until a P wave is not followed by a QRS complex.
P waves come at regular intervals so PP intervals are normal. Following the missed QRS
complex, the PR interval returns to its baseline duration. A pacemaker is not recommended in
patients with second degree Mobitz type I heart block who are asymptomatic. It is
recommended in symptomatic patients, however, and is guided by electrophysiologic studies.
Q10-A 50-year-old male with hypertension who is not at increased risk for gastrointestinal
bleeding should begin low-dose aspirin at what 10-year risk levelfor cardiovascular disease?
A) 1%
B) 7.5%
C) 10%
D) 15%
E) 20%

ANSWER: C
Low-dose aspirin therapy is recommended by the U.S. Preventive Services Task Force for the
primary prevention of cardiovascular disease (CVD) in patients 50– 59 years of age who have
a risk of CVD 10% (USPSTF B recommendation). The recommendation statement adds that
the patient should have a life expectancy of at least 10 years, should be willing to take daily
aspirin for at least 10 years, and should not be at increased risk for gastrointestinal bleeding.
The decision to start aspirin therapy for patients 60–69 years of age should be based on
individual considerations (USPSTF C recommendation). For adults younger than 50 or age 70
or older, the evidence is insufficient to assess the balance of benefits and harms (C
recommendation). The recent Aspirin in Reducing Events in the Elderly (ASPREE) trial
indicated that daily aspirin use in those over age 70 did not significantly lower the risk of
cardiovascular disease, and did not increase disability-free survival.

According to the recommendations of the American Heart Association, which one of the
following patients requires endocarditis prophylaxis?

A)A 10-year-old female with a previous history of Kawasaki disease without valvular
dysfunction
B)A 22-year-old female who underwent surgical repair of a ventricular septaldefect 1 year ago
C)A 28-year-old female with mitral valve prolapse without regurgitation
D)A 35-year-old female with a history of infectious endocarditis in her 20s that was related to
intravenous drug use
E)A 42-year-old female with a history of rheumatic fever with chorea who has normal
cardiovascular findings
ANSWER: D
The American Heart Association and the American College of Cardiology have decreased the
number of indications for antibiotic prophylaxis prior to dental procedures. Currently
antibiotics are indicated for prosthetic cardiac valves, previous infective endocarditis,
unrepaired cyanotic congenital heart disease or a repaired congenital defect with a residual
shunt, and a cardiac transplant with valve regurgitation due to a structurally abnormal valve.
Amoxicillin, 2 g, is the antibiotic prophylaxis of choice.

Q11-A healthy 55-year-old white male with a family history of coronary artery disease sees
you for a routine health maintenance visit. He asks you what he could do to decrease his risk
of cardiovascular disease. He is a nonsmoker, does not drink alcohol, and has no history of
substance abuse. His BMI is normal and the physical examination is otherwise unremarkable.
His vital signs include a heart rate of 80 beats/min, a blood pressure of 119/70 mm Hg, a
respiratory rate of 15/min, and a temperature of 37.0°C (98.6°F).
Laboratory Findings
Fasting glucose 92 mg/dL , Total cholesterol 190 mg/dL , LDL-cholesterol 98 mg/dL
, HDL-cholesterol 50 mg/dL , Triglycerides 145 mg/dL
His calculated 10-year risk for cardiovascular disease is 5.4%. Which one of the following has
the best evidence to prevent cardiovascular disease in a patient such as this?
A) Moderate-intensity exercise, 150 minutes weekly
B) A low-dose statin
C) Aspirin, 81 mg daily
D) Fish oil supplements
E) Niacin supplements
ANSWER: A
A systematic evidence review released by the U.S. Preventive Services Task Force (USPSTF)
noted that the most active people had median cardiovascular risk reductions of about 30%–
35% when compared with the least active. Statins are beneficial for both primary and
secondary prevention of cardiovascular disease, but the benefit is greater when the baseline
risk is greater. Current guidelines would not support statin therapy for a patient with a 10-year
risk of atherosclerotic cardiovascular disease (ASCVD) <5%. Fish oil supplements have not
proven to be useful for primary prevention of ASCVD. Aspirin is recommended for the
prevention of cardiovascular disease in adults 50–59 years of age with a
>10% 10-year ASCVD risk who are not at increased risk of bleeding, are expected to live at
least 10 years, and are willing to take low-dose daily aspirin for 10 years (USPSTF B
recommendation). Niacin is no longer recommended for cardiovascular risk reduction due to a
lack of evidence for benefit.
Q12-A patient is being discharged from the hospital following an acute non–ST- elevation
myocardial infarction. He is currently being treated with aspirin, lisinopril (Prinivil, Zestril),
and metoprolol. An echocardiogram performed in the hospital was normal and a lipid panel
included a total cholesterol level of 200 mg/dL and a triglyceride level of 225 mg/dL. On
examination he has a pulse rate of 68 beats/min and a blood pressure of 130/80 mm Hg.
Which one of the following additional medications has been shown to improve survival in
patients like this?
A) Amlodipine (Norvasc)
B) Atorvastatin (Lipitor)
C) Gemfibrozil (Lopid)
D) Isosorbide mononitrate
E) Spironolactone (Aldactone)
ANSWER: B
Statins such as atorvastatin, antiplatelet drugs such as aspirin, angiotensin inhibitors such as
lisinopril, and -blockers such as metoprolol have all been shown to increase survival after an
acute coronary artery event. Calcium channel blockers such as amlodipine, fibrates such as
gemfibrozil, and nitroglycerins such as isosorbide mononitrate have not been shown to
increase survival.
Spironolactone has been shown to increase survival in patients with heart failure and reduced
ejection fractions but not in those with ischemic heart disease withpreserved ejection fractions.
Q13-An 80-year-old male sees you for the first time. He is asymptomatic except for some fatigue.
His pulse rate is 50 beats/min. An EKG shows a prolonged PR interval.
Which one of the following medications in his current regimen is the most likely explanation
for these findings?
A) Donepezil (Aricept)
B) Escitalopram (Lexapro)
C) Lisinopril (Prinivil, Zestril)
D) Memantine (Namenda)
E) Zolpidem (Ambien)
ANSWER: A
The 2015 American Geriatrics Society Beers Criteria for potentially inappropriate medication
use in older adults more than or equal 65 years of age states that donepezil use should be
avoided in patients with syncope, due to an increased risk of bradycardia (Moderate Evidence
Level; Strong Strength of Recommendation). Donepezil is a cholinesterase inhibitor. Due to
their cholinergic effect, these medications have a vagotonic effect on the sinoatrial and
atrioventricular nodes. This can cause bradycardia or heart block in patients with or without
underlying cardiac conduction abnormalities. Syncope has been reported with these
medications. Memantine is an N-methyl-D-aspartate receptor antagonist and is not associated
with bradycardia. Escitalopram, lisinopril, and zolpidem are also not associated with
bradycardia.
Q14-A 75-year-old male with a history of hypertension, TIA, and atrial fibrillation sees you
for follow-up. Ten days ago he was on vacation in another state when he developed chest pain.
He went to a local hospital where he was diagnosed with an ST-elevation myocardial
infarction (STEMI) and was taken immediately for cardiac catheterization. He had a drug-
eluting stent placed in his left anterior descending artery. He brings some discharge paperwork
with him, including a medication list, but has not yet seen a local cardiologist. He is concerned
that he is taking too many blood thinners. He feels well and does not have any chest pain,
shortness of breath, or excessive bleeding or bruising.
Prior to his STEMI the patient was taking lisinopril (Prinivil, Zestril), 10 mg daily; warfarin
(Coumadin), 2.5 mg daily; and metoprolol succinate (Toprol-XL), 25 mg daily. Upon
discharge he was instructed to continue all of those medications and to add clopidogrel
(Plavix), 75 mg daily, and aspirin, 81 mg daily.
The patient’s vital signs and physical examination are normal except for an irregularly
irregular rhythm on the cardiovascular examination. His INR is 2.5. Which one of the
following would be most appropriate at this time?
A) Continue the current regimen
B) Discontinue aspirin
C) Discontinue clopidogrel
D) Discontinue warfarin
E) Decrease warfarin with a goal INR of 1.5–2.0

ANSWER: A
Current guidelines recommend that patients with an ST-elevation myocardial infarction
(STEMI) who also have atrial fibrillation take dual antiplatelet therapy such as aspirin plus
clopidogrel and a vitamin K antagonist, with a goal INR of 2.0–
3.0. If a patient was already taking a direct-acting oral anticoagulant (DOAC) instead of
warfarin for atrial fibrillation, the patient should continue with the DOAC in addition to dual
antiplatelet therapy. The duration of triple therapy should be as short as possible, and aspirin
can often be discontinued after 1–3 months. However, this patient’s STEMI occurred less than
2 weeks ago and heshould continue triple therapy.
ABFM2016
14. A 25-year-old male presents with pleuritic chest pain. He has not had a fever or symptoms
of a respiratory infection. He has no risk factors for thromboembolism, and no past medical or
family history of thromboembolism. His vital signs and examination are normal, including
clear lungs on auscultation and no chest wall tenderness. Laboratory findings include a normal
CBC and a normal D-dimer level. A chest radiograph is also normal.
Which one of the following is the most appropriate next step in this patient’s management?
A) An NSAID
B) An anticoagulant pending further imaging
C) C-reactive protein and antinuclear antibody levels, and corticosteroids
D) A rib belt
ANSWER: A
Causes of pleuritic chest pain include pneumonia, chest wall trauma, pulmonary embolus, and
vasculitis. If these conditions are deemed unlikely based on the history, physical examination,
and limited laboratory studies, a chest radiograph is obtained. If this is within normal limits
then viral pleuritic pain is most likely, and can be treated with an NSAID. Given that the
history and physical findings are not suspicious for thromboembolism and a D-dimer is
negative, anticoagulation is inappropriate. With no other systemic symptoms or findings of
collagen-vascular disease, corticosteroids are not indicated. Since there is no rib tenderness
and no radiographic findings of an acute rib fracture, a rib belt is not indicated.

Q15-A 54-year-old male develops chest pain while running. He is rushed to the emergency
department of a hospital equipped for percutaneous coronary intervention. An EKG shows 3
mm of ST elevation in the anterior leads. He is diaphoretic and cool with ongoing chest pain.
His blood pressure is 80/50 mm Hg, his pulse rate is 116 beats/min, and his oxygen saturation
is 98% on room air.
You would immediately administer
A. Alpha -blocker
B. Dual antiplatelet therapy and an anticoagulant
C. Intravenous fibrinolytic therapy
D. An intravenous vasopressor.
ANSWER: B
This patient is likely experiencing an acute anterior wall myocardial infarction with possible
incipient cardiogenic shock. Along with initiating the hospital’s protocol for myocardial
infarction, immediate treatment should include dual antiplatelet therapy with a 325-mg dose of
non enteric aspirin, a P2Y12 inhibitor (clopidogrel, prasugrel, or ticagrelor),
and an anticoagulant (unfractionated heparin or bivalirudin). Given the possibility of
cardiogenic shock, -blockers should not be used. Unless more than a 2-hour delay in
percutaneous coronary intervention is expected, fibrinolytics should not be administered. An
intravenous vasopressor is not indicated.

Q16-A 67-year-old male presents to your office because of fatigue and a syncopal episode. His
vital signs in the office are normal. An examination reveals a harsh systolic murmur best heard
over the second right intercostal space radiating to the neck. Echocardiography confirms your
suspected diagnosis.
Which one of the following is the only treatment that improves mortality with this condition?
A. -Blockers
B. Antimicrobial prophylaxis for bacterial endocarditis
C. Aortic valve replacement
D. Mitral valve repair
E. Ventricular septal defect closure.

ANSWER: C
This patient has symptomatic severe aortic stenosis. The only treatment that improves this
condition is aortic valve replacement (SOR B). Transcutaneous aortic valve replacement may
be an alternative for patients who are not candidates for surgery. -Blockers must be used with
caution due to the risk of depressing left ventricular systolic function. They have not been
shown to improve mortality. Antimicrobial prophylaxis is not indicated unless a patient has
undergone valve replacement or has a history of endocarditis (SOR C). Atrial fibrillation is
common in patients with aortic stenosis and rate control is important. Symptomatic mitral
valve regurgitation may require mitral valve intervention. However, these murmurs are
holosystolic, high pitched, and best heard at the cardiac apex. A ventricular septal defect can
cause a loud holosystolic murmur with an associated thrill heard best at the third/fourth
interspace along the sternal border.
Q17-Which one of the following is an indication for considering atrial ventricular nodal
ablation in a patient with atrial fibrillation?
A. Hemodynamic instability
B. Successful cardioversion to sinus rhythm
C. Inability of the patient to be anticoagulated
D. Atrial fibrillation refractory to medical therapy
E. Persistent atrial fibrillation in a patient who has been successfully rate controlled and
anticoagulated for several years
ANSWER: D
Atrial ventricular nodal ablation is recommended for patients whose atrial fibrillation is
refractory to medical therapy, and requires that patients be anticoagulated for at least 1 month
prior to the procedure and for several months afterward (SOR C). Patients with atrial
fibrillation who are hemodynamically unstable should be considered for emergent
cardioversion (SOR C). Atrial ventricular nodal ablation is not necessary for patients
successfully converted to sinus rhythm or for those who are successfully treated with medical
interventions for rate control and anticoagulation.
Q18- A 25-year-old male presents with pleuritic chest pain. He has not had a fever or
symptoms of a respiratory infection. He has no risk factors for thromboembolism, and no past
medical or family history of thromboembolism.
His vital signs and examination are normal, including clear lungs on auscultation and no chest
wall tenderness. Laboratory findings include a normal CBC and a normal D-dimer level. A
chest radiograph is also normal.
Which one of the following is the most appropriate next step in this patient’s management?
A) An NSAID
B) An anticoagulant pending further imaging
C) C-reactive protein and antinuclear antibody levels, and corticosteroids
D) A rib belt
ANSWER: A
Causes of pleuritic chest pain include pneumonia, chest wall trauma, pulmonary embolus, and
vasculitis. If these conditions are deemed unlikely based on the history, physical examination,
and limited laboratory studies, a chest radiograph is obtained. If this is within normal limits
then viral pleuritic pain is most likely, and can be treated with an NSAID. Given that the
history and physical findings are not suspicious for thromboembolism and a D-dimer is
negative, anticoagulation is inappropriate. With no other systemic symptoms or findings of
collagen-vascular disease, corticosteroids are not indicated. Since there is no rib tenderness
and no radiographic findings of an acute rib fracture, a rib belt is not indicated.
Q19- A 52-year-old healthy female nonsmoker who has a family history of coronary artery
disease presents with episodes of left-sided chest pain that last 10–15 minutes but are unrelated
to activity. A resting EKG is normal. She is on medication for hypertension and is in good
physical condition.
Which one of the following is the best study to order at this time?
A) Exercise treadmill testing
B) Stress echocardiography
C) Coronary CT angiography
D) Stress myocardial perfusion imaging
E) Dobutamine echocardiography
ANSWER: A
Early studies of ischemic heart disease included mostly male subjects. More recently there has
been a determined effort to understand the special considerations associated with this problem
and its management in women. In 2014 the American Heart Association published a
consensus statement summarizing the research on how to best evaluate women with suspected
ischemic heart disease. Its recommendations focused on the level of pretest risk for ischemic
heart disease (low, intermediate, and high), a normal or abnormal resting EKG, the ability of
the subject to exercise, and potential risks of radiation exposure. The patient in this scenario
would be considered low to intermediate risk for ischemia due to her age and risk factors,
along with a history of atypical chest pain. Since her resting EKG is normal and she is
physically fit, she should undergo an exercise treadmill test without imaging.

Q20-A healthy 45-year-old female presents to your office to establish care. She has no
significant past medical history and is up to date on her immunizations. She has no chest pain,
shortness of breath, or exercise intolerance. She does not take any prescribed medications but
does take a low-dose aspirin daily for prevention of coronary artery disease. She does not
smoke and she exercises by walking for 45 minutes 4–5 times a week. She is concerned
because her mother had a fatal cardiac arrest at age 63 and her father was recently diagnosed
with end-stage renal disease at age 75.
On examination the patient’s blood pressure is 120/75 mm Hg and her BMI is 22.1 kg/m2.
Her cardiovascular and pulmonary examinations are unremarkable. Which one of the following
would you recommend to this patient?
A) A resting EKG
B) An exercise EKG
C) Discontinuing aspirin therapy
D) Increasing aspirin to 325 mg daily
E)A basic metabolic panel to screen for chronic kidney disease
ANSWER: C
Although this patient has a family history of coronary artery disease, she is under the age of 50
and thus aspirin therapy as primary prevention is not recommended and may increase the risk
for gastrointestinal bleeding, regardless of the dosage. The U.S. Preventive Services Task
Force found insufficient evidence for screening for chronic kidney disease even in individuals
with a positive family history.
Neither a resting nor exercise EKG is recommended for asymptomatic individuals to detect or
prevent coronary artery disease.

Q21-A 52-year-old male was admitted to the hospital with evidence of an acute myocardial
infarction. He underwent cardiac catheterization and percutaneous coronary intervention with
placement of two drug-eluting stents.

Echocardiography revealed a left ventricular ejection fraction of 30%. By the time of his
discharge the patient had been started on several new medications, including aspirin.
Which one of the following medications is more useful for symptom control than for improving
mortality in this situation?
A) Atorvastatin (Lipitor)
B) Clopidogrel (Plavix)
C) Lisinopril (Prinivil, Zestril)
D) Metoprolol tartrate (Lopressor)
E) Nitroglycerin.

ANSWER: E
Evidence-based guidelines for the treatment of patients with acute coronary syndrome support
several medications in the subacute period. Dual antiplatelet therapy, such as clopidogrel
combined with aspirin, has been shown to reduce cardiovascular mortality (SOR B). ACE
inhibitors and statins should be initiated immediately after a myocardial infarction and
continued indefinitely to reduce mortality and the risk of repeat infarction (SOR A). β-
Blockers have been shown to improve mortality in patients with a left ventricular ejection
fraction <40% (SOR A). Nitroglycerin is often used to manage angina but has no
demonstrated mortality benefit.
Q22-One of your patients is admitted to the hospital with an acute myocardial infarction and
requires a drug-eluting stent in his left circumflex artery. Which one of the following
antiplatelet regimens would be most appropriate for at least the next 6–12 months?
A) Clopidogrel (Plavix) alone
B) Clopidogrel plus aspirin, 81 mg daily
C) Aspirin alone, 325 mg daily
D) Aspirin/dipyridamole (Aggrenox)
E) Prasugrel (Effient) plus aspirin, 325 mg daily.

ANSWER: B
Evidence strongly supports the use of dual antiplatelet therapy with aspirin and a P2Y12
inhibitor such as clopidogrel for a minimum of 6–12 months in patients with a drug-eluting
stent. Aspirin at a dosage of 81 mg is as effective as 325 mg, if not more effective. It is also
associated with a lower bleeding risk. Prasugrel is a second-line antiplatelet agent and can be
used when there is a contraindication to clopidogrel or if the patient is resistant to clopidogrel.
When it is used, it should be used with an aspirin dosage of 81 mg daily. Aspirin/dipyridamole
is effective for secondary stroke prevention but does not have a role in the treatment of
ischemic cardiovascular disease.

Q23- In a patient presenting with unstable angina, which one of the following findings would
denote the highest risk for death or myocardial infarction?
A) New-onset angina beginning 2 weeks to 2 months before presentation
B) Angina with hypotension
C) Angina provoked at a lower threshold than in the past
D) Increased anginal frequency

ANSWER: B

Unstable angina patients at high risk include those with at least one of the following:
Angina at rest with dynamic ST-segment changes ³1 mm
• Angina with hypotension
• Angina with a new or worsening mitral regurgitation murmur
• Angina with an S3 or new or worsening crackles
• Prolonged (>20 min) anginal pain at rest
• Pulmonary edema most likely related to ischemia
ABFM2015
Q24-A 65-year-old male comes to your office to establish care after hospitalization for an acute
myocardial infarction. While reviewing his hospital record you see that he has normal renal
function and had an echocardiogram showing a left ventricular ejection fraction of 40%. His
current medications include metoprolol succinate (Toprol-XL), lisinopril (Prinivil, Zestril),
atorvastatin (Lipitor), and aspirin. In your office today his blood pressure is 132/84 mm Hg
and he is still feeling somewhat weak. He has 1+ pitting edema in his legs and mild dyspnea
with exertion.
Which one of the following, when added to his current regimen, has evidence to support its
use in preventing all-cause mortality?
A) Chlorthalidone
B) Spironolactone (Aldactone)
C) Ezetimibe (Zetia)
D) Losartan (Cozaar)
E) Fish oil.

ANSWER: B
Spironolactone is an aldosterone antagonist. This class of drugs has been found to reduce all-
cause mortality and cardiac death when initiated after a myocardial infarction in patients with
a low left ventricular ejection fraction (LVEF) and signs of heart failure. Guidelines from the
American College of Cardiology and the American Heart Association recommend the use of
aldosterone blockers in patients who have heart failure or diabetes mellitus, have an LVEF
£40%, are receiving ACE inhibitors and $-blockers, and have a serum potassium level <5.0
mEq/L (5.0 mmol/L) and a creatinine level >2.5 mg/dL in men or >2.0 mg/dL in women.
None of the other medications listed has this level of evidence to support its use.

Q25- Which one of the following NSAIDs is safest for patients with a previous history of
myocardial infarction?
A) Ibuprofen
B) Celecoxib (Celebrex)
C) Diclofenac (Zorvolex)
D) Meloxicam (Mobic)
E) Naproxen (Naprosyn.

ANSWER: E
All oral NSAIDs increase the risk of myocardial infarction (relative risk versus placebo from
1.5 for ibuprofen to 1.7 for celecoxib), with the exception of naproxen. Cardiac risks are greater in
older patients, those with a history of cardiac events, and with higher dosages.

Q26-A 60-year-old male is scheduled for coronary revascularization. Which one of the
following would reduce his cardiovascular risk the most when given perioperatively?
A) $-Blockers
B) Calcium channel blockers
C) Statins
D) Aspirin
E) Warfarin (Coumadin

ANSWER: C
Statins are the drugs of choice to reduce perioperative cardiovascular risks (level of evidence
1). In addition to lowering cholesterol, they also reduce vascular inflammation, improve
endothelial function, and stabilize atherosclerotic plaques. For the most protection, statins
should be started 4 weeks prior to the procedure and continued after surgery (SOR A). $-
Blockers and aspirin are beneficial, but lessso than statins

Q27-The most common source of chest pain in children is


A) pulmonary
B) cardiac
C) musculoskeletal
D) gastroesophageal
E) psychogenic
ANSWER: C
Chest pain is a common presenting complaint in children and certainly can result from serious
cardiac pathology. However, the majority of chest pain in children is benign, and determining
clinically which patients need a cardiac workup is therefore paramount. Patients and families
overestimate the prevalence of cardiac causes of chest pain and underestimate the prevalence
of more benign causes. The most common cause of chest pain in children is musculoskeletal
(50%–60%) followed by psychogenic (10%–30%) and respiratory causes (3%–12%). Cardiac
conditions account for 0%–5% of cases of chest pain in children.Red flags that suggest a
cardiac etiology include a patient history of palpitations with the chest pain, an abnormal
cardiac physical examination (rubs or gallops), exertional chest pain without another more
likely etiology such as asthma, and a positive family history. When any of the red flags is
present, the patient should be referred to a pediatric cardiologist. This patient has no red flags
and the most likely etiology of her chest pain is therefore musculoskeletal.
Q28-A 56-year-old male complains of daily early awakening and low energy for the past 3
weeks. Six weeks ago he had a myocardial infarction treated with a coronary artery stent.
During that hospitalization his CBC, fasting glucose level, and thyroid function were normal. A
recent phone note from the cardiac rehabilitation nurse indicates that he became apathetic and
stopped
attending his rehabilitation sessions. He admits to a feeling of hopelessness. He denies chest
pain, dyspnea, orthopnea, and palpitations. His vital signs and physical examination are
remarkable for a healing radial artery catheterization wound. In addition to resumption of
cardiac rehabilitation,
which one of the following would be most appropriate at this point?

A) Reassurance and a follow-up appointment in 6 weeks


B) A Patient Health Questionnaire 9 (PHQ-9)
C) Polysomnography
D) A BNP level
E) An exercise thallium stress test.

ANSWER: B
Depression affects up to 9% of U.S. patients and can cause significant disability. The U.S.
Preventive
Services Task Force recommends screening for depression in adults in practices that have
systems in place
to ensure accurate diagnosis and treatment with follow–up. Brief validated depression
screening tools are readily available to assist in the diagnosis of depressed patients. In his
history, this patient gave the equivalent of positive answers to the two-question Patient Health
Questionnaire (PHQ-2), a screening instrument that is specific for depression. In other words,
depression can be ruled out when the responses are negative. Because the PHQ-2 questions are
positive in this patient,the next step is confirmation with the PHQ-9, a questionnaire that
includes the two questions in the PHQ-2 plus seven additional questions.
Cardiovascular testing may be indicated in the future for this patient, but not for these
symptoms. The patient’s sleep disturbance, viewed in the context of his other depressive
symptoms and positive PHQ-2, is not likely to be due to a sleep disorder, so polysomnography
is not indicated at this point. Untreated depression is associated with worse outcomes in
coronary artery disease, so postponing further evaluation would be inappropriate for this
patient
Q29-A 62-year-old male comes to your office as a new patient. He has a past history of a
myocardial
infarction and is currently in stage C heart failure according to the American Heart Association
classification. His ejection fraction is 30%.
Which one of the following medications that the patient is currently taking is potentially
harmful
and should be discontinued if possible?
A) Diltiazem (Cardizem)
B) Lisinopril (Prinivil, Zestril)
C) Carvedilol (Coreg)
D) Spironolactone (Aldactone)
E) Atorvastatin (Lipitor)
ANSWER: A
ACE inhibitors or angiotensin receptor blockers should be used in all patients with a history of
myocardial infarction and reduced ejection fraction. Aldosterone receptor antagonists are
indicated in patients who have a left ventricular ejection fraction £35%. Nondihydropyridine
calcium channel blockers with negative inotropic effects (verapamil and diltiazem) may be
harmful in patients with low left ventricular ejection fractions. Statin therapy is recommended
in all patients with a history of myocardial infarction. Evidence-based $-blockers (carvedilol
or metoprolol succinate) should be used in all patients with a history of myocardial infarction.

Q30- A 70-year-old male presents to your office for follow-up after he was hospitalized for
acute coronary syndrome. He has not experienced any pain sincedischarge and is currently in a
supervised cardiac rehabilitation exercise program. His medications include aspirin, lisinopril
(Prinivil, Zestril), and metoprolol, but he was unable to tolerate atorvastatin (Lipitor), 40 mg
daily, because he developed muscle aches.
Which one of the following would you recommend?
A. Evolocumab (Repatha)
B. Ezetimibe/simvastatin (Vytorin)
C. Fenofibrate (Tricor)
D. Niacin
E. Omega-3 fatty acid supplements.
ANSWER: B
High-intensity statin therapy is recommended for patients younger than 75 years of age with
known coronary artery disease. For those who are intolerant of high- intensity statins, a trial of
a moderate-intensity statin is appropriate. There is evidence to support ezetimibe plus a statin
in patients with acute coronary syndrome or chronic kidney disease. Omega-3 fatty acids,
fibrates, and niacin should not be prescribed for primary or secondary prevention of
atherosclerotic cardiovascular disease because they do not affect patient-oriented outcomes.
.
PCSK9 inhibitors such as evolocumab are injectable monoclonal antibodies that lower LDL-
cholesterol levels significantly and have produced some promising results, but more studies
are needed to determine when this would be cost effective

Q31-A 63-year-old male sees you after carotid ultrasonography at a local health fair showed a
50% occlusion of his left proximal internal carotid artery. He has no significant past medical
history and has never had a TIA or stroke.
In addition to a healthy diet and exercise, you would recommend
A. No further treatment or follow-up
B. Observation, and repeat ultrasonography in 1 year
C. Statin therapy, and repeat ultrasonography in 1 year
D. Statin therapy and referral to a vascular surgeon for consideration of a carotid artery stent
E. Statin therapy and referral to a vascular surgeon for consideration of carotid
endarterectomy.

ANSWER: C
Asymptomatic carotid artery disease is considered a coronary artery disease risk equivalent;
therefore, statin therapy is indicated. Repeating ultrasonography annually to monitor for
progression of the disease and to guide intervention is also considered reasonable. According to
the 2014 guidelines for the primary prevention of stroke issued by the American Heart
Association/American Stroke Association, prophylactic carotid artery stenting might be
considered in highly selected asymptomatic patients with >70% carotid stenosis, but the
effectiveness of this intervention compared with statin therapy alone is not well established.
The guidelines also state that it is reasonable to consider carotid endarterectomy for
asymptomatic patients with >70% stenosis if the risks of perioperative complications are low.
Nocturnal enuresis

Pretest 2019
Q1- You are caring for a 7-year-old boy with enuresis. His physical examination and initial
testing are all normal. His parents want to try lifestyle modifications to improve the situation.
Which of the following is NOT an effective lifestyle intervention for this child?

a.Discontinue pull ups


b.Include child in clean up if he wets the bed at night
c.Use positive reinforcement like stickers for dry nights
d.Set an alarm for 3 hours after he goes to bed, and wake him up to urinate at that time
e.Discontinue fluids after 5 pm

The answer is e.
Lifestyle modifications should be tried for 3 to 6 months prior to initiating pharmacologic
therapy. Effective interventions include:
•Goal-setting for the child to get up and urinate at night, using an alarm Stopping use of
diapers or pull-ups so the child can notice the sensation of wetness • Improving access to the
toilet, including the use of a bedside potty
• Emptying bladder before bedtime
• Including the child in cleanup activities, as long as it’s done non-punitively
• Positive reinforcement with the use of stickers or other methods
Fluid restriction after dinner is often advocated, but not effective. However, avoiding excessive
fluids at night is appropriate.

Q1.2-The child described in the question above has tried the lifestyle modifications for 6
months with only slight improvement. His mother is not interested in pharmacologic therapies,
but would like to discuss using a moisture- sensitive alarm. Which of the following is true
regarding the use of these alarms for nocturnal enuresis?

a. The goal of this alarm is to wake the child just after the initiation of urination. b. The success
rate is greater for boys than for girls.
c. The success rate is less than 50%.
d. If the process will be successful, it only takes 3 to 4 weeks on average.
e. The alarms are easier for families because the child takes responsibility for the treatment.
The answer is a.
Moisture-sensitive alarms can be a very successful behavioral treatment for nocturnal
enuresis. The first drops of urine complete a circuit, activating an alarm that will wake the child
(and the parents). The parents then help the child complete voiding in the toilet. Over time, a
conditioned response develops, and the child awakens voluntarily with the sensation of a full
bladder. There is no gender difference in success rates, and with appropriate use and parent
involvement, success rates are around 66%. It may take weeks or months to be successful, and
requires a sizeable commitment from the parents and child involved. The child should not take
responsibility for this treatment, because without parental involvement, success rates drop.

Q3-You are seeing a 7-year-old girl whose parents brought her in to have her bed- wetting
evaluated. She has been toilet trained during the day since the age of 4, but still wets the bed at
night. Her father wet the bed until the age of 8 years. Her physical examination reveals no
abnormalities and her urinalysis is normal. Whichof the following is the appropriate next step?
a. Reassurance with no testing or treatment
b. Obtain a post-void residual using US
c. Obtain a blood count and serum chemistry
d. Obtain a renal US
e. Obtain a voiding cystourethrogram (VCUG)

The answer is b.

While the cause of nocturnal enuresis is unknown, it is felt to be due to decreased production of
nocturnal antidiuretic hormone. Risk factors include family history (there is a 5-7X risk if one
parent was eneuretic), preterm birth, male gender (3X more common in males), and smaller
bladder capacity. Most children presenting will have a normal physical examination, and in
that case, the only testing necessary is a urinalysis and an estimation of bladder capacity. This
estimate is most easily done by measuring a post-void residual. The normal capacity is age (in
ounces) plus 2, and a post-void residual should be less than 10% of maximum bladder
capacity.
Q4-You are evaluating a 5-year-old boy. His mother has brought him in because he wets the
bed. He has never been dry at night and his parents are starting to get concerned. You obtain a
thorough voiding history, and find the child to be completely normal on physical examination.
He is otherwise developmentally normal. His urinalysis is normal and his post- void residual
is also normal. Whatshould be the next step in the workup of this patient?

a. Observation
b. X-rays of the lumbar and sacral spine c. Renal US
d. VCUG
e. Both renal US and VUCG

The answer is a.
In the child with monosymptomatic nocturnal enuresis, no further evaluation is needed, other
than a thorough voiding history, physical examination, urinalysis, and potentially a post-void
residual. X-rays of the lumbar and sacral spine are indicated if there is suspicion of spina
bifida occulta, and renal US/VCUG are indicated if there are suspected anatomic
abnormalities that would lead to enuresis. Observation is appropriate because treatment should
not be considered until the age of 7, or when the child can be relied upon to be an active
participant in his/her therapy.
Q4.1-The patient in the question above is now 8 years old and has had no improvement. He
has never been consistently dry through the night, and his physical examination continues to
be normal. Which of the following has been shown to be the most effective intervention for this
condition?

a. Frequent nighttime wakening to encourage voiding


b. Use of an alarm that wakes the child when he wets at night
c. Use of desmopressin (synthetic DDAVP)
d. Use of tricyclic antidepressant medications (for example, imipramine) e. Use of an
anticholinergic antispasmodic (for example, oxybutynin)

The answer is b.
Enuresis alarms have been shown to be an effective treatment for nocturnal enuresis. The
alarms need to be used appropriately, with parental involvement in order to be effective.
Frequent night-time wakening may be effective, but compliance is a barrier to effectiveness.
DDAVP can also be effective, but relapse rate is high once the medication is discontinued.
Tricyclics have a lower initial cure rate and a high relapse rate. They can also be lethal, if
overdosed and are therefore not used for this purpose. Oxybutynin has a high relapse rate and
has not been proven to be efficacious when compared with placebo.

ABFM2016
Q1- You are evaluating a 7-year-old male for long-standing nighttime bed wetting. He has no
daytime symptoms and no other health problems. A thorough physical examination is
normal.Which one of the following would be most appropriate at this point?
A) Urinalysis alone
B) Urinalysis and a serum metabolic panel
C) Urinalysis and a lumbosacral radiograph
D) Urinalysis and renal ultrasonography.

ANSWER: A
This child has monosymptomatic enuresis, meaning there are no other symptoms except
nighttime bed wetting. His physical examination is normal. In this setting, the recommended
initial workup is a urinalysis alone. If the results are normal and there are no other symptoms
suggestive of underlying behavioral or medical conditions, reassurance and instruction on
possible behavioral interventions is appropriate without further evaluation.
ABFM2015
Q2-A mother brings her 7-year-old son in for a well child check and you find thattheir main
concern is bedwetting. He has never achieved consistent nighttime continence.
He currently wets
the bed about 4 nights per week but has no difficulty maintaining continence during the day
and
reports no symptoms such as dysuria or urinary frequency. The parents have triedlimiting his
evening fluid intake but this has not helped. He is otherwise healthy. The patient wants to stop
wearing nighttime diapers.
Which one of the following interventions has the best evidence of long-term success in
addressing this condition?
A) A reward system for achieving dry nights
B) Use of a bed alarm
C) Desmopressin (DDAVP)
D) Imipramine (Tofranil)
E) Oxybutynin

ANSWER: B
This patient has primary monosymptomatic enuresis, the most common type of nocturnal
enuresis. Primary refers to a child who has never achieved 6 months of continuous dry nights.
Monosymptomatic refers to
the absence of daytime symptoms such as dysuria or urinary frequency. Children with daytime
urinary symptoms have a higher incidence of urinary tract pathology and require further
diagnostic evaluation.
Primary monosymptomatic enuresis has a spontaneous annual remission rate of about 15% and
does not require treatment unless the patient (not just the parent) is concerned about the issue.
Treatment requires
participation from both the child and the parents, so ensuring interest from both parties is key.
Bed alarms have the best evidence for long-term success in that they train children via
classical conditioning to awaken at the onset of urination and get up to finish voiding into the
toilet. Reward systems for achieving dry nights have some evidence of benefit but it is
difficult to determine if they are superior to the spontaneous remission rate. Medications such
as desmopressin, imipramine, and oxybutynin have a role in addressing nocturnal enuresis if
bed alarm use is unsuccessful or if parents and children are not willing to engage in the
activities necessary to implement the therapy. Medications may work well while they are used,
but enuresis commonly recurs when they are stopped.
Obecity

Pretest 2019
For questions Q1 to Q3, use the following answer key:
a. Normal weight
b. Overweight
c. Obesity Class I
d. Obesity Class II
e. Obesity Class III

Q1- You are discussing weight management with a 28-year-old Caucasian man. He does not
exercise in any form, and has a strong family history of obesity. His height and weight 2 make
his BMI 27.8 kg/m .

Q2- You are seeing a Hispanic woman for a work physical examination. She is 31 years old, 2
generally inactive, and has no other medical conditions. Her BMI is
33.4 kg/m .

Q3- You are caring for a 56-year-old woman with diabetes and hyperlipidemia. Her BMI is 2

35.8 kg/m .

1 The answer is b. 2 The answer is c.3 The answer is d.


More than 97 million adults in the United States are overweight or obese. The percentage of
overweight adults in the United States is around 66.3%. Obesity is a disorder of excess body
fat resulting in an increased risk for adverse health conditions. In 1997, the World Health
Organization recommended adoption of BMI as a standard for the assessment of body fat. It is
calculated by dividing a
person’s weight in kilograms by their height in meters squared. The classifications for obesity
are:

• Normal weight: BMI between 18.5 and 24.9 kg/m2


• Overweight: BMI between 25 and 29.9 kg/m2
• Obesity Class I: A BMI between 30 and 34.9 kg/m2
• Obesity Class II: A BMI between 35 and 39.9 kg/m2
• Obesity Class III: A BMI greater than or equal to 40 kg/m .
BMI by itself cannot estimate obesity in some subgroups of individuals. These include
competitive athletes and bodybuilders (who may have a misleadingly high BMI but a low total
body fat), pregnant women, children, and adolescents (excessive calorie intake will usually
manifest in additional height and weight, so appropriate weight-to-height ratios must be
assessed using age and gender specific tables).

Q4- You are caring for a 48-year-old Caucasian woman who has a significant family history
of obesity and obesity-related health conditions. She reports that she was at her “ideal” weight
in college, where she majored in finance and accounting. Currently, she is single and lives a
fairly active life, but she does have a high-stress, executive-level position at work. Given what
you know about this patient, which characteristics described in her history put her at risk for
overweight or obesity?

a. Her race
b. Her age
c. Her socioeconomic status d. Her level of education
d. Her marital status

The answer is b.
Risk factors for obesity include race, age, inactivity, socioeconomic status, and marital status.
Regarding race, higher percentages of black and Hispanic Americans meet the criteria of
overweight or obesity than in their non-Hispanic white counterparts. In fact, 76% of blacks
over 20 meet the criteria for being overweight, 76% of Mexican Americans meet the criteria
for being overweight, and 64% of non-Hispanic whites meet the criteria for being overweight.
Regarding socioeconomic status and education level, in industrialized countries, a higher
prevalence of obesity is seen in those with lower educational levels and low income.
Regarding marital status, a tendency to gain weight is seen after marriage and parity. Regarding
age, the prevalence of obesity increases with age, and is particularly apparent between the
ages of 40 and 60.
Q5- The patient described in the question above is interested in a diet that will help her lose
weight. Of the following, which type of diet has been consistently shown to be superior to
others if long-term weight loss is the desired outcome?
a. Lower-fat diets.
b. The “Adkins” diet (low carbohydrate).
c. The “South Beach” diet (low glycemic index).
d. The “Zone” diet (40% carbohydrates, 30% protein, 30% fat). e. None have been shown to
be superior to others.

The answer is e.
Diet control is the cornerstone of obesity management. There are many dietary approaches to
controlling one’s diet, but no specific diet has consistently been shown to be superior to others
related to long-term weight loss outcomes. The key is to create a calorie deficit below what is
needed to maintain weight—and as long as that occurs, the composition of the diet is
secondary.

Q6- A 33-year-old woman is seeing you for weight management. At 5 ft 6 in tall and 230 lb,
she reports a history of having difficulty with weight since her teenage years. The rest of her
medical history is unremarkable. Using conventional dietary techniques, what is her chance of
losing 20 lb and maintaining that weight loss for 2 years?

a. 1%
b. 5%
c. 10%
d. 20%
e. 50%

The answer is d.

Unfortunately, only 20% of patients will lose 20 lbs and maintain the weight loss for 2 years
using conventional dietary techniques. Only 5% can maintain a 40-lb weight loss. Those who
are successful report continued close contact with their health care provider. Most successful
programs are multidisciplinary and include a low-calorie diet, behavior modification, exercise,
and social support.
Q7 -You are discussing weight management with an overweight 33-year-old woman. She has
tried for years to lose weight, but despite multiple attempts, remains overweight. Which of the
following is indicated in the workup of her weight concerns?

a. History and physical alone b. CBC


c. TSH
d. Serum electrolytes

e. LH to FSH ratio

The answer is a.
The history and physical examination are of utmost importance when evaluating the obese
patient. Less than 1% of obese patients have a secondarynonpsychiatric cause for their obesity.
Hypothyroidism and Cushing syndrome are important examples that can generally be detected
by history and physical (but would need additional testing if historical features or physical
findings point in that direction). However, laboratory evaluation is necessary to assess the
medical consequences of obesity. Testing should include fasting glucose, LDL, HDL, and
triglyceride levels. Should the patient present with global symptoms such as skin changes, hair
loss, abnormal menstrual cycles, or atypical fat distribution, laboratory evaluation should
include a workup of endocrine disorders.
Q8-You are caring for an obese 30-year-old woman who would like to consider
pharmacotherapy for the treatment of her obesity. Which of the following Food and Drug
Administration (FDA)-approved options has been shown to result in the highest weight loss
percentage?

a.Orlistat (Xenical)
b.Lorcaserin (Belviq)
c.Phentermine hydrochloride and topiramate (Qsymia)
d.Naltrexone and bupropion hydrochloride (Contrave)
e.Liraglutide (Saxenda)
The answer is c.
There are five FDA-approved medications for weight loss. Orlistat blocks fat absorption in the
gut, and can result if a 2 to 4 kg greater weight loss than placebo. In other studies, Lorcaserin,
a selective serotonin receptor agonist, leads to weight loss of about 3% of initial weight. The
combination of naltrexone and bupropion demonstrates a 2% to 4% weight reduction
compared with placebo at 1 year. Liraglutide is an injectable incretin, and has been shown a
3.7% to 4.5% weight loss compared to placebo at 1 year. In clinical trials, the combination of
phentermine hydrochloride and topiramate resulted in up to 9.8% more weight loss compared
to placebo. All of these options have side effects, and the choice of medication used should
depend on specific patient factors.

Q9- You are evaluating a patient whose BMI is 44 kg/m . You would like the patient to
consider weight-loss surgery, specifically a Roux-en-Y gastric bypass. Which of the following
is true regarding this procedure?

a. The operative mortality rate for this procedure in the first 30 days is near 5%.
b. Complications from this procedure occur in approximately 40% of the cases.
c. The procedure can be expected to help the patient lose up to 30% of initial body weight.
d. Nutritional deficiencies after surgery are rare.
e. This surgery is reserved for people with BMI greater than 30 kg/m .

The answer is b
Bariatric surgery is an increasingly more common treatment option for severe obesity. In the
United States, the most common procedure performed is the Roux-en- Y gastric bypass. The
procedure can result in substantial weight loss, up
to 50% of the initial weight in some studies. Complications are common and occur with about
40% of the cases. Operative mortality is actually quite low, (0% to 1% in the first 30 days).
Nutritional deficiencies are common postoperatively, and patients require life-long
supplementation. Because of the risks of the surgery, bariatric surgery is limited to those with
a BMI more than 40 kg/m2, or more than 35 kg/m2 if there are obesity-related comorbidities
present.
ABFM 2020
A 56-year-old female with a BMI of 37 kg/m2 seeks your advice regarding weight loss.
Which one of the
following medications in her current regimen is most likely to contribute to weight gain?
A. Bupropion (Wellbutrin)
B. Lisinopril (Prinivil, Zestril)
C. Metformin (Glucophage)
D. Mirtazapine (Remeron)
E. Naproxen
ANSWER: D
Mirtazapine, an antidepressant, is associated with weight gain. Lisinopril and naproxen are
weight neutral. Bupropion and metformin may promote weight loss.

157. A 38-year-old male presents with a 2-month history of increased postprandial nausea and
non-bloody, painless, loose stools. He feels well otherwise. Ten months ago, he underwent
bariatric surgery ,which involved creating a small stomach pouch. There were no
complications from the surgery and he does not take any medications. A review of his diet
reveals that he has three small meals and two snacks daily .He has one serving of vegetables
or bread with each meal. He has 16 oz of coffee with breakfast, 12 oz of soda with lunch, 12
oz of beer with dinner, and a cup of water or milk with snacks. His snack is usually cheese or
peanut butter and a cracker. An examination is unremarkable. You are concerned with his diet
habits and your recommendations include:
A. No fluid for 15 minutes before or after meals and snacks
B. Limiting carbonated beverages to 8 oz per meal or snack
C. Increasing daily servings of fibrous vegetables and whole grain breads
D. Eating one meal and three snacks daily
E. Eating a diet lower in fat
ANSWER: A
Hundreds of thousands of Americans have undergone bariatric surgery, and family physicians
are often asked to provide long-term postoperative management. Many bariatric surgery
procedures create a small stomach pouch. Dietary compliance is essential to minimize feeding
intolerance symptoms such as postprandial nausea, emesis, and diarrhea. Post bariatric surgery
diet recommendations typically include the following: ▪ Avoid fluid 15 minutes before and
after meals. Fluids with meals will move food more quickly through the pouch and decrease
the feeling of fullness. ▪ Avoid carbonated beverages entirely. ▪ Eat three small protein-rich
meals and one or two snacks daily. Lower fat diets are not typically recommended. ▪ Whole
grains and fibrous vegetables often exacerbate symptoms so there is no need to increase these
foods.
159. Which one of the following clinical features is a component of the STOP-Bang
questionnaire used as a screening tool for obstructive sleep apnea?
A. Blood pressure
B. Pulse rate
C. Resting oxygen saturation
D. Smoking status
E. Waist circumference
ANSWER: A
The STOP-Bang questionnaire is a screening tool to help identify patients with obstructive
sleep apnea. In the questionnaire S = snoring, T = tiredness, O = observed apnea, P = high
blood pressure, B = BMI >35 kg/m2, A = age >50 years, N = neck circumference >40 cm, and
G = male gender. For each question, answering “yes” scores 1, answering “no” scores 0, and
the total score can range from 0 to 8, with a higher score indicating a higher probability of
obstructive sleep apnea. Pulse rate, resting oxygen saturation, smoking
ABFM 2018
Q1- Referral for bariatric surgical evaluation is indicated for patients with a BMI of
A) 35 kg/m2 and mild cognitive impairment
B) 36 kg/m2 and type 2 diabetes mellitus
C) 37 kg/m2 and no other medical problems
D) 40 kg/m2 and active alcohol abuse
E) 42 kg/m2 and uncontrolled schizophrenia
ANSWER: B
Inclusion criteria for bariatric surgery include a BMI 40 kg/m2 without coexisting medical
problems or a BMI 35 kg/m2 with one or more severe obesity related comorbidities such as
diabetes mellitus. Exclusion criteria include active substance abuse, uncontrolled severe
psychiatric illness, severe cardiopulmonary disease that makes the surgical risk prohibitive,
and lack of cognitive function to comprehend the associated risks, benefits, and required
lifestyle changes.

Q2- patient has a past medical history that includes a sleeve gastrectomy for weight loss.
Which one of
the following medications should be AVOIDED in this patient
A) Acetaminophen
B) Gabapentin (Neurontin )
C) Hydrocodone
D) Ibuprofen
E) Tramadol (Ultram)
ANSWER: D
NSAIDs such as ibuprofen are thought to increase the risk of anastomotic ulcerations or
perforations in patients who have had bariatric surgery and should be completely avoided after
such surgery if possible (C Recommendation, Level of evidence 3). It is also recommended
that alternative pain medications that can be used are identified prior to the surgery (D
Recommendation).
Options such as acetaminophen, gabapentin, hydrocodone, and tramadol can be considered in
patients who have had bariatric surgery if the medications are clinically appropriate otherwise.

ABFM 2016
Q2-A 55-year-old female with hypertension sees you for a routine follow-up visit. A physical
examination is normal except for a BMI of 34.0 kg/m2. Laboratory findings are notable for an
ALT (SGPT) of 53 U/L (N 7–35), and an AST (SGOT) of 28 U/L (N 10–34). She had a
similar elevation a month ago when she was seen in the emergency department. Her
comprehensive metabolic panel is otherwise normal, and a CBC is also normal. She has had
negative hepatitis B surface antigen and hepatitis C antibody testing in the past. She does not
drink alcohol or take medications that affect liver function.
Which one of the following is the most appropriate test for evaluating this patient?
A) Anti–smooth muscle antibody
B) Ceruloplasmin
C) Hepatitis D antibody
D) Iron studies
E) A liver biopsy.

ANSWER: D
This patient is obese and has an isolated elevation of ALT. The initial history and workup,
including testing for chronic hepatitis, a medication review, and alcohol screening, have all
been conducted. Current guidelines suggest iron studies to screen for hemochromatosis as part
of the initial evaluation (SOR C). Anti– smooth-muscle antibody and ceruloplasmin levels can
detect more rare causes of hepatocyte inflammation (autoimmune hepatitis and copper
deposition from Wilson’s disease). These tests can be considered in persistent transaminitis
based on clinical judgment (SOR C). Hepatitis D is a superinfection seen in patients with
hepatitis B, and is not a consideration in this patient. A liver biopsy may also be considered in
persistent transaminitis, especially in the setting of clinical or serologic evidence of advanced
liver fibrosis, but is not recommended at this stage (SOR C). Patients with fatty liver disease
are at increased risk for developing liver cirrhosis.
Q4-A 35-year-old female who is approximately 90 kg (200 lb) above her ideal body weight
comes to you for weight loss recommendations. Her mother, who had a BMI of 37.0 kg/m2,
recently suffered a fatal heart attack and the patient would like to avoid this. She has no other
medical problems except for well-controlled hypertension. Her medication list includes
lisinopril (Prinivil, Zestril), 20 mg daily, and an etonogestrel subdermal (Nexplanon) implant
for contraception.
Which one of the following strategies would be most effective for reducing her cardiac risk?

A) A low-fat diet
B) A high-protein diet
C) Orlistat (Xenical), 120 mg 3 times daily with meals
D) Phentermine (Suprenza), 30 mg daily
E) Referral for bariatric surgery.

ANSWER: E
Bariatric surgery has been shown to reduce all-cause mortality in patients with morbid obesity,
mostly from reduced myocardial infarctions. Although orlistat and phentermine, along with
other weight loss drugs, have been shown to be associated with moderate weight loss, there is
no evidence that any of these agents reduce morbidity or mortality. A low-carbohydrate diet
has been associated with increased HDL-cholesterol levels and decreased triglyceride levels
when compared to a low-fat diet, which may indicate a reduction in cardiac risk. No particular
diet strategy has been shown to be more effective for weight loss than any other strategy.
ABFM 2015
Q5-A 4-year-old male has a BMI of 17.5 kg/m2, which places him between the 90th and 95th
percentiles for BMI. According to the CDC, he should be classified as being
A) at a healthy weight
B) overweight
C) obese
D) morbidly obese.

ANSWER: B
The recommended terminology for weight classification in children is based on age and either
BMI (for children ages 2–18 years) or weight-for-length ratio (for children ages 0–2 years).
Children under the age
of 2 years are identified as being overweight when their weight-for-length ratio exceeds the
95th percentile for their sex. The term obese is not used for children under the age of 2 years.
Children age 2–18 years
are appropriately classified as underweight when their BMI falls below the 5th percentile,
healthy weight when their BMI is between the 5th and 85th percentile, overweight when their
BMI is between the 85th
and 94th percentile, and obese when their BMI is in the 95th or greater percentile. There is
currently no standard definition of childhood morbid obesity, but obesity is sometimes
classified as severe or extreme
when a child’s BMI is at the 99th percentile or greater.

Q6-A 35-year-old female asks you about options for weight loss. She weighs 104 kg (229 lb)
and
has a BMI of 34 kg/m2. Her health problems include hypertension anddepression.
According to the U.S. Preventive Services Task Force, which one of the following is the most
appropriate initial recommendation for weight-loss management in this patient?
A) A high-protein diet
B) A low-carbohydrate diet
C) Behavioral counseling
D) Bariatric surgery
ANSWER: C
The U.S. Preventive Services Task Force found that the most effective behavioral counseling
interventions for obesity management were comprehensive and of high intensity (12–26
sessions in a year), and involved multiple behavioral management activities, such as group and
individual sessions, setting weight-loss goals, addressing barriers to change, and active use of
self-monitoring. Low- carbohydrate diets are minimally effective over the long term without
behavioral interventions.
ABFM 2014
Q7-A 52-year-old female with morbid obesity is incidentally noted to have mildly elevated
AST (SGOT) levels. She does not consume alcohol and denies using recreational drugs. A
workup for chronic viral hepatitis and hemochromatosis is negative
Which one of the following is most likely to improve her hepatic condition?
A) Pentoxifylline
B) Simvastatin (Zocor)
C) L-carnitine
D) Vitamin E
E) Weight loss

ANSWER: E
Nonalcoholic fatty liver disease is characterized by the accumulation of fat in hepatocytes. It is
associated with insulin resistance, central adiposity, increased BMI, hypertension, and
dyslipidemia. An incidentally discovered elevated AST level in the absence of alcohol or drug-
induced liver disease strongly suggests the presence of nonalcoholic fatty liver disease. The
goal of therapy is to prevent or reverse hepatic injury and fibrosis. Diabetes mellitus,
hypertension, dyslipidemia,and other comorbid conditions should be appropriately managed.
A healthy diet, weight loss, and exercise are first-line therapeutic measures to reduce insulin
resistance in patients with nonalcoholic fatty liver disease. Weight loss has been shown to both
normalize AST levels and improve hepatic histology. Vitamin E has been shown to improve
AST levels but has no impact on liver histology, and pentoxifylline, simvastatin, and L-
carnitine have not been shown to consistently improve either AST levels or liver histology
(SOR B

Q8-A 53-year-old obese female presents with left calf swelling and tenderness. Using the
Wells criteria you determine that she is at intermediate risk for deep vein thrombosis.
Which one of the following is the most appropriate next step in the evaluation and treatment of
this patient?
A) Anticoagulation
B) D-dimer assessment
C) Compression ultrasonography
D) Impedance plethysmography
E) Contrast venography.
ANSWER: C
In patients at intermediate to high risk for deep vein thrombosis, compression ultrasonography
is the initial diagnostic test of choice. In low-risk patients, a negative D-dimer has a high
enough negative predictive value to essentially rule out deep vein thrombosis. It is not
sensitive or specific enough for evaluating intermediate-risk patients. Initiating treatment
based on pretest probability would be inappropriate. Impedance plethysmography is not
readily available, andcontrast venography is too invasive to be used routinely.

Q9-The National Weight Control Registry includes individuals who have lost substantial
weight without surgery, and have maintained the weight loss for an average of 5 years. Which
one of the following behaviors is typical of these individuals?
A) Eating breakfast every day
B) Taking daily vitamin and mineral supplements
C) Weighing themselves daily
D) Being physically active >2 hours a day
E) Eating a low-protein diet.

ANSWER: A
Individuals on the National Weight Control Registry typically eat a low-fat diet rich in
complex carbohydrates, eat breakfast daily, weigh themselves at least once a week, and are
physically active for 60–90 minutes a day.
STD
Pretest2019
Q1. A 48-year-old man is seeing you to discuss sexual concerns. He complains of being unable
to achieve an erection, despite having strong interest in sexual activity. Which of the following
is true?
a. This is most often because of an unrecognized mood disorder.
b. This is most often because of a lack of attraction for his partner.
c. This is most often because of stressors in the home and interpersonal conflict.
d. This is most often because of a vascular problem.
e. This is most often because of alcohol abuse.

The answer is d.
The sexual response is divided into four phases. The first is libido (or desire/interest). This
phase requires androgens and an intact sensory system. The second phase is arousal (or
excitement) and in men, involves erection. Vascular arterial or inflow problems are by far the
most common cause, though mood disorders, stressors, and alcohol abuse may all play a role.
Lack of attraction to a partner would represent a disorder of desire, not arousal.

Q2. A 36-year-old man sees you to discuss a lack of sexual interest. He is not having sexual
fantasies and is unmotivated to begin sexual activity. He does not report depressive symptoms
and has no other physical complaints. His physical examination is normal. Which of the
following laboratory tests is most appropriate?
a. Total testosterone
b. Free testosterone
c. Thyroid-stimulating hormone (TSH)
d. Prolactin
e. Prostate-specific antigen (PSA)

The answer is b.
In patients with decreased sex drive, laboratory workup should be
directed by the history and physical examination findings. In a male patient with no other
complaints and no physical examination findings, assessment of hormone status is indicated.
Testosterone levels should be checked in the morning, when they peak. Free testosterone is a
more accurate measure of androgen status, as it is a measure of bioavailable testosterone. The
TSH and prolactin levels may be indicated in the presence of other complaints or physical
findings. PSA would not be helpful. If this were a female patient, both androgen and estrogen
status should be evaluated.
Q3. You have diagnosed a 30-year-old woman with depression. She is concerned that medical
treatment may cause sexual dysfunction. In order to avoid sexual side effects, which
antidepressant would be the best choice?
a. Amitriptyline
b. Paroxetine
c. Citalopram
d. Sertraline
e. Bupropion
The answer is e.
Tricyclics and SSRIs frequently cause sexual dysfunction due to a raised threshold for orgasm.
Conversely, bupropion actually decreases the orgasm threshold and is least likely to cause
sexual dysfunction.

Q4. A 23-year-old man comes to your office to discuss premature ejaculation. He has had this
condition since beginning sexual activity at 17 years of age. He has tried behavioral methods,
but these have not been successful. Which of the following medications is most likely to help
this condition?
a. Alprostadil
b. Fluoxetine
c. Bupropion
d. Sildenafil
e. Atenolol
The answer is b.
Premature ejaculation is the most common sexual dysfunction in men, affecting about 29% of
men in the general population. Alprostadil is used for erectile dysfunction, but would not
positively affect premature ejaculation.
Fluoxetine raises the threshold for orgasm, making it an effective treatment option. Bupropion
and sildenafil may decrease the orgasmic threshold and further exacerbate premature
ejaculation. Atenolol may cause erectile dysfunction, but would likely not treat premature
ejaculation.
Q5. You are evaluating a 47-year-old man with erectile dysfunction. After a thorough history
and physical examination, you order a morning free testosterone level. His level was low.
What is the most appropriate next step?
a.Begin testosterone injections
b.Begin topical testosterone replacement
c.Obtain follicle-stimulating hormone (FSH), luteinizing hormone (LH), and prolactin levels
d.Confirm the low level with repeat testosterone testing
e.Perform a nocturnal penile tumescence evaluation

The answer is d.
In men with erectile disorders, obtaining a serum free testosterone level in the morning is
appropriate. If the level is low, the level should be confirmed with a second level before
testosterone replacement is considered. After a second low measurement is obtained, the next
step is to obtain a FSH, LH, and prolactin level. If the FSH and LH are low, but the prolactin is
normal, the diagnosis is pituitary or hypothalamic failure. If the FSH and LH are high and the
prolactin is normal, the diagnosis is testicular failure. The nocturnal penile tumescence
evaluation would be done to eliminate psychologic factors that inhibit arousal in the setting of
ED, but would not be helpful to follow up an abnormal testosterone level.

Q6. You are caring for a woman who describes primary orgasmic dysfunction and comes to
you for advice. Which of the following therapies has shown to be the most effective in treating
this condition?
a. Directed self-stimulation
b. The “stop-start” technique
c. Group therapy
d. Hypnotherapy
e. Sensate focus
The answer is a.
Treatment for sexual dysfunction has been studied in many settings by
many people. The most effective treatment program found to date for women with primary
orgasmic dysfunction is directed self-stimulation. Beginning with basic education in anatomy
and physiology, women progress through the stages of tactile and visual self-exploration and
manual stimulation. The stop-start technique is a treatment program for premature ejaculation
in men. Group therapy can help counteract sexual myths and correct sexual misconceptions,
but generally is not used for orgasmic dysfunction. Hypnotherapy may be helpful in situations
where relaxation interferes with sexual functioning. Sensate focus involves guided touch of a
partner in areas other than the genital area. This is helpful for couples therapy.
ABFM2018
A 25-year-old female who recently moved to the area comes in for a well woman visit. She
reports that she has had yearly Papanicolaou (Pap) tests and sexually transmitted infection
(STI) screening since age 21 with no abnormal results. She has had a total of six sexual
partners. She is asymptomatic and does not have any history of STIs or new partners in the
past year. Your nurse informs her that STI screening can be done, but a Pap test is not
necessary at this time. ?The patient is concerned about not having a Pap test this year and asks
you why it is not recommended. You explain that the most important reason is that
A) she has no history of STIs
B) she has had several normal Pap tests in a row
C) she is in a low-risk group for HPV infection
D) Pap test abnormalities would require no further evaluation in a patient her age
E) the risk of harm from unnecessary procedures and treatment exceeds the potential benefit at
her age

ANSWER: E
Annual HPV screening in patients age 21-29 years has very little effect on cancer prevention
and leads to an increase in procedures and treatments without significant benefit. In this age
group there is a high prevalence of high-risk HPV infections but a low incidence of cervical
cancer. If this patient were due for a Papanicolaou (Pap) test and results were ASC-US with a
positive high-risk HPV or a higher grade abnormality, colposcopy would be recommended.
Current recommendations are for a Pap test with cytology every 3 years for women age 21-29
years with normal results, and the frequency does not change with an increased number of
normal screens. HPV is the most common sexually transmitted infection (STI) and up to 79%
of sexually active women contract HPV infection in their lifetime, so the lack of other STIs
does not preclude the possibility of an HPV infection.

A 30-year-old female presents with an episode of recurrent, painful vesicular lesions on the
labia. She noted a tingling, burning sensation a few days before the lesions appeared. A few
years ago she had a similar outbreak just before the birth of her second child.
Which one of the following is indicated for this patient?
A) Doxycycline
B) Fluconazole (Diflucan)
C) Metronidazole
D) Penicillin G benzathine (Bicillin L-A) E) Valacyclovir (Valtrex)

ANSWER: E
This patient has a recurrent outbreak of genital herpes, and valacyclovir is the preferred
treatment. Penicillin G benzathine is a treatment for syphilis, which usually begins as a
painless papule that transforms into the classic chancre. Fluconazole and metronidazole are
treatments for yeast vaginitis and bacterial vaginitis; these conditions present with itching and
a vaginal discharge but not vesicular lesions.
Doxycycline is a treatment for Chlamydia infection, which is often completely asymptomatic
and detected only with screening.
ABFM2015
Q1-A 23-year-old healthy male is sexually active with other men and does not use condoms.
He is interested in reducing his risk of contracting HIV by using a daily oral antiretroviral
medication.
Which one of the following laboratory tests should be done no more than 7 days before
initially prescribing pre-exposure prophylaxis with emtricitabine/tenofovir disoproxil
(Truvada)?
A) A CD4 cell count
B) Antibody testing for HIV
C) Hemoglobin concentration
D) A platelet count
E) An ALT level

ANSWER: B
It is of critical importance that patients have a documented negative HIV antibody test (from
serum or point-of-care fingerstick) prior to starting pre-exposure prophylaxis (PrEP) to avoid
inadvertent treatment of HIV infection with emtricitabine/tenofovir. This is the only
medication currently approved in the United States for PrEP, but it is inadequate for HIV
treatment. Using this treatment by itself in HIV-positive patients increases the risk of HIV
strains developing resistance to these antiviral agents. Other recommended testing prior to
PrEP use includes creatinine clearance calculation, hepatitis B antibody testing, screening for
sexually transmitted diseases, and pregnancy testing in females capable of pregnancy. The
CDC does not recommend testing liver function, hemoglobin, or platelet levels prior to PrEP
use in otherwise healthy individuals
Q2-24-year-old female presents with a painless ulcer on her labia, which has been present for a
week. You suspect primary syphilis, but a rapid plasma reagin (RPR) test is negative. Which
one of the following is the best strategy for confirming or ruling out syphilis in this situation?
A) A spinal fluid analysis
B) A serum fluorescent treponemal antibody absorption (FTA-ABS) test now
C) A Treponema pallidum particle agglutination (TPPA) test now
D) A Venereal Disease Research Laboratory (VDRL) test now
E) Repeating the RPR test in 2 weeks

ANSWER: E
A nontreponemal test, such as the rapid plasma reagin (RPR) test or Venereal Disease
Research Laboratory (VDRL) test, is the initial step for evaluating a patient with suspected
syphilis. These tests become positive
within 3 weeks of the appearance of the primary chancre, so they may be negative in patients
with an early infection. Darkfield microscopy of material obtained from a swab of the lesion is
often useful in this situation, but it requires special equipment and experienced technicians. If
there is a strong suspicion of syphilis, a repeat nontreponemal test in 2 weeks is indicated.
Patients with a positive nontreponemal test should be tested with a specific treponemal test for
confirmation. These tests may lack reactivity in early primary syphilis, however, and are not
indicated for use in the initial evaluation. Spinal fluid analysis is used only for the evaluation
of tertiary syphilis
Pnuemonia
Pretest2019
Q1. You are seeing an otherwise healthy 18-year-old man who has had an acute cough for 2
weeks. It started like a typical “cold,” but has persisted. Over the last 3 days, he reports
dyspnea and some wheezing. On examination, his temperature is 38.3°C, but his other vital
signs are normal. Chest auscultation reveals crackles in the left lower lobe. What is the
appropriate next step?
a. Supportive therapy without antibiotics
b. Treatment with a bronchodilator
c. A 10-day course of amoxicillin
d. A 5-day course of azithromycin
e. A posteroranterior (PA) and lateral chest film

The answer is e.
The causes of cough can range from a self-limited viral upper respiratory infection to severe
infections. Patient signs and symptoms will define the approach to take. The first factor to
consider is whether or not the patient has symptoms, signs, or risk factors that would warrant a
chest radiograph. Symptoms that a chest radiograph is necessary include dyspnea, high fever,
rigors, pleuritic chest pain, and altered mental status. Signs include a temperature greater than
38°C, heart rate greater than 100 beats/min, a respiratory rate greater than 24 breaths/min, and
an abnormal lung examination. Risk factors include elderly patients, those with known COPD,
heat failure, renal failure, or diabetes. Since the patient in this question has dyspnea, a high
fever, and an abnormal lung examination, the next thing to do would be to obtain a PA and
lateral chest x-ray to confirm the diagnosis of pneumonia.

Q2. In the patient described above, which of the following pathogens is the most likely
culprit in this case?
a. Chlamydia pneumoniae
b. Mycoplasma pneumoniae
c. H influenzae
d. Legionella
e. Respiratory syncytial virus
The answer is b.
The American College of Obstetrics and Gynecology recommends treating asymptomatic
bacteriuria in pregnancy, as 20% to 35% of the cases eventually develop into overt UTIs. In
the other cases above, treatment of asymptomatic bacteriuria is not indicated, as it has not been
shown to decrease morbidity and may increase the likelihood of developing resistant
microorganisms.
ABFM 2020
2 0 0 . A previously healthy 62-year-old female presents to your office with a 3-day history of
fever and a cough productive of purulent sputum. On examination she has a temperature of
39.2°C (102.6°F), a blood pressure of 110/70 mm Hg, a pulse rate of 92 beats/min, a
respiratory rate of 25/min, and an oxygen saturation of 94% on room air. She shows no signs
of confusion. An examination is significant for crackles at the right lower lung base and a
chest radiograph confirms an infiltrate in the same location.
Which one of the following treatment settings would be most appropriate at this time for this
patient’s community-acquired pneumonia?
1. Outpatient
2. The emergency department
3. A regular hospital inpatient floor
4. The intensive-care unit
ANSWER: A
The CRB-65 (confusion, respiratory rate, blood pressure, 65 years of age) rule is a validated
tool for risk stratification in the primary care setting. It can be used to determine who is a good
candidate for outpatient treatment of community-acquired pneumonia. Patients are given 1
point for each of the following signs or symptoms: new-onset confusion, a respiratory rate
>30/min, a blood pressure <90 mm Hg systolic or
<60 mm Hg diastolic, and an age of 65 years or older. Patients with 0 points, such as this
patient, are at low risk and can be managed in the outpatient setting unless there are other
significant comorbidities or social factors that make outpatient treatment contraindicated.
Patients with a score of 1–2 are at moderate risk and should be hospitalized in most cases.
Patients with a score of 3–4 are at high risk and should be considered for hospitalization in an
intensive-care unit.

137. In patients hospitalized with acute respiratory infections, procalcitonin levels are useful
for:
A. Nutritional status assessment
B. Reducing inappropriate use of antibiotics
C. Ruling out pulmonary embolism
D. Early identification of the syndrome of inappropriate secretion of antidiuretic hormone
(SIADH)
E. Identifying acute respiratory distress syndrome
ANSWER: B
Procalcitonin is a biomarker for the presence of severe bacterial infections such as pneumonia
and sepsis. Its utility in clinical decision-making is reducing unnecessary antibiotic use, which
reduces antibiotic resistance. Its use in guiding treatment is associated with reduced mortality
(SOR A) and with fewer antibiotic treatment days and fewer antibiotic complications (SOR
B). Procalcitonin levels are not useful in assessing nutritional status, ruling out pulmonary
embolism, early identification of the syndrome of inappropriate secretion of antidiuretic
hormone, or identifying acute respiratory distress syndrome.

ABFM2019
Q1-A 67-year-old male is admitted to the hospital for community-acquired pneumonia. An
examination reveals a temperature of 40.0°C (104.0°F), a respiratory rate of 50/min, a pulse
rate of 110 beats/min, a blood pressure of 90/50 mm Hg, and an oxygen saturation of 88% on
room air. The patient is confused and requires aggressive fluid resuscitation for hypotension
and he is transferred to the intensive-care unit. He has no known additional risk factors or
exposures.
In addition to treatment with ceftriaxone and azithromycin (Zithromax), which one of the
following medications is most likely to result in improved outcomes?
A. Clindamycin (Cleocin)
B. Levofloxacin (Levaquin)
C. Methylprednisolone (Medrol)
D. Oseltamivir (Tamiflu)

ANSWER: C
This patient has severe community-acquired pneumonia based on clinical criteria, including an
elevated respiratory rate, confusion, and hypotension requiring aggressive fluid resuscitation.
Corticosteroids such as methylprednisolone have been shown to improve clinical outcomes
such as length of stay, duration of antibiotic treatment, and the risk of developing adult
respiratory distress syndrome. The preferred choice of antibiotic treatment for patients in the
intensive-care unit is a -lactam antibiotic (ceftriaxone, cefotaxime) or ampicillin/sulbactam,
plus a macrolide alone or a macrolide and a respiratory fluoroquinolone. The addition of
levofloxacin is not necessarily preferred over just ceftriaxone and azithromycin. Clindamycin
is not indicated in the absence of risk factors for anaerobic infection such as aspiration or
alcoholism. Oseltamivir is not indicated in the absence of known or suspected influenza
infection.
Q2- A 69-year-old female presents to your office with a 5-day history of cough and low-grade
fever. She has a past history of hypertension and obstructive sleep apnea. Her daughter brought
her in this morning because of worsening symptoms. The patient’s temperature is 37.4°C
(99.3°F), her blood pressure is 110/74 mm Hg, her pulse rate is 88 beats/min, her respiratory
rate is 36/min, and her oxygen saturation is 95% on room air. She is alert and oriented to
person, place, and time. A CBC and basic metabolic panel are normal except for an elevated
WBC count of 12,500/mm3 (N 4300–10,800). A chest radiograph shows a right lower lobe
infiltrate.
This patient has a higher risk of mortality and should be considered for inpatient treatment due
to her
A. Female sex
B. Underlying hypertension
C. Respiratory rate
D. Elevated WBC count
E. Abnormal chest radiograph

ANSWER: C
There are several decision support tools to assist in predicting 30-day mortality for patients
with community-acquired pneumonia. Calculating the number of high-risk markers can aid in
deciding whether to admit the patient to the hospital. The risk of mortality increases with a
respiratory rate 30/min, hypotension, confusion or disorientation, a BUN level 20 mg/dL, age
>65 years, male sex, or thepresence of heart failure or COPD.

Q3- A 75-year-old male nursing home resident is brought to the emergency department with a
cough and fever. His past medical history is significant for coronary artery disease, COPD,
hypertension, and osteoarthritis. On examination he has a blood pressure of 145/90 mm Hg, a
pulse rate of 84 beats/min, and an oxygen saturation of 89% on room air. A physical
examination is remarkable for mildly labored breathing and crackles in his left lower lung
field. A chest radiograph confirms left lower lobe pneumonia. He is admitted to the hospital
for intravenous antibiotics.
Which one of the following would be the most appropriate antibiotic treatment?
A. Cefdinir only
B. Piperacillin/tazobactam (Zosyn) only
C. Ceftriaxone and azithromycin (Zithromax)
D. Cefixime (Suprax) and vancomycin (Vancocin)
E. Piperacillin/tazobactam, vancomycin, and ciprofloxacin (Cipro)
ANSWER: C
Current recommendations state that nursing home–acquired pneumonia should be treated as
community-acquired pneumonia unless patients have severe illness, chronic wounds, foreign
bodies in the airway, a history of antibiotic use in the last 90 days or recent hospitalization,
colonization with multidrug-resistant pathogens, or very low functional status, or reside in a
facility with a high prevalence of multidrug-resistant pathogens. Community-acquired
pneumonia should be treated with either a respiratory fluoroquinolone or an advanced
macrolide plus a -lactam antibiotic. Doxycycline could also be used in place of the macrolide.

Q4- A 25-year-old male presents to your office with a nearly 3-month history of a persistent,
mildly productive cough. He does not have any fevers, myalgias, or night sweats but is often
congested. He has never smoked and is normotensive.
An examination is notable for a normal
weight and vital signs, clear lungs, a mildly erythematous oropharynx, and pale, edematous
nasal mucosa. There is no lymphadenopathy.
Which one of the following is the most likely cause of this patient’s chronic cough?
A. Bronchogenic carcinoma
B. Chronic aspiration
C. Obstructive sleep apnea
D. Tuberculosis
E. Upper airway cough syndrome

ANSWER: E
The most common causes of chronic cough in adults include upper airway cough syndrome,
tobacco use, GERD, asthma, and ACE inhibitor use. The physical examination of this patient
is most consistent with upper airway cough syndrome, previously referred to as postnasal drip
syndrome. Given the patient’s lack of tobacco use and normal blood pressure, bronchogenic
carcinoma and obstructive sleep apnea are less likely. There are no risk factors in this patient’s
history to suggest chronic aspiration or tuberculosis.
Q5-A 28-year-old male presents to your office in January with a 1-day history of cough and
nasal congestion. He has not had any fever, shortness of breath, or chest pain. An examination
reveals some rhinorrhea and hoarseness. A lung examination is normal. The patient asks you
to prescribe albuterol (Proventil, Ventolin) because it seemed to help a friend who had similar
symptoms.

Which one of the following would be the most appropriate evidence-based response to the
patient’s request?
A. Do not prescribe albuterol
B. Prescribe albuterol alone
C. Prescribe albuterol and oral corticosteroids
D. Prescribe albuterol and inhaled corticosteroids

ANSWER: A
In a Cochrane review of five trials of 2-agonist therapy in adults, there was no significant
difference between 2-agonists and placebo in cough reduction. Based on reports of adverse
effects, the number needed to harm is 2. It is important toset reasonable expectations for cough
duration after an acute respiratory illness.

Q6- .A 60-year-old patient is admitted to the hospital for a COPD exacerbation. For this
patient, which one of the following would be the most appropriate prednisone dosage?
A. 40 mg daily for 5 days
B. 40 mg daily for 10 days
C. 60 mg daily, tapered over 6 days
D. 60 mg daily for 10 days

ANSWER: A
For patients with a COPD exacerbation, systemic glucocorticoids can improve FEV1, improve
oxygenation, shorten recovery time, and reduce the length of hospitalization (level of evidence
A). Prednisone, 40 mg daily for 5 days, is recommended for COPD exacerbations. Studies
have shown that oral administration is equally efficacious compared to the intravenous route.
The duration of therapy should not be longer than 5–7 days (level of evidence A). Q7-A 58-
year-old female presents to your office to discuss a new diagnosis of COPD. She has a 40-
pack-year smoking history, and she quit using tobacco 18 months ago. Spirometry performed
last week showed an FEV1/FVC ratio of 0.62 with an FEV1 that is 75% of predicted. She was
first treated for an exacerbation
last month and has never required hospitalization for any respiratory illness. You administer a
COPD assessment test and she rates herself at 16/40, which is a moderately high score.
Which one of the following inhaled medications would be appropriate to initiate today?
A. Albuterol (Proventil, Ventolin)
B. Fluticasone (Flovent)
C. Fluticasone/salmeterol (Advair)
D. Ipratropium (Atrovent)
E. Tiotropium (Spiriva)

ANSWER: E
This patient has airflow obstruction consistent with a diagnosis of COPD, evidenced by an
FEV1/FVC ratio <0.7. An evidence-based standard approach to COPD is found in the annual
guidelines published by the Global Initiative for Chronic Obstructive Lung Disease (GOLD).
According to the 2019 report, this patient’s FEV1 of 75% of predicted puts her in the GOLD
grade 2 (moderate) category of airflow limitation. Her symptoms and risk of exacerbations
places her in GOLD group B. Patients in this category have symptoms that bother them
regularly without having frequent COPD exacerbations. Patients in this category benefit from
daily use of long-acting bronchodilators, either long-acting -agonists (LABAs) or long-acting
muscarinic agents (LAMAs). Long-acting agents such as tiotropium (a LAMA) or salmeterol
(a LABA) are preferred over the short-acting agents ipratropium and albuterol for patients in
this category of disease severity. Patients with persistent symptoms while using one of these
agents may benefit from a combination of a LABA and a LAMA.
Monotherapy with inhaled corticosteroids has not been shown to improve mortality or prevent
a long-term decline in FEV1. The combination of an inhaled corticosteroid plus a LABA has
evidence of superiority over either agent alone for improving lung function and health status,
and for reducing exacerbations in patients who have more severe disease. No improvement in
all-cause mortality has been noted.
ABFM 2018
Q8-A 68-year-old male presents to your office with a 2-day history of headache, muscle aches,
and chills. His wife adds that his temperature has been up to 104.1°F and he seems confused
sometimes. His symptoms have not improved with usual care, including ibuprofen and
increased fluid intake. He and his wife returned from a cruise 10 days ago but don’t recall
anyone having a similar illness on the ship. This morning he started to cough and his wife was
concerned because she saw some blood in his sputum. He also states that he experiences
intermittent shortness of breath and feels nauseated. His blood pressure is 100/70 mm Hg,
heart rate 98/min, temperature 39.4°C (102.9°F), and oxygen saturation 95% on room air.
Which one of the following would be the preferred method to confirm your suspected
diagnosis of Legionnaires’ disease?

A) Initiating azithromycin (Zithromax) to see if symptoms improve


B) A chest radiograph C) Legionella polymerase chain reaction (PCR) testing
D) A sputum culture for Legionella
E) Urine testing for Legionella pneumophila antigen

ANSWER: E
A urine test for Legionella pneumophila antigen is the preferred method to confirm
Legionnaires’ disease. This test is rapid and will only detect Legionella pneumophila antigen.
A sputum culture is the gold standard for the diagnosis of Legionnaires’ disease but it requires
48–72 hours. A chest radiograph does not confirm the diagnosis but may show the extent of
disease. Responding to antibiotic treatment does not confirm a specific diagnosis.

Q9-A previously healthy 57-year-old patient who smokes is hospitalized and treated with a
fluoroquinolone for community-acquired pneumonia. Which one of the following could be
expected with a 5-day course of antibiotics compared to a longer course in patients such as
this?
A) Slower clinical improvement
B) Higher hospital readmission rates
ABFM2017
Q10 -A 12-year-old male has a 1-week history of fever, headache, sore throat, and a mildly
productive cough. He also began having ear pain yesterday. On examination he does not
appear to be toxic. He has a temperature of 37.8°C (100.0°F). Examination of his ears shows a
bulla on the right tympanic membrane, as well as mild to moderate erythema of the posterior
pharynx. The neck is supple. The lungs have a few scattered crackles. The remainder of the
examination is unremarkable. A chest radiograph reveals thickened bronchial shadows, as well
as interstitial infiltrates in the lower lobes.
The most appropriate treatment at this time would be
A) amoxicillin
B) azithromycin (Zithromax)
C) ceftriaxone (Rocephin)
D) cefuroxime (Zinacef)
E) vancomycin

ANSWER: B
Community-acquired pneumonia in children is treated based on age. The most likely etiologic
agents in a school-age child are Mycoplasma pneumoniae, Chlamydia pneumoniae, and
Streptococcus pneumoniae. Group A Streptococcus and Haemophilus influenzae are less
common causes. Staphylococcus aureus that is methicillin-resistant has become increasingly
common. The preferred treatmentfor community-acquired
In children ages 5–16, Mycoplasma pneumonia tends to have a gradual onset of symptoms and
seldom causes respiratory distress. Signs and symptoms may vary. The patient may develop a
rash, musculoskeletal symptoms, or gastrointestinal symptoms. Radiographs may reveal
bronchopneumonia, nodular infiltrates, hilar adenopathy, pleural effusions, or plate-like
atelectasis. Ear pain may be due to bullous myringitis, although this may be viral as well.
Laboratory findings may not be helpful, as the WBC count may be normal or slightly elevated.
There may be thrombocytosis, an elevated erythrocyte sedimentation rate, an elevation of cold
agglutinins, or an elevated reticulocyte count. A Coombs test is seldom needed, although it
might be helpful at times. The diagnosis is generally made on a clinical basis.
Q11- .A 56-year-old male presents with a 2-day history of a fever and productive cough. He
has mild dyspnea with exertion and has pain in his right side when he takes a deep breath. On
examination
his temperature is 38.4°C (101.1°F), his respiratory rate is 24/min, his pulse rate is 92
beats/min, and his oxygen saturation is 92% on room air. He has crackles in the right lower
lung posteriorly. The remainder of the examination is normal. The most likely diagnosis is
A) upper respiratory infection
B) community-acquired pneumonia
C) heart failure
D) pulmonary embolus
E) acute leukemia

ANSWER: B
This patient has pneumonia based on the clinical presentation and the physical findings of
fever, cough, and abnormal lung findings. A fever would not be a typical finding in pulmonary
embolus or heart failure. An upper respiratory infection is unlikely given the abnormal lung
findings that suggest a lower respiratory tract infection. This would not be a typical
presentation for acute leukemia.

Q12-A 74-year-old male with a history of diabetes mellitus, hypertension, and heart failure
presents to the clinic with shortness of breath with an unknown etiology. Laboratory results
that show an elevated procalcitonin level would help differentiate systolic heart failure from
A) diastolic heart failure
B) bacterial pneumonia
C) acute coronary syndrome
D) pulmonary embolus
Q13-17-A 67-year-old female sees you because of a cough she has had for the past few days
and a fever that started today. She is short of breath and generally does not feel well. She has
no history of lung disease and is a nonsmoker. Her medical history is significant for
hypertension, hyperlipidemia, and type 2 diabetes mellitus, all of which are well managed
with medications and diet.
A physical examination reveals a mildly ill-appearing female with a temperature of 38.2°C
(100.8°F), a pulse rate of 90 beats/min, a respiratory rate of 21/min, a blood pressure of
110/60 mm Hg, and an oxygen saturation of 98% on room air.
Her heart has a regular rhythm and her respirations appear unlabored. She has rhonchi in the
left lower lung field but has good air movement overall. A chest radiograph reveals a left
lower lobe infiltrate.
Which one of the following is the most appropriate setting for the management of this patient’s
pneumonia?
A) Home with close monitoring
B) An inpatient medical bed without telemetry monitoring
C) An inpatient medical bed with telemetry monitoring
D) An inpatient intensive care bed.

ANSWER: A
For community-acquired pneumonia, an important decision point is the severity of illness that
indicates the need for inpatient care. There are multiple tools for evaluation of pneumonia
severity, including SMART-COP (predicts the likelihood of the need for invasive ventilation
or vasopressor support), the Pneumonia Severity Index (predicts the risk of 30-day mortality
and the need for admission to the intensive-care unit), and CURB-65 or CRB-65. In an
outpatient setting, CURB- 65 and CRB-65 are easy to use, although they have weaker
predictive values for30-day mortality. In addition, clinical judgment should always be used. In
this scenario, the patient does not clinically appear markedly ill, and her vital signs and
physical examination do not fit any criteria for increased risk in any of the scoring systems. -
65 or CRB-65 can be managed as outpatients.
Q14- A 62-year-old male sees you the day after returning from a 4- day cruise. He says he
developed a fever and a productive cough on the day before the ship returned to Los Angeles
following a trip down the coast of Baja California. He tells you that several other passengers
had similar symptoms. The examination is remarkable for tachypnea and you hear crackles in
both lungs. This patient’s history should raise concerns about infection with which one of the
following pathogens?
A) Asian avian influenza A virus
B) Coxiella burnetii
C) Hantavirus
D) Histoplasma capsulatum
E) Legionella species

ANSWER: E
Legionella should be considered as a pathogen for community- acquired pneumonia when the
patient has a history of a hotel stay or cruise ship travel within the past couple of weeks.
Travel to or residence in Southeast Asia or East Asia is a risk factor for avian influenza,
exposure to farm animals or parturient cats is a risk factor for Coxiella burnetii infection,
exposure to bird or bat droppings is a risk factor for Histoplasma capsulatum infection, and
travel to or residence in desert Southwest states with deer mouse exposure is a risk factor for
Hantavirus infection.

ABFM 2015
Q16 : A 4-week-old white male is brought to your office with a 2-week history of increasing
dyspnea, cough, and poor feeding. The child appears nontoxic and is afebrile. On examination
you note conjunctivitis, and a chest examination reveals tachypnea and crackles. A chest film
shows hyperinflation and diffuse interstitial infiltrates and a WBC count reveals eosinophilia.
What is the most likely etiologic agent?
A) Staphylococcus species
B) Chlamydia trachomatis
C) Respiratory syncytial virus
E)Parainfluenza virus
ANSWER: B
Chlamydial pneumonia is usually seen in infants 3–16 weeks of age, and these patients
frequently have been sick for several weeks. The infant appears nontoxic and is afebrile, but is
tachypneic with a prominent cough. The physical examination will reveal diffuse crackles
with few wheezes, and conjunctivitis is present in about 50% of cases. A chest film will show
hyperinflation and diffuse interstitial or patchy infiltrates.
Staphylococcal pneumonia has a sudden onset. The infant appears very ill and has a fever, and
initially may have an expiratory wheeze simulating bronchiolitis. Signs of abdominal distress,
tachypnea, dyspnea, and localized or diffuse bronchopneumonia or lobar disease may be
present. The WBC count will show a prominent leukocytosis.
Respiratory syncytial virus infections start with rhinorrhea and pharyngitis, followed in 1–3
days by a cough and wheezing. Auscultation of the lungs will reveal diffuse rhonchi, fine
crackles, and wheezes, but the chest film is often normal. If the illness progresses, coughing
and wheezing increase, air hunger and intercostal retractions develop, and evidence of hyper
expansion of the chest is seen. In some infants the course of the illness may be similar to that
of pneumonia. Rash or conjunctivitis may occur occasionally, and fever is an inconsistent
sign. The WBC count will be normal or elevated, and the differential may be normal or shifted
either to the right or left. Chlamydial infections can be differentiated from respiratory syncytial
virus infections by a history of conjunctivitis, the subacute onset and absence of fever, and the
mild wheezing. There may also be eosinophilia. Parainfluenza virus infection presents with
typical cold symptoms. Eight percent of infections affect the upper respiratory tract. In
children hospitalized for severe respiratory illness, parainfluenza viruses account for about
50% of the cases of laryngotracheitis and about 15% each of the cases of bronchitis,
bronchiolitis, and pneumonia. Respiratory syncytial virus infections start with rhinorrhea and
pharyngitis, followed in 1–3 days by a cough and wheezing. Auscultation of the lungs will
reveal diffuse rhonchi, fine crackles, and wheezes, but the chest film is often normal. If the
illness progresses, coughing and wheezing increase, air hunger and intercostal retractions
develop, and evidence of hyper expansion of the chest is seen. In some infants the course of
the illness may be similar to that of pneumonia. Rash or conjunctivitis may occur
occasionally, and fever is an inconsistent sign. The WBC count will be normal or elevated,
and the differential may be normal or shifted either to the right or left. Chlamydial infections
can be differentiated from respiratory syncytial virus infections by a history of conjunctivitis,
the subacute onset and absence of fever, and the mild wheezing. There may also be
eosinophilia. Parainfluenza virus infection presents with typical cold symptoms. Eight percent
of infections affect the upper respiratory tract. In children hospitalized for severe respiratory
illness, parainfluenza viruses account for about 50% of the cases of laryngotracheitis and
about 15% each of the cases of bronchitis, bronchiolitis, and pneumonia.
Q18-A 3-year-old female is brought to your office with coughing and a tactile fever. Her only
other symptom is mild rhinorrhea. She has a temperature of 38.2°C (100.8°F) and is mildly
tachypneic. Her vital signs are otherwise normal and she appears to be well and in no
respiratory
distress. Her examination is unremarkable except for decreased breath sounds and crackles in
the right lower lung field. She has no allergies to medications. Which one of the following
would be the most appropriate treatment?
A) Amoxicillin
B) Azithromycin (Zithromax)
C) Cefdinir
D) Moxifloxacin (Avelox)
E) Ceftriaxone (Rocephin)
ANSWER: A
Amoxicillin is the recommended first-line treatment for previously healthy infants and school-
age children with mild to moderate community-acquired pneumonia (CAP) (strong
recommendation; moderate-quality evidence). The most prominent bacterial pathogen in CAP
in this age group is Streptococcus pneumoniae, and amoxicillin provides coverage against this
organism. Azithromycin would be an appropriate choice in an older child because
Mycoplasma pneumoniae would be more common. Moxifloxacin should not be used in
children. Ceftriaxone and cefdinir can both be used to treat CAP, but they are broader
spectrum antibiotics and would not be a first-line choice in this age group.

Q19-A 25-year-old male presents with a 3-day history of cough, chills, and fever. The patient
was previously healthy and has no chronic medical problems. He has no known drug allergies.
On examination he is alert and oriented, and has a temperature of 38.4°C (101.1°F), a pulse
rate of 88 beats/min, a blood pressure of 120/70 mm Hg, a respiratory rate of 16/min, and an
oxygen saturation of 98%.
Auscultation of the lungs reveals no wheezing and the presence of right basilar crackles. A
chest radiograph shows a right lower lobe infiltrate.
There is a low rate of macrolide-resistant pneumococcus in the community. Which one of the
following is the most appropriate initial management of this patient?
A) Outpatient treatment with azithromycin (Zithromax)
B) Outpatient treatment with cefuroxime (Ceftin)
C) Inpatient treatment on the medical floor with ceftriaxone (Rocephin) and azithromycin
D) Inpatient treatment on the medical floor with piperacillin/tazobactam (Zosyn) and
levofloxacin
Inpatient treatment in the intensive-care unit with ceftriaxone, levofloxacin, and vancomycin
(Vancocin)
ANSWER: A
In patients with community-acquired pneumonia it is necessary to decide on both the antibiotic
regimen and the treatment setting. The decision regarding site of care is based on the severity
of illness, which can be assessed with tools such as the CURB-65 score, which take into
account factors such as respiratory rate, blood pressure, uremia, confusion, and age. Patients
who have only mild symptoms can be treated with azithromycin on an outpatient basis if there
is a low level of macrolide resistance in the community. If there is a high level of resistance in
the community, if the patient has comorbidities such as diabetes mellitus or COPD, or if there
is a history of use of an immunosuppressing drug or recent use of an antibiotic, the patient can
still be treated as an outpatient but should be treated with levofloxacin. Patients with more
severe symptoms, such as an elevated pulse rate or respiratory rate, should be treated on an
inpatient basis with ceftriaxone or azithromycin. Patients who have more severe symptoms
along with bronchiectasis should be treated with piperacillin/tazobactam plus levofloxacin.

Q20-Patients with the most severe symptoms, including hypotension, a more elevated pulse
rate, low oxygen saturation, and confusion, should be treated in the intensive-care unit with
levofloxacin and vancomycin.

Which one of the following is associated with treatment of COPD with inhaled corticosteroids?
A) An increased risk of monilial vaginitis
B) An increased risk of bruising
C) Consistent improvement in FEV1
D) A decreased risk of pneumonia
E) Decreased mortality

ANSWER: B

Inhaled corticosteroids increase the risk of bruising, candidal infection of the oropharynx, and
pneumonia. They also have the potential for increasing bone loss and fractures. They decrease
the risk of COPD exacerbations but have no benefit on mortality and do not improve FEV1 on
a consistent basis..
Q21-A 63-year-old female with corticosteroid-dependent COPD has developed pneumonia.
Which one of the following pathogens should the antibiotic regimen cover in this patient that
would be
unlikely in someone with pneumonia and otherwise healthy lungs?
A) Streptococcus pneumoniae
B) Mycoplasma pneumoniae
C) Haemophilus influenzae
D) Staphylococcus aureus
E) Pseudomonas aeruginosa

ANSWER: E
All of the pathogens listed can cause pneumonia in any patient. However, in patients with
chronic lung disease who are taking corticosteroids, Pseudomonas is more common than in
those with otherwise healthy lungs. The antibiotics chosen empirically should cover this
pathogen.

Q22-A 58-year-old male with COPD presents with a 5-day history of increased dyspnea and
purulent sputum production. He is afebrile. His respiratory rate is 24/min, heart rate 90
beats/min, blood pressure 140/80 mm Hg, and oxygen saturation 90% on room air. Breath
sounds are equal, and diffuse bilateral rhonchi are noted. He is currently using
albuterol/ipratropium by metered-dose inhaler three times daily. In addition to antibiotics,
which one of the following would be most appropriate for treating this exacerbation?

A) A single dose of intramuscular dexamethasone Oral prednisone for 5 days


B) Daily inhaled fluticasone (Flovent)
C) Hospital admission for intravenous methylprednisolone sodium succinate(Solu-Medrol)
D) No corticosteroids at this time
ANSWER: B
This patient most likely has a mild to moderate COPD exacerbation. His vital signs do not
indicate a serious condition at this time, so he can be treated as an outpatient. Since he is
already on a reasonable
dose of an inhaled bronchodilator/anticholinergic combination, he should be treated with an
oral antibiotic and an oral corticosteroid. Intravenous corticosteroids offer no advantages over
oral therapy, provided there are no gastrointestinal tract limitations such as poor motility or
absorption . Oral corticosteroid therapy initiated early in a COPD exacerbation reduces the rate
of treatment failure,
decreases hospitalization rates, improves hypoxia and pulmonary function, and shortens the
length of stay for patients requiring hospitalization. Short courses of oral corticosteroids (5–7
days) are as effective as longer ones (SOR A). Inhaled corticosteroids are ineffective in the
treatment of a COPD exacerbation.
Intramuscular dexamethasone has no role in treating COPD.

ABFM 2014
Q23-A 25-year-old male presents with a 3-day history of cough, chills, and fever. The patient
was previously healthy and has no chronic medical problems. He has no known drug allergies.
On examination he is alert and oriented, and has a temperature of 38.4°C (101.1°F), a pulse
rate of 88 beats/min, a blood pressure of 120/70 mm Hg, a respiratory rate of 16/min, and an
oxygen saturation of 98%.
Auscultation of the lungs reveals no wheezing and the presence of right basilar crackles. A
chest radiograph shows a right lower lobe infiltrate.
There is a low rate of macrolide-resistant pneumococcus in the community. Which one of the
following is the most appropriate initial management of this patient?

A) Outpatient treatment with azithromycin (Zithromax)


B) Outpatient treatment with cefuroxime (Ceftin)
C) Inpatient treatment on the medical floor with ceftriaxone (Rocephin) and azithromycin
D) Inpatient treatment on the medical floor with piperacillin/tazobactam (Zosyn) and
levofloxacin
E) Inpatient treatment in the intensive-care unit with ceftriaxone, levofloxacin, and
vancomycin (Vancocin)
ANSWER: A
In patients with community-acquired pneumonia it is necessary to decide on both the antibiotic
regimen and the treatment setting. The decision regarding site of care is based on the severity
of illness, which can be assessed with tools such as the CURB-65 score, which take into
account factors such as respiratory rate, blood pressure, uremia, confusion, and age. Patients
who have only mild symptoms can be treated with azithromycin on an outpatient basis if there
is a low level of macrolide resistance in the community. If there is a high level of resistance in
the community, if the patient has comorbidities such as diabetes mellitus or COPD, or if there
is a history of use of an immunosuppressing drug or recent use of an antibiotic, the patient can
still be treated as an outpatient but should be treated with levofloxacin. Patients with more
severe symptoms, such as an elevated pulse rate or respiratory rate, should be treated on an
inpatient basis with ceftriaxone or azithromycin. Patients who have more severe symptoms
along with bronchiectasis should be treated with piperacillin/tazobactam plus levofloxacin
Patients with the most severe symptoms, including hypotension, a more elevated pulse rate,
low oxygen saturation, and confusion, should be treated in the intensive-care unit with
levofloxacin and vancomycin.

Q24- While making rounds on the rehabilitation floor of your hospital, you see a 62-year-old
female who was recently transferred from the acute-care section of the hospital where she was
admitted for urosepsis. She is a liver-transplant recipient and her specialist has been tapering
her immunosuppressive drug regimen for the last 2 months. According to the nursing staff the
patient became hypoxic suddenly and had a low-grade fever and cough. You note that she
looks ill and uncomfortable, and has an increased respiratory rate. A chest radiograph reveals
diffuse bilateral interstitial infiltrates.
Which one of the following is the most likely diagnosis?
A) Pneumococcal pneumonia
B) Staphylococcal pneumonia
C) Pneumocystis pneumonia
D) Pulmonary tuberculosis
E) Pneumothorax
ANSWER: C
The most likely diagnosis is Pneumocystis pneumonia. Initially named Pneumocystis carinii,
the causative organism has been reclassified and renamed Pneumocystis jiroveci. It causes
disease IN immunocompromised patients. In non–HIV-infected patients, the most significant
risk factors are defects in cell- mediated immunity, glucocorticoid therapy, use of
immunosuppressive agents (especially when dosages are being lowered), hematopoietic stem
cell or solid organ transplant, cancer, primary immunodeficiencies, and severe malnutrition.
The clinical presentation in patients without HIV/AIDS is typically an acute onset of hypoxia
and respiratory failure, associated with a dry cough and fever.
Characteristic radiographic findings include diffuse bilateral interstitial infiltrates.
Pneumococcal pneumonia typically presents with fever, chills, cough, and pleuritic chest pain.
A sudden onset of severe hypoxia is less common. Radiologic findings typically include lobar
infiltrates or bronchopneumonia (with a segmental pattern of infiltrate), whereas diffuse
bilateral infiltrates are much less common. Staphylococcal pneumonia usually has radiologic
findings of focal, multiple infiltrates or cavitary lesions.
Pulmonary tuberculosis presents most commonly with pleuritic or retrosternal chest pain.
Fever is present in about 25% of patients. Cough is actually less common, and a sudden onset
of acute hypoxia would be a very rare presentation. Radiographs typically reveal hilar
adenopathy and pleural effusion. Diffuse bilateral interstitial infiltrates would be a very rare
finding.
Spontaneous pneumothorax does present with an acute onset of hypoxia, tachypnea, and
respiratory distress. However, fever would be unlikely and the radiologic findings in this
patient are not consistent with pneumothorax.

Q24- A 25-year-old male presents to your office after recently being diagnosed with HIV
infection at the health department. You obtain blood work and note that his CD4+ count is 180
cells/mm3.
This patient should receive prophylaxis against which one of the following opportunistic
infections?
A) Histoplasma capsulatum
B) Microsporidiosis
C) Mycobacterium avium-intracellulare complex
D) Pneumocystis
E) Toxoplasma gondii
ANSWER: D
Patients with HIV infection and severe immunodeficiency are at risk for certain opportunistic
infections. Susceptibility to opportunistic infections can be measured by CD4+ T lymphocyte
counts. Patients with a CD4+ count <200 cells/mm3 should receive
trimethoprim/sulfamethoxazole for prevention of Pneumocystis pneumonia, and prophylaxis
against Toxoplasma gondii should also be given if the CD4+ level is <100 cells/mm3.
Azithromycin is used to prevent infection with Mycobacterium avium-intracellulare complex
when CD4+ counts are <50 cells/mm3. Itraconazole is used to prevent Histoplasma
capsulatum infection when the CD4+ count is £150 cells/mm3 if the patient is at risk due to
occupational exposure or living in a community with a hyperendemic rate of histoplasmosis
(>10 cases per 100 patient years). There is no recommendation for prophylaxis against
microsporidiosis.
Q25-A 28-year-old previously healthy male nonsmoker has a 3-day history of fever and a
productive cough. He presents to the urgent care clinic for evaluation after developing pain in
the right lower chest when breathing deeply. He has not sought medical care for over 5 years
and has never been immunized for influenza. On examination you note a temperature of
38.6°C (101.4°F), a blood pressure of 136/74 mm Hg, a pulse rate of 90 beats/min, an oxygen
saturation of 93% on room air, and a respiratory rate of 20/min. The patient appears
uncomfortable but is not in significant distress. The presence of crackles over the right lower
anterior chest prompts an order for chest radiography, which reveals an air bronchogram and a
patchy alveolar infiltrate involving the medial middle lobe.
Which one of the following treatment options would be most appropriate at this time?
A) Outpatient treatment with oral azithromycin (Zithromax)
B) Outpatient treatment with oral ciprofloxacin (Cipro)
C) Outpatient treatment with oseltamivir (Tamiflu)
D) Inpatient treatment with intravenous ceftriaxone (Rocephin) and oral azithromycin
E) Inpatient treatment with intravenous ceftriaxone and ciprofloxacin
ANSWER: A
This patient’s presentation is consistent with community-acquired pneumonia (CAP).
Pathogens commonly involved include viruses such as influenza, as well as Mycoplasma
pneumoniae and Streptococcus pneumoniae. This patient’s history and findings are most
consistent with early lobar pneumonia, given the sputum production, presence of rales, and
radiographic findings, and empiric antibiotic treatment is most appropriate. His premorbid
history of good health and the lack of findings such as confusion, tachypnea, hypotension, or
multilobar infiltrates that would indicate severe CAP make outpatient antibiotic treatment the
most appropriate option. He is outside of the time frame when anti-influenza treatments would
be expected to be effective, even if influenza seemed likely. For previously healthy
individuals who have not taken antibiotics in the previous 3 months the most appropriate
treatment for CAP is empiric treatment with an oral macrolide such as azithromycin,
clarithromycin, or erythromycin (level I evidence) or doxycycline (level III evidence). In the
presence of comorbidities such as diabetes, alcoholism, or chronic heart, lung, liver, or renal
diseases, the treatment of CAP should provide broader coverage with dual antibiotic treatment
regimens including combinations of fluoroquinolones, $-lactam drugs, and macrolide options,
and hospitalization is often indicated
Q26-A 6-month-old male is seen in the clinic for a cough, fever, and decreased feeding for the
past 2 days. He has no medical problems and no known drug allergies, and has not had nausea
or vomiting. On examination he is fussy but consolable and his mucous membranes are moist.
His temperature is 38.8°C (101.8°F), pulse rate 92 beats/min, respirations 56/min, oxygen
saturation 96% on room air, and blood pressure 90/50 mm Hg. He has mild intercostal
retractions and crackles are heard at the right lung base. His heartbeat is regular without
murmurs. Capillary refill is normal.
Which one of the following is the best initial treatment for this child?
A) High-dose oral amoxicillin
B) Oral azithromycin (Zithromax)
C) Ceftriaxone (Rocephin) intravenously or intramuscularly every 24 hours
D) Trimethoprim/sulfamethoxazole (Bactrim)
E) Erythromycin

ANSWER: A
Oral antibiotics are preferred for community-acquired pneumonia if the patient is able to take
them. The antibiotic of choice is amoxicillin for children 60 days to 5 years of age because of
its activity against Streptococcus pneumoniae.
Azithromycin is the antibiotic of choice for children 5–16 years old because of its activity
against Mycoplasma pneumoniae and Chlamydia pneumoniae, and it can be used in children
between the ages of 60 days and 5 years who are allergic to penicillin. Ceftriaxone is not a
preferred antibiotic

Q27- A 2-month-old female is brought to your office with tachypnea and a staccato cough.
She is afebrile. A chest radiograph shows hyperinflation and bilateral infiltrates, and a CBC
reveals eosinophilia.
Which one of the following is the most likely etiologic agent?
A) Chlamydia trachomatis
B) Listeria pneumoniae
C) Streptococcus pneumoniae
D) A gram-negative bacteria
A) Respiratory syncytial virus
ANSWER: A
This infant has the typical findings of chlamydial pneumonia, which usually develops 1–3
months after birth and should be suspected in a young infant who has tachypnea, a staccato
cough, and no fever (SOR A). Radiographs often show hyperinflation and infiltrates, and a
CBC will reveal eosinophilia
Q28- A 3-year-old female is brought to your office with coughing and a tactile fever. Her only
other symptom is mild rhinorrhea. She has a temperature of 38.2°C (100.8°F) and is mildly
tachypneic. Her vital signs are otherwise normal and she appears to be well and in no
respiratory distress. Her examination is unremarkable except for decreased breath sounds and
crackles in the right lower lung field. She has no allergies to medications.
Which one of the following would be the most appropriate treatment?

A) Amoxicillin
B) Azithromycin (Zithromax)
C) Cefdinir
D) Moxifloxacin (Avelox)
E) Ceftriaxone (Rocephin)

ANSWER: A
Amoxicillin is the recommended first-line treatment for previously healthy infants and school-
age children with mild to moderate community-acquired pneumonia (CAP) (strong
recommendation; moderate-quality evidence). The most prominent bacterial pathogen in CAP
in this age group is Streptococcus pneumoniae, and amoxicillin provides coverage against this
organism. Azithromycin would be an appropriate choice in an older child because
Mycoplasma pneumoniae would be more common. Moxifloxacin should not be used in
children. Ceftriaxone and cefdinir can both be used to treat CAP, but they are broader
spectrum antibiotics and would not be a first-line choice in this age group.
Thyroid
Pretest2019

Q1. A 45-year-old woman presents to your office for follow-up. She has a long history of
hypothyroidism and is taking thyroid replacement therapy. She has been on 50 µg of
levothyroxine daily for around 1 year but reports that she’s recently felt fatigued and has
gained some weight. You obtain a TSH test, and it comes back high at 7.2 mIU/mL. You
decide to increase her dosage of levothyroxine in an attempt to get her euthyroid. When
should you recheck the TSH to gauge the new dosage?
a. 2 weeks after the dose adjustment.
b. 6 weeks after dose adjustment.
c. 3 months after dose adjustment.
d. 6 months after dose adjustment.
e. TSH should not be used to gauge dose adjustments.
The answer is b.
Thyroid disorders are common, with hypothyroidism being much
more common than hyperthyroidism, nodular disease, or thyroid cancer. The incidence of
hypothyroidism is 7% of women and 3% of men aged 60 to 89 years. Once diagnosed, the
TSH level is the most important measure to gauge the dose of thyroid replacement therapy.
Dose of thyroid replacement therapy will vary depending on age, weight, cardiac status, and
severity of the condition. Once therapy is adjusted, a TSH level should be checked in 6 weeks
and therapy shouldbe titrated to keep the TSH in the normal range.

Q2. You are caring for a 35-year-old man who is complaining of fatigue and an inability to
gain weight. Laboratory evaluation reveals a TSH of 6.0 mIU/L (high) but a normal free T4.
Which of the following is the best next step?
a. Test for antithyroid peroxidase
b. Test for thyroid autoantibodies
c. Treat with levothyroxine
d. Treat with levothyroxine and T3
e. Monitor at yearly intervals
The answer is e.
Subclinical hypothyroidism is distinguished by an elevated TSH and a normal free T4. This
condition occurs in 4% to 8% of the general population. It will progress to clinical
hypothyroidism at a rate of 2% to 5% per year. Risk for progression includes the presence of
thyroid autoantibodies, old age, a female gender, and a TSH level greater than mIU/L. Patients
who do not progress are considered euthyroid with a reset thyrostat, and they should be
monitored clinically and biochemically on an annual basis.
Antithyroid peroxidase levels would help to diagnose autoimmune thyroiditis, but would not
be helpful in this case. Starting thyroxine would be appropriate if the TSH was greater than 10
mIU/L, but not if the levels were only slightly elevated.

Q3. A 26-year-old woman presents with weight gain, lethargy, dry skin, sweatiness, cold
intolerance, and thinning hair. You suspect hypothyroidism and order the appropriate
laboratory tests. Her TSH is high, and her free T3 and freeT4 are both low. Which of the
following is the most likely diagnosis?
a. Primary hypothyroidism
b. Secondary hypothyroidism
c. Iodine deficiency
d. Thyroid hormone resistance
e. Subclinical hypothyroidism

The answer is a.
Primary hypothyroidism is common, usually a result of Hashimoto thyroiditis or after Graves’
disease. In this case, the TSH would be elevated, and the free T3 and T4 would be low.
Secondary hypothyroidism is related to hypothalamic or pituitary dysfunction. Iodine
deficiency is a cause of primary hypothyroidism.
Subclinical hypothyroidism is when the TSH is elevated, but the T3 and T4 are normal.
Thyroid hormone resistance would present with the TSH, T3, and T4 allbeing elevated.

Q4. You are screening a 35-year-old woman who presents with tachycardia, nervousness,
tremor, palpitations, heat intolerance, and weight loss. You suspect Graves’ disease. What
would you expect to find on radionucleotide uptake scanning of the thyroid with Graves’
disease?
a. Diffusely decreased uptake
b. Patchy uptake
c. Nodular areas of increased uptake
d. Areas of increased uptake surrounded by hypoactivity
e. Diffusely increased uptake

The answer is e.
Once hyperthyroidism is identified, radionucleotide uptake and scanning of the thyroid is
useful to determine whether the problem is due to Graves’ disease or another cause. In scans
of patients with Graves’ disease, there is increased uptake on radionucleotide imaging with
diffuse hyperactivity.
Nodules demonstrate limited areas of uptake on a radionucleotide scan with surrounding
hypoactivity. In subacute thyroiditis, uptake is patchy and decreased overall.
Q5. You have confirmed that the patient above has Graves’ disease. Which of the followingis
the treatment of choice?
a. Radioactive iodine
b. Methimazole
c. Propylthiouracil
d. β-blockade
e. Surgery
The answer is a.
Radioactive iodine is the treatment of choice for Graves’ disease in adults who are not
pregnant. Methimazole is the drug of choice in patients who choose antithyroid drug therapy,
except during the first trimester of pregnancy when propylthiouracil is preferred. β-Blockade
is not a treatment, but can be given to patients with symptomatic thyrotoxicosis or patients
with resting heart rates of more than 90 beats/min. Surgery is reserved for patients in whom
medication and radioactive iodine ablation are not acceptable, or in patients where a large
goiter is compressing nearby structures or is disfiguring.

Q6. You note a thyroid nodule in a 35-year-old African-American woman who is not
demonstrating clinical features of thyroid dysfunction. Which of the following patient
characteristics, or features (if present), would increase your suspicion of malignancy?
a. The patient’s age
b. The patient’s race
c. The patient’s sex
d. If the nodule were slowly growing on follow-up
e. If the patient complained of persistent hoarseness

The answer is e.
Several features of the history are associated with an increased risk of malignancy in a thyroid
nodule. These include prior history of head and neck irradiation, or a family history of
endocrine cancers. Also included are age less than 20 years or more than 70 years, male
gender, and rapid growth of the nodule. Physical findings that raise clinical suspicion include
a nodule with a firm consistency, cervical adenopathy, and symptoms like persistent
hoarseness, dysphonia, or dysphagia.
Q7. When examining a 35-year-old man, you notice a firm 3-cm thyroid nodule. His thyroid
studies are normal, and he is clinically euthyroid. An ultrasound is performed, and the nodule
appears suspicious for malignancy. What should thenext step in the workup be?
a. Follow thyroid studies and continue workup if abnormal
b. Measure thyroid peroxidase antibodies
c. Suppress thyroid with levothyroxine therapy
d. Perform a fine needle aspirate
e. Perform a surgical biopsy

The answer is d.
Once a thyroid nodule is found, the next step in the workup is obtaining a TSH assay and
ultrasonography of the thyroid. If the nodule appears suspicious on the basis of shape, size,
margins, position, or echogenic pattern, a fine needle aspiration(FNA) should be performed
irrespective of the patient’s TSH level. If the patient had an elevated TSH and the nodule did
not appear suspicious, thyroid peroxidase antibodies can be measured, and treatment of
hypothyroidism can be initiated. If the FNA reveals malignant cells, surgical intervention is
indicated.

ABFM 2020
A72-year-old female presents with bothersome palpitations. She is other wise healthy and
isnot taking any medications. A physical examination is normal, including thyroid and eye
examinations. Laboratory studies reveal a serum TSH level of 0.2 U/mL (N 0.4–4.0) and
normal T3 and free T4 levels. An EKG reveals frequent premature atrial contractions but is
otherwise normal. Ultrasonography of the thyroid does not reveal any nodules, thyroid
scintigraphy shows diffuse uptake, and an anti–thyrotropin-receptor (thyroid-stimulating
immunoglobulin) antibody level is significantly elevated.
Which one of the following is the most likely diagnosis?
A. Central hypothyroidism
B. Graves' disease
C. Iodine deficiency
D. Solitary toxic thyroid nodule
E. Toxic multinodular goiter
ANSWER: B
This patient has subclinical hyperthyroidism caused by Graves' disease. A positive anti–
thyrotropin-receptor (thyroid-stimulating immunoglobulin) antibody result is virtually
diagnostic of Graves' disease. Central hypothyroidism is associated with a low TSH level and
low T3 and T4 levels. Iodine deficiency is associated with goiter and hypothyroidism. Nodular
thyroid disease is unlikely given the imaging results. Treatment of this patient’s mild Graves
disease is probably indicated, given her age and cardiac symptoms.

148. A 34-year-old female comes to your office for follow-up after an emergency department
visit because of anxiety. She notes persistent anxiety, poor focus, and palpitations. She also
reports that she is not hungry and has lost several pounds. She reports “odd things happening”
such as sudden weakness in her legs, falling, and getting lost. When taking her history ,you
note that the patient is hyperverbal and displays tangential speech. She has a temperature of
37.4°C(99.3°F),a heart rate of 134beats/min, a respiratory rate of 20/min, and a blood pressure
of 117/69 mm Hg. A physical examination reveals an anxious-appearing female who is
tremulous at rest. A HEENT examination shows exophthalmos but no thyromegaly. A cardiac
examination is unremarkable aside from tachycardia. A pulmonary examination reveals faint
bi-basilar crackles. An EKG shows sinus tachycardia. Laboratory results are as follows: CBC
within normal limits Basic metabolic panel within normal limits TSH 19.0
pg/dL (N 2.3–4.2) Which one of the following would be most appropriate at this point?
A. Start methimazole (Tapazole)
B. Check for thyroid receptor antibody
C. Obtain a radioactive iodine uptake scan
D. Refer her to endocrinology
E. Admit her to the hospital

ANSWER: E
Hyperthyroidism is a common condition with a generally favorable prognosis. However, it is
important to remember that life-threatening complications such as thyrotoxicosis, also known
as thyroid storm, can occur. Symptoms of thyroid storm include fever, central nervous system
dysfunction, gastrointestinal or liver dysfunction, and cardiovascular complications such as
tachycardia and heart failure. The diagnosis is made using the Burch-Wartofsky Point Scale,
which produces a total score based on the presence or absence of various diagnostic criteria. In
this case, the patient has a score of 45, which is highly suggestive of thyroid storm. This acute,
life-threatening condition typically requires care in an intensive-care unit. It would therefore
be inappropriate to start treatment with an agent such as methimazole prior to hospitalization.

While a thyroid receptor antibody test may be useful in identifying the cause of the condition
it should not delay hospitalization. A radioactive iodine uptake test is also useful for
identifying the underlying cause of hyperthyroidism but should be avoided until the thyroid
storm has resolved. This patient requires hospitalization, so a referral to endocrinology would
not be most appropriate at this time.

An 87-year-old female comes to your office for an annual health maintenance visit. She
appears cachectic and tells you that for the past 6 months she has had a decreased appetite and
generalized muscle weakness. The patient is alert and oriented to person and place. She has a
10% weight loss, dry mucous membranes, and tenting of the skin on the extensor surface of
her hands. While inflating the blood pressure cuff on her right arm you observe carpopedal
spasms.
Which one of the following is the most likely electrolyte disturbance?
A. Hypercalcemia
B. Hypocalcemia
C. Hypokalemia
D. Hypernatremia
E. Hyponatremia
ANSWER: B
A Trousseau sign, defined as spasmodic contraction of muscles caused by pressure on the
nerves that control them, is present in up to 94% of patients with hypocalcemia.
Hypercalcemia is more likely to present with hyperreflexia. Patients with hypokalemia,
hypernatremia, or hyponatremia may present with weakness and confusion, but tetany is not a
common sign of either sodium or potassium imbalance.

111. A50-year-old female sees you for a routine health maintenance visit.Onexamination you
note a thyroid nodule and ultrasonography confirms a solid 1.5-cm nodule. Her TSH level is
normal. Which one of the following would be the most appropriate next step in the
management of this patient?
A. Antithyroid antibody titers
B. A thyroid scan
C. A fine-needle aspiration biopsy
D. Surgical excision of the nodule
E. Repeat ultrasonography in 6 months
ANSWER: C
Thyroid nodules more than or equal 1 cm that are solid or have suspicious features require a
fine-needle aspiration biopsy to rule out malignancy. Fine-needle aspiration should not be
performed on nodules <1 cm. The evaluation of a single thyroid nodule does not call for
testing of antithyroid antibody titers. If the patient had a low TSH level then a radionuclide
thyroid uptake scan (nuclear medicine thyroid scan) is indicated to look for a toxic nodule.
These hyperfunctioning nodules are seldom malignant. Surgical nodule excision is not
indicated since needle aspiration is diagnostic. Repeat ultrasonography in 6 months is not
indicated because this would not change management.

ABFM2019
Q1-A 46-year-old female with a history of hyperthyroidism controlled with methimazole
(Tapazole), 10 mg daily, returns to your office after an absence of several years. She has new
symptoms of palpitations, heat intolerance, and hoarseness. A physical examination reveals an
enlarged thyroid and a radioactive iodine uptake scan shows accumulation of tracer in multiple
areas.Which one of the following is the appropriate definitive treatment for this patient?
A. Methimazole alone, 20 mg daily
B. Methimazole, 20 mg daily, plus propranolol, 80 mg twice daily
C. Propylthiouracil alone, 50 mg 3 times daily
D. Radioactive iodine
E. Thyroidectomy

ANSWER: E
This patient has a medical history, physical examination, and radioactive iodine uptake scan
consistent with toxic multinodular goiter, which is the second most common cause of
hyperthyroidism in the United States. Although the addition of propranolol and an increase in
methimazole may control her palpitations and other symptoms of hyperthyroidism, these
measures will not permanently eliminate the problem. Radioactive iodine ablation and
thyroidectomy with subsequent thyroid hormone replacement are both appropriate treatments
for toxic multinodular goiter, but thyroidectomy is indicated for this patient because she has
compressive symptoms from the goiter itself.

Q2-A nulliparous 34-year-old female comes to your office for evaluation of fatigue, hair loss,
and anterior neck pain. These symptoms have been gradually worsening for the past few
months. Her past medical history is unremarkable. She has gained 5 kg (11 lb) since her last
office visit 18 months ago. Examination of the thyroid gland reveals tenderness but no discrete
nodules. Her TSH level is 7.5 U/mL (N 0.4–4.2), her T4 level is low, and her thyroid
peroxidase antibodies are elevated.

Which one of the following would be the most appropriate next step?
A.Continue monitoring TSH every 6 months
B.Begin thyroid hormone replacement and repeat the TSH level in 6–8 weeks
C.Begin thyroid hormone replacement and repeat the TSH level along with a T3 level in 6–8
weeks
D.Order ultrasonography of the thyroid
E.Order fine-needle aspiration of the thyroid

ANSWER: B
This patient has thyroiditis with biochemical evidence for autoimmune (Hashimoto’s)
thyroiditis. The most appropriate plan of care is to begin thyroid hormone replacement and
monitor with a repeat TSH level 6–8 weeks later. It is not necessary to include a T3 level
when assessing the levothyroxine dose. There is no need to routinely order thyroid
ultrasonography when there are no palpable nodules on a thyroid examination. Fine-needle
aspiration may be necessary to rule out infectious thyroiditis when a patient presents with
severe thyroid pain and systemic symptoms.

Q3-Which one of the following laboratory tests is best for assessing thyroidfunction?
A. Free T3
B. Reverse T3
C. Free T4
D. TSH
E. Thyroid peroxidase antibody
ANSWER: D
The best laboratory assessment of thyroid function is a serum TSH test, which is the preferred
test for diagnosing primary hypothyroidism. If an elevated serum TSH level is detected and
hypothyroidism is suspected, then a free T4 measurement would be indicated

Q4-A 30-year-old gravida 2 para 1 in her second trimester is evaluated for hypothyroidism.
A. A.The normal TSH range in pregnancy is lower than in the non-pregnant state
B.higher than in the non-pregnant state
C.the same as in the non-pregnant state
D.not useful for evaluating hypothyroidism after the first trimester
ANSWER: A
The TSH reference range is lower during pregnancy because of the cross-reactivity of the -
subunit of hCG. Levels of hCG peak during weeks 7–13 of pregnancy, and hCG has mild
TSH-like activity, leading to slightly high free T4 levels in early pregnancy. This leads to a
feedback decrease in TSH.
Q6-A 32-year-old female presents with heat intolerance, excessive weight loss, and anxiety.
She gave birth 6 months ago and recently stopped breastfeeding. On examination her thyroid
gland is slightly diffusely enlarged and nontender.
Laboratory studies reveal a decreased TSH level and elevated free T3 and T4 levels. You
suspect that she has postpartum thyroiditis.
Which one of the following tests would be most useful to confirm the diagnosis?
A. Radioactive iodine uptake
B. Thyroid peroxidase antibody levels
C. Thyroid ultrasonography
D. Thyrotropin receptor antibody levels
ANSWER: A
Postpartum thyroiditis is defined as a transient or persistent thyroid dysfunction that occurs
within 1 year of childbirth, miscarriage, or medical abortion. Release of preformed thyroid
hormone in the bloodstream initially results in hyperthyroidism. During the hyperthyroid
phase, radioactive iodine uptake will be low, which can help to confirm the diagnosis.
Pregnancy and breastfeeding are contraindications to radionuclide imaging. Thyroid
peroxidase antibody levels are elevated with chronic autoimmune thyroiditis (Hashimoto’s
thyroiditis), and patients present with symptoms of hypothyroidism. The Endocrine Society
and American Association of Clinical Endocrinologists do not recommend routine thyroid
ultrasonography in patients with abnormal thyroid function tests if there is no palpable
abnormality of the thyroid gland. Thyrotropin receptor antibody levels are positive in Graves'
disease.

ABFM2018
Q7-A 62-year-old female who is a new patient requests a thyroid evaluation because she has a
history of abnormal thyroid test results. You obtain a copy of her records, which include a
TSH level of 0.2 U/mL (N 0.4–4.2) and a free T4 level of 2.0 ng/dL (N 0.8–2.7) from 3 years
ago. She reports feeling well and has no other health conditions. She does not take any
medications .
A physical examination reveals normal vital signs, a BMI of 23.0 kg/m2, no neckmasses ,
a normal thyroid size, and normal heart sounds. Laboratory studies reveal a TSH level of
0.1 U/mL, a free T4 level of 2.5 ng/dL, and a free T3 level of 3.1 pg/mL (N 2.3–4.2) .
Treatment for this condition would be indicated if the patient has an abnormal
A) calcium level
B) DXA scan
C) glucose level
D) lipid level
E) thyroid ultrasonography study
ANSWER: B
This patient has subclinical hyperthyroidism as evidenced by her low TSH level with normal
free T4 and free T3 levels. Common causes of subclinical hyperthyroidism include Graves
disease, autonomous functioning thyroid adenoma, and multinodular toxic goiter. Subclinical
hyperthyroidism may progress to overt hyperthyroidism; this is more likely in patients with
TSH levels <0.1 U/mL. Even in the absence of overt hyperthyroidism these patients are at
higher risk for several health conditions, including atrial fibrillation, heart failure, and
osteoporosis. For this reason it is important to assess for these conditions and consider treating
the underlying thyroid condition, as well as the complication. The American Thyroid
Association recommends treating patients with complications who are either over age 65 or
have a TSH level <0.1 U/mL. Lipid and glucose abnormalities are not known to be related to
subclinical hyperthyroidism.Calcium levels may be abnormal in hyperparathyroidism but not
hyperthyroidism.Thyroid ultrasonography may be helpful to determine the cause of
hyperthyroidism but is not used to help decide when to treat subclinical hyperthyroidism.

Q8-A 69-year-old male presents for follow-up of hypertension treated with spironolactone
(Aldactone) and amlodipine (Norvasc). His past medical history is remarkable only for a
kidney stone several years ago. A physical examination is unremarkable. A comprehensive
metabolic panel is unremarkable except for a calcium level of 12.0 mg/dL (N 8.0–10.0) .
Which one of the following is the most likely cause of his elevated calcium level ?
A) Excessive ingestion of calcium supplements
B) His current medication regimen
C) Occult malignancy
D) Primary hyperparathyroidism
E) Vitamin D deficiency

ANSWER: D
The most common cause of hypercalcemia is hyperparathyroidism. This is seldom
symptomatic and is often discovered through routine blood testing.
Hypercalcemia due to cancer can be caused by secretion of the parathyroid hormone–related
protein and by osteoclastic bone resorption. Other causes of hypercalcemia include thiazide
diuretics, lithium, vitamin D intoxication, hyperthyroidism, milk alkali syndrome from
excessive calcium antacid ingestion, adrenal insufficiency, and lymphoma.
Q9-A 21-year-old gravida 1 para 0 is diagnosed with overt hyperthyroidism early in the first
trimester. The most appropriate management at this time is
A) observation only
B) methimazole (Tapazole)
C) propylthiouracil
D) radioactive iodine
E) thyroidectomy .

ANSWER: C
Overt hyperthyroidism during pregnancy is associated with adverse effects to the mother and
fetus, so treatment is required. Since methimazole is associated with birth defects when used in
the first trimester, propylthiouracil is preferred.
Methimazole should be considered after the first trimester because the risk of congenital
anomalies is less than the risk of liver failure associated with propylthiouracil. Surgery and
radioactive iodine should only be used if there is a clear indication, and radioactive iodine
would not be appropriate during pregnancy.

ABFM2017
Q10-A 45-year-old female comes to your office for follow-up of palpitations and anxiety. Her
pulse rate is 112 beats/min and her heart rate is regular with no audible murmurs. Her thyroid
isnontender with no palpable nodules. You note bilateral exophthalmos as well as pretibial
myxedema. Her TSH level is 0.05 U/mL (N 0.5–5.0); free T4 and total T3 levels are elevated.
A pregnancy test is negative. Which one of the following would be the best option for limiting
the progression of this patient’s ophthalmopathy?
A) Atenolol (Tenormin)
B) Cholestyramine (Questran)
C) Methimazole (Tapazole)
D) Prednisone
E) Radioactive iodine
F)
ANSWER: C
Based on her clinical presentation and classic ophthalmopathy, this patient has Graves disease.
Unlike radioactive iodine, methimazole has been shown to decrease the risk of development or
progression of ophthalmopathy in Graves disease (SOR B). Atenolol is used for symptomatic
control in hyperthyroidism. Cholestyramine can help lower thyroid hormone acutely but is not
a long-term treatment. Prednisone is used for severe hyperthyroidism and not long-term
treatment. Atenolol, cholestyramine, and prednisone do not have any effect on the long-term
complications of Graves disease.
Q11-A 40-year-old female presents with a 4-week history of a persistent sore throat despite
supportive treatment for a viral upper respiratory infection provided by an urgent care facility.
She reports palpitations, weight loss, frequent bowel movements, and anxiety with insomnia
for the past month.On examination she has a mildly enlarged thyroid gland. Laboratory
evaluation is notable for a suppressed TSH level along with elevated free T4 and total T3
levels. A radioactive iodine uptake scan shows low uptake. Which one of the following is the
most likely diagnosis?
A) Factitious thyrotoxicosis
B) Graves disease
C) Multinodular goiter
D) Subacute thyroiditis
E) TSH-secreting pituitary adenoma

ANSWER: D
The initial ―destructive‖ phase of subacute thyroiditis presents with signs, symptoms, and
laboratory findings of overt hyperthyroidism; however, a radioactive iodine uptake scan is
negative in this phase. Graves disease and toxic multinodular goiter also present with overt
hyperthyroidism, but radioactive iodine uptake is high. Factitious thyrotoxicosis is associated
with low TSH and elevated or normal free T4 and total T3, but a goiter is not present. A TSH-
secreting pituitary adenoma results in elevated TSH, free T4, and total T3.

Q12-Which one of the following coexisting conditions could require the use of a much higher
than
expected dose of levothyroxine (Synthroid) to adequately treat hypothyroidism?
A) Chronic kidney disease
B) Diabetes mellitus
C) Helicobacter pylori gastritis
D) Hepatitis C infection
E) Hyperparathyroidism

ANSWER: C
Absorption of levothyroxine is impaired by several gastrointestinal conditions, including
atrophic gastritis, chronic proton pump inhibitor use, and Helicobacter pylori infection.
Treatment of H. pylori infection reverses this effect, and following eradication of the infection
a reduction of the levothyroxine dosage by 30% or more will often be required.
Q13-A 66-year-old asymptomatic male presents with a 6-month history of ongoing pruritus. A
physical examination is normal. Laboratory studies are also normal except for an LDL-
cholesterol level of 150 mg/dL, a free T4 level of 1.4 ng/dL (N 0.9–2.3), and a TSH level of
6.22 U/mL (N 0.4–4.5).
Which one of the following would be most appropriate at this point?
a. No treatment at this time and a repeat TSH level in 3 months
b. Testing for thyroid antibodies and treatment with levothyroxine if the test is negative
c. A 131I uptake and scan with no treatment pending results
d. Liothyronine (Cytomel)
e. Levothyroxine (Synthroid)

ANSWER: A
This patient has subclinical hypothyroidism and should have a TSH level repeated in 1–3
months, as TSH may fluctuate in patients without thyroid disease and return to normal on
subsequent testing. In a patient with a normal free T4 the TSH level must be >10 U/mL for a
diagnosis of hypothyroidism. Mild TSH elevations may be a normal manifestation of aging.
Q14-Which one of the following coexisting conditions could require the use of a much higher
than
expected dose of levothyroxine (Synthroid) to adequately treat hypothyroidism?
A) Chronic kidney disease
B) Diabetes mellitus
C) Helicobacter pylori gastritis
D) Hepatitis C infection
E) Hyperparathyroidism
ANSWER: C

Absorption of levothyroxine is impaired by several gastrointestinal conditions, including


atrophic gastritis, chronic proton pump inhibitor use, and Helicobacter pylori infection.
Treatment of H. pylori infection reverses this effect, and following eradication of the infection
a reduction of the levothyroxine dosage by 30% or more will often be required.
Q15-A 66-year-old asymptomatic male presents with a 6-month history of ongoing pruritus. A
physical examination is normal. Laboratory studies are also normal except for an LDL-
cholesterol level of 150 mg/dL, a free T4 level of 1.4 ng/dL (N 0.9–2.3), and a TSH level of
6.22 U/mL (N 0.4–4.5).
Which one of the following would be most appropriate at this point?
a. No treatment at this time and a repeat TSH level in 3 months
b. Testing for thyroid antibodies and treatment with levothyroxine if the test is negative
c. A 131I uptake and scan with no treatment pending results
d. Liothyronine (Cytomel)
e. Levothyroxine (Synthroid)

ANSWER: A
This patient has subclinical hypothyroidism and should have a TSH level repeated in 1–3
months, as TSH may fluctuate in patients without thyroid disease and return to normal on
subsequent testing. In a patient with a normal free T4 the TSH level must be >10 U/mL for a
diagnosis of hypothyroidism. Mild TSH elevations may be a normal manifestation of aging.

ABFM2015
Q16-Patients being treated with amiodarone (Cordarone) should be monitored periodically
with serum levels of
A) cortisol
B) creatine phosphokinase
C) creatinine
D) LDH
E) TSH.

ANSWER: E
Patients on amiodarone can develop either hyperthyroidism or hypothyroidism. It is
recommended that a patient on amiodarone have baseline thyroid function tests (free T4, TSH)
with follow-up testing every 6 months to monitor for these conditions. Hyperadrenalism and
hypoadrenalism are not associated with amiodarone treatment
Q17-While performing a routine physical examination on a 42-year-old female you discover an
apparent nodule in the left lobe of the thyroid measuring approximately 1 cm in diameter,
which is confirmed on ultrasonography. The most appropriate next step in the evaluation of this
finding is a
A) serum calcitonin level
B) serum free T3 level
C) serum TSH level
D) serum thyroglobulin level
E) radionuclide thyroid scan
ANSWER: C
Thyroid nodules >1 cm that are discovered incidentally on examination or imaging studies
merit further evaluation. Nodules <1 cm should also be fully evaluated when found in patients
with a family history of thyroid cancer, a personal history of head and neck irradiation, or a
finding of cervical node enlargement. Reasonable first steps include measurement of TSH or
ultrasound examination. The American Thyroid Association’s guidelines recommend that
TSH be the initial evaluation (SOR A) and that this be followed by a radionuclide thyroid scan
if results are abnormal. Diagnostic ultrasonography is recommended for all patients with a
suspected thyroid nodule, a nodular goiter, or a nodule found incidentally on another imaging
study (SOR A). Routine measurement of serum thyroglobulin or calcitonin levels is not
currently recommended

Q18-A 25-year-old female who is 3 months post partum presents with multiple complaints,
including increasing weakness and fatigue, intolerance to warm environments, a weight loss of
30 lb despite an increased appetite, difficulty sleeping, awareness that her heart is beating
faster and “pounding” in her chest, increasing restlessness and difficulty concentrating,
increased tremulousness, and a significant swelling in her neck. She takes no medication, has
experienced no recent trauma, and has not ingested large amounts of iodine. When you
examine her you find no exophthalmos or lid lag and no pretibial edema, but her skin is warm,
smooth, and moist. You also find a smooth, non- nodular, nontender, enlarged thyroid gland,
clear lungs, a resting tremor, and hyperactive reflexes. Laboratory testing reveals a low TSH
level, elevated free T3 and free T4, and high uptake on a radioactive iodine uptake scan.
Which one of the following is the most likely diagnosis?

A) Postpartum thyroiditis
B) Silent thyroiditis
C) Subacute thyroiditis
D) Graves disease
E) Exogenous thyroid ingestion.
ANSWER: D
This patient has symptoms consistent with hyperthyroidism, which could be caused by any of
the options listed. TSH is suppressed and free T4 and free T3 are elevated in all of these
conditions. Only Graves disease, however, will cause high radioactive iodine uptake on a
thyroid scan. Uptake will be low in the other conditions.

Q19-Which one of the following is the recommended first-line test for investigating suspected
hyperor hypothyroidism?
A) Free T3
B) Free T4
C) TSH
D) Antithyroglobulin
E) Antithyroid peroxidase

ANSWER: C
When investigating presumed hyper- or hypothyroidism, TSH is the first-line test (SOR A). If
the patient is found to have an abnormal TSH level, free T4 is the next test to order. A free T3
test can also be helpful, but the free T4 assay is not affected by changes in iodothyronine-
binding proteins, and T3 is often a peripheral product and can be abnormal due to nonthyroid
diseases or medications. Occasionally, free T4 and T3 tests are performed as second-line tests,
even if the TSH is normal, if the results do not match the clinical picture. Other second- and
third-line tests include measurement of thyroid antibodies, such as antithyroid peroxidase and
antithyroglobulin. Routine thyroid screening tests are not indicated for asymptomatic adults
(SOR A).

Q20-A mother brings in her 10-year-old son because of a swollen area in his neckthat she first
noticed yesterday. He has also had symptoms of an upper respiratory infection.
On examination
the child has a runny nose but otherwise appears well. Palpation reveals a soft, 1.5-cm, slightly
tender mass, inferior to the angle of the mandible and anterior to the sternocleidomastoid
muscle.
The most likely diagnosis is
A) thyroglossal duct cyst
B) dermoid cyst
C) branchial cleft cyst
D) thyroid tumor
ANSWER: C
In children, neck masses usually fall into one of three categories: developmental,
inflammatory/reactive, or neoplastic. The history and physical examination can help narrow
the diagnosis, with location of the mass being particularly helpful. Branchial cleft cysts make
up approximately 20% of neck masses in children. They commonly present in late childhood or
adulthood, when a previously unrecognized cyst becomes infected. They are most frequently
found anterior to the sternocleidomastoid muscle, but can also be preauricular. Thyroglossal
duct cysts are located in the midline over the hyoid bone. Frequently, they elevate when the
patient swallows. Dermoid cysts are usually mobile, moving with the overlying skin. They can
be located in the submental or midline region. Thyroid tumors are also usually located in the
midline. Malignant masses are usually hard, irregular, nontender, and fixed.

ABFM2014
Q21-1-A 25-year-old female with hypothyroidism sees you for preconception counseling. Her
thyroid problem has been well managed with levothyroxine (Synthroid), 75 :g daily, but she
asks your advice about changing her treatment to something more natural now that she is
planning to become pregnant. Which one of the following is the best recommendation for this
patient?
A)Continue the current dosage of levothyroxine
B)Reduce the current dosage of levothyroxine to 50 :g daily
C)Change to a comparable dosage of combination levothyroxine/L- triiodothyronine
D)Change to a comparable dosage of desiccated thyroid

ANSWER: A
Untreated hypothyroidism during pregnancy impairs fetal development and increases the risk
of spontaneous miscarriage, prematurity, preeclampsia, gestational hypertension, and
postpartum hemorrhage. These risks are mitigated by appropriate levothyroxine treatment.
Levothyroxine/Ltriiodothyroxine combinations and desiccated thyroid preparations have the
potential to correct maternal hypothyroidism, but the T4 level may still be too low to provide
the transplacental delivery necessary for optimal fetal health. The most appropriate pregnancy
planning advice is to continue the current dosage of levothyroxine with a plan for monthly
monitoring of TSH and T4 during pregnancy, with the expectation that an increase in dosage
may be required as the pregnancy progresses
Q22-A 70-year-old white female with hypertension and atrial fibrillation has been chronically
anticoagulated. A higher dosage of warfarin (Coumadin) would be required to achieve a
therapeutic INR if the patient were found to have
A)malnutrition
B)hypothyroidism
C)heart failure
D)acute kidney injury
E)progressive nonalcoholic cirrhosis
ANSWER: B
Medical conditions that decrease responsiveness to warfarin and reduce the INR include
hypothyroidism, visceral carcinoma, increased vitamin K intake, diabetes mellitus, and
hyperlipidemia. Conditions that increase responsiveness to warfarin, the INR, and the risk of
bleeding include vitamin K deficiency caused by decreased dietary intake, malabsorption,
scurvy, malnutrition, cachexia, small body size, hepatic dysfunction, moderate to severe renal
impairment, hypermetabolic states, fever, hyperthyroidism, infectious disease, heart failure,
and biliary obstruction (SOR B, SOR C).

Q23-In a patient with symptoms of thyrotoxicosis and elevated free T4, the presence of
thyroid TSH receptor site antibodies would indicate which one of the following as the cause of
thyroid gland enlargement?
A) Toxic multinodular goiter
B) Toxic adenoma
C) Hashimoto’s (lymphadenoid) thyroiditis
D) Subacute (giant cell) thyroiditis
E) Graves disease

ANSWER: E
When there is a question about the cause of goiter and thyrotoxicosis, the presence of TSH
receptor immunoglobulins indicates Graves disease. The prevalence of specific forms of TSH
receptor site antibodies can distinguish Graves disease from Hashimoto’s disease. Both are
autoimmune diseases, but in Graves disease there is a predominance of TSH receptor
antibodies. In Hashimoto’s disease TSH receptor–blocking antibodies are more predominant.
These immunoglobulins tend to disappear with therapy
Q24-A 45-year-old female had myalgias, a sore throat, and a fever 2 weeks ago. She now has
anterior neck tenderness and swelling, with pain radiating up to her ears. Your examination
reveals a tender goiter.
Which one of the following would support a diagnosis of subacute granulomatous thyroiditis?
A) Pretibial myxedema
B) Exophthalmos
C) Multiple nodules on ultrasonography
D) Low radioactive iodine uptake (<5%).

ANSWER: D
Subacute granulomatous thyroiditis is the most common cause of thyroid pain. Free T4 is
elevated early in the disease, as it is in Graves disease; however, later in the disease T4
becomes depressed and then returns to normal as the disease resolves. Pretibial myxedema,
exophthalmos, and a thyroid thrill or bruit can all be found in Graves disease, but are not
associated with subacute granulomatous thyroiditis. Multiple nodules on ultrasonography
suggests multinodular goiter rather than subacute granulomatous thyroiditis. Patients with
subacute granulomatous thyroiditis will have a low radioactive iodine uptake (RAIU) at 24
hours, but patients with Graves disease will have an elevated RAIU (SOR C).

Q25-A 44-year-old female with a history of type 2 diabetes mellitus that is well controlled
with metformin (Glucophage) is noted to have mild anterior neck fullness during an annual
physical examination. A review of systems is unremarkable. Ultrasonography of the neck
shows a normal-sized thyroid gland with a 1.2-cm nodule in the right lobe. Her TSH leveL is
normal.
Which one of the following is the most appropriate next step in the management of this
patient’s thyroid nodule?
A) A repeat TSH level in 3 months
B) A radionuclide uptake thyroid scan
C) Noncontrast CT of the neck
D) Fine-needle aspiration of the nodule
E) No further workup
ANSWER: D
Thyroid nodules are often detected either during a clinical examination or incidentally on an
imaging study. The first step in the evaluation of a thyroid nodule is to order a TSH level. If
the TSH level is suppressed, radionuclide scintigraphy should be ordered to rule out a
hyperfunctioning nodule. If the TSH level is either normal or high, the current
recommendation is to biopsy only nodules >1 cm. Clinical follow-up is recommended for
nodules 1 cm.
URTIs
Pretest 2019
Q1- You are seeing an otherwise healthy 18-year-old man who has had an acute cough for 2
weeks. It started like a typical “cold,” but has persisted. Over the last 3 days, he reports
dyspnea and some wheezing. On examination, his temperature is 38.3°C, but his other vital
signs are normal. Chest auscultation reveals crackles in the left lower lobe. What is the
appropriate next step?
a. Supportive therapy without antibiotics
b. Treatment with a bronchodilator
c. A 10-day course of amoxicillin
d. A 5-day course of azithromycin
e. A posteroranterior (PA) and lateral chest film

Answer is e.
The causes of cough can range from a self-limited viral upper respiratory infection to severe
infections. Patient signs and symptoms will define the approach to take. The first factor to
consider is whether or not the patient has symptoms, signs, or risk factors that would warrant a
chest radiograph. Symptoms that a chest radiograph is necessary include dyspnea, high fever,
rigors, pleuritic chest pain, and altered mental status. Signs include a temperature greater than
38°C, heart rate greater than 100 beats/min, a respiratory rate greater than 24 breaths/min, and
an abnormal lung examination.
Risk factors include elderly patients, those with known COPD, heat failure, renal failure, or
diabetes. Since the patient in this question has dyspnea, a high fever, and an abnormal lung
examination, the next thing to do would be to obtain a PA and lateral chest x-ray to confirm the
diagnosis of pneumonia.

Q2-You are seeing a 21-year-old college student who complains of congestion, headache,
sinus pressure, and tooth pain for more than 2 weeks. She is otherwise healthy, but feels like
she’s “having trouble shaking this cold.” She has used over- the-counter decongestants with
limited relief. A CT scan of her sinuses demonstrates acute sinusitis. Which of the following is
the most common organism causing her symptoms?
a. Moraxella catarrhalis
b. S aureus
c. Group A β-hemolytic streptococcal species
d. Streptococcus pneumoniae
e. A polymicrobial mixture of many organisms
Answer is d.
S pneumoniae is the most common bacterial pathogen in bacterial sinusitis. Other causes
include H influenzae, M catarrhalis, and group A β-hemolytic streptococci. Pathogens vary
regionally in both prevalence and drug resistance. Complicating the situation further is that if
the sinus aspirates of healthy adults are cultured, bacteria colonization will be found in around
one quarter of them.

Q3-A 19-year-old male patient presented to your office with a 3-day history of fatigue, sore
throat, and low-grade fevers. On examination, his temperature was 100.3°F, and you noted an
exudative pharyngitis with anterior and posterior cervical adenopathy. You sent a throat
culture and started him on amoxicillin prophylactically. Two days later, he presents for
follow-up with continued symptoms and a diffuse, symmetrical erythematous maculopapular
rash. Which of the following is the most likely cause of his symptoms?
a. Scarlet fever
b. Allergic reaction to amoxicillin
c. Viral exanthem
d. Mononucleosis
e. Rubella

Answer is d.
Mononucleosis is often mistaken for streptococcal pharyngitis. Both have symptoms of sore
throat, fatigue, fever, and adenopathy. Lymphadenopathy is more widespread in
mononucleosis and usually confined to the anterior nodes in streptococcal pharyngitis. If
patients with mononucleosis are given ampicillin (and other penicillin derivatives), up to 100%
may develop the rash described above, sometimes confused as an allergic reaction to
penicillin. The rash of scarlet fever is more confluent, and has a sandpaper-like texture. The
rash of rubella is similar in appearance to that of mononucleosis, but exudative pharyngitis is
absent.
Q4-An 11-year-old patient has a history of recurrent pharyngitis with repeatedly positive
streptococcal RADTs. You test him when he is asymptomatic and find that the test continues to
be positive. Assuming that the patient has no allergies, what would be the best treatment
for him?
a. No antibiotics are required.
b. Penicillin (Pen VK).
c. Amoxicillin (Amoxil), using high dosages.
d. Azithromycin (Zithromax).
e. Clindamycin (Cleocin).
Answer is a.
Approximately 20% of school age children are carriers of group A β-hemolytic Streptococcus.
In the past, it was felt that these children needed to be treated to eradicate the bacteria. The
antibiotics used to eliminate group A streptococcal carriage from oropharyngeal secretions
were oral respiratory quinolones or oral clindamycin. Recent studies have shown that these
carriers do not need to be identified or treated, as they do not develop complications from
infection and have not been found to be important in the spread of group A β-hemolytic
Streptococcus to others.

Q5-You are caring for a 15-year-old who came in for evaluation of his sore throat. He has
been ill for 24 hours. In addition to the sore throat, he has a fever, but no cough. On physical
examination, his temperature is 101°F, tender anterior cervical adenopathy, and tonsillar
exudate. His RADT is positive. He denies allergies to medication. Which of the following is
the appropriate next step?
a. Reassurance and observation
b. Perform a throat culture
c. Treat with oral penicillin (Pen VK)
d. Treat with oral azithromycin (Zithromax)
e. Treat with cephalexin (Keflex).
Answer is c.
The goal of antibiotic treatment for GABHS is to decrease infectivity and prevent
complications, especially rheumatic fever. Even without treatment, symptoms generally
improve in 3 to 5 days. Delay of treatment does not appear to increase the risk of rheumatic
fever, but early treatment does reduce infectivity, lessen morbidity, and promote early return
to normal activities. The choice of antibiotic is determined by the bacteriology of the GABHS,
clinical efficacy, patient adherence, adverse effects, and cost. Penicillin remains the antibiotic
of choice for GABHS pharyngitis. There has been no reported resistance to penicillin. First-
generation cephalosporins and macrolides can be used in patients allergic to penicillin. There
is no need to observe once a test is positive, and a throat culture is not necessary to confirm a
positive rapid antigen test.

Q6-A 7-year-old boy comes to see you for a sore throat. He reports fevers, chills, myalgias, and
pain on swallowing. On examination, you note anterior adenopathy, erythematous tonsils, and
edema of his uvula. He has no drug allergies. Which of the following would be the best next
step?
a. Symptomatic care
b. Antiviral therapy
c. Penicillin (Pen VK)
d. RADT
e. Throat culture.
Answer is d.
It is necessary to obtain microbiologic confirmation of infection before treating GABHS
pharyngitis. If the clinical symptoms suggest GAHBHS, an RADT is the appropriate next step.
A culture would not be necessary unless the rapid test were negative. The RADT has high
specificity but low sensitivity. The use of the RADT has allowed 247 clinicians to begin
antibiotic therapy early in those with a positive test, decreasing the risk of spread and allowing
an earlier return to school or work. Students can return to school 24 hours after starting
antibiotics.

Q7-You are seeing a 15-year-old patient complaining of sore throat. She complains of 3 days
of symptoms that have included hoarseness and cough. Shedenies any medication allergies. On
examination, she is afebrile, has erythematous tonsils and cervical adenopathy. Which of the
following would be the best next step?
a. Symptomatic care
b. Antiviral therapy
c. Treatment with penicillin (Pen VK)
d. Perform a rapid antigen detection test (RADT)
e. Perform a throat culture.
Answer is a.
is common and accounts for more than 7 million outpatient visits for children annually. The
estimated total cost for pharyngitis in children is $540M per year. Around 30% of pharyngitis
in children is caused by group A β-hemolytic Streptococcus, while another 30% to 40% are
caused by viruses. Rhinovirus is the most common of the viral causes. The type of pharyngitis
cannot be identified by history alone, and if treatment is to be initiated with antibiotics,
microbiologic confirmations are necessary. However, if clinical symptoms suggest viral
pharyngitis and the pretest probability of GABHS infection is low, a diagnostic test need not be
performed. In this question, the patient’s hoarseness and cough are more likely associated with
a viral cause, therefore symptomatic care is appropriate. There is no need to treat with
antibiotics or antivirals, and testing is unnecessary. A modified Centor score can be used to
help evaluate probability. Patients are given one point each for
• Absence of cough
• Swollen and tender anterior cervical lymph nodes
• Temperature more than 100.4°F
• Tonsillar exudate or swelling
If the patient has an age of 3 to 14 years, a point is added. For people between 15 and 44 years
old, no points are added, and for patients 45 years old and older, a point is taken away.
Patients with a score of zero or 1 are at very low risk for streptococcal infection. Patients with
a score of 2 or 3 should be tested. Patients with a score of 4 or higher are at high risk. In those
high-risk patients, empiric treatment may be considered.
Q8-You are evaluating an 17-year-old man with a sore throat. It has been present for 3 days
175 and began with chills. He now has fever (to 103°F), aches, and fatigue. On examination,
he has enlarged tonsils, exudative pharyngitis, soft palate petechiae, and anterior and posterior
cervical adenopathy. Which of the following would most likely be found, if testing occurs?
a. Erythrocytosis
b. Neutropenia
c. Atypical lymphocytes
d. A positive heterophile antibody test
e. A positive culture for group A β-hemolytic
Answer is c.
Infectious mononucleosis should be suspected when a teen or young adult (ages 15-25)
present with the classic triad of severe sore throat, lymphadenopathy, and fever (which can be
up to 104°F). In general, mononucleosis begins with a prodrome of chills,
sweats, and malaise. Clinical signs include enlarged tonsils, cervical adenopathy (posterior and
anterior), and sometimes hepatosplenomegaly. A CBC would show absolute lymphocytosis
with more than 10% atypical lymphocytes. It would not show erythrocytosis, and although
neutropenia can be seen with viral infections, atypical lymphocytes are more likely. A throat
culture would be negative, unless the person was a carrier, and the heterophile antibody test
would not be positivefor 2 to 3 weeks after symptoms began.

Q9-You are caring for a patient with recurrent bouts of acute sinusitis. She has had multiple
courses of analgesics, decongestants, and occasional rounds of antibiotics. Among other
remedies, she has tried nasal steroids without relief from her acute symptoms. Which of the
following statement is true regarding imaging in this case?
a. No imaging will be helpful in this case.
b. Sinus films may be helpful in this case.
c. CT of the sinuses is only helpful for chronic sinusitis.
d. MRI is the most sensitive in the evaluation of sinusitis.
e. US is recommended based on cost and sensitivity in this case.
Answer is b.
In general, most patients with acute sinusitis do not benefit from imaging studies. However,
imaging may be helpful in uncertain or recurrent cases, as in this question. Sinus films, though
not helpful for initial evaluation, may be abnormal in acute rhinosinusitis, but are actually most
helpful when negative. A normal series has a negative predictive value of 90% to 100%. The
positive predictive value is around 80%, but its sensitivity is only around 60%. CT scans have
superior sensitivity (95% to 98%) when compared with sinus films, so can be valuable in
acute settings. That said, they are particularly valuable in establishing the diagnosis of chronic
sinusitis or in equivocal cases before starting long-term antibiotic therapy. MRI is used when
fungal sinusitis or tumors are suspected, but are not used for routine evaluation. US is not
helpful.
Q10-You are seeing a 21-year-old college student who complains of congestion, headache,
sinus pressure, and tooth pain for more than 2 weeks. She is otherwise healthy, but feels like
she’s “having trouble shaking this cold.” She has used over- the-counter decongestants with
limited relief. A CT scan of her sinuses demonstrates acute sinusitis. Which of the following is
the most common organism causing her symptoms?
a. Moraxella catarrhalis
b. S aureus
c. Group A β-hemolytic streptococcal species
d. Streptococcus pneumoniae
e. A polymicrobial mixture of many organisms.

Answer is d.
S pneumoniae is the most common bacterial pathogen in bacterial sinusitis. Other causes
include H influenzae, M catarrhalis, and group A β-hemolytic streptococci. Pathogens vary
regionally in both prevalence and drug resistance. Complicating the situation further is that if
the sinus aspirates of healthy adults are cultured, bacteria colonization will be found in around
one quarter of them.

Q11-You are caring for a 42-year-old woman with a 10 days of congestion, purulent nasal
discharge, headache, and tooth pain. Based on clinical grounds, you diagnose her with
uncomplicated rhinosinusitis. According to studies, which of the following statements
regarding treatment of this condition is true?
a. Treatment with amoxicillin is superior to placebo.
b. Treatment with erythromycin is superior to placebo.
c. Treatment with ciprofloxacin is superior to placebo.
d. Treatment with trimethoprim-sulfamethoxazole is superior to placebo.
e. No significant difference has been demonstrated between any antibiotic and placebo.

Answer is e.
Between 40% and 70% of patients with acute bacterial rhinosinusitis improve
symptomatically within 2 weeks without antibiotic therapy. Antibiotic treatment is
controversial in uncomplicated cases of clinically diagnosed sinusitis because only around 5%
of patients will note a shorter duration of symptoms with treatment, and antibiotics have
nearly twice the number of adverse events as compared with placebo. Antibiotics should be
considered when symptoms are severe, or when cases are complicated. In these cases, 249
antibiotics reduce theincidence of clinical failure by 50%.
Q12 - You are caring for a 32-year-old female smoker with a history of allergic rhinitis who
presents to discuss upper respiratory symptoms. She reports congestion, facial pressure, nasal
discharge, tooth pain, and headache. Her symptoms have been present for 5 days and they
have not improved with decongestants. Sinus palpation causes significant pain. Which of the
following is true regarding the criteria for diagnosing acute bacterial rhinosinusitis in this
adult?
a.Pain with sinus palpation is one of the minor criterion.
b.Purulent nasal drainage is one of the minor criterion.
c.Facial pain is one of the minor criterion.
d.Tooth pain is one of the major criterion.
e.There are no agreed upon criteria for the diagnosis of acute bacterial rhinosinusitis in an
adult.

Answer is e.
Despite the fact that acute bacterial rhinosinusitis affects more than 20 million Americans per
year, there are no agreed-upon criteria for the diagnosis of acute bacterial rhinosinusitis in
adults. This is because so many of the symptoms mimic other diseases ranging from a common
cold to allergic rhinitis. Some feel that the presence of two major criteria or one major and two
minor criteria allow a clinician to diagnosis sinusitis in adults. Major criteria are:
• Purulent anterior nasal discharge or postnasal discharge
• Nasal congestion or obstruction
• Facial congestion or fullness
• Facial pain or pressure
• Hyposmia/anosmia and fever
Minor criteria include:
• Headache
• Ear pain/pressure
• Halitosis
• Dental pain
• Cough
• Fatigue
Bacterial rhinosinusitis can sometimes be distinguished from viral rhinitis by persistence of
symptoms for more than 10 days after onset, or worsening symptoms within 10 days after an
initial improvement. This bimodal presentationcan be helpful in diagnosis.
ABFM 2020

174. A 46-year-old male who injects heroin daily presents with a 6-month history of
progressive dyspnea on exertion, a productive cough, and fatigue. He does not have any fever,
chills, malaise, or hemoptysis. He has not had any sick contacts and has never smoked. On
physical examination he shows no signs of distress, has a normal oxygen saturation on room
air, and has normal breath sounds. A chest radiograph reveals bilateral perihilar shadowing.
A subsequent lung biopsy will most likely show
1. Adenocarcinoma
2. Branching hyphae
3. Foreign body granulomas
4. Caseating granulomas
5. Noncaseating granulomas
ANSWER: C
Although persons who inject drugs are at high risk for a variety of pulmonary infectious
diseases, this patient’s presentation, including the relatively slow development of symptoms,
is most consistent with pulmonary foreign body granulomas. These result from the injection of
crushed pills, talc, or other foreign substances, which are then deposited in the vasculature of
the lungs. Adenocarcinoma is not as likely given the patient’s age and nonsmoking history.
Branching hyphae would be seen in aspergillosis but this patient does not have fevers or
malaise. Caseating granulomas are seen in tuberculosis, which is less likely given the absence
of fever and hemoptysis. Noncaseating granulomas, seen in sarcoidosis, would not be more
likely in this patient than in the general population.

187. A 35-year-old male first presented to your office 4 months ago with a persistent chronic
cough. He is a nonsmoker with no significant past medical history. Over the past few months,
he has been evaluated for GERD, asthma, eosinophilic bronchitis, and upper airway cough
syndrome without symptomatic relief or diagnosis.
Which one of the following is recommended for chronic refractory cough in this otherwise
healthy male?
1. Cyclobenzaprine
2. Duloxetine (Cymbalta)
3. Gabapentin (Neurontin)
4. Lorazepam (Ativan)
5. Propranolol
ANSWER: C

In randomized, controlled trials, gabapentin has demonstrated benefit for treating a refractory
chronic cough after 4 weeks of treatment (SOR C). Chronic cough may be due to a
hypersensitivity of the cough reflex, either centrally or peripherally. Cyclobenzaprine,
duloxetine, lorazepam, and propranolol have not proven to be beneficial in reducing or
eliminating chronic cough.

194. A 3-year-old female is brought to your office by her mother with a 3-day history of
cough, and you diagnose a common cold. The mother asks for a recommendation to relieve
the cough.
You tell her that the safest and most effective management for cough would be
A. Honey
B. Codeine
C. Dextromethorphan
D. Diphenhydramine
E. Ibuprofen
ANSWER: A
Safe and effective options to treat cough in young children include nasal saline irrigation, a
menthol rub, and honey (in children 12 months of age or older). Codeine should no longer be
used for cough in anyone under 18 years of age. Over-the-counter cough and cold medications
are not recommended for children under 4 years of age due to the lack of evidence of benefit
and the significant side effects. Ibuprofen has not been shown to be effective for cough.

During rounds at the nursing home, you are informed that there are two residents on the
unit with laboratory-confirmed influenza. According to CDC guidelines, who
should receive chemoprophylaxis for influenza?
A. Only symptomatic residents on the same unit
B. Only symptomatic residents in the entire facility
C. All asymptomatic residents on the same unit
D. All residents of the facility regardless of symptoms
E. All staff regardless of symptoms

ANSWER: C
In long-term care facilities, an influenza outbreak is defined as two laboratory-confirmed cases
of influenza within 72 hours in patients on the same unit. The CDC recommends
chemoprophylaxis for all asymptomatic residents of the affected unit. Any resident exhibiting
symptoms of influenza should be treated for influenza and not given chemoprophylaxis
dosing. Chemoprophylaxis is not recommended for residents of other units unless there are
two laboratory-confirmed cases in those units. Facility staff of the affected unit can be
considered for chemoprophylaxis if they have not been vaccinated or if they had a recent
vaccination, but chemoprophylaxis is not recommended for all staff in the entire facility.
An 18-month-old female is brought to your office by her mother for evaluation of a cough.
The patient has had low-grade fevers and a runny nose for 2 days. She now has a cough that is
worse at night. On examination she has a temperature of 37.5°C (99.5°F), a pulse rate of 120
beats/min, a respiratory rate of 30/min, and an oxygen saturation of 92% on room air. She is
noted to have hoarseness, mild inspiratory stridor, and a barking cough. She does not have
drooling or a muffled voice.
Which one of the following should be ordered to confirm the diagnosis?
A. No further testing
B. A CBC
C. A viral culture
D. Rapid antigen testing
E. A radiograph of the neck
ANSWER: A
This patient has croup, which is diagnosed clinically and no further testing is usually
indicated. A CBC is nonspecific and is usually only indicated if a bacterial cause of stridor is
suspected, such as bacterial tracheitis, epiglottitis, retropharyngeal abscess, or peritonsillar
abscess. Viral cultures and rapid antigen testing should be reserved for instances in which the
patient fails to respond as expected to initial treatment. A neck radiograph is not indicated in
the absence of findings that suggest possible epiglottitis, such as drooling or a muffled voice.

ABFM2019
Q1-A 6-month-old male is brought to the urgent care center with a 3-day history of rhinorrhea,
cough, and increased respiratory effort. His temperature is 37.5°C (99.5°F), his heart rate is
120 beats/min, his respiratory rate is 42/min, and his oxygen saturation is 96% on room air.
On examination the child appears well hydrated with clear secretions from his nasal passages,
there is diffuse wheezing heard bilaterally, and there is no nasal flaring or retractions. The
mother states that the child has a decreased appetite but is drinking a normal amount of fluids.
Which one of the following would be the most appropriate management for this patient?
A. Supportive therapy only
B. Bronchodilators
C. A corticosteroid taper
D. Epinephrine
E. Nebulized hypertonic saline

ANSWER: A
This patient’s symptoms and the examination suggest viral bronchiolitis. Supportive therapy,
including adequate hydration, is recommended for treatment. Treatment with bronchodilators,
epinephrine, hypertonic saline, or corticosteroids is not indicated (SOR A)
Q2-A 25-year-old male presents to your office with a nearly 3-month history of a persistent,
mildly productive cough. He does not have any fevers, myalgias, or night sweats but is often
congested. He has never smoked and is normotensive. An examination is notable for a normal
weight and vital signs, clear lungs, a mildly erythematous oropharynx, and pale, edematous
nasal mucosa. There is no lymphadenopathy. Which one of the following is the most likely
cause of this patient’s chronic cough?
A. Bronchogenic carcinoma
B. Chronic aspiration
C. Obstructive sleep apnea
D. Tuberculosis
E. Upper airway cough syndrome

ANSWER: E
The most common causes of chronic cough in adults include upper airway cough syndrome,
tobacco use, GERD, asthma, and ACE inhibitor use. The physical examination of this patient
is most consistent with upper airway cough syndrome, previously referred to as postnasal drip
syndrome. Given the patient’s lack of tobacco use and normal blood pressure, bronchogenic
carcinoma and obstructive sleep apnea are less likely. There are no risk factors in this patient’s
history to suggest chronic aspiration or tuberculosis.

Q3- A 2-year-old male is brought to your office by his mother. The child has a 2- day history
of a runny nose and mild cough associated with a subjective fever. The cough worsened last
night. The patient has had a reduced appetite but a good intake of fluids.
On examination the child has an axillary temperature of 37.4°C (99.3°F), a heart rate of 120
beats/min, a respiratory rate of 26/min, a weight of 16 kg (35 lb), and an oxygen saturation of
96% on room air. He appears mildly ill but is alert and does not show any signs of distress,
and has a prominent high-pitched barking cough. You note that he has clear rhinorrhea, the
tympanic membranes are normal, and the oropharynx is moist and clear. Auscultation reveals
inspiratory stridor, but there are no signs of respiratory distress. The patient’s skin has good
turgor with no rash. Which one of the following would be the most appropriate next step in the
management of this child?
A.Administration of dexamethasone, 0.6 mg/kg orally in a single dose
B.Initiation of oral amoxicillin, 40 mg/kg twice daily
C.Administration of albuterol, 2.5 mg/3 mL via nebulizer
D.Administration of racemic epinephrine 2.25% solution (Asthmanefrin), 0.5 mL via nebulizer
E.Transfer to a hospital emergency department for stabilization and hospital admission.
ANSWER: A
This patient has mild croup based on the clinical findings. A single dose of dexamethasone is
recommended in all cases of croup (SOR A). Hospitalization is not necessary if the child is
stable. Racemic epinephrine, which has been shown to reduce symptoms at 30 minutes but not
at 2 hours or 6 hours, is recommended for the treatment of moderate to severe croup when
patients are being observed in a medical setting such as the emergency department or hospital
(SOR A). Amoxicillin and albuterol are not indicated in the management of croup.

Q4-A 70-year-old female comes to your office with a 10-day history of a subjective fever at
home, facial and tooth pain, sinus pressure, and a green nasal discharge. There has been no
change in her symptoms. The patient has a history of allergic rhinitis treated with
immunotherapy, and a history of developing hives while taking penicillin. On examination her
temperature is 38.1°C (100.6°F).
In addition to nasal saline and analgesics, which one of the following would be the most
appropriate management?
A.Standard-dose amoxicillin/clavulanate (Augmentin)
B.High-dose amoxicillin/clavulanate
C.Azithromycin (Zithromax)
D.Doxycycline
ANSWER: D
This patient has a history of symptoms consistent with acute bacterial rhinosinusitis that have
persisted for 10 days, warranting empiric antibiotic therapy (SOR B). Doxycycline is an
appropriate alternative to amoxicillin/clavulanate for a patient with a history of a reaction to
penicillin. Macrolides and trimethoprim/sulfamethoxazole are not recommended as empiric
therapy because of high rates of resistance.

Q5-A 15-year-old male presents with a 2-day history of dark-colored urine, lower extremity
edema, and fatigue. Approximately 2 weeks ago he said he had a “bad sore throat” that was
treated empirically with amoxicillin. On examination his blood pressure is 144/92 mm Hg, his
pulse rate is 76 beats/min, and his other vital signs are normal. Other than mild dependent
edema there are no additional significant physical examination findings. A urinalysis dipstick
shows 3+hematuria.
Which one of the following findings on microscopic evaluation of the urine sediment would
help to confirm the diagnosis in this patient?
A. Gram-positive cocci in chains
B. RBC casts
C. WBC casts
D. Eosinophils
E. Oxalate crystals
ANSWER: B
This is a classic presentation for acute post-streptococcal glomerulonephritis (APSGN), with
the onset of gross hematuria associated with hypertension and systemic edema. This is most
commonly seen in school-age children, usually 1–2 weeks after an episode of pharyngitis or 3–
4 weeks after an episode of impetigo, caused by so-called nephritogenic strains of Group A -
hemolytic Streptococcus. The hematuria is caused by immune complex–mediated glomerular
injury. Bacteriuria may be seen in both upper and lower urinary tract infections, but may also
be a spurious finding, especially with the combined presence of epithelial cells. The classic
finding on microscopic urinalysis for acute glomerulonephritis is the presence of RBC casts.
WBC casts are seen with acute pyelonephritis. The presence of urinary eosinophils indicates
acute interstitial nephritis. Calcium oxalate makes up the most common type of kidney stones.
Antibiotics prescribed for antecedent pharyngitis do not prevent APSGN. Treatment is
supportive, controlling blood pressure and edema with a thiazide or a loop diuretic. The
prognosis for resolution and full recovery of the vast majority of patients with APSGN is
excellent, especially in the pediatric age group.

Q6-A 32-year-old female presents with a 4-month history of nasal drainage, congestion, and
loss of her sense of smell. She reports having a cold about 4 months ago that never resolved.
On examination the nasal turbinates are swollen and you note mucopurulent drainage on the
right. Which one of the following is the most likely cause of her symptoms?
A. Chronic rhinosinusitis
B. Granulomatosis with polyangiitis (Wegener’s granulomatosis)
C. Nasal polyposis
D. Sarcoidosis
E. Seasonal allergic rhinitis
ANSWER: A
The American Academy of Otolaryngology defines chronic rhinosinusitis as the presence of
two of four cardinal symptoms, which include nasal drainage, nasal obstruction, facial pain or
pressure, and hyposmia or anosmia, along with objective signs on examination or radiographic
studies. This patient has three cardinal symptoms of chronic rhinosinusitis and objective
evidence on the physical examination. No nasal polyps were seen on the examination.
Granulomatosis with polyangiitis and sarcoidosis can both present similarly but are
uncommon causes of chronic rhinosinusitis. Allergic rhinitis can be associated with chronic
rhinosinusitis but would also present with allergic symptoms.
Q7-You are caring for a 36-year-old woman who is plagued by frequent viral upper respiratory
infections during cold and flu season. She schedules an appointment with you and
would like to discuss the possibility of using Echinacea to help decrease her incidence of
illness. Of the following, which advice is best supported by theevidence?

a.There is no evidence to suggest that Echinacea will have any effect on illness incidence.
b.Echinacea has been shown to have a significant placebo effect, but has not been found to be
effective in reducing illness incidence.
c.Echinacea has been shown to decrease the risk for upper respiratory infections in well-
designed clinical studies.
d.The risk for using Echinacea outweighs the benefit for most people.
e.Echinacea has not been studied adequately enough for clinicians to render an opinion
regarding its use

answer is c.
Echinacea originated in Eastern North America and its plants were used extensively by Native
Americans. Their use continues to be popular today, with preparations made from roots,
stems, leaves, flowers, or combinations of parts. There have been dozens of studies using
Echinacea preparations for the prevention and treatment of upper respiratory tract infections.
A 2014 Cochrane review evaluated 12 studies for the prevention of colds and found that
although none of the trials individually showed a reduction in colds, when the data were
pooled, there was a 10% to 20% risk reduction in the treatment group. A meta- analysis in
2015 included six clinical studies with a total of 2458 participants and found a relative risk of
0.65 for upper respiratory tract infection in the experimental group. In 2016, there was a large
double-blind, randomized, placebo-controlled prevention trial in which 755 healthy people
were followed while either taking Echinacea or placebo. Those in the Echinacea arm
experienced 58% fewer days of cold symptoms. There are few risks in using Echinacea, and
although some trials show a placebo effect, there are well-designed studies that support its use.
ABFM 2018
Q8-A 52-year-old male smoker presents to your office in January with worsening respiratory
symptoms over the past 24 hours, along with a rapid onset of fever and chills, nausea, myalgias,
and sore throat. He has a history of mild chronic bronchitis and hypertension, and his
medications include tiotropium (Spiriva) inhaled daily; lisinopril/hydrochlorothiazide
(Zestoretic), 20/12.5 mg daily; and albuterol (Proventil, Ventolin) as needed.
On examination the patient has a temperature of 38.8°C (101.8°F), a heart rate of 102
beats/min, a respiratory rate of 24/min, and an oxygen saturation of 94% on room air. He is ill-
appearing and pale. Examination of his throat reveals mild erythema, and chest auscultation
reveals bilateral bronchovesicular breath sounds with no crackles or wheezing.
The examination is otherwise unremarkable. Laboratory and radiology services are not
available.
Which one of the following would be most appropriate at this point?
A) Observation only, with follow-up in a few days
B) Azithromycin (Zithromax)
C) Oseltamivir (Tamiflu)
D) Penicillin VK
E) Prednisone

ANSWER: C
This patient has findings consistent with influenza, including a rapid onset of fever, nausea, and
sore throat, and negative pulmonary findings. Influenza is considered a clinical diagnosis and
confirmation of the diagnosis with laboratory testing is not required. Treatment of influenza is
recommended for individuals at a high risk of influenza-related complications. High-risk
individuals include those with chronic lung disease; cardiovascular (excluding hypertension),
renal, hepatic, hematologic, or neurologic disease; or age >65. Children on long-term aspirin
therapy, and pregnant and postpartum women are also considered high risk. This patient should
be treated with antiviral medication because of his chronic pulmonary disease. While
pneumonia and streptococcal pharyngitis should be considered in the differential diagnosis,
these are less likely given the examination findings, and antibiotics are not recommended.
Prednisone is not indicated for influenza-like illness and may cause harm.
Q9-A 7-month-old male is admitted to the hospital for respiratory syncytial virus
bronchiolitis. His temperature is 37.9°C (100.2°F), pulse rate 160 beats/min, respiratory rate
70/min, and oxygen saturation 92% on room air. Auscultation of the lungs reveals diffuse
wheezing and crackles accompanied by nasal flaring and retractions.
Which one of the following interventions would most likely be beneficial?
A) Bronchodilators
B) Corticosteroids
C) Epinephrine
D) Nasogastric fluids
E) Oxygen supplementation to maintain O2 saturation above 95%

ANSWER: D
The mainstay of therapy for acute respiratory syncytial virus bronchiolitis is supportive care,
and maintaining hydration is important. Infants with respiratory rates >60/min may have poor
feeding secondary to difficulty breathing and oral rehydration may increase the risk of
aspiration. In these cases, nasogastric or intravenous fluids should be administered. Oxygen
saturation of 90% or more on room air is sufficient for infants with bronchiolitis, and using
supplemental oxygen to maintain higher oxygen saturations only prolongs hospitalization
because of an assumed need for oxygen. Bronchodilators should not be administered to infants
with bronchiolitis, because they have not been shown to have any effect on the need for
hospitalization, oxygen saturation, or disease resolution. In addition, there is no evidence to
support the use of epinephrine or corticosteroids in the inpatient setting.

Q10-A 34-year-old female presents with a 3-month history of a minimally productive cough.
She has never smoked. She does not have any fever, weight loss, rhinorrhea, congestion, or
heartburn. She does not have a known history of allergies or asthma and has tried over-the-
counter cold remedies, cough syrups, and cough drops without significant relief. She is
otherwise healthy and takes no medications. On examination her vital signs are normal. An
ear, nose, and throat examination is remarkable for swollen nasal turbinates. A lung
examination is normal. Given the duration of the cough, you order a chest radiograph, which
is normal as well. ?Which one of the following would be most appropriate at this point
A) A trial of an intranasal corticosteroid
B) A trial of an inhaled bronchodilator
C) A trial of a proton pump inhibitor
D) A sinus radiograph
E) Referral for allergy testing
ANSWER: A
According to the CDC, cough is the most common symptom resulting in primary care visits.
Chronic cough in adults is defined as one that lasts 8 weeks or more. The workup should include
a history focusing on potential triggers, as well as the identification of any red flags. If the
physical examination is normal and the patient's history does not indicate the cause of the
cough, a chest radiograph is appropriate . The most common cause of chronic cough in adults
is upper airway cough syndrome. Patients might have nasal symptoms such as rhinorrhea or
congestion. Physical findings can include swollen turbinates and posterior pharyngeal
cobblestoning, or they can be unremarkable. Initial treatment may include the use of
decongestants, oral or intranasal antihistamines, intranasal corticosteroids, or saline nasal rinses
(SOR C). Symptoms should resolve within a few weeks, and referral for allergy testing can be
considered if they are not resolved within 2 months. CT of the sinuses can be considered as
well, but sinus radiographs are more specific . Other common causes of chronic cough include
asthma, nonasthmatic eosinophilic bronchitis, and GERD. If asthma is suspected, spirometry
is indicated. If spirometry is positive for asthma, a trial of an inhaled bronchodilator is
indicated. If there are other indications of GERD such as heartburn, globus sensation, or
hoarseness, an antacid or a trial of a proton pump inhibitor is indicated.

Q11-A 22-year-old female presents to your office for evaluation of nasal and sinus
congestion,frequent sneezing, and itchy red eyes. These symptoms have been present 5–7 days
per week for the past 6 months. She has had similar symptoms in the past but they have never
lasted this long. She moved into a new home 2 months ago. There are no animals in the house.
She has tried over-the-counter fexofenadine (Allegra) with only partial relief of symptoms.
Which one of the following would be the most appropriate recommendation at this time?
A) Use of a mite-proof impermeable pillow cover
B) Intranasal saline irrigation
C) Intranasal azelastine (Astepro)
D) Intranasal budesonide (Rhinocort)
E) CT of the sinuses
ANSWER: D
This patient has symptoms consistent with allergic rhinitis, and the presence of symptoms
more than 4 days per week and for more than 4 weeks places her into the persistent symptoms
category. In addition to allergen avoidance and patient education, an intranasal corticosteroid
should be the first-line treatment for allergic rhinitis with persistent symptoms (SOR A). The
Choosing Wisely recommendations from the American Academy of Otolaryngology-Head and
Neck Surgery Foundations include avoiding sinonasal imaging in patients with symptoms
limited to a primary diagnosis of allergic rhinitis. Impermeable pillow or mattress covers are
often recommended but there is no evidence of any benefit (SOR A).
Intranasal saline irrigation is beneficial and can be used as monotherapy for mild intermittent
symptoms, but intranasal corticosteroids are likely to provide more benefit for more persistent
symptoms. Intranasal antihistamines such as azelastine are more expensive, less effective, and
more likely to produce adverse effects than intranasal corticosteroids, so they are not
recommended as first-line therapy (SOR B).

Q12-A 36-year-old female singer presents with a 10-day history of hoarseness. She has never
smoked and does not take any medications. Her vital signs are normal. An oropharyngeal
examination is normal, her chest is clear to auscultation, and there is no cervical adenopathy
and no masses. She is anxious to be able to sing again as soon as possible.
Which one of the following would you advise at this time?
A) No talking, whispering, or throat clearing for 48 hours
B) No singing or loud talking for 5–7 days
C) Nebulized hypertonic saline treatments 3 times daily for 2–3 days
D) Nebulized ribavirin twice daily for 3 days
E) Inhaled corticosteroids twice daily for 5 days

ANSWER: A
Complete vocal rest, including no whispering or throat clearing, is the most effective and
quickest initial remedy for short-duration laryngitis, whether viral or due to vocal overuse or
abuse. Limiting voice use or whispering, as opposed to complete vocal rest, will likely
prolong and possibly worsen hoarseness. Clearing the throat of mucus should also be avoided
for the same reason. Inhaled corticosteroids and antibiotics are not effective treatments for
laryngitis.
Hypertonic saline nebulization treatments would likely cause violent coughing fits that would
worsen the condition. Nebulized ribavirin is never indicated for use in adults.

Q13-A 34-year-old male has a 3-day history of a runny nose, postnasal drainage, sinus
congestion, and left-sided facial pain. He also reports a mild cough and difficulty sleeping due
to the congestion. He is afebrile and the examination reveals inflammation of the nasal
mucosa, purulent rhinorrhea, and mild left maxillary sinus tenderness to percussion.
Which one of the following would be the most appropriate pharmacotherapy?
A) Amoxicillin/clavulanate (Augmentin)
B) Levofloxacin (Levaquin)
C) Loratadine (Claritin)
D) Mometasone (Nasonex)
ANSWER: D
This patient presents with symptoms of acute rhinosinusitis. In the first 3–4 days, viral and
bacterial rhinosinusitis are indistinguishable. Guidelines from the American Academy of
Otolaryngology—Head and Neck Surgery suggest that antibiotics should not be routinely
prescribed for acute mild to moderate sinusitis unless symptoms persist for 7 days or worsen
after initial improvement. Watchful waiting without antibiotic treatment is appropriate when
follow-up is accessible (SOR A). In this scenario antibiotic therapy is not indicated.
Amoxicillin with or without clavulanate is appropriate for symptoms lasting 7 or more days
without improvement and is the first-line antibiotic treatment for acute bacterial rhinosinusitis
(SOR A). Due to the risk of adverse effects and no benefit over -lactams, respiratory
fluoroquinolones are not considered first-line antibiotic therapy. Symptomatic treatment is
recommended within the first 10 days of the onset of symptoms and may be continued if
antibiotics are started. Intranasal corticosteroid use has a modest therapeutic benefit for
patients with acute rhinosinusitis. Decongestants and antihistamines have not been proven
effective for the treatment of acute rhinosinusitis.

Q14- A 36-year-old male presents with a 4-month history of persistent nasal congestion and
rhinorrhea. On examination he has clear nasal discharge and edema of the nasal mucosa but no
nasal polyps are noted. His current medications include intranasal fluticasone (Flonase).
Which one of the following would be the most appropriate management of his chronic
symptoms?
A) Recommend neti pot nasal irrigation
B) Add oral amoxicillin
C) Add oral prednisone
D) Replace fluticasone with budesonide (Rhinocort) nasal spray

ANSWER: A
In addition to nasal corticosteroids, saline irrigation is a mainstay in the treatment of chronic
rhinosinusitis. Low-pressure, high-volume irrigation, such as with a neti pot, is superior to
nasal saline spray (SOR B). There is no evidence that one nasal corticosteroid is better than
another. The role of antibiotics in the treatment of chronic rhinosinusitis is unclear. Antibiotics
may be helpful in patients with signs of bacterial infection, such as mucopurulent drainage or
acute worsening of symptoms. Oral corticosteroids are an option for the short-term
improvement of severe symptoms in patients with nasal polyps who are already on
maintenance therapy with both nasal saline irrigation and an intranasal corticosteroid spray.
Q15-A 30-year-old white male presents to the emergency department with a 4- day history of
fever to 101°F, a sore throat, rhinorrhea, and cough. An examination reveals rhinorrhea and a
boggy nasal mucosa, but is otherwise unremarkable. A chest radiograph shows a questionable
infiltrate.
Which one of the following would help determine if antibiotic treatment wouldbe
appropriate?
A) A C-reactive protein level
B) A procalcitonin level
C) A WBC count with differential
D) An erythrocyte sedimentation rate
E) CT of the chest

ANSWER: B
Using a procalcitonin-guided therapy algorithm reduces antibiotic use by 3.47 days without
increasing either morbidity or mortality in adults with acute respiratory infections. If the
procalcitonin level is <0.10 mg/dL, a bacterial infection is highly unlikely and it is strongly
recommended that antibiotics not be prescribed. If the procalcitonin level is 0.10–0.24 mg/dL
a bacterial infection is still unlikely and it is recommended that antibiotics not be used. If the
level is 0.25–
0.50 mg/dL a bacterial infection is likely and antibiotics are recommended. It is strongly
recommended that antibiotics be given if the level is >0.50 mg/dL, because a bacterial
infection is very likely.

ABFM2016
Q16- A 2-year-old male is brought to the emergency department by his frightened mother
following the sudden onset of nasal stuffiness and a harsh, barking, nonproductive cough. The
child does not appear significantly distressed. His temperature is 37.9°C (100.2°F) orally.
Mild nasal flaring is present. The nasopharyngeal mucus appears mildly edematous and
injected, and upper airway noises are heard, but good air movement is evident on auscultation
of his chest.
Which one of the following is the most appropriate treatment for this patient?
A) A bedside humidifier
B) A single dose of oral dexamethasone
C) Amoxicillin for 7 days
D) Nebulizer treatment with racemic epinephrine
E) Tracheal intubation and oxygen administration
ANSWER: B
Croup is a common, self-limited illness caused by viral infection of the upper respiratory tract.
The diagnosis is based primarily on the clinical history and examination findings. A history of
the abrupt onset of a barking cough, inspiratory stridor, and hoarseness in a 2-year-old child is
typical of croup, although it can present at any age between 6 months and 12 years. Low-grade
fever, a barking cough, and varying degrees of respiratory distress (nasal flaring, retractions,
or stridor) are typically present on examination. Findings such as a toxic appearance,
wheezing, drooling, and difficulty swallowing are not consistent with the diagnosis.
Treatment of mild disease with a corticosteroid has proven benefit (SOR A) even when
administered as a single oral dose (SOR B). Nebulized epinephrine has been shown to improve
outcomes in children with moderate to severe croup (SOR A). Humidification therapy in the
emergency department setting provides no benefit (SOR A). This child has mild croup and a
single dose of dexamethasone (0.15–0.60 mg/kg, usually given orally) followed by close
observation is the most appropriate treatment.

ABFM2015
Q17-You are the medical director of a long-term-care facility that has 60 residents. Several
patients experience fever, cough, and upper respiratory symptoms. Two of these patients test
positive for influenza A (H1N1) virus. Which one of the following is recommended by the
Centers for Disease Controland Prevention (CDC) for this situation?
A)Chemoprophylaxis with appropriate medications for all residents
B)Treatment initiated on an individual basis once testing confirms that a resident has influenza
C)Prophylaxis only for staff who have had direct patient contact with a resident with
laboratory-confirmed infection
D)No chemoprophylaxis for staff or residents who have been appropriately vaccinated.

ANSWER: A
The occurrence of two or more laboratory-confirmed cases of influenza A is considered an
outbreak in a long-term care facility. The CDC has specific recommendations for managing an
outbreak, which include chemoprophylaxis with an appropriate medication for all residents
who are asymptomatic and treatment for all residents who are symptomatic, regardless of
laboratory confirmation of infection or vaccination status. All staff should be considered for
chemoprophylaxis regardless of whether they have had direct
patient contact with an infected resident or have received the vaccine. Requesting restriction
of visitation is recommended; however, it cannot be strictly enforced due to residents’ rights
Q18- A 30-year-old female with a history of prolonged QT syndrome presents with severe
acute bacterial sinusitis. Which one of the following antibiotics should be avoided?
A) Amoxicillin
B) Clarithromycin (Biaxin)
C) Amoxicillin/clavulanate (Augmentin)
D) Moxifloxacin (Avelox)
E) Cefuroxime (Ceftin)

ANSWER: B
A number of medications can cause or exacerbate prolonged QT syndrome, which can lead to
torsades de pointes. This can be associated with syncope or degenerate into a sustained
ventricular tachycardia or ventricular fibrillation. Clarithromycin interferes with the delayed
rectifier potassium current, which results in the accumulation of potassium ions in cardiac
myocytes and thereby delays cardiac repolarization. This leads to prolongation of the QT
interval and therefore the risk of fatal arrhythmia. Clarithromycin is metabolized by the
cytochrome P450 3A enzyme. When using clarithromycin it is important to avoid any other
medications that may inhibit this enzyme, leading to higher clarithromycin levels. The other
antibiotics listed do not have this effect.

Q19- Which one of the following is true regarding respiratory syncytial virus (RSV) infection?
A) Most infections in the United States occur between August and December
B) Corticosteroids should be a routine part of treatment
C) The diagnosis is usually based on positive serology
D) It is rarely associated with bacterial co-infection.

ANSWER: D
Respiratory syncytial virus (RSV) is a common cause of respiratory tract infections in children.
The infections are usually self-limited and are rarely associated with bacterial co-infection, but
in very young infants, prematurely born infants, or those with pre-existing heart/lung
conditions, the infection can be severe. In North America, RSV season is November to April.
Treatment is primarily supportive, including a trial of bronchodilators, with continued use only
if there is an immediate response. Corticosteroids and antibiotics are not routinely indicated
(SOR B). Routine laboratory and radiologic studies should not be used in making the
diagnosis, as it is based on the history and physical examination (SOR C).
Q20-Which one of the following is true regarding the live attenuated intranasal influenza
vaccine?
A) It is preferred in all children >6 months of age
B) It is more effective in children age 2–6 years than the inactivated vaccine
C) It is more effective in children >6 years of age than in younger children
D) It is the vaccine of choice for pregnant women
E) It is less effective in adults age 18–49 than the inactivated vaccine

ANSWER: B
The live attenuated intranasal influenza vaccine is recommended for healthy nonpregnant
persons 2–49 years of age. It is more effective than the inactivated vaccine in children 2–6
years of age; for patients 6–49 years of age either the live attenuated intranasal or the
inactivated vaccine is recommended. The live intranasal vaccine is contraindicated in
pregnancy and in patients with asthma or COPD. Patients older than 49 years should receive
the inactivated vaccine

Q21-A 26-year-old male presents with a sore throat and a temperature of 38.3°C (101.0°F). On
examination you note muffling of the voice and unilateral tonsillar swelling with a shift of the
uvula away from the affected tonsil. A rapid test for Streptococcus pyogenes is negative.
Which one of the following would be most appropriate at this point?
A)Laboratory testing for infectious mononucleosis
B)Immediate tonsillectomy
C)Initiation of antibiotics with close clinical follow-up
D)Culture of the throat and delayed initiation of antibiotics pending results
ANSWER: B

This patient has examination findings that strongly suggest a peritonsillar abscess, which is the
most common deep infection of the head and neck in young adults.
Although antibiotics are indicated in this case, the cornerstone of management is drainage of
the abscess either by needle drainage or by incision and drainage.
Immediate tonsillectomy is less favored, as it is a less cost-effective option.
Q22-A 32-year-old male smoker presents with a 4-day history of progressive hoarseness. He
is almost unable to speak, and associated symptoms include a cough slightly productive of
yellow sputum, as well as tenderness over the ethmoid sinuses. He is afebrile and has normal
ear and lung examinations. His oropharynx is slightly red with no exudate, and examination of
his nasal passages reveals mucosal congestion.
Which one of the following would be the most appropriate treatment?
A) Amoxicillin for 10 days
B) Omeprazole (Prilosec), 40 mg daily
C) Azithromycin (Zithromax) for 5 days
D) Symptomatic treatment only

ANSWER: D
Acute laryngitis most often has a viral etiology and symptomatic treatment is therefore most
appropriate. A Cochrane review concluded that antibiotics appear to have no benefit in treating
acute laryngitis. Proton pump inhibitors such as omeprazole can be of benefit in treating
chronic laryngitiscaused by acid reflux, but not for an acute problem such as the one describe

Q23-Which one of the following is recommended in all patients with croup, including those
with mild disease?
A) Humidification therapy
B) Oral dexamethasone as a single dose
C) Oral diphenhydramine (Benadryl) every 6 hours until improvement
D) Subcutaneous epinephrine as a single dose
E) Intramuscular ceftriaxone (Rocephin) as a single dose

ANSWER: B
A single dose of dexamethasone (0.15–0.60 mg/kg, usually given orally) is recommended in
all patients with croup, including those with mild disease. Humidification therapy has not been
proven beneficial. Nebulized epinephrine is an accepted treatment in patients with moderate to
severe croup. Subcutaneous epinephrine, diphenhydramine, and ceftriaxone are not
recommended treatments
A24- An 18-month-old previously healthy infant is admitted to the hospital with bronchiolitis.
Pulse oximetry on admission is 92% on room air.
Which one of the following should be included in the management of this patient?
A)Tracheal suction to clear the lower airways
B)Nasal suction to clear the upper airway
C)Chest physiotherapy
D)Corticosteroids
E)Azithromycin (Zithromax)
ANSWER: B
Recommendations for the treatment of hospitalized infants with bronchiolitis include nasal
suctioning via bulb or neosucker to clear the upper airway. Deep suction (beyond the
nasopharynx) is not recommended. Oxygen is recommended for infants with a persistent
oxygen saturation <90%. Bronchodilators should not be used routinely in the management of
bronchiolitis, and corticosteroids, antibiotics, nasal decongestants, and chest physiotherapy are
not recommended. A single trial of inhaled epinephrine or albuterol for respiratory distress
may be considered, but only if there is a history of asthma, atopy, or allergy.

ABFM2014
Q25- A healthy 24-year-old male presents with a sore throat of 2 days’ duration. He reports
mild congestion and a dry cough. On examination his temperature is 37.2°C (99.0°F). His
pharynx is red without exudates, and there are no anterior cervical nodes. His tympanic
membranes are normal, and his chest is clear.
Which one of the following would be most appropriate at this point?
A) Analgesics and supportive care only
B) A rapid strep test
C) A throat culture and empiric treatment with penicillin
D) Azithromycin (Zithromax
ANSWER: A
The Centers for Disease Control and Prevention (CDC) assembled a panel of national health
experts to develop evidence-based guidelines for evaluating and treating adults with acute
respiratory disease. According to these guidelines, the most reliable clinical predictors of
streptococcal pharyngitis are the Centor criteria. These include tonsillar exudates, tender
anterior cervical lymphadenopathy, absence of cough, and history of fever. The presence of
three or four of these criteria has a positive predictive value of 40%–60%, and the absence of
three or four of these criteria has a negative predictive value of 80%. Patients with four
positive criteria should be treated with antibiotics, those with three positive criteria should be
tested and treated if positive, and those with 0–1positive criteria should be treated with
analgesics and supportive care only. This patient has only one of the Centor criteria, and
should therefore not be tested or treated with antibiotics
Q26-Which one of the following ethnic groups in the United States is at greatest risk for
complications from influenza?
A) African-American
B) Asian-American
C) Mexican-American
D) Native American
E) Scandinavian-American
ANSWER: D
While anyone, even previously healthy individuals, may benefit from treatment of symptomatic
clinical influenza infection with antiviral agents, not everyone who has been exposed but is
asymptomatic requires chemoprophylaxis. However, persons at higher risk for complications
from influenza should be considered for preventive treatment. Those at highest risk include
children under the age of 2 years, pregnant women (including women less than 2 weeks post
partum), adults over the age of 65, the morbidly obese (BMI >40 kg/m2), and Native or
Alaskan Americans. If persons at high risk for influenza complications are not treated
prophylactically with antiviral agents after exposure, then they should receive prompt
treatment as soon as possible after developing signs and symptoms of influenza infection
Q27-The Infectious Diseases Society of America recommends which one of the following as
the drug of choice for group A streptococcal pharyngitis?
A) Azithromycin (Zithromax)
B) Cefadroxil
C) Cephalexin (Keflex)
D) Clindamycin (Cleocin)
E) Penicillin

ANSWER: E
The Infectious Diseases Society of America recommends that penicillin remain the treatment
of choice for group A streptococcal pharyngitis because of its proven efficacy, safety, narrow
spectrum, and low cost. Penicillin-resistant group A Streptococcus has never been
documented. Amoxicillin is often used in place of penicillin V as oral therapy for young
children, primarily because of acceptance of the taste of the suspension. The other options
listed are all possible regimens for group A streptococcal pharyngitis but penicillin is still
considered the treatment of choice
Q28--A 25-year-old female kindergarten teacher comes to your office for evaluation of a
cough she has had for 2 weeks. The preceding week she had symptoms of rhinorrhea, mild
malaise, low-grade fever, and lacrimation. She reports that episodes of coughing are so severe
that vomiting is induced. She was evaluated at a walk-in clinic 1 week ago and was diagnosed
with bronchitis. Treatment with hydrocodone cough syrup and amoxicillin has not helped. On
examination she has mild rhinorrhea and injected conjunctivae, but her lungs are clear. A chest
radiograph is normal and her laboratory results reveal a mild lymphocytosis.
Which one of the following is the most appropriate next step in the managementof this patient?
A)Corticosteroid therapy
B)A sputum culture
C)A nasopharyngeal culture and polymerase chain reaction testing
D)Direct fluorescent antibody testing
E)Serologic testing

ANSWER: C
Whooping cough has reemerged over the past few years. The initial catarrhal stage is
manifested by nonspecific symptoms similar to those of a viral upper respiratory illness. This
stage is usually 1–2 weeks in duration, and the patient is highly contagious. The paroxysmal
stage is manifested by severe coughing spells that occur in paroxysms and may be followed by
the inspiratory whoop (much more likely in children). Post-tussive emesis is another classic
sign. There are no characteristic findings on examination other than signs induced by extreme
coughing. The CDC recommends both a nasopharyngeal culture and polymerase chain reaction
testing to confirm the diagnosis. Serologic testing is useful only in research settings, and direct
fluorescent antibody testing is not recommended. Azithromycin should be used as initial
therapy, but this is to decrease transmission of the illness and does not improve symptoms

Q29--A 2-year-old female is brought to the urgent care center with a fever and cough. Her
symptoms started suddenly a few hours ago with a runny nose and fever to 101°F. On
examination the child is crying and has a persistent barking cough but has no stridor or
significant respiratory distress. Her lungs are clear to auscultation. Her skin is warm, pink, and
well perfused, and her oxygen saturation is 99% on room air. A chest radiograph is normal.

Which one of the following treatments has been shown to improve outcomes for this problem?
A)Humidified air
B)Nebulized albuterol (Proventil, Ventolin)
C)Oral azithromycin (Zithromax)
D)Oral dexamethasone
E)Oxygen therapy
ANSWER: D
This patient presents with a typical case of mild to moderate croup. This is a viral infection that
results in swelling in the larynx. It rarely is severe enough to cause respiratory collapse or
require intubation and must be differentiated from more severe conditions such as epiglottitis,
retropharyngeal abscess, or pneumonia.
There is no reason to treat this viral infection with an antibiotic. The condition is usually
benign and self-limiting, with the worst symptoms occurring at night. Cool and/or humidified
air has traditionally been recommended, but studies have not confirmed any significant benefit
from these interventions. Since this child is not in respiratory distress and oxygenation is
normal, supplemental oxygen therapy is not indicated. Studies have confirmed the benefits of
treating croup with a single dose of either an oral or intramuscular corticosteroid. Specifically,
dexamethasone is recommended due to its 72-hour length of effect. Inhaled racemic
epinephrine has been shown to reduce the need for intubation in cases of moderate to severe
croup. Albuterol, however, is not indicated

Q30--An 18-month-old male with a history of prematurity at 36 weeks gestation but no


baseline lung disease is brought to the emergency department with a fever of 38.3°C (100.9°F),
rhinorrhea, cough, wheezing, mild tachypnea, and an oxygen saturation of 88%. A chest
radiograph reveals perihilar infiltrates, and a nasal swab is positive for respiratory syncytial
virus (RSV) antigen.
Which one of the following management options has evidence of benefit for this patient?
A) Aerosolized ribavirin
B) Supplemental oxygen
C) Intravenous corticosteroids
D) Macrolide antibiotics
ANSWER: B
Respiratory syncytial virus (RSV) bronchiolitis is responsible for approximately 2.1 million
health care encounters annually in the United States. The child in this case has a typical
presentation of RSV bronchiolitis. The diagnosis can be made clinically, although specific
testing for RSV is often used in the hospital setting to segregate RSV-infected patients from
others. Management is primarily supportive, especially including maintenance of hydration
and oxygenation.
Bronchodilators, corticosteroids, and antiviral agents do not have a significant impact on
symptoms or the disease course. Ribavirin is not recommended for routine use due to its
expense, conflicting data on effectiveness, and potential toxicity to exposed health care
workers. Antibiotics are of no benefit in the absence of bacterial superinfection
Q31-A 2-year-old female is brought in by her father for evaluation of a cough. Her cough
started 10 days ago along with a runny nose and a low-grade fever. The runny nose and fever
are no longer present but a dry-sounding cough persists.
On examination the patient appears well and has a normal heart rate and respiratory rate. You
note no retractions and lung sounds are also normal. Which one of the following would be an
appropriate management option?
A) Buckwheat honey
B) Albuterol (Proventil, Ventolin)
C) Azithromycin (Zithromax)
D) Dextromethorphan
E) Diphenhydramine (Benadryl
ANSWER: A
This patient’s symptoms are most consistent with a viral upper respiratory infection. There is
no curative treatment so management should be focused on symptoms. Most over-the-counter
cough and cold preparations, including the ingredients dextromethorphan and
diphenhydramine, have no evidence of benefit and carry a risk of harm in children and should
not be recommended. Albuterol is only helpful for cough in patients with wheezing.
Buckwheat honey has limited evidence of effectiveness but appears to carry no risk of harm
and may be recommended for symptom management sore throat, a runny nose, and loss of her
voice. She is still symptomatic and the father is concerned about the longevity of this illness
and requests antibiotic therapy.
On examination the patient is afebrile with normal tympanic membranes, moderate
mucopurulent posterior pharyngeal drainage, and a normal cardiopulmonary examination. She
appears alert and active in the office, with no signs of acute distress.
Which one of the following is the most appropriate management at this time?
A) Reassurance and supportive care
B) Diphenhydramine (Benadryl)
C) Amoxicillin
D) Azithromycin (Zithromax)
E) Cefdinir
ANSWER: A
This patient has a diagnosis of laryngitis (a viral infection causing inflammation of the vocal
cords lasting less than 3 weeks). Symptoms of laryngitis can include loss/muffling of the
voice, as well as other classic symptoms of an upper respiratory tract infection. A Cochrane
study has shown that antibiotic therapy does not decrease the duration of laryngitis symptoms
or hasten the return of vocal patency (SOR A). The U.S. Food and Drug Administration no
longer recommends the use of over-the-counter cough and cold medications for children under
the age of 2 years, because of their serious and potentially life-threatening adverse effects,
including respiratory arrest due to hypopnea (SOR B )
LRTI+ COVID-19
ABFM 2018
A 35-year-old white female presents with recurrent wheezing and coughing over the past few
weeks, and recent production of brown sputum plugs. She is a regular patient of yours and has
a long history of asthma and multiple allergies. She has been treated four times in the last 3
months for asthma exacerbations and generally feels better the first day she takes her
corticosteroid, but any attempt at tapering leads to a recurrence of symptoms. She previously
had good control of her asthma, although she has required regular use of a high-dose inhaled
corticosteroid and a long-acting -agonist. In spite of just completing a course of levofloxacin
(Levaquin) for suspected pneumonia she returns today with a recurrence of the same
symptoms.
A physical examination is unremarkable with the exception of diffuse expiratory wheezing.
She has no fever or other abnormal vital signs. A chest radiograph shows opacities in the
upper and middle lobes and a CBC is concerning for eosinophilia.
Which one of the following is the most likely diagnosis?
A) Allergic bronchopulmonary aspergillosis
B) Community-acquired pneumonia
C) Pulmonary embolism
D) Medication nonadherence

ANSWER: A
Allergic bronchopulmonary aspergillosis (ABPA) affects 1%–12% of immunocompetent
patients with asthma and is important to consider in patients with recurrent exacerbations
because it can cause permanent lung damage if it is undetected and untreated. The symptoms
alone are insufficient for a diagnosis, but this clinical presentation should prompt
consideration of the diagnosis, and some of the symptoms and findings noted are included in
the diagnostic criteria. The major diagnostic criteria include the presence of asthma or cystic
fibrosis and immediate skin reactivity to Aspergillus antigens, peripheral eosinophilia,
transient pulmonary infiltrates or opacities, central bronchiectasis on a chest radiograph or CT,
serum precipitating antibodies to Aspergillus fumigatus , and elevated Aspergillus IgE- and/or
IgG-specific antibodies. Minor criteria that support the diagnosis include production of
brownish mucus plugs, identification of Aspergillus in the sputum, and delayed skin
sensitivity to Aspergillus.
Pneumonia is unlikely in this case given recent treatment with a respiratory fluoroquinolone
and a lack of common symptoms such as fever, chills, tachycardia, tachypnea, and pleuritic
chest pain, along with a cough productive of mucopurulent sputum. The most common
symptoms of pulmonary embolism include dyspnea, chest pain, syncope, tachypnea, and a
cough. While medication nonadherence may increase asthma exacerbations and wheezing, it
would be unlikely to be related to the new brown mucus production.

A 45-year-old male presents to your office with a 2-month history of a nonproductive cough,
mild shortness of breath, fatigue, and a 5-lb weight loss. On examination his lungs are clear. A
PPD skin test is negative. A chest radiograph shows bilateral hilar adenopathy and his
angiotensin converting enzyme level is elevated. A biopsy of the lymph node shows a
noncaseating granuloma.
Which one of the following would be the most appropriate initial treatment?
A) Azathioprine (Imuran)
B) Fluconazole (Diflucan)
C) Isoniazid
D) Levofloxacin (Levaquin)
E) Prednisone
ANSWER: E
This patient has sarcoidosis that has been confirmed by a biopsy. He is symptomatic so
treatment would be indicated. The recommended initial treatment for sarcoidosis is oral
corticosteroids. Anti-infective agents are not appropriate treatment for sarcoidosis.
Immunosuppressants are second- and third-line therapy for sarcoidosis and would not be
recommended as first-line treatment.

A 68-year-old male presents to your office with a 2-day history of headache, muscle aches,
and chills. His wife adds that his temperature has been up to 104.1°F and he seems confused
sometimes. His symptoms have not improved with usual care, including ibuprofen and
increased fluid intake. He and his wife returned from a cruise 10 days ago but don’t recall
anyone having a similar illness on the ship. This morning he started to cough and his wife was
concerned because she saw some blood in his sputum. He also states that he experiences
intermittent shortness of breath and feels nauseated. His blood pressure is 100/70 mm Hg,
heart rate 98/min, temperature 39.4°C (102.9°F), and oxygen saturation 95% on room air.
Which one of the following would be the preferred method to confirm your suspected
diagnosis of Legionnaires’ disease?
A) Initiating azithromycin (Zithromax) to see if symptoms improve
B) A chest radiograph C) Legionella polymerase chain reaction (PCR) testing
D) A sputum culture for Legionella
E) Urine testing for Legionella pneumophila antigen

ANSWER: E
A urine test for Legionella pneumophila antigen is the preferred method to confirm
Legionnaires’ disease. This test is rapid and will only detect Legionella pneumophila antigen.
A sputum culture is the gold standard for the diagnosis of Legionnaires’ disease but it requires
48–72 hours. A chest radiograph does not confirm the diagnosis but may show the extent of
disease. Responding to antibiotic treatment does not confirm a specific diagnosis.

A 7-month-old male is admitted to the hospital for respiratory syncytial virus bronchiolitis.
His temperature is 37.9°C (100.2°F), pulse rate 160 beats/min, respiratory rate 70/min, and
oxygen saturation 92% on room air. Auscultation of the lungs reveals diffuse wheezing and
crackles accompanied by nasal flaring and retractions.
Which one of the following interventions would most likely be beneficial?
A) Bronchodilators
B) Corticosteroids
C) Epinephrine
D) Nasogastric fluids
E) Oxygen supplementation to maintain O2 saturation above 95%
ANSWER: D
The mainstay of therapy for acute respiratory syncytial virus bronchiolitis is supportive care,
and maintaining hydration is important. Infants with respiratory rates >60/min may have poor
feeding secondary to difficulty breathing and oral rehydration may increase the risk of
aspiration. In these cases, nasogastric or intravenous fluids should be administered. Oxygen
saturation of 90% or more on room air is sufficient for infants with bronchiolitis, and using
supplemental oxygen to maintain higher oxygen saturations only prolongs hospitalization
because of an assumed need for oxygen. Bronchodilators should not be administered to infants
with bronchiolitis, because they have not been shown to have any effect on the need for
hospitalization, oxygen saturation, or disease resolution. In addition, there is no evidence to
support the use of epinephrine or corticosteroids in the inpatient setting.

An 18-month-old female is brought to your office in January for evaluation of a cough and
fever. She has no chronic medical conditions. She abruptly developed a barking cough and
hoarseness with a low-grade fever 2 days ago. The cough is worse at night. She has been
drinking normally but is not interested in eating. On examination she is alert and resists the
examination. Her respiratory rate and effort are normal. She has no stridor or wheezing.
Which one of the following would be most appropriate at this point?
A) A nasal swab for influenza testing
B) A chest radiograph
C) A single dose of oral dexamethasone
D) Azithromycin (Zithromax)
E) Oseltamivir (Tamiflu)
ANSWER: C
This patient has symptoms consistent with croup, a lower respiratory infection that is common
in the winter months in children ages 6 months to 3 years. The diagnosis is clinical and should
be suspected in children with a history of a sudden onset of a deep cough, hoarseness, and a
low-grade fever. Randomized studies have shown that even with mild croup (an occasional
barking cough with no stridor at rest), oral corticosteroids provide some benefit.

A 42-year-old male presents with a fever, cough, and chest pain. A rapid influenza test is
positive.
Which one of the following would be most appropriate for the management of this patient’s
pleuritic chest pain?
A) Colchicine (Colcrys)
B) Hydrocodone
C) Ibuprofen
D) Prednisone
E) Tramadol (Ultram)

ANSWER: C
NSAIDs such as ibuprofen should be used as first-line treatment for the control of pleuritic
pain (SOR B). While NSAIDs do not have the analgesic potency of narcotics, they do not
cause respiratory suppression or change the patient’s sensorium. Corticosteroids should be
reserved for patients who cannot take NSAIDs.

ABFM 2016
23. A 6-month-old male is brought to the urgent care center with a 3-day history of rhinorrhea,
cough, and increased respiratory effort. His temperature is 37.5°C (99.5°F), his heart rate is
120 beats/min, his respiratory rate is 42/min, and his oxygen saturation is 96% on room air.
On examination the child appears well hydrated with clear secretions from his nasal passages,
there is diffuse wheezing heard bilaterally, and there is no nasal flaring or retractions. The
mother states that the child has a decreased appetite but is drinking a normal amount of fluids.
Which one of the following would be the most appropriate management for this patient?
A. Supportive therapy only
B. Bronchodilators
C. A corticosteroid taper
D. Epinephrine
E. Nebulized hypertonic saline

ANSWER: A
This patient’s symptoms and the examination suggest viral bronchiolitis. Supportive therapy,
including adequate hydration, is recommended for treatment. Treatment with bronchodilators,
epinephrine, hypertonic saline, or corticosteroids is not indicated (SOR A)

Vaginal discharge+ Abnormal Vaginal Bleeding:


ABFP 2020
1- A 24-year-old female presents with a 2-day history of mild to moderate pelvic pain. She has
had two male sex partners in the last 6 months and uses oral contraceptives and sometimes
condoms. A physical examination reveals a temperature of 36.4°C (97.5°F) and moderate
cervical motion and uterine tenderness. Urine hCG and a urinalysis are negative. Vaginal
microscopy shows only WBCs.
The initiation of antibiotics for treatment of pelvic inflammatory disease in this patient
A. Is appropriate at this time
B. Requires an elevated temperature, WBC count, or C-reactive protein level
C. Should be based on the results of gonorrhea and Chlamydia testing
D. Should be based on the results of pelvic ultrasonography

ANSWER: A
Pelvic inflammatory disease (PID) is a clinical diagnosis, and treatment should be
administered at the time of diagnosis and not delayed until the results of the nucleic acid
amplification testing (NAAT) for gonorrhea and Chlamydia are returned. The clinical
diagnosis is based on an at-risk woman presenting with lower abdominal or pelvic pain,
accompanied by cervical motion, uterine, or adnexal tenderness that can range from mild to
severe. There is often a mucopurulent discharge or WBCs on saline microscopy. Acute phase
indicators such as fever, leukocytosis, or an elevated C-reactive protein level may be helpful
but are neither sensitive nor specific. A positive NAAT is not required for diagnosis and
treatment because an upper tract infection may be present, or the causative agent may not be
gonorrhea or Chlamydia. PID should be considered a polymicrobial infection. Pelvic
ultrasonography may be used if there is a concern about other pathology such as a tubo-
ovarian abscess.
ABFP 2018
A 30-year-old female with anovulatory uterine bleeding asks about treatment options. An
examination is normal and blood testing is negative. She is unmarried and is undecided about
having children. Which one of the following would be the most appropriate treatment for this
patient?
A) Oral progestin during the luteal phase
B) A levonorgestrel-releasing IUD
C) Endometrial ablation
D) Hysterectomy

ANSWER: B

Few treatments for dysfunctional uterine bleeding have been studied. NSAIDs, oral
contraceptive pills, and danazol have not been shown to have sufficient evidence of effect for
the treatment of dysfunctional uterine bleeding. Progestin is effective when used on a 21-day
cycle, but not if used only during the luteal phase. Hysterectomy and ablation are very
effective, but both eliminate fertility. In a young woman unsure about having children, the
levonorgestrel-releasing IUD is the most effective treatment that preserves fertility (SOR A).

A 75-year-old female sees you because of a bulge at the vaginal opening. A pelvic
examination confirms descent of the vaginal wall to just beyond the hymen. This protrusion is
bothering her and interfering with her quality of life. She has had two vaginal deliveries. She
is sexually active and has not had any pelvic surgery.
Which one of the following would be the most appropriate initial treatment for this problem?
A) Kegel exercises
B) A ring pessary
C) A space-occupying pessary
D) Hysteropexy
E) Hysterectomy

ANSWER: B
Pessaries are considered first-line treatment for pelvic organ prolapse (SOR C). Ring pessaries
provide support and are the initial choice in most circumstances. Sexual intercourse can still
occur with a ring pessary, which can be inserted and removed by the patient. Space-occupying
pessaries are associated with more vaginal discharge and irritation and do not allow for sexual
intercourse. While they can improve stress and urge urinary incontinence, Kegel exercises do
not treat pelvic organ prolapse. Surgery, including hysterectomy or hysteropexy that conserves
the uterus, can be considered after first-line treatment with a pessary.

ABFP 2017
178. A 47-year-old female is concerned about a change in her menstrual pattern. Her monthly
periods continue, but for the past several months they have been heavier than usual and have
been lasting a few days longer. Last month she also noted some spotting for several days prior
to the onset of her menses. Her pelvic examination is normal.
Which one of the following would be most appropriate at this time?
A) Observation only, and reexamination in 3 months
B) A serum FSH level
C) Transvaginal ultrasonography
D) Progestin-only therapy to normalize bleeding
E) Cyclic estrogen-progestin therapy to normalize bleeding

Answer : C
Abnormal uterine bleeding can be a sign of endometrial cancer in premenopausal women, who
account for 20% of cases of endometrial cancer. The American College of Obstetricians and
Gynecologists recommends that women with abnormal uterine bleeding should be evaluated
for endometrial cancer if they are older than 45 years or if they have a history of unopposed
estrogen exposure (SOR C). Most guidelines recommend either transvaginal ultrasonography
or endometrial biopsy as the initial study in the evaluation of endometrial cancer. Transvaginal
ultrasonography is often preferred as the initial study because of its availability, cost-
effectiveness, and high sensitivity. If bleeding persists despite normal transvaginal
ultrasonography a tissue biopsy should be performed. The listed hormonal treatment options
may be appropriate once cancer is ruled out. An FSH level can help determine whether
someone is menopausal or approaching menopause, in which case they will likely be missing
periods. Continued observation would only delay the diagnosis.

ABFP 2016
47. A 35-year-old female has had a foul-smelling vaginal discharge for the past week since
having sexual intercourse with a new male partner. It does not seem thicker than her usual
discharge. She does not have any itching or pain with intercourse.
Which one of the following are you most likely to find on examination?
A) Evidence of excoriation of the vulva
B) Vulvar erythema
C) Vaginal atrophy
D) A vaginal pH of 4.0–4.5
E) A vaginal pH >4.5
ANSWER: E
This patient is more likely to have bacterial vaginosis than other causes of vaginitis, based on
the presence of a foul-smelling discharge in the absence of pruritus or dyspareunia. The odor
of bacterial vaginitis is more likely to appear in the presence of semen because of the increase
in vaginal alkalinity. Bacterial vaginosis is associated with a vaginal pH >4.5. Vulvovaginal
candidiasis is associated with vulvar erythema, excoriation of the vulva, and a normal vaginal
pH (4.0–4.5). Vaginal atrophy is seen in women with estrogen deficiency, and atrophic
vaginitis is unlikely in this patient due to her age and lack of pruritus and dyspareunia.

Menstrual cycle + Menstrual abnormalities


ABFP 2020
13years old female is brought to your office by her adoptive mother. Theydonot know the
patient’s biological family history. They are concerned because, unlike all of her friends, she
has not yet started to menstruate. Breast development began 2 years ago. On examination her
breasts show a secondary mound from the areola and nipple above the contour of her breast.
She has dark, coarse hair covering the mons pubis consistent with a stage 4 sexual maturity
rating. If her sexual development continues to be normal, at what age should you recommend
evaluation for primary amenorrhea?
A. 13
B. 14
C. 15
D. 16
E. 17
ANSWER: C
Delayed puberty in girls is defined as the absence of breast development by age 13. Typically,
menarche starts 2.5 years after the onset of breast development, with an average age of 12.5
years (normal range 9–15 years). In girls with otherwise normal sexual development, the
absence of menarche by 15 years of age should prompt an evaluation for primary amenorrhea

ABFP 2018
A 35-year-old female presents with fatigue. She has been falling asleep at work for the past 6
weeks. She is married with two children and works as a nurse at the community hospital.
Since she returned to work 12 weeks ago after maternity leave, her infant has had multiple
respiratory infections and has not slept well through the night. Her menstrual cycle has been
irregular and heavy for the past several months. A CBC and TSH level are normal.
Which one of the following laboratory tests would be appropriate at this visit?
A) 25-Hydroxyvitamin D
B) -hCG
C) D-dimer
D) A serum antibody test for Lyme disease

ANSWER: B
25-Hydroxyvitamin D levels should not be measured in patients presenting with fatigue (SOR
A). A serum antibody test for Lyme disease or a D-dimer would not be indicated for this
patient based on her history and symptoms. Because of the patient’s history of irregular
menses, a -hCG level would be indicated.

A 25-year-old female sees you because of irregular menses, hirsutism, and moderate acne. She
is sexually active in a monogamous relationship with a male, has never been pregnant, and
prefers not to become pregnant at this time. ?Which one of the following is considered first-
line therapy
A) Clomiphene (Clomid )
B) Letrozole (Femara)
C) Levonorgestrel/ethinyl estradiol
D) Metformin (Glucophage )
E) Spironolactone (Aldactone )

ANSWER: C
The Endocrine Society recommends hormonal contraception as the first-line medication for
women diagnosed with polycystic ovary syndrome (PCOS) who are experiencing irregular
menses, acne, and hirsutism and do not desire pregnancy (SOR A). Metformin may help
regulate menses but has not been shown to be as effective as oral hormone therapy. In a 2015
Cochrane review, oral contraceptives were recommended as the most effective treatment for
hirsutism. Either letrozole or clomiphene is appropriate for women diagnosed with PCOS who
want to become pregnant.

A 17-year-old female comes to your office with an 8-month history of amenorrhea.


Menarche occurred at age 12 and her menses were regular until the past year. On examination
the patient’s vital signs are in the normal range for her age but she has a BMI of 16.1 kg/m2,
which is below the third percentile for her age. She is a high school senior who dances with
the local ballet company. She practices dance several hours a day and works out regularly. She
tells you that she follows a strict 800-calorie/day diet to keep in shape for ballet. You order a
CBC, a comprehensive metabolic panel, a urine -hCG level, FSH and LH levels, and a TSH
level. Which one of the following is also recommended as part of the workup?
A) An EKG
B) Pelvic ultrasonography
C) Abdominal/pelvic CT
D) A DXA scan
E) A nuclear bone scan

ANSWER: D
Relative energy deficiency in sport (RED-S), formerly known as the female athlete triad, is a
relatively common condition in female athletes, and is characterized by amenorrhea,
disordered eating, and osteoporosis. It is more common in sports that promote lean body mass.
Female athletes should be screened for the disorder during their preparticipation evaluations.

Individuals who present with one or more components of RED-S should be evaluated for the
other components. This patient has a low BMI for her age, which indicates an eating disorder,
and secondary amenorrhea, and should be screened for osteoporosis using a DXA scan. The
International Society for Clinical Densitometry recommends using the Z-score, rather than the
T-score, when screening children or premenopausal women. The T-score is based on a
comparison to a young adult at peak bone density, whereas the Z-score uses a comparison to
persons of the same age as the patient. A Z-score less than –2.0 indicates osteoporosis. The
American College of Sports Medicine defines low bone density as a Z score of –1.0 to –2.0.
An EKG is not required in this patient since she has normal vital signs. Pelvic ultrasonography
is not necessary unless an abnormal finding is identified on a pelvic examination.
Abdominopelvic CT would be inappropriate given the patient’s age and lack of
abdominopelvic symptoms such as pain or a mass. A nuclear bone scan likewise is not
recommended, as it is not used to diagnose osteoporosis (SOR C).

A 33-year-old gravida 2 para 2 presents with a 1-year history of amenorrhea, hot flashes, and
vaginal dryness. She previously had normal menses and takes no medications. Her past
medical and surgical histories are negative. The patient is 178 cm (70 in) tall and her BMI is
22.0 kg/m2. Her vital signs are normal. A physical examination is normal except for vaginal
dryness. Laboratory studies reveal a negative urine pregnancy test, normal TSH and prolactin
levels, and elevated LH and FSH levels.
The most likely diagnosis is
A) intrauterine synechiae (Asherman syndrome)
B) functional hypothalamic amenorrhea
C) polycystic ovary syndrome
D) primary ovarian insufficiency
E) Turner’s syndrome

ANSWER: D
This patient presents with secondary amenorrhea. The differential diagnosis includes
polycystic ovary syndrome (PCOS), intrauterine synechiae (Asherman syndrome), functional
hypothalamic amenorrhea, hypothyroidism, hyperprolactinemia, and primary ovarian
insufficiency (also known as premature ovarian failure). This patient’s presentation and the
laboratory findings are most consistent with a diagnosis of primary ovarian insufficiency. This
is defined as menopause before the age of 40 due to ovarian folliculardepletion. Laboratory
findings will usually reveal a low serum estradiol and elevated FSH and LH levels.
This condition is different than menopause because of the age of presentation and the
unpredictability of long-term ovarian function (up to 10% of cases spontaneously remit and
patients have a temporary returnof fertility).

Patients with PCOS typically present with obesity, difficulty conceiving, and normal or low
FSH and LH levels. This patient’s normal weight and prior history of normal menses make
this diagnosis less likely.
Intrauterine synechiae is characterized by scar tissue inside the uterus. Risk factors include
intrauterine procedures, pregnancy, inflammation, and infection. Patients present with
abnormal uterine bleeding, recurrent pregnancy loss, dysmenorrhea, and infertility. FSH and
LH levels are usually normal.
Functional hypothalamic amenorrhea is characterized by suppression of the hypothalamic-
pituitary-ovarian axis, usually due to extreme stress, excessive exercise, marked weight loss,
and/or dysfunctional eating.
LH and FSH levels are usually low or low-normal. Turner’s syndrome is caused by the 45,X
genotype, and patients have short stature, a webbed neck, a low hairline, and cardiac
abnormalities. This is unlikely in a patient who is 178 cm (70 in) tall and has a normal
examination.

ABFP 2017
83-A 13-year-old female sees you for the first time. Her mother has brought her in because the
patient has never menstruated. She feels well and has no other health concerns. Her medical
history is unremarkable, and she takes no medications.
A physical examination is significant for the absence of both breast development and pubic
hair. Her height is below the 2nd percentile and her weight is at the 6th percentile for her age.
-hCG, prolactin, and TSH levels are all normal. LH and FSH levels are elevated.
Which one of the following would you order at this point?
A) A trial of combined oral contraceptive pills
B) A serum testosterone level
C) Pelvic ultrasonography
D) Karyotyping
E) MRI of the brain and brainstem
ANSWER: D
Brain MRI would be useful in ruling out brain and pituitary tumors, but this patient has no
brain tumor symptoms and normal prolactin and TSH levels. A trial of combined oral
contraceptive pills may precipitate withdrawal bleeding, but this has poor sensitivity as a
marker of ovarian function (SOR C) and will not lead to a diagnosis.
There are no signs or symptoms of hyperandrogenism, so a serum testosterone level would not
be helpful. Pelvic ultrasonography to confirm the presence of a uterus is not indicated by the
physical findings at this point.
Short stature and the lack of pubertal changes are characteristics of primary ovarian failure
and Turner syndrome. A karyotype to rule out Turner syndrome and the presence of Y
chromosome material is indicated at this point.

ABFP 2016
126. A 25-year-old female presents to your office to discuss abnormal menstrual periods. She
says that her cycles have always been irregular but she has not had any bleeding in 3 months.
She also says she has gained 20 lb over the past 6 months. She is not taking any medications.
You perform an examination and order laboratory tests. Her blood pressure is 110/72 mm Hg,
heart rate 84 beats/min, respiratory rate 12/min, and weight 78.0 kg (172 lb) with a BMI of
29.5 kg/m2. She is noted to have moderate cystic acne. Her examination is otherwise
unremarkable. A serum hCG measurement is negative and TSH, FSH, and LH levels are
normal. Follow-up laboratory evaluation is significant for a total testosterone level 3 times the
upper limit of normal and a normal 17-hydroxyprogesterone level.
Which one of the following would be most appropriate at this point?
A) An estradiol level
B) A dexamethasone suppression test
C) Karyotyping
D) CT of the abdomen and pelvis
E) MRI of the brain
ANSWER: D
This patient presents with mild symptoms of hyperandrogenism. Her initial laboratory results
rule out pregnancy, thyroid disorders, and primary ovarian failure. The follow-up laboratory
evaluation indicates significant hyperandrogenism. The rapid onset and high testosterone level
suggest an ovarian or adrenal tumor that should be evaluated by abdominal/pelvic imaging.
MRI of the brain is not helpful in evaluating hyperandrogenism. It would be appropriate in
evaluating hypothalamic and pituitary causes of secondary amenorrhea such as the female
athlete triad or other causes of stress and malnutrition that are associated with weight loss. A
karyotype would be helpful in identifying the cause of primary amenorrhea. An estradiol level
is not helpful in evaluating hyperandrogenism. A dexamethasone suppression test aids in the
diagnosis of Cushing syndrome, which generally does not cause amenorrhea and is usually
associated with stigmata of hypercortisolism, which this patient does not have.

60. A 19-year-old female sees you for evaluation of severe dysmenorrhea. She tells you she is
not sexually active and has never had intercourse. A physical examination is unremarkable
and you determine that a pelvic examination is not necessary.
Which one of the following is the treatment of choice for this patient?
A) Acetaminophen
B) Hydrocodone
C) Medroxyprogesterone acetate (Depo-Provera)
D) An NSAID
E) An oral contraceptive
ANSWER: D

NSAIDs should be used as first-line treatment for primary dysmenorrhea (SOR A). A
Cochrane review that included 73 randomized, controlled trials demonstrated strong evidence
to support NSAIDs as the first-line treatment for primary dysmenorrhea. Since no NSAID has
been proven more effective than others, the choice of NSAID should be based on effectiveness
and tolerability for each patient. The medication should be taken 1–2 days before the expected
onset of the menstrual period and continued on a fixed schedule for 2–3 days. Oral
contraceptives may be effective for relieving symptoms of primary dysmenorrhea but the
evidence is limited. Hydrocodone, acetaminophen, and medroxyprogesterone acetate are not
appropriate choices.
Autism:
ABFM 2020
A 37-year-old male sees you to discuss some mental health concerns. He states that he has
recently been unable to meet deadlines at work and has been reprimanded by his boss. He
feels he cannot complete the tasks during his work hours and is worried he is going to lose his
job. Your evaluation indicates that he might have adult attention-deficit/hyperactivity disorder
(ADHD).
Which one of the following is true regarding adult ADHD?
A. Symptoms of ADHD must be present before age 6
B. Inattention does not persist into adulthood
C. Hyperactivity symptoms worsen into adulthood
D. Stimulants should not be prescribed in adults over the age of 30
E. Adults with ADHD should be screened for co-existing psychiatric
disorders
ANSWER: E
Attention deficit/hyperactivity disorder (ADHD) is a common neuro-developmental disorder
in children and adolescents. Approximately 30% of children carry the diagnosis into
adulthood. Individuals with ADHD should be screened for coexisting psychiatric disorders
because they are at a higher risk for problems such as anxiety and depression and are more
likely than the general population to have substance use disorders. The DSM-5 criteria for the
diagnosis of adult ADHD require symptoms to have been present before age 12. Inattention
often persists into adulthood, while hyperactivity and impulsivity usually improve with
time. Compared to children with ADHD, hyperactivity in adults often presents as
talkativeness, irritability, and restlessness. Along with behavioral strategies, stimulants have
been found to be effective for treatment of adult ADHD. Although there are no age restrictions
for the use of stimulants, adults should have their blood pressure and heart rate monitored
during use to monitor for potential cardiac complications.
ABFM 2018
Intensive behavioral intervention has more benefit than other treatment modalities in treating
children who have been diagnosed with
A) attention-deficit/hyperactivity disorder
B) autism
C) depression
D) obsessive-compulsive disorder
E) posttraumatic stress disorder

ANSWER: B
The only evidence-based treatment that confers significant benefits to children with autism is
intensive behavioral interventions, which should be initiated before 3 years of age. Attention-
deficit/hyperactivity disorder can be treated with cognitive-behavioral therapy (CBT) but
medication is often required. CBT is as effective, if not more effective, than medication for
treating anxiety, depression, and trauma-related disorders.

ABFM 2017
91- A 36-month-old male has persistent deficits in social communication and interaction
across multiple contexts. He displays restricted and repetitive patterns of behavior, interest,
and activities. According to the DSM-5, which one of the following is the most appropriate
diagnosis?
A) Asperger syndrome
B) Autism spectrum disorder
C) Autistic disorder
D) Childhood disintegrative disorder
E) Pervasive developmental disorder not otherwise specified
ANSWER: B
Key diagnostic features of autism spectrum disorder include deficits in social communication
and interaction across multiple contexts and restricted, repetitive patterns of behavior,
interests, or activities. The DSM-5, which came out in 2013, created an umbrella diagnosis of
autism spectrum disorder to consolidate four previously separate disorders: autistic disorder,
Asperger’s disorder, childhood disintegrative disorder, and pervasive developmental disorder
not otherwise specified. Any individuals with a previous diagnosis of one of these disorders
should now be given a diagnosis of autism spectrum disorder.

ABFM 2016
40. The American Academy of Pediatrics recommends administration of an autism-specific
screening tool at which well child visits?
A) 18 and 24 months
B) 2 and 3 years
C) 3 and 5 years
D) 5 and 10 years
E) 10 and 12 years
ANSWER: A
Since early intensive behavioral therapy can improve functional outcomes for children with
autism and autism spectrum disorders, early diagnosis is critical. The American Academy of
Pediatrics recommends screening all children with an autism-specific screening instrument at
18 and 24 months in addition to surveillance of developmental issues at all well child visits.

Abdominal Pain Approach


2019 pretest
A 38-year-old female has a 4-year history of intermittent abdominal pain occurring several
days per week. She went to an urgent care clinic over the weekend and has brought her
laboratory results for you to review. A comprehensive metabolic panel, CBC, and TSH level
are all normal. She states that her stools have become harder and difficult to pass, with
frequent straining and pain. She reports now having a bowel movement only 2–3 times a
week. She tells you that she tried psyllium but it resulted in abdominal bloating. She is
concerned because she has been missing work because of her symptoms.
Her past and family medical histories are otherwise negative. Her vital signs are normal and
her weight is unchanged from a previous wellness visit 6 months ago. An abdominal
examination is notable for generalized pain with deep palpation. She does not have any
masses, hepatosplenomegaly, or rebound tenderness. A rectal examination is also normal.
Which one of the following is the most likely diagnosis?
A. Celiac disease
B. Colon cancer
C. Hyperparathyroidism
D. Inflammatory bowel disease
E. Irritable bowel syndrome

ANSWER: E
The Rome IV criteria for irritable bowel syndrome are symptom-based diagnostic criteria used
to identify patients with irritable bowel syndrome (IBS). According to these criteria, IBS is
defined as recurrent abdominal pain at least 1 day per week for at least 3 months. At least two
of the following must be present: abdominal pain related to defecation, a change in stool form,
and/or a change in stool frequency. The diagnosis can be made from the patient history, a
physical examination, and a minimal laboratory evaluation. This patient meets the criteria.
This patient does not have any “alarm” symptoms (a positive family history of colorectal
cancer, rectal bleeding in the absence of documented bleeding hemorrhoids or anal fissures,
unintentional weight loss, or anemia) so colon cancer would be unlikely. Celiac disease and
inflammatory bowel disease are unlikely in a patient with constipation-predominant IBS.
Serologic and inflammatory marker testing would be warranted in patients with predominant
diarrhea or mixed types of IBS. Hyperparathyroidism does not usually present with this
clinical picture, and calcium levels would likely be elevated.

ABFM 2020
A 60-year-old male presents with left lower quadrant abdominal pain. His medical and
surgical histories are remarkable only for a history of hypertension controlled with
hydrochlorothiazide and lisinopril (Prinivil, Zestril), and no polyps seen on screening
colonoscopy 5 years ago. He is afebrile, and a physical examination is notable only for mild
abdominal tenderness in the left lower quadrant without peritoneal signs.A urinalysis is
normal. You diagnose mild diverticulitis.
Which one of the following management options would be indicated at this time?
1. Rest and clear liquids
2. Avoidance of seeds, nuts, and popcorn
3. Abdominal CT
4. Referral for colonoscopy
5. Hospital admission for intravenous fluids and intravenous antibiotics
ANSWER: A
In patients with mild diverticulitis, outpatient management with rest and oral fluids is
preferred. Avoidance of seeds, nuts, and popcorn does not reduce recurrence rates. CT of the
abdomen may be indicated if the diagnosis is uncertain or if complications are suspected.
Colonoscopy is contraindicated acutely and is only necessary for follow-up when age-
appropriate cancer screening is indicated, or in cases of complicated disease. Antibiotics may
not be necessary in all cases, and hospital admission is unnecessary for mild cases.

94. A 17-year-old male presents to your office to be screened for celiac disease. His older
sister was recently diagnosed with biopsy-confirmed celiac disease. He has had intermittent
abdominal bloating but no other symptoms.

Which one of the following is the current best practice to screen for celiac disease in this
situation?
A. No screening and treatment based on family history
B. Antigliadin antibodies
C. Endomysial antibodies
D. Serum total IgA levels and IgA tTG antibodies
E. Esophagogastroduodenoscopy with small bowel biopsies

ANSWER: D
Celiac disease affects about 10% of first degree relatives of a person with celiac disease and a
heightened suspicion should be maintained in this higher-than-average risk group. Many
patients are asymptomatic or only minimally symptomatic.

Several serologic tests are available for initial screening, and should be followed by a small
bowel biopsy for confirmation. IgA tTG antibody testing is currently the test of choice and
should be paired with serum total IgA levels, as IgA deficiency is 10–15 times more common
in patients with celiac disease than in the general population.
Because only up to 10% of first-degree relatives are affected, committing this patient to a
lifelong dietary restriction without confirmatory testing may be unnecessary. Antigliadin
antibody testing has low sensitivity and specificity. Endomysial antibody testing has higher
specificity and sensitivity, but it is very costly. An initial esophagogastroduodenoscopy is
likely too invasive and expensive for screening purposes in this patient.

A 65-year-old female presents to the emergency department with a 24-hour history of


abdominal pain, nausea, and vomiting. She reports that her last bowel movement was 2 days
ago and she has not passed any flatus. The pain is diffuse and she rates it as 4 on a scale of 10
but says that it reaches a level of 8 at times. She has hypertension, which has been controlled
with amlodipine (Norvasc), 5 mg daily. She had an appendectomy at age 25 and had two
normal vaginal deliveries. She has been postmenopausal since age 52. An examination reveals
a blood pressure of 120/80 mm Hg, a pulse rate of 110 beats/min, a respiratory rate of 16/min
and nonlabored, and an oxygen saturation of 95% on room air. Examination of the heart and
lungs is normal. On examination of the abdomen you note high-pitched bowel sounds, and the
abdomen is distended and tympanic, and diffusely tender with no appreciable masses. A
radiograph of the abdomen shows a small bowel obstruction.
Which one of the following is the most likely cause of this patient’s small bowel obstruction?
A. Constipation
B. Intestinal adhesions
C. A neoplasm
D. Ventral herniation
E. Volvulus
ANSWER: B
Intestinal adhesions are the most common cause of small bowel obstruction, accounting for
60%–75% of cases. Less common causes include neoplasms in 13%–20% of cases, herniation
in 2%–15% of cases, and volvulus in <5% of cases. Severe constipation is a rare cause of
small bowel obstruction. Further evaluation with CT of the abdomen and pelvis would be
indicated to rule out ischemia or perforation and to determine the etiology of this patient’s
small bowel obstruction. Initial management includes nasogastric tube decompression and
intravenous fluid resuscitation. Surgical consultation is recommended. Immediate surgery is
indicated for unstable patients, a closed loop obstruction such as volvulus that cannot be
reduced, intestinal ischemia, or perforation. Most cases resolve with conservative
management, but surgery is recommended if the obstruction has not resolved after 3–5 days.

ABFM 2018
An 8-year-old male is brought to your office because of acute lower abdominal pain. He is not
constipated and has never had abdominal surgery. You suspect acute appendicitis. Which one
of the following would be most appropriate at this point?
A) Plain radiography
B) Ultrasonography
C) CT without contrast
D) CT with contrast
E) MRI

ANSWER: B
Ultrasonography is recommended as the first imaging modality to evaluate acute abdominal
pain in children. It avoids radiation exposure and is useful for detecting many causes of
abdominal pain, including appendicitis. After ultrasonography, CT or MRI can be used if
necessary to diagnose appendicitis. Abdominal radiography is helpful in patients with
constipation, possible bowel obstruction, or a history of previous abdominal surgery. The
American Academy of Pediatrics Choosing Wisely recommendation on the evaluation of
abdominal pain states that CT is not always necessary. The American College of Surgeons
Choosing Wisely recommendation on the evaluation of suspected appendicitis in children says
that CT should be avoided until after ultrasonography has been considered as an option.

You see a 3-year-old female with a 2-day history of intermittent abdominal cramps, two
episodes of emesis yesterday, and about five watery, nonbloody stools each day. She does not
have a fever, her other vital signs are normal, and she has not traveled recently. Today she has
tolerated sips of fluid but still has mild fatigue and thirst. An examination is normal except for
mildly dry lips. A friend at preschool had a similar illness recently. Which one of the
following would be the most appropriate initial management of this
patient?
A) A sports drink and food on demand
B) Half-strength apple juice and food on demand
C) Ginger ale and no food yet
D) Water and no food yet
E) A bolus of intravenous normal saline and no food yet

ANSWER: B
Family physicians often see patients with diarrheal illnesses and most of these are viral.
Patients sometimes have misconceptions about preferred fluid and feeding recommendations
during these illnesses. The World Health Organization recommends oral rehydration with low
osmolarity drinks (oral rehydration solution) and early refeeding.

Half-strength apple juice has been shown to be effective, and it approximates an oral
rehydration solution. Its use prevents patient measurement errors and the purchase of
beverages with an inappropriate osmolarity. Low osmolarity solutions contain glucose and
water, which decrease stool frequency, emesis, and the need for intravenous fluids compared
to higher osmolarity solutions like soda and most sports drinks. Water increases the risk of
hyponatremia in children. This patient is not ill enough to need intravenous fluids. Early
refeeding has been shown to decrease the duration of illness.

A 30-year-old male presents with intermittent right upper quadrant pain after meals. He has
been in moderate pain for the past 3 hours. On examination the patient’s vital signs are normal
except for a temperature of 39.2°C (102.6°F). He appears toxic. Examination of the abdomen
reveals a positive Murphy’s sign.
Laboratory Findings
WBCs ----------------------------- 3000/mm3 (N 4300–10,800)
ALT (SGPT) --------------------- 132 U/L (N 10–55)
AST (SGOT) --------------------- 123 U/L (N 9–25)
Alkaline phosphatase -----------200 U/L (N 45–115)
Bilirubin ----------------------------- 2.6 mg/dL (N 0.0–1.0)
Lipase--------------------------------15 U/dL (N 3–19)
Ultrasonography reveals cholelithiasis. Other findings include an enlarged gallbladder,
thickening of the gallbladder wall, and a common bile duct diameter of 11 mm.
Which one of the following is the most likely cause of this patient’s symptoms?
A) Acute cholangitis
B) Acute viral hepatitis
C) Cholangiocarcinoma
D) Gallstone pancreatitis

ANSWER: A
Gallstones are often asymptomatic and found incidentally on imaging. However, they may
become symptomatic, which usually causes pain in the right upper quadrant or epigastrium.
Most patients with symptomatic gallstones present with chronic cholecystitis, which causes
recurrent attacks of pain. The pain is constant, increases in severity at the beginning, and lasts
from 1 to 5 hours. It often starts during the night after a fatty meal and may be associated with
nausea and vomiting. Abdominal ultrasonography is the initial imaging method.
The two main complications of choledochal stones are cholangitis and pancreatitis. Acute
cholangitis is a bacterial infection. Bacterial growth is enhanced by obstruction of the duct. It
may present as a mild self-limited disease but can also lead to sepsis. Cases typically present
with fever, pain, and jaundice.

Laboratory findings include an elevated WBC count and elevated bilirubin, transaminases, and
alkaline phosphatase. Ultrasonography will show a dilated bile duct in many cases, although it
might not be dilated in acute obstruction.
Patients with pancreatitis present with pain, nausea, and vomiting. The pain is usually
epigastric and radiates to the back. It reaches its maximum intensity within 1 hour and may
last for days. The physical examination may reveal tachycardia, hypotension, tachypnea, and
fever. The abdomen may be distended and is typically tender to palpation. The diagnosis
requires two of three primary features: abdominal pain, elevation of serum amylase or lipase,
and findings on imaging studies that are consistent with the diagnosis.
Ultrasonography can show pancreatic enlargement or edema, and visualization of gallstones
will suggest choledocholithiasis as the cause of the pancreatitis.

A 34-year-old female sees you because of cramping diarrhea for the past several months
following resection of her terminal ileum as treatment for Crohn’s disease. She is not aware of
any exposure to individuals with similar symptoms. She has not had any fever, chills, nausea,
vomiting, or myalgias, and she has not noticed any blood in her stool. She is passing several
loose stools daily, mostly after meals. She has not been able to identify any clear relationship
to the type of food she eats.
Which one of the following would be the best initial treatment option for this patient?
A) A dairy-free diet
B) Cholestyramine (Questran) daily
C) A Lactobacillus probiotic supplement (Lactinex) 4 times daily
D) Loperamide (Imodium) as needed
E) Psyllium fiber (Metamucil) twice daily

ANSWER: B
Diarrhea that develops in patients with ileal Crohn’s disease or following ileal resection is
usually due to increased amounts of bile acid remaining in the stool. This affects colonic
secretion and motility and various protein factors in the gut, resulting in the development of
bile acid diarrhea (BAD). Although various tests can be performed to evaluate the stool, gut
flora, and bowel function, a therapeutic trial with a bile acid sequestrant such as
cholestyramine is most often used for both the diagnosis and treatment of BAD. Reducing fat
intake may also be beneficial. Loperamide can lessen the diarrhea in some patients but should
not be the primary treatment because chronic use can result in constipation. Fiber
supplementation may help to produce a more formed stool and could be used as an adjunct
treatment when appropriate.

A 5-year-old male is brought to your office after passing an intestinal worm. He lives on a
farm with cattle, pigs, and dogs. He has never traveled very far from home. He does not have
any respiratory symptoms or diarrhea, but has experienced some abdominal bloating. A
picture of the worm taken by his mother is shown below.
Which one of the following is the infecting organism?
A) Ascaris lumbricoides (roundworm)
B) Enterobius vermicularis (pinworm)
C) Giardia lamblia
D) Necator americanus (hookworm)
E) Taenia solium (tapeworm)

ANSWER: A
This case and image are consistent with Ascaris lumbricoides infestation. A. lumbricoides is a
large roundworm that typically infects the ileum. Symptoms are variable but large infections
can lead to intestinal obstruction. Pinworms are much smaller and typically present with anal
pruritus. Tapeworms can be large, but are flat and segmental in appearance, and are typically
found in the stool as segments called proglottids. Hookworms are also round, but are typically
6–12 mm in length. They are a significant cause of anemia in children globally. Giardia
lamblia is a microscopic protozoan parasite that is not visible on gross examination.

ABFM 2016
4. An 82-year-old male nursing home resident is sent to the emergency department with
crampy lower abdominal pain and bloody diarrhea. On examination he has mild abdominal
tenderness and slightly decreased bowel sounds. He has a previous history of vascular
dementia, peripheral artery disease, hypertension, and hyperlipidemia. On examination he is
afebrile, and a nasogastric aspirate is negative for evidence of bleeding.
Which one of the following is the most likely cause of this patient’s bleeding?
A) Peptic ulcer disease
B) Ischemic colitis
C) Diverticular bleeding
D) Angiodysplasia
E) Infectious colitis

ANSWER B
This patient most likely has ischemic colitis, given his abdominal pain, bloody diarrhea, and
cardiovascular risk factors. Peptic ulcer disease is unlikely because the nasogastric aspirate
was negative. Diverticular bleeding and angiodysplasia are painless. Infectious colitis is
associated with fever.

35. A 40-year-old male presents to the emergency department with a fever and acute, severe
abdominal pain. A complete history and careful physical examination fail to localize the
source of his pain, and basic blood and urine tests also prove to be nondiagnostic. His pain is
diffuse, and you are concerned about a potentially serious pathology.
Which one of the following would be most appropriate to order next?
A) Plain radiographs of the abdomen
B) An upper gastrointestinal series with small-bowel follow-through
C) Ultrasonography of the abdomen
D) CT of the abdomen and pelvis
E) MRI of the abdomen
ANSWER: D
The differential diagnosis of acute nonlocalized abdominal pain is broad. CT is typically the
imaging modality of choice if there is significant concern about serious pathology or if the
diagnosis is unclear from the history, physical examination, and laboratory testing. In this
instance, the American College of Radiology recommends CT of the abdomen and pelvis,
preferably with contrast. One prospective study of patients with nontraumatic abdominal pain
in an emergency department setting found that CT results changed the leading diagnosis in
49% of patients and the management plan in 42% of patients.
Irritable bowel disease
ABFM 2020
A 25-year-old male presents with a 4-month history of crampy abdominal pain, diarrhea, and
fatigue. His symptoms began gradually but have become more severe and he is now
experiencing rectal bleeding. He says that his abdominal pain seems to temporarily improve
after eating. He has smoked five cigarettes per day for the past 8 years. He is surprised to learn
that he has lost 7 kg (15 lb) when he is weighed today.
His vital signs include a blood pressure of 116/70 mm Hg, a heart rate of 76 beats/min, a
respiratory rate of 12/min, and a temperature of 37.7°C (99.9°F). A physical examination
reveals abdominal tenderness and mild distention. An anorectal examination is significant for
a perianal fistula. A laboratory evaluation is notable for mild anemia. His kidney and liver
function are normal.
Which one of the following is the most likely diagnosis?
A. Celiac disease
B. Chronic pancreatitis
C. Crohn’s disease
D. Irritable bowel syndrome
E. Ulcerative colitis

ANSWER: C
Crohn’s disease may present insidiously with diarrhea, abdominal pain, rectal bleeding, fever,
weight loss, and fatigue. Red-flag symptoms include perianal lesions, a first degree relative
with inflammatory bowel disease, weight loss of 5% of the patient’s usual weight, abdominal
pain for more than 3 months, nocturnal diarrhea, fever, the absence of abdominal pain for 30–
45 minutes after eating, and the absence of rectal urgency. This patient exhibits symptoms
consistent with Crohn’s disease. While anemia is also common in celiac disease, rectal
bleeding is not. Chronic pancreatitis does not generally present with improved pain after
eating. Irritable bowel syndrome is not associated with fever, rectal bleeding, anemia, or
perianal fistulas. Ulcerative colitis is not associated with perianal lesions.

ABFM 2018
A 34-year-old female sees you because of cramping diarrhea for the past several months
following resection of her terminal ileum as treatment for Crohn’s disease. She is not aware of
any exposure to individuals with similar symptoms. She has not had any fever, chills, nausea,
vomiting, or myalgias, and she has not noticed any blood in her stool. She is passing several
loose stools daily, mostly after meals. She has not been able to identify any clear relationship
to the type of food she eats.
Which one of the following would be the best initial treatment option for this patient?

A) A dairy-free diet
B) Cholestyramine (Questran) daily
C) A Lactobacillus probiotic supplement (Lactinex) 4 times daily
D) Loperamide (Imodium) as needed
E) Psyllium fiber (Metamucil) twice daily

ANSWER: B
Diarrhea that develops in patients with ileal Crohn’s disease or following ileal resection is
usually due to increased amounts of bile acid remaining in the stool. This affects colonic
secretion and motility and various protein factors in the gut, resulting in the development of
bile acid diarrhea (BAD). Although various tests can be performed to evaluate the stool, gut
flora, and bowel function, a therapeutic trial with a bile acid sequestrant such as
cholestyramine is most often used for both the diagnosis and treatment of BAD. Reducing fat
intake may also be beneficial. Loperamide can lessen the diarrhea in some patients but should
not be the primary treatment because chronic use can result in constipation. Fiber
supplementation may help to produce a more formed stool and could be used as an adjunct
treatment when appropriate.

ABFM 2016
67. Which one of the following is more characteristic of Crohn’s disease, as opposed to
ulcerative colitis?
A) Bloody diarrhea
B) Perianal manifestations
C) Involvement of the rectum
D) Proximal progression
E) Associated inflammatory arthropathies
ANSWER: B
Crohn’s disease typically spares the rectum. Ulcerative colitis usually has rectal involvement,
progresses proximally, and rarely has perianal or systemic manifestations (SOR A). Both
conditions may cause bloody diarrhea as well as inflammatory arthropathies, eye
inflammation such as uveitis, and skin findings such as erythema nodosum.
Contraception
ABFM 2020
129. A healthy 26-year-old female would like to start using regular contraception and asks
about her options. Her menses are normal and her last menstrual period was 10 days ago.
If initiated today, which one of the following contraceptive methods does NOT require the use
of backup contraception?
A. Oral contraceptives
B. An etonogestrel subdermal implant (Nexplanon)
C. A copper-containing IUD (ParaGard)
D. A levonorgestrel IUD (Mirena)

ANSWER: C
A copper-containing IUD may be placed at any time during the menstrual cycle and does not
require the use of backup contraception. If oral contraceptives are started the day of the
appointment, then backup contraception is needed for the first week. If oral contraceptives will
be started with the next menstrual cycle, then backup contraception is needed until the pills are
started. Backup contraception is needed if a hormonal implant is placed more than 5 days from
the start of the last menstrual period, and if a levonorgestrel IUD is placed more than 7 days
from the start of the last menstrual period.

ABFM 2018
A 25-year-old female sees you because of irregular menses, hirsutism, and moderate acne. She
is sexually active in a monogamous relationship with a male, has never been pregnant, and
prefers not to become pregnant at this time. ?Which one of the following is considered first-
line therapy
A) Clomiphene (Clomid )
B) Letrozole (Femara)
C) Levonorgestrel/ethinyl estradiol
D) Metformin (Glucophage )
E) Spironolactone (Aldactone )

ANSWER: C
The Endocrine Society recommends hormonal contraception as the first-line medication for
women diagnosed with polycystic ovary syndrome (PCOS) who are experiencing irregular
menses, acne, and hirsutism and do not desire pregnancy (SOR A). Metformin may help
regulate menses but has not been shown to be as effective as oral hormone therapy. In a 2015
Cochrane review, oral contraceptives were recommended as the most effective treatment for
hirsutism. Either letrozole or clomiphene is appropriate for women diagnosed with PCOS who
want to become pregnant.

A 38-year-old patient wishes to start contraception. She currently takes lisinopril (Prinivil,
Zestril) for hypertension and also takes sumatriptan (Imitrex) occasionally for migraines at the
first sign of flashing lights or zigzagging lines in her vision. Her medical, family, and social
histories are otherwise unremarkable. An examination is notable only for a blood pressure of
130/80 mm Hg and a BMI of 36.0 kg/m2.
The patient is interested in using either the vaginal ring or the contraceptive patch. Which one
of the following would you recommend?
A) Transdermal norelgestromin/ethinyl estradiol (Ortho Evra) B) The etonogestrel/ethinyl
estradiol vaginal ring (NuvaRing)
C) Neither method due to her migraines
D) Neither method due to her age
E) Losing weight before starting either method
ANSWER: C
Family physicians are often asked to provide contraception and need to be familiar with the
current methods and contraindications. Estrogen-containing products, including the
contraceptive patch and the vaginal ring, are contraindicated in smokers >35 years of age and
in patients with migraine with aura.
ABFM 2017
43-A 34-year-old sexually active female consults you about contraception options. She has
late-stage kidney disease and her nephrologist has notified you that she will likely be
recommended for kidney transplantation soon. Which one of the following would you
recommend for safety and efficacy?
A) Condoms
B) Combined oral contraceptive pills
C) Medroxyprogesterone acetate (Depo-Provera) injections
D) An IUD
E) An etonogestrel/ethinyl estradiol vaginal ring (NuvaRing)
ANSWER: D
Family physicians are often asked to provide primary care for organ transplant recipients.
Pregnancy should be avoided during the 12 months following transplantation because of the
increased risk of preterm delivery and graft rejection. Female fertility typically increases post
transplant. The use of an IUD avoids interactions with medications, does not increase the risk
of infection, and is not affected by typical immunosuppressive therapies. The remaining
options are incorrect because of their higher failure and discontinuation rates. The CDC cites
failure rates with typical use of 9% for combined oral contraceptives and the
etonogestrel/ethinyl estradiol vaginal ring, 6% for injectable progesterone, 0.2% for
levonorgestrel IUDs, and 0.08% for the copper IUD. Barrier method failure rates exceed 18%.
Combined oral contraceptives and the vaginal ring also have potential estrogen-related side
effects, and injectable progesterone use increases the risk for osteoporosis.

62-A 20-year-old female presents to your office with questions about her contraceptive
method. She has been using a combined oral contraceptive pill for the past 2 years without any
complications. She has learned that several of her friends recently switched to an IUD. She is
concerned about the efficacy of her current method and asks about the failure rate. You tell her
that with typical use, the annual failure rate of a combined oral contraceptive pill is
A) 0.2%
B) 2%
C) 9%
D) 18%
E) 22%

ANSWER: C
The annual failure rate of combined oral contraceptive pills with typical use is 9%. Typical
failure rates for other contraceptive methods are 0.2% for the levonorgestrel IUD, 6% for
injectable progestin, 18% for male condoms, and 22% for the withdrawal method.

170. A 20-year-old female visits your office for advice regarding contraceptives. She
expresses interest in long-acting reversible contraception (LARC).
Which one of the following is the recommended timing for LARC placement?
A) Any time during the menstrual cycle
B) 14 days before the anticipated onset of menses
C) 7 days before the anticipated onset of menses
D) 7 days after the onset of menses
E) 14 days after the onset of menses
Answer : A
Long-acting reversible contraception (LARC) includes the copper IUD, levonorgestrel IUDs,
and subdermal implants. LARCs can be placed at any point in the patient’s menstrual cycle
(SOR A). There should be evidence that the patient is not pregnant prior to placement.

194. A 37-year-old female who smokes 1 pack of cigarettes per day has just had her third
child and
requests contraception. She does not intend to have any more children.
Which one of the following is the safest option for this patient?
A) Traditional combined oral contraceptive pills
B) Extended-cycle combined oral contraceptive pills
C) The contraceptive patch (Ortho Evra)
D) The contraceptive vaginal ring (NuvaRing)
E) The etonogestrel implant (Nexplanon)
ANSWER: E

ABFM 2016
131. The American College of Obstetricians and Gynecologists and the American Academy of
Pediatrics support the advance provision of emergency contraception to sexually active
women. Evidence shows that this practice
A) Decreases pregnancy rates on a population level
B) Decreases the time from unprotected sex to use of emergency contraception
C) Decreases contraception use by the patient prior to sexual activity
D) Increases rates of sexually transmitted infection
E) Increases rates of unprotected intercourse
ANSWER: B
Multiple studies, including randomized, controlled trials, have compared standard access to
emergency contraception (EC) with advance provision in which the patient is given a
prescription for the EC and encouraged to have it filled in order to have it immediately
available in case of unprotected intercourse. In multiple populations in the United States and
Europe advance provision compared to standard access has been shown to increase the rate of
use of EC and to reduce the interval between intercourse and use of EC.
However, no change has been shown for several outcomes when advance provision was
compared to standard access to EC. These include rates of sexually transmitted infections,
unprotected intercourse, use of routine contraceptives prior to sexual activity, and pregnancy
within the population studied.

239. Which one of the following is an absolute contraindication to combined oral


contraceptives in a 42-year-old female?
A) Varicose veins
B) Obesity (BMI >30.0 kg/m2)
C) Sickle cell disease
D) A history of ovarian cancer
E) Smoking 1 pack of cigarettes/day

ANSWER: E

In a female ≥35 years old, smoking 15 or more cigarettes per day poses an unacceptable health
risk with the use of combined oral contraceptives. Patients with varicose veins are not at
increased risk for deep vein thrombosis (DVT)/pulmonary embolism, which would be an
unacceptable health risk with the use of combined oral contraceptives. Combined oral
contraceptives actually lower the risk of ovarian cancer, and women may continue to use them
while awaiting treatment for ovarian cancer. Women with a BMI ≥30.0 kg/m2 who take oral
contraceptives are more likely to develop DVTs than those who do not use them, but the
advantages of oral contraceptives are considered to be greater than the disadvantages in these
patients, and obesity is not an absolute contraindication. A patient with sickle cell disease is at
a higher risk of adverse events from an unintended pregnancy than from the use of combined
oral contraceptives.
Irritable bowel syndrome:
ABFM 2018
You suspect a 45-year-old female may have irritable bowel syndrome. She has a 6-month
history of crampy, diffuse abdominal pain associated with defecation. Her symptoms occur
several days per week. According to the Rome IV criteria, an associated symptom that would
help in making this diagnosis is
A) a change in stool frequency
B) increased gas and bloating
C) pain brought on by eating
D) waking up at night to defecate
E) weight loss of 5 lb (2 kg)

ANSWER: A
The Rome IV criteria are widely used as guidelines to diagnose suspected irritable bowel
syndrome. These criteria specify that there should be recurrent abdominal pain associated with
two or more additional symptoms at least 1 day per week in the last 3 months. These
symptoms include pain related to defecation, a change in stool frequency, or a change in stool
form. Pain brought on by eating and increased gas and bloating are observed in irritable bowel
syndrome but are not included in the Rome IV criteria. Weight loss and waking at night to
defecate are not typically seen in this disorder.
A 68-year-old female presents with a history of episodic severe lower abdominal pain relieved
by defecation. She has had a long history of constipation with normal to very firm stools. Her
history and a physical examination are otherwise normal. A colonoscopy 3 years ago was
normal. You diagnose constipation-predominant irritable bowel syndrome. ?Which one of the
following agents would be the most appropriate treatment for this patient :
A) Lactulose
B) Magnesium citrate
C) Milk of magnesia
D) Polyethylene glycol
E) Sodium phosphate

ANSWER: D
Hypertonic osmotic laxatives such as milk of magnesia, magnesium citrate, and sodium
phosphate draw water into the bowel and should be used with caution in older adults and those
with renal impairment because of the risk of electrolyte abnormalities and dehydration in
patients with irritable bowel syndrome (IBS). Lactulose, also an osmotic laxative, should be
avoided in patients with IBS because it is broken down by colonic flora and produces
excessive gas.

Polyethylene glycol, a long-chain polymer of ethylene oxide, is a large molecule that causes
water to be retained in the colon, which softens the stool and increases the number of bowel
movements. It is approved by the FDA for short-term treatment in adults and children with
occasional constipation and is commonly prescribed for patients with IBS. It is considered
safe and effective for moderate to severe constipation when used either daily or as needed.

ABFM 2017
46-A 33-year-old female comes to your office for follow-up of irritable bowel syndrome. You
ruled out other causes of her abdominal bloating, abdominal pain, and diarrhea at earlier visits.
She has no change in symptoms, such as constipation or blood in her stool. She has resisted
treatment in the past, but her symptoms are becoming more frequent and she would now like
to consider treatment.
Evidence shows that which one of the following would most likely be beneficial for this
patient?
A) Acupuncture
B) Increased insoluble fiber in her diet
C) Fluoxetine (Prozac), 20 mg daily
D) Neomycin, 1000 mg every 6 hours for 7 days
E) Polyethylene glycol (MiraLAX), 17 g daily

ANSWER: C
This patient has diarrhea-predominant irritable bowel syndrome (IBS). There are many
treatments available, with varying degrees of evidence. SSRIs, along with tricyclic
antidepressants, have been shown to decrease abdominal pain and improve global assessment
scores in those with IBS. Polyethylene glycol is a treatment for constipation and would not
help this patient. Acupuncture has not been shown to be superior to sham acupuncture in
improving IBS symptoms. Neomycin has been shown to improve symptoms in constipation-
predominant IBS but would not be helpful in diarrhea-predominant IBS. Soluble fiber such as
psyllium improves symptoms and decreases abdominal pain scores in patients with IBS.
Insoluble fiber has not been shown to improve any IBS outcomes.
ABFM 2016
170. Which one of the following therapies is most likely to reduce abdominal cramping in a
patient with irritable bowel syndrome?
A) Peppermint oil
B) Polyethylene glycol (MiraLAX)
C) Probiotics
D) Wheat bran

ANSWER: A

Peppermint oil is an antispasmodic that has moderate evidence of benefit in the relief of
abdominal cramping in patients with irritable bowel syndrome (IBS). Probiotics as a group
have some evidence of benefit in improving global IBS symptoms, but the studies have used
different probiotic preparations so it is difficult to draw definite conclusions about their
effectiveness or to recommend a specific preparation. Polyethylene glycol is an osmotic
laxative that has evidence of benefit for bowel symptoms, including improvement in stool
frequency and consistency, but it does not improve abdominal pain. Wheat bran is an
insoluble fiber that contains fructans that can exacerbate IBS symptoms and should be avoided
in patients with IBS.
Well Baby Care + Infantile Colic
ABFM 2020
191. Which one of the following developmental milestones would be expected in a typical 12-
month-old child?
1. Standing independently
2. Identifying at least two body parts
3. Using three words other than names
4. Scribbling spontaneously
5. Building a three-cube tower
ANSWER: A
Standing independently is an expected developmental milestone for a 12-month-old child.
Identifying at least two body parts, using three words other than names, scribbling
spontaneously, and building a three-cube tower are expected milestones for older children.
98. A healthy 2-month-old female born at term is brought to your office for a well child check.
The parents report that the child is exclusively breastfed and ask whether they should be
providing any additional nutrition.Which one of the following would you advise?
A. No supplementation until the child is 6 months old
B. Water
C. Fluoride
D. Vitamin D
E. Rice cereal

ANSWER: D
It is recommended that all infants should be given 400 IU of vitamin D because of generally
decreased sun exposure in today’s living situations. Additional water consumption can
decrease milk intake and cause electrolyte disturbances. Fluoride supplementation and the
introduction of cereal are not recommended until 6 months of age.

143. A6-month-old male is brought to your office by his parents for a well child check. He is
their first child. On examination you note that he has two erupted teeth. The family lives in a
town with fluoridated water and the parents would like to know how to care for their child’s
teeth. Which one of the following is the most effective preventive measure for this child’s
newly erupted teeth?
A. No care is needed at this point
B. A parent wiping or brushing the teeth with water twice daily
C. A parent brushing the teeth with a smear of low-fluoride toothpaste twice daily
D. A fluoride supplement daily
E. Topical sealants

ANSWER: C
Early and consistent dental care in infants has been shown to reduce the rate of early
childhood caries. The American Academy of Pediatric Dentistry recommends the use of low-
fluoride toothpaste for tooth cleaning, starting with newly erupted teeth. According to the
CDC, dental caries is one of the most prevalent chronic conditions among children in the
United States. Dental caries can start soon after eruption of the first teeth. Brushing an infant’s
teeth twice daily with a “smear” of fluoridated toothpaste is recommended. Fluoridated
toothpaste has been found to be safe and effective for infants’ teeth. The use of fluoride in
toothpaste is more effective for the prevention of caries than wiping or brushing the teeth with
water. A daily fluoride supplement is not appropriate for this patient because the family’s local
water source already contains fluoride. Topical sealants are intended for molar teeth, not
incisors.

ABFM 2018
You perform the initial newborn examination on a male on his first day of life, following an
uncomplicated vaginal delivery at an estimated gestational age of 37 weeks and 6 days. The
prenatal course was significant for the initial presentation for prenatal care at 22 weeks
gestation. You note that the infant’s upper lip is thin and the philtrum is somewhat flat. Which
additional finding would increase your concern for fetal alcohol syndrome?
A) Curvature of the fifth digit of the hand (clinodactyly)
B) A supernumerary digit of the hand
C) Flattening of the head (plagiocephaly)
D) Metatarsus adductus in one foot
E) Syndactyly of the toes (webbed feet)

ANSWER: A
In addition to clinodactyly, fetal alcohol syndrome is associated with camptodactyly (flexion
deformity of the fingers), other flexion contractures, radioulnar synostosis, scoliosis, and
spinal malformations. It is also associated with many neurologic, behavioral, and
cardiovascular abnormalities, as well as other types of abnormalities. Plagiocephaly,
supernumerary digits, syndactyly, and metatarsus adductus are common in newborns but are
not related to fetal alcohol spectrum disorders.

In addition to group B Streptococcus (GBS), which one of the following is the most common
cause of neonatal sepsis?
A) Escherichia coli
B) Group A Streptococcus
C) Listeria monocytogenes
D) Staphylococcus aureus
E) Streptococcus pneumonia

ANSWER: A
Newborns with sepsis may have focal signs of infection such as pneumonia or respiratory
distress syndrome, but they also may have nonfocal signs and symptoms. In the newborn
period the two most common causes of neonatal sepsis are group B Streptococcus and
Escherichia coli. Listeria monocytogenes was once a more common cause but it is now
uncommon. Streptococcal pneumonia is an uncommon cause of sepsis in neonates.
Staphylococcus aureus and group A Streptococcus are not as common but should be
considered in newborns with cellulitis.

An 18-month-old male is brought to your office for a well child check. He is walking only
with assistance. You and the parents are concerned about gross motor delay.
Which one of the following findings would be most suggestive of muscular dystrophy in this
patient?
A) A cross-legged “scissoring” posturing
B) Head lag when sitting up
C) Hyperreflexia in the legs
D) Partial hemiparesis of the lower extremities
E) Toe walking

ANSWER: B
Some abnormal gross motor developmental findings suggest muscular dystrophy. Signs of
increased muscular tone, such as cross-legged posturing, neck stiffness, and hyperreflexia,
suggest a central cause of motor delay such as cerebral palsy. Head lag due to neck muscle
weakness in infants is a classic early finding of muscular dystrophies. Hemiparesis similarly
suggests a central nervous system abnormality. Toe walking can be seen with both central and
peripheral neuromuscular abnormalities, including muscular dystrophy, but is less specific,
and therefore less helpful, in differentiating the cause of motor delay. In muscular dystrophies
it is a sign of quadriceps weakness.

The mother of a newborn infant is concerned because her baby’s eyes are sometimes crossed.
Assuming the intermittent eye crossing persists, which one of the following is the most
appropriate age for ophthalmologic referral?
A) 10–14 days
B) 6 months
C) 12 months
D) 24 months
ANSWER: B
In many normally developing infants there may be imperfect coordination of eye movements
and alignment during the early days and weeks of life, but proper coordination should be
achieved by age 4–6 months. Persistent deviation of an eye in an infant requires evaluation.

A mother brings in her 2-week-old infant for a well child check. She reports that she is
primarily breastfeeding him, with occasional formula supplementation.
Which one of the following should you advise her regarding vitamin D intake for her baby?
A) Breastfed infants do not need supplemental vitamin D
B) He does not need supplemental vitamin D if he is taking at least 16 oz of formula per day
C) Vitamin D supplementation should not be started until he is at least 6 months old
D) He should be given 400 IU of supplemental vitamin D daily
E) Intake of vitamin D in excess of 200 IU/day is potentially toxic

ANSWER: D
The American Academy of Pediatrics recommends a daily intake of 400 IU of vitamin D in
infants, children, and adolescents (SOR C). Breastfeeding does not provide adequate levels of
vitamin D. Exclusive formula feeding probably provides adequate levels of vitamin D, but
infants who consume less than 1 liter of formula per day need supplementation with 400 IU of
vitamin D daily. Vitamin D supplementation should be started within the first 2 months of life.

ABFM 2016
195. Risk factors for sudden infant death syndrome include which one of the following?
A) Side sleeping
B) Being large for gestational age at birth
C) A history of an apparent life-threatening event
D) Pacifier use
E) Asian or Pacific Islander ethnicity
ANSWER: A
Proper sleep position is highly important in reducing the risk of sudden infant death syndrome
(SIDS). Side sleeping and prone sleeping increase the risk for SIDS (SOR A). Bed sharing
also increases the risk. The “Back to Sleep” campaign of the American Academy of Pediatrics
(AAP) was associated with a 53% decrease in the incidence of SIDS in the United States in
the 1990s. The use of pacifiers during sleep decreases the risk for SIDS, as does exclusive
breastfeeding. The AAP recommends offering a pacifier for use during sleep for infants in
whom breastfeeding is well established. There are also multiple invariable risk factors for
SIDS. These include male sex, Native American ethnicity, birth weight <2500 g, and birth
before 37 weeks gestation. Large for gestational age infants are not at increased risk for SIDS,
and infants of Asian or Pacific Islander ethnicity actually have a lower than average risk of
SIDS. Apparent life-threatening events are not a risk factor for SIDS and are not related to
SIDS (SOR B).

Gastroenteritis

ABFM 2020
150. A58-year-old male with uncontrolled type 2 diabetes sees you for follow-up after a recent
hospitalization for urosepsis treatment with intravenous antibiotic therapy. His hospital course
was complicated by Clostridioides (Clostridium) difficile colitis, and he completed oral
vancomycin (Vancocin)therapy 2weeks ago. He was nearly back to his baseline but has had
recurrent watery diarrhea for the past 3 days. You confirm a recurrent infection. Which one of
the following would be the most appropriate treatment?
A. Probiotics
B. Fidaxomicin (Dificid), 200 mg twice daily for 10 days
C. Metronidazole (Flagyl), 500 mg three times daily for 10 days
D. Vancomycin, 125 mg four times daily for 10 days
E. Fecal microbiota transplantation

ANSWER: B
This patient presents with his first recurrence of Clostridioides (Clostridium) difficile
infection, which was previously treated with vancomycin. Initial episodes can be treated with
vancomycin (strong recommendation, high quality of evidence), fidaxomicin (strong
recommendation, high quality of evidence), or metronidazole if the other two treatments are
unavailable (weak recommendation, high quality of evidence). However, fidaxomicin is
recommended for recurrent infection if vancomycin was prescribed for the initial episode
(weak recommendation, moderate quality of evidence). If available, a prolonged tapered
course of vancomycin could be used if a 10-day course was prescribed initially (weak
recommendation, low quality of evidence). Vancomycin is only recommended for a first
recurrent episode if metronidazole was used initially (weak recommendation, low quality of
evidence). Metronidazole is not recommended for recurrent episodes. Probiotic administration
is not recommended due to insufficient data. Fecal microbiota transplantation is only
recommended for a second or subsequent recurrent infection (strong recommendation,
moderate quality of evidence).
130. In a patient with documented Helicobacter pylori infection, which one of the following
studies should be performed to document clearance of the infection?
A. An H. pylori IgG level
B. An H. pylori IgM level
C. A stool antigen test 1 week after the completion of antibiotic therapy
D. A urea breath test 1 month after the completion of antibiotic therapy
E. A gastric biopsy

ANSWER: D

After treatment for a Helicobacter pylori infection it is essential to document clearance of the
infection. This is typically done with a stool antigen test or a urea breath test performed 1
month after the completion of antibiotic therapy. If the patient is taking a proton pump
inhibitor it should be discontinued prior to the test. H. pylori IgG or IgM levels and gastric
biopsies are not appropriate for documenting clearance of H. pylori infection.

ABFM 2017
223. A 12-year-old male is brought to your office with a 3-day history of nausea, vomiting,
and fever. His fever has resolved but he continues to have vomiting and diarrhea. He has had
large-volume, loose, nonbloody stools, as well as abdominal cramping. A stool culture is
positive for Salmonella. Which one of the following would be the most appropriate treatment?
A) Supportive care only
B) Ampicillin
C) Ceftriaxone (Rocephin)
D) Ciprofloxacin (Cipro)
E) Trimethoprim/sulfamethoxazole (Bactrim)

ANSWER: A
Salmonella is a common cause of gastroenteritis. Transmission is most often associated with
eggs, poultry, undercooked ground meat, and dairy products from contaminated animals, or
produce contaminated by their waste. Salmonella infection is usually associated with nausea,
vomiting, diarrhea, and fever starting 6–48 hours after ingestion of contaminated water or
food. Stools are usually moderate-volume, loose, and nonbloody, although they can be large-
volume watery stools with blood. While Salmonella can cause severe infection, it is usually
self-limited. Antibiotics should not be routinely used to treat uncomplicated Salmonella
gastroenteritis and may prolong the duration of Salmonella excretion in stool. Antibiotic
treatment should be reserved for patients who are severely ill or suspected of being
bacteremic. The threshold for treatment should also be decreased in those who are considered
to be at higher risk for severe illness and invasive disease, such as infants, the elderly, patients
with sickle cell disease, and immunosuppressed patients. Chronic fecal carriers of Salmonella
may also benefit from treatment. If treatment is required, ciprofloxacin, ampicillin,
ceftriaxone, and trimethoprim/sulfamethoxazole are all treatment options.
ABFM 2016
chemoreceptor trigger zone, making it an effective antiemetic for patients with gastroenteritis?
A) Diphenhydramine (Benadryl)
B) Meclizine (Antivert)
C) Metoclopramide (Reglan)
D) Ondansetron (Zofran)
E) Scopolamine (Transderm Scop)
ANSWER: C
Dopamine antagonists, such as metoclopramide, block dopamine stimulation in the
chemoreceptor trigger zone, thereby limiting emetic input to the medullary vomiting center
(SOR C). SSRIs, such as ondansetron, also work in the chemoreceptor trigger zone. They
inhibit serotonin at the 5-HT3 receptor in the small bowel, vagus nerve, and chemoreceptor
trigger zone. Antihistamines and anticholinergics limit stimulation of the vomiting center
through inhibition of the H1 receptor and acetylcholine, respectively. These medications are
particularly beneficial in vestibular-mediated nausea, such as motion sickness.
Adrenal gland disorders
2019 pretest
181. A 39-year-old female presents to your office with progressive swelling in the right leg
that has spread over the past 2 days. She feels well otherwise. Her past medical history is
significant for adrenal insufficiency treated with oral prednisone, 5 mg daily, and
fludrocortisone, 0.1 mg daily. An examination reveals a temperature of 37.8°C (100.0°F), a
pulse rate of 88 beats/min, a blood pressure of 105/62 mm Hg, and redness, warmth, and
swelling in the right lower extremity. You diagnose cellulitis and prescribe appropriate
antibiotics.
Which one of the following would be the most appropriate management of her corticosteroid
regimen?
A. Continue prednisone and fludrocortisone at the current dosages
B. Increase prednisone to 15 mg daily and continue fludrocortisone at the current dosage
C. Increase prednisone to 15 mg daily and increase fludrocortisone to 0.3 mg daily
D. Stop prednisone until the infection resolves but continue fludrocortisone at the current
dosage
E. Stop both prednisone and fludrocortisone until the infection resolves

ANSWER: B
Patients with chronic adrenal insufficiency, either primary or secondary, may not be able to
mount a stress response to infection or surgical procedures. Common practice during minor
infections is to increase the corticosteroid supplementation (SOR C). Fludrocortisone should
be continued, but stress dosing is not necessary.
ABFM 2020
138. A 36-year-old male sees you for follow-up of progressive fatigue and lightheadedness
that has worsened over the past 3 months. He has lost 5 kg (11 lb) during this time. On
examination he has a BMI of 21 kg/m2, a blood pressure of 88/48 mm Hg, and a pulse rate of
66 beats/min. A skin examination is notable for patches of nonpigmented skin on the hands.
Initial laboratory testing is significant for a sodium level of 132 mEq/L (N 135–145) and a
potassium level of 5.3 mEq/L (N 3.5–5.0).
Which one of the following tests would confirm the most likely diagnosis?
A. 17-hydroxyprogesterone
B. ACTH stimulation
C. Dexamethasone suppression
D. Late night salivary cortisol
E. Plasma renin and aldosterone
ANSWER: B

This clinical case is consistent with Addison’s disease, or adrenal insufficiency. This case is
most likely the result of autoimmune disease, given the concurrent vitiligo, but it may also be
idiopathic or secondary to cancers such as lymphoma or infections such as tuberculosis.
Adrenal insufficiency is suggested by a low morning cortisol level, but the test of choice to
confirm this diagnosis is the ACTH stimulation test.
17-Hydroxyprogesterone deficiency causes congenital adrenal hyperplasia, which typically
presents in childhood. This test is part of newborn screening in the United States. Acquired
17- hydroxyprogesterone deficiency can present in adulthood as adrenal insufficiency but a
low 17- hydroxyprogesterone level does not confirm adrenal insufficiency. Late night salivary
cortisol is an initial test for corticosteroid excess (Cushing syndrome) and the dexamethasone
suppression test is used to confirm that disorder. Renin and aldosterone levels can be helpful
to characterize mineralocorticoid deficiency but they are not diagnostic.

ABFM 2018
You see a patient with a serum sodium level of 122 mEq/L (N 135–145) and a serum
osmolality of 255 mOsm/kg H2O (N 280–295). Which one of the following would best
correlate with a diagnosis of syndrome of inappropriate antidiuresis?

A) A fractional excretion of sodium below 1%


B) Elevated urine osmolality
C) Elevated serum glucose
D) Elevated BUN
E) Low plasma arginine vasopressin

ANSWER: B
The syndrome of inappropriate antidiuresis (SIAD, formerly SIADH) is related to a variety of
pulmonary and central nervous system disorders in which hyponatremia and hypo-osmolality
are paradoxically associated with an inappropriately concentrated urine. Most cases are
associated with increased levels of the antidiuretic hormone arginine vasopressin (AVP).
Making a diagnosis of SIAD requires that the patient be euvolemic and has not taken diuretics
within the past 24–48 hours, and the urine osmolality must be high in conjunction with both
low serum sodium and low osmolality. The BUN should be normal or low and the fractional
excretion of sodium >1%. Fluid restriction (<800 cc/24 hrs) over several days will correct the
hyponatremia/hypo-osmolality, but definitive treatment requires eliminating the underlying
cause, if possible. In the case of severe, acute hyponatremia with symptoms such as confusion,
obtundation, or seizures, hypertonic (3%) saline can be slowly infused intravenously but might
have dangerous neurologic side effects. Elevated serum glucose levels may cause a factitious
hyponatremia, but not SIAD.

A 28-year-old female presents with a 3-month history of fatigue and postural lightheadedness.
On examination she is diffusely hyperpigmented, especially her skin creases and areolae. A
CBC and basic metabolic panel are normal except for an elevated potassium level. You order
a corticotropin stimulation test. Prior to the corticotropin injection, you should order which
one of the following tests to confirm that this patient has a primary insufficiency and not a
secondary (pituitary) disorder?
A) ACTH
B) Aldosterone
C) Melanocyte-stimulating hormone
D) Renin
E) TSH
ANSWER: A
A plasma ACTH level is recommended to establish primary adrenal insufficiency. The sample
can be obtained at the same time as the baseline sample in the corticotropin test. A plasma
ACTH greater than twice the upper limit of the reference range is consistent with primary
adrenal insufficiency. Aldosterone and renin levels should be obtained to establish the
presence of adrenocortical insufficiency, but these do not differentiate primary from secondary
adrenal insufficiency. The hyperpigmentation of Addison’s disease is caused by the
melanocyte-stimulating hormone (MSH)–like effect of the elevated plasma levels of ACTH.
ACTH shares some amino acids with MSH and also produces an increase in MSH in the
blood. TSH is not part of the feedback loop of adrenal insufficiency.

In a patient presenting with truncal obesity, hypertension, type 2 diabetes mellitus, hirsutism,
osteopenia, and skin fragility, which one of the following tests is needed to confirm the
diagnosis of Cushing syndrome?
A) A dexamethasone suppression test
B) Inferior petrosal sinus sampling
C) Plasma corticotropin
D) Plasma free cortisol
E) Urinary free cortisol
ANSWER: E In a patient presenting with obesity, hypertension, type 2 diabetes mellitus, and
hirsutism, who also has thin skin and osteopenia, an elevated 24-hour collection showing high
urinary free cortisol confirms the presence of Cushing syndrome. The dexamethasone
suppression test, though still commonly used, no longer has a place in the diagnosis and
treatment of patients with Cushing syndrome. Corticotropin-dependent and corticotropin-
independent causes of Cushing syndrome can be separated by measuring plasma corticotropin.
Plasma free cortisol measurements should be obtained only to determine the success or failure
of transsphenoidal microadenomectomy or adrenalectomy. Inferior petrosal sinus sampling is
used to confirm the source of corticotropin secretion before surgical intervention.

ABFM 2017
159. A 26-year-old female presents with a 3-month history of abdominal pain, lightheadedness
with standing, and some hyperpigmentation. Her CBC is normal, but a basic metabolic panel
reveals a slightly low sodium level and a slightly high potassium level. Which one of the
following would be the most appropriate next step in your evaluation of this patient?
A) A serum aldosterone level
B) A corticotropin (ACTH) stimulation test
C) Paired morning cortisol and melanocyte stimulating hormone levels
D) 21-hydroxylase antibodies and 17-hydroxyprogesterone levels

ANSWER: B
This patient has signs and symptoms consistent with primary adrenal insufficiency (PAI). In
Western countries autoimmunity is responsible for 90% of these cases. Because the
corticotropin (ACTH) stimulation test has a higher degree of sensitivity and specificity than
morning cortisol and ACTH concentrations, it is the preferred test in all patients with possible
primary adrenal insufficiency. Serum aldosterone paired with plasma renin activity is used to
screen for adrenal hyperplasia in hypertensive patients and also for establishing the existence
of mineralocorticoid insufficiency in patients with PAI. Once the diagnosis is established, 21-
hydroxylase antibodies and 17-hydroxylase progesterone levels are used to determine the
etiology of PAI.
Obsessive – compulsive disorder
None
Peripheral vascular disease
ABFM 2020
171. A72-year-oldfemalecomestoyourofficetodiscussnewlydiagnosedperipheralartery disease.
She has a history of long-standing type 2 diabetes, hypertension, and hyperlipidemia. Her
current medications include atorvastatin (Lipitor), 80 mg daily; metformin (Glucophage XR),
2000 mg daily; lisinopril (Prinivil, Zestril), 40 mg daily; and aspirin, 81 mg daily. On
examination she has a blood pressure of 114/62 mm Hg. Laboratory studies reveal an LDL-
cholesterol level of 68 mg/dL and a hemoglobin A1c of 7.7%.
If added to her current regimen, which one of the following medications would most likely
improve her walking distance?
A. Cilostazol
B. Clopidogrel (Plavix)
C. Ezetimibe (Zetia)
D. Liraglutide (Victoza)
E. Warfarin (Coumadin)

ANSWER: A
In patients with peripheral artery disease (PAD), cilostazol, a phosphodiesterase inhibitor with
antiplatelet and vasodilatory properties, increases the maximal walking distance on a treadmill
by approximately 25% compared with placebo. The side effects include tachycardia, diarrhea,
and increased bleeding tendency, and it is contraindicated in patients with heart failure or a
low ejection fraction. A supervised exercise program is also an important component of a
comprehensive approach to PAD. In one trial a supervised exercise program resulted in an
increase of 2.1 minutes of mean peak walking time at 6 months.
Clopidogrel is an antiplatelet agent that has been shown to be slightly more effective than
aspirin in reducing the risk of a composite outcome of ischemic stroke, myocardial infarction,
or death from vascular causes. Dual antiplatelet therapy can be considered to reduce the risk of
cardiovascular events though it carries an increased risk of bleeding. Warfarin has no evidence
of benefit in PAD and adds to bleeding risk. Statins are associated with a lower risk of
cardiovascular events in patients with PAD. Ezetimibe has no evidence of additional benefit.
ACE inhibitors appear to be the preferred blood pressure agent for patients with PAD given
the findings of the HOPE trial, in which ramipril resulted in a lower risk of adverse
cardiovascular outcomes compared to placebo over 5 years of follow-up. Blood pressure
targets are not clear in PAD but this patient’s blood pressure is already <120/75 mm Hg and
she is unlikely to benefit from additional blood pressure lowering. Intensifying therapy for this
patient’s type 2 diabetes by adding liraglutide also has no evidence of benefit for PAD.

Palpitation
ABFM 2020
188. A 35-year-old female comes to your office for follow-up of an emergency department
(ED) visit for palpitations. She tells you that she was driving on the highway with her three
small children when she suddenly felt her heart racing, along with chest tightness,
lightheadedness,andsevere anxiety. She pulled over and called 911. While she waited for EMS
she took diltiazem (Cardizem), 30 mg orally, which had been prescribed following a similar
episode several years ago. Her symptoms lasted about 10 minutes and had improved by the
time EMS arrived. An examination, EKG, and chest radiograph in the ED were all normal.
Which one of the following findings in her previous medical record would confirm your
diagnosis?
A) A Generalized Anxiety Disorder–7 (GAD-7) score of 6
B) An elevated TSH level
C) C. P waves hidden within a narrow QRS complex on an event recorder
D) Mitral valve prolapses on an echocardiogram
E) Atherosclerotic plaque seen on carotid ultrasonography
ANSWER: C
This patient presents with a history consistent with typical atrioventricular nodal reentrant
tachycardia, which is the most common type of supraventricular tachycardia (SVT). She is
also using “pill-in-the-pocket” treatment, which is effective for infrequent SVT. Because the
symptoms are episodic and the tachycardia is paroxysmal, patients generally present with
normal examination and EKG findings. Further evaluation with event monitoring may identify
a narrow-complex tachycardia with P waves hidden within the QRS complex or identified
early after it. Most patients with SVT have structurally normal hearts.
An elevated Generalized Anxiety Disorder–7 (GAD-7) score is consistent with a diagnosis of
generalized anxiety disorder (GAD). However, GAD is a common misdiagnosis in patients
with SVT, particularly females. While hyperthyroidism is associated with tachycardia,
hypothyroidism usually is not. Mitral valve prolapse is not specifically associated with SVT.
Carotid atherosclerosis is not associated with SVT either, but knowledge of its presence may
help determine treatment.

59. A 32-year-old female comes to your office because of palpitations. She reports a sensation
of her heart racing that lasts for several seconds, occurs at rest, and has been occurring daily
for the past couple of weeks. She has not had any loss of consciousness or other associated
symptoms, has no history of recent stressors or anxiety,and does not drink caffeine or take any
Illicit drugs.She is other wise healthy and takes no medications.An examination is
unremarkable and an EKG and basic laboratory studies are all normal.
Which one of the following would be the most appropriate next step in your evaluation?

A. Reassurance only
B. A 24-hour Holter monitor
C. A 30-day cardiac event monitor
D. An exercise stress test
E. Transthoracic echocardiography
ANSWER: B
This patient does not have any obvious metabolic or psychiatric explanations for her
palpitations, raising the clinical suspicion for an underlying cardiac cause. Reassurance
without further investigation of a possible cardiac cause would be inappropriate. Given that
her symptoms are occurring daily, a 24-hour Holter monitor would be the most appropriate
next step in the evaluation. If her symptoms occurred less frequently, a 30-day cardiac event
monitor, which stores data after being activated, could be used. An exercise stress test would
be indicated as the next step if her symptoms were exertional. Transthoracic echocardiography
would be indicated as the next step if there were a clinical suspicion for structural heart
disease based on a previous history of cardiac disease or worrisome signs and symptoms such
as dyspnea, crackles, lower extremity edema, or elevated jugular venous pressure.
Rheumatoid Arthritis
ABFM 2019
19. A 57-year-old male with diabetes mellitus and hypertension presents with a 1-month
history of pain in his hands and elbows. His hands are shown below. On examination they are
tender and he has soft swelling of the wrists, metacarpophalangeal (MCP) joints, and proximal
interphalangeal (PIP) joints.
Plain films show mild, diffuse bony erosions in the MCP and PIP joints.
Which one of the following is the most likely diagnosis?
A. Dermatomyositis
B. Osteoarthritis
C. Psoriatic arthritis
D. Rheumatoid arthritis
E. Systemic lupus erythematosus
ANSWER: D
This patient’s clinical findings and radiographs indicate a diagnosis of inflammatory arthritis,
most likely rheumatoid arthritis. Symmetric small-joint inflammatory arthritis is typical of
rheumatoid arthritis and systemic lupus erythematosus (SLE), but bony erosions are not seen
in SLE. Psoriatic arthritis can also affect small joints but is typically not symmetric.
Dermatomyositis can present with a thick, bright red rash over the metacarpophalangeal
(MCP) and interphalangeal joints (Gottron’s sign) but is typically associated with proximal
muscle weakness rather than joint pain or erosions that can be seen on radiographs.
Osteoarthritis does not typically cause the soft-tissue swelling seen in the image. It usually
affects the distal and proximal interphalangeal joints while sparing the MCP joints, and it
results in osteophytes and joint space narrowing that can be seen on radiographs.

Approach to head truma with CT SS


ABFM 2020
149. A 19-year-old football player presents to your office after being removed from a game
when he sustained a head injury. He has vomited twice since the injury and is disoriented. You
determine that imaging is indicated. Which one of the following would be most appropriate at
this time?
A. Plain radiography of the skull
B. CT of the head
C. MRI of the brain
D. Functional MRI of the brain

ANSWER: B
Evaluation of uncomplicated concussions does not require imaging. When a more severe
injury is evident, the modality of choice is CT. Plain radiographs have minimal utility in the
evaluation of head trauma. MRI is most appropriate for evaluation of prolonged symptoms.
Functional MRI is still a research tool for evaluating brain injuries.

Approach to Limping Child


ABFM 2020
95. A 3-year-old male is brought to your office by his mother because he has been limping.
The mother reports that she saw him fall while playing at the park with some other children
yesterday, but that he seemed fine afterward. This morning, however, he did not want to put
any weight on the leg when he got up. The child has also had a fever off and on for 3–5 days.
His mother states that he had a temperature of 102°F today. The child does not want to bear
weight on the affected extremity. Plain film imaging of both lower extremities is negative.

Which one of the following would be the most appropriate next step in the workup?
A. A CBC, erythrocyte sedimentation rate, and C-reactive protein level
B. Joint aspiration
C. Ultrasonography
D. Bone scintigraphy
E. MRI

ANSWER: A
In this scenario trauma is a consideration due to the patient’s history of falling. However, with
a history of fever >38.5°C (101.3°F) it is appropriate to order a CBC, erythrocyte
sedimentation rate (ESR), and C-reactive protein (CRP) level as the next step in the
evaluation.

An oral temperature >38.5°C, refusal to bear weight on the affected leg, an ESR >40 mm/hr, a
peripheral WBC count >12,000/mm3, or a CRP level >20 mg/L should raise suspicion for
septic arthritis (SOR C). Septic arthritis of the hip should be suspected in this patient. As the
evaluation progresses, joint aspiration may be considered to evaluate for septic arthritis or
transient synovitis. MRI may also be appropriate as the evaluation progresses after a CBC,
ESR, and CRP level. Ultrasonography and bone scintigraphy would not be a consideration at
this time. For hip effusions in children, ultrasonography is recommended over plain
radiography, but it does not differentiate between sterile, purulent, and hemorrhagic effusions.

Failure to thrive and Short Stature


ABFM 2017
1- A 3-year-old male is brought to your office for a well child visit. The history indicates that
the child has a vocabulary of about 100 single words and has not begun to speak in 2-word
phrases. An ear examination is normal and the parents have no concerns regarding the child’s
hearing. They report that the child is interacting with others normally.
Which one of the following would you propose?
A) Reassessment in 6 months
B) Limiting screen time to 2 hours per day
C) A 4-week intensive parent-administered reading program
D) Referral to a local early developmental intervention program

ANSWER: D
In a 3-year-old, red flags that would suggest the need for immediate speech-language
evaluation include the inability to understand prepositions or action words or the child having
a vocabulary that consists of less than 200 words (SOR C). A child should use 2-word phrases
by 2½ years of age. The evaluation should be performed through a local early developmental
intervention program or a speech-language pathologist. The therapeutic response to parent-
administered programs varies greatly, with programs lasting longer than 8 weeks having more
success. Limiting screen time would not address this child’s problem. Many family physicians
would implement a parent-completed developmental survey such as Ages and Stages.

112- An adolescent male is concerned because he is the shortest boy in his class. His age is
14.3 years and his parents are of normal height. He has a negative past medical history and no
symptoms.
On examination he is 151 cm (59 in) tall. The average height for his age is 165 cm. His weight
is 43 kg (95 lb). His sexual maturity rating is 3 for genitalia and 2 for pubic hair. A wrist
radiograph shows a bone age of 12.2 years. The average height for this bone age is 152 cm.
On the basis of this evaluation you can tell the patient and his parents that
A) he should have a growth hormone stimulation test
B) his adult height will be below average
C) his sexual development is about average for his age
D) he will begin to grow taller within approximately a year
E) an underlying nutritional deficiency may be the cause of his short stature
ANSWER: D
Constitutional growth delay, defined as delayed but eventually normal growth in an
adolescent, is usually genetic. If evaluation of the short adolescent male reveals no evidence of
chronic disease, if his sexual maturity rating is 2 or 3, and if his height is appropriate for his
skeletal age he can be told without endocrinologic testing that he will begin to grow taller
within a year or so. His adult height may be below average but cannot be predicted reliably.
Average sexual maturity ratings for a male of 14.3 years are 4 for genitalia and 3–4 for pubic
hair. The history and physical examination would have given clues to any illnesses or
nutritional problems.
76-A 7-month-old infant is hospitalized for the third time with lower-lobe bronchopneumonia.
Findings include a weight and height below the 10th percentile. A sibling died of sudden
infant death syndrome. Laboratory testing reveals a hemoglobin level of 9.0 g/dL (N 10.5–
14.0), a mean corpuscular volume of 85 m3 (N 72–88), and a serum calcium level of 9.0
mg/dL (N 9.0–10.5). A sweat chloride level is 20 mEq/L (N <60).
Which one of the following is the most likely cause of this infant’s failure to thrive?
A) Cystic fibrosis
B) DiGeorge’s syndrome
C) Battered child syndrome
D) Gastroesophageal reflux
E) β-Thalassemia

ANSWER: D
Gastroesophageal reflux accounts for a significant number of cases of failure to thrive, crib
death, and recurrent pneumonia. Features of gastroesophageal reflux include a history of
recurrent pneumonia, a low growth curve, a family history of sudden infant death syndrome,
and normocytic anemia. A sweat chloride level of 20 mEq/L rules out cystic fibrosis. Normal
serum calcium excludes DiGeorge’s syndrome. The battered child generally presents with
more than just a single recurring medical problem. β –Thalassemia would be indicated by a
microcytic anemia.
126- A mother who recently immigrated to the United States from Mexico brings her 4½-year-
old son to your clinic for his pre- kindergarten examination. The child’s examination is normal
except for a hemangioma located on his left arm. His mother says that the lesion appeared at
about 4 weeks of age, continued to grow until he was about 5 months of age, and then began
to flatten, shrink, and fade. She is concerned because it has not improved in the past 18
months. When you examine the lesion more closely you note telangiectasia, fibrofatty tissue,
dyspigmentation, and scarring where involution has occurred.
Which one of the following would be the most appropriate management?
A) Oral propranolol
B) Corticosteroid injection
C) Referral for laser therapy
D) Referral for surgical excision

ANSWER: D
Infantile hemangiomas usually appear by 4 weeks of age and stop growing by 5 months of
age. As manyas 70% leave residual skin changes, including telangiectasia, fibrofatty tissue,
redundant skin, atrophy, dyspigmentation, and scarring. Systemic corticosteroids were the
mainstay of treatment for hemangiomas during infancy until 2008, when the FDA approved
oral propranolol for this indication. Intralesional corticosteroids can be effective for small,
bulky, well localized lesions in infants. Laser therapy can also be used to treat early lesions or
residual telangiectasia. Once involution is complete, however, as is the case with this child,
elective surgical excision is the treatment of choice, producing better outcomes.
Children Emergencies
2019 pretest
176. A 2-year-old male is brought to your office by his mother. The child has a 2-day history
of a runny nose and mild cough associated with a subjective fever. The cough worsened last
night. The patient has had a reduced appetite but a good intake of fluids.
On examination the child has an axillary temperature of 37.4°C (99.3°F), a heart rate of 120
beats/min, a respiratory rate of 26/min, a weight of 16 kg (35 lb), and an oxygen saturation of
96% on room air. He appears mildly ill but is alert and does not show any signs of distress,
and has a prominent high-pitched barking cough. You note that he has clear rhinorrhea, the
tympanic membranes are normal, and the oropharynx is moist and clear. Auscultation reveals
inspiratory stridor, but there are no signs of respiratory distress. The patient’s skin has good
turgor with no rash.
Which one of the following would be the most appropriate next step in the management of this
child?
A. Administration of dexamethasone, 0.6 mg/kg orally in a single dose
B. Initiation of oral amoxicillin, 40 mg/kg twice daily
C. Administration of albuterol, 2.5 mg/3 mL via nebulizer
D. Administration of racemic epinephrine 2.25% solution (Asthmanefrin), 0.5 mL via
nebulizer
E. Transfer to a hospital emergency department for stabilization and hospital admission

ANSWER: A
This patient has mild croup based on the clinical findings. A single dose of dexamethasone is
recommended in all cases of croup (SOR A). Hospitalization is not necessary if the child is
stable. Racemic epinephrine, which has been shown to reduce symptoms at 30 minutes but not
at 2 hours or 6 hours, is recommended for the treatment of moderate to severe croup when
patients are being observed in a medical setting such as the emergency department or hospital
(SOR A). Amoxicillin and albuterol are not indicated in the management of croup.

ABFM 2020
190. A fully immunized 7-month-old male is brought urgently to your office after his parents
noted a possible seizure. The mother says that the infant began to “shake all over” for about 3–
4 minutes and then promptly fell asleep for about 20 minutes. When he awoke he was alert but
fussy and crying. He has been ill for the last few days with a cough, congestion, decreased oral
intake, and fevers up to 101°F. On examination he has an oral temperature of 38.3°C
(100.9°F), a heart rate of 170 beats/min, a respiratory rate of 50/min, and an oxygen saturation
of 97% on room air. The infant is fussy but consolable. His mucous membranes are moist, his
tympanic membranes are clear, and he has a normal oropharynx.

He has clear rhinorrhea. Examination of the heart is normal, and examination of the lungs
reveals rhonchi and wheezes. He is moving all of his extremities normally.
Which one of the following would be the most appropriate initial step in the evaluation of this
child?
1. A basic metabolic panel
2. Radiography of the chest
3. MRI of the brain
4. Electroencephalography
5. A lumbar puncture

ANSWER: B
Children who have a simple febrile seizure and appear neurologically intact do not require
routine testing except to determine the source of their fever (SOR C). This child has signs of
possible pneumonia so a chest radiograph would be warranted to look for the source of
infection that triggered the fever.
Routine laboratory testing is not indicated in the workup of simple febrile seizures. There is a
low risk that these children will have low sodium or glucose levels, and this would not predict
seizure recurrence. Routine neuroimaging such as MRI is not recommended for febrile
seizures. Electroencephalography is not useful for predicting the recurrence of simple febrile
seizures and would not be indicated in the workup of these seizures. A lumbar puncture is
indicated only in cases where the child has neurologic findings suggestive of meningitis, but
that is not the case for this child.

179. A 6-month-old female is brought to the emergency department by her parents with a 4-
day historyoffever,congestion,cough,shortnessofbreath,anddecreasedappetite. Onexamination
of the lungs you note wheezes and crackles throughout. You also note subcostal retractions, an
oxygen saturation of 91% on room air, and a respiratory rate of 42/min. A chest radiograph
reveals peri-bronchial markings with no infiltrates. The parents say that the patient’s teenage
cousin visited recently and was having “cold symptoms.” You suspect bronchiolitis.
Which one of the following would you recommend next?
1. Maintaining hydration and keeping oxygen saturation >90%
2. Deep nasal suctioning
3. Albuterol via nebulizer
4. Broad-spectrum antibiotics
5. Systemic corticosteroids
ANSWER: A
Bronchiolitis is a common lower respiratory tract infection in young children and infants.
Respiratory syncytial virus (RSV) is the most common cause. Supportive care with hydration
and maintenance of oxygen saturation is important in the treatment of RSV bronchiolitis.

Infants with respiratory rates >60/min are often unable to manage oral hydration due to the
risk of aspiration. In these cases, intravenous or nasogastric feeds are acceptable. An oxygen
saturation >90% is sufficient in RSV bronchiolitis and use of supplemental oxygen to achieve
higher levels of oxygen saturation may prolong hospital stays. There is no clear advantage to
deep nasal suctioning, which may also be associated with prolonged hospital stays. Routine
nasal suctioning is indicated, however. Bronchodilators are not recommended in the treatment
of RSV (level of evidence A). Antibiotics are only indicated with a confirmed bacterial co-
infection (level of evidence B). Systemic corticosteroids have shown no benefit in the
treatment of bronchiolitis.

178. A healthy 30-month-old male is brought to your office because of right elbow pain that
began acutely today when his father grabbed his wrist as he fell off a stool at home. There are
no other symptoms. An examination is notable for the patient’s resistance to move his right
arm, which he holds partially flexed and pronated at the elbow. The affected arm is otherwise
normal.
Which one of the following would you recommend at this time?
A) Splinting and close follow-up
B) Attempted reduction of the subluxed radial head now
C) C. A radiograph of the right elbow
D) Referral to physical therapy
E) Referral to an orthopedic surgeon

ANSWER: B
Nursemaid’s elbow is a common injury in young children, often occurring with sudden
upward traction on the arm that subluxes the radial head. Patients typically report acute elbow
pain. Many children hold the elbow partially flexed and pronated and decline to move the
elbow. A clinical diagnosis is made when the history and examination are typical for the
condition, as is the case here. Manual reduction in the office is the initial treatment and
succeeds more than 70% of the time. Splinting is not a first-line treatment. Imaging and
referral are not indicated before attempting reduction.
170. A 20-year-old female with no significant past medical history is brought to your office
with a sudden onset of hives, shortness of breath, dizziness, and vomiting. The patient is in
anaphylaxis. Intramuscular administration of epinephrine should be given immediately at
which one of the following sites?
A. Mid-outer aspect of the shoulder
B. Mid-outer aspect of the buttocks
C. Mid-outer aspect of the thigh
D. Mid-outer aspect of the calf
E. Lower medial abdominal wall

ANSWER: C

In the event of anaphylaxis, epinephrine should be given in the mid-outer aspect of the thigh
(anterolateral aspect of the vastus lateralis, mid-muscle belly). Epinephrine is more quickly
absorbed and produces higher tissue and plasma levels when injected in the vastus lateralis
than in other muscles. It should not be injected subcutaneously.
CSF analysis
None
Antenatal care
ABFM 2020
-A 25-year-old female at 28 weeks gestation comes to your office for a routine prenatal visit.
She received Tdap vaccine 3 years ago after she was bitten by a horse. Which one of the
following would be most effective to reduce the newborn’s risk of contracting pertussis?
A. Tdap vaccination of the mother now
B. Tdap vaccination of the mother post- partum
C. No Tdap vaccination of the mother now or post-partum, and Tdap vaccination of
other family members
D. No Tdap vaccination of the mother now or post-partum, and Tdap vaccination
of the newborn within 72 hours of birth
E. No Tdap vaccination of the mother now or post-partum, and a recommendation for
good hand hygiene to protect the newborn

ANSWER: A
The Advisory Committee on Immunization Practices recommends Tdap vaccination during
each pregnancy, regardless of the time interval since the last booster, primarily to confer
immunity against pertussis to the infant. For that reason, administration of the vaccine is
recommended between 27- and 36-weeks' gestation to maximize the concentration of pertussis
antibody transferred to the fetus. Postpartum vaccination of the mother with Tdap does not
provide as much protection as vaccination of the mother during pregnancy. Newborns should
not receive Tdap but should begin pertussis immunization at 2 months of age with the
firstdose of DTaP. There is no recommendation to vaccinate infants earlier to reduce the risk
of vertical transmission. Family members and close contacts should be vaccinated against
pertussis at least 2 weeks prior to contact with the infant, but this strategy alone is less
effective than maternal immunization. Good hand hygiene is important when caring for an
infant, but on its own is not an effective strategy to reduce the risk of pertussis transmission.

Which one of the following should NOT be consumed during pregnancy due to a potentially
high mercury content?
A. Catfish
B. Crawfish, shrimp, and lobster
C. Flounder and haddock
D. Salmon and trout
E. Shark and swordfish

ANSWER: E
Larger ocean fish that consume other fish may accumulate mercury levels that can cause
neurologic problems when consumed, so these fish should be avoided by children and
pregnant or nursing women. Shark and swordfish are among the fish with the highest mercury
content. Catfish, crawfish, shrimp, lobster, flounder, haddock, salmon, and trout have the least
amount of mercury.

-You are advising a group of medical students who are planning a tobacco cessation program
for expectant mothers. The medical students want to build an advertising program that touts
the pregnancy benefits of tobacco cessation. You tell the medical students that evidence
suggests that tobacco cessation in pregnant women:
A. Decreases the risk for cesarean delivery
B. Decreases the risk for preeclampsia
C. Decreases the need for epidural anesthesia
D. Increases infant birth weight
E. Increases the risk for preterm delivery

ANSWER: D
Smoking during pregnancy increases the risk for fetal growth restriction. The U.S. Preventive
Services Task Force identified evidence that tobacco cessation increases infant birth weight
and decreases the risk for preterm delivery. There is no evidence that tobacco cessation in
pregnant women decreases the risk for cesarean delivery, the risk for preeclampsia, or the
need for epidural anesthesia.

Infertility
None
Menopause
ABFM 2020
146. A65-year-old female sees you to establish primary care. Her past medical history is
significant only for hypertension, which is well managed on hydrochlorothiazide, and
menopausal symptoms managed with oral estrogen/progestogen for the past 10 years. She has
no current health concerns and her vital signs are normal. She requests a refill of her hormone
medication. Which one of the following would be most appropriate at this time?
A. Refill her hormone medication, but with a tapering schedule to end the therapy
B. Recommend switching from oral hormone therapy to vaginal estrogen and oral
progestogen
C. Discuss the possible risks of continued hormone therapy before deciding whether she
should continue
D. Order a bone density test and use the results to determine whether to continue hormone
therapy
E. Recommend discontinuation of hormone therapy as soon as possible ,as she has already
been on the treatment for more than 5 years

ANSWER: C
This patient has been on menopausal hormone therapy for an extended period of time. A
discussion about cessation is warranted as it is common practice to use hormones for the
shortest duration possible. The American College of Obstetricians and Gynecologists does not
recommend discontinuation based on age or duration of treatment alone, however,
recommending instead that the patient history and symptoms be taken into account. Tapering
hormones versus abrupt discontinuation of hormones is not well studied, and patients can do
well with either plan. Vaginal estrogen should be used with similar cautions as oral estrogen
and is primarily indicated for genitourinary symptoms of menopause, so switching from oral
estrogen to vaginal estrogen would not be indicated in this case. Menopausal hormone therapy
does decrease the risk for hip fractures, but the American Academy of Family Physicians, like
the U.S. Preventive Services Task Force, does not recommend using hormone therapy for
prevention of chronic conditions, so the patient’s bone density should not be a consideration.

32-A 50-year-old female reports vaginal dryness, burning, and pain with penetration during
sexual intercourse. On examination she is noted to have pale, dry vaginal epithelium that is
smooth and shiny with loss of most rugation. Which one of the following treatments is most
likely to be effective for her sexual dysfunction?
A) Cognitive-behavioral therapy
B) Vaginal estrogen
C) Testosterone therapy
D) Bupropion (Wellbutrin)
E) Sildenafil (Viagra)

ANSWER: B
This patient has genitourinary syndrome of menopause (formerly termed vulvovaginal
atrophy) based on her symptoms and examination. Estrogen therapy is highly effective for
dyspareunia related to genitourinary syndrome of menopause, with the vaginal route preferred
over systemic therapy if vaginal dryness is the primary concern. Bupropion and sildenafil may
benefit women with sexual dysfunction induced by antidepressant medications. Data on the
benefit of testosterone therapy is limited and inconsistent and lacks long-term information
about safety. Cognitive-behavioral therapy has been shown to effectively treat low sexual
desire, but does not affect the physiologic changes associated with genitourinary syndrome of
menopause.

40-A 34-year-old female consults you because of excessive body and facial hair. She has a
normal body weight, no other signs of virilization, and regular menses. She had a bilateral
tubal ligation 4 years ago. Which one of the following would be the most appropriate
treatment for her mild hirsutism?
A) Leuprolide
B) Metformin (Glucophage)
C) Prednisone
D) Spironolactone (Aldactone)

ANSWER: D
Antiandrogens such as spironolactone, along with oral contraceptives, are recommended for
the treatment of hirsutism in premenopausal women (SOR C). Women should avoid becoming
pregnant while on spironolactone because of the potential for teratogenic effects. In addition
to having side effects, prednisone is only minimally helpful for reducing hirsutism by
suppressing adrenal androgens. Leuprolide, although better than placebo, has many side
effects and is expensive. Metformin can be used to treat patients with polycystic ovary
syndrome, but this patient does not meet the criteria for this diagnosis.

Semen analysis
ABFM 2019
2. An otherwise healthy 57-year-old male presents with mild fatigue, decreased libido, and
erectile dysfunction. A subsequent evaluation of serum testosterone reveals hypogonadism.
Which one of the following would you recommend at this time?
A. No further diagnostic testing
B. A prolactin level
C. A serum iron level and total iron binding capacity
D. FSH and LH levels
E. Karyotyping
ANSWER: D
In men who are diagnosed with hypogonadism with symptoms of testosterone deficiency and
unequivocally and consistently low serum testosterone concentrations, further evaluation with
FSH and LH levels is advised as the initial workup to distinguish between primary and
secondary hypogonadism. If secondary hypogonadism is indicated by low or inappropriately
normal FSH and LH levels, prolactin and serum iron levels and measurement of total iron
binding capacity are recommended to determine secondary causes of hypogonadism, with
possible further evaluation to include other pituitary hormone levels and MRI of the pituitary.
If primary hypogonadism is found, karyotyping may be indicated for Klinefelter’s syndrome.

Nail disorders
None
Hair disorders
ABFM 2020
152. A48-year-old female presents with concerns about hair loss. She has noticed gradual
thinning of the hair on the top of her head for the last year. Her scalp is now visible through
the hair. She is not taking any oral medications. A levonorgestrelIUD (Mirena) was placed 4
years ago. She has not had any recent illnesses or stressors and are view of systems is negative
.Her mother had similar hair loss starting in her fifties .On examination you note thin hair on
the top of the scalp, an intact frontal hair line, and growth of thin, wispy hairs on her crown.
Which one of the following would be most appropriate for this patient?
A. Removal of the levonorgestrel IUD
B. Spironolactone (Aldactone), 50 mg daily for 6 months
C. Finasteride (Proscar), 5 mg daily indefinitely
D. Minoxidil (Rogaine) 2% solution for 6 months
E. Minoxidil 5% foam indefinitely

ANSWER: E

Hair thinning on the crown of the head with the presence of small, wispy hairs among the
regular hair is characteristic of female pattern hair loss (FPHL). A family history of similar
issues is often present but not necessary for the diagnosis. Topical minoxidil is the mainstay of
treatment for FPHL (SOR A). It is available in a 2% solution or 5% foam for women (the 5%
solution is indicated only for men). Treatment for FPHL as well as the male equivalent,
androgenic alopecia, must be continued long term. With treatment there is often an initial
period of increased hair loss. Regrowth is noticeable around 6 months. Discontinuation of
treatment results in loss of regrown hair. There is no clear association between hormone status
and FPHL. Removal of this patient’s levonorgestrel IUD is unlikely to affect hair loss.
Spironolactone has also been used for FPHL but evidence is lacking regarding its
effectiveness. Finasteride is approved by the FDA only for males with hair loss. There is a
high risk of teratogenicity with its use. It has been used in women but evidence of efficacy is
minimal.
122. The initial treatment of choice for head lice is :
A. lindane 1% shampoo
B. Permethrin 1% shampoo (Nix)
C. Pyrethrum 0.3%/piperonyl butoxide 4% shampoo (Rid)
D. Ivermectin 0.5% lotion (Sklice)
E. Malathion 0.5% lotion (Ovide)
ANSWER: B
Permethrin 1% shampoo is recommended as first-line treatment for head lice. Lindane should
not be used because of neurotoxicity. Pyrethrum 0.3%/piperonyl butoxide 4% shampoo,
ivermectin 0.5% lotion, and malathion 0.5% lotion are alternative treatments but should not be
used unless two treatments with permethrin are unsuccessful.

Liver function test interpretation + Hepatitis markers


ABFM 2020
A 42-year-old male sees you because of abnormal laboratory results from a recent health fair.
Laboratory Findings:

Based on these results, which one of the following is the most likely cause of his elevated liver
transaminase levels?
A. Acute viral hepatitis
B. Alcoholic liver disease
C. Cholestasis
D. Nonalcoholic fatty liver disease
ANSWER: B
An AST/ALT ratio >2 is highly suggestive of alcoholic liver disease. Acute viral hepatitis
usually presents with AST and ALT levels >25 times the upper limit of normal. Cholestasis is
unlikely to be the cause in a patient with normal alkaline phosphatase and bilirubin levels.
Although nonalcoholic fatty liver disease is the most common cause of asymptomatic elevated
liver transaminase levels in the United States, it is usually associated with an AST/ALT ratio

118. A 67-year-old female with a history of type 2 diabetes, hypertension, osteoarthritis, and
hyperlipidemia sees you for follow-up of laboratory results after hepatic steatosis and mild
splenomegaly were noted on abdominal ultrasonography performed during a recent
emergency department visit for right upper quadrant pain. She is currently asymptomatic. Her
vital signs include a blood pressure of 128/72 mm Hg and a BMI of 34 kg/m2. An
examination is significant only for trace bilateral lower extremity edema. A cardiovascular
examination is otherwise normal and neurologic and abdominal examinations are normal.
Laboratory Findings
Platelets 112,000/mm3 (N 150,000–450,000)
Creatinine 0.8mg/dL(N0.6–1.2)
HemoglobinA1c 6.6%
AST 68 U/L (N 8–48)
ALT 55 U/L (N 7–55)
Albumin 3.2g/dL (N3.5–5.0)
Bilirubin 0.5mg/dL (N0.1–1.2)
INR 1.0 (N <1.2)

Which one of the following medications should be AVOIDED in this patient to reduce the
likelihood of complications of her medical condition?
A. Acetaminophen
B. Atorvastatin (Lipitor)
C. Ibuprofen
D. Lisinopril (Prinivil, Zestril)
E. Metformin (Glucophage)
ANSWER: C
This patient has previously unrecognized compensated hepatic cirrhosis. While the diagnosis
of cirrhosis should be confirmed and assessed by methods such as transient elastography, the
family physician should recognize this as presumed cirrhosis based on the splenomegaly and
laboratory findings. NSAIDs such as ibuprofen should be avoided in patients with cirrhosis
due to the risk of renal insufficiency (SOR B). While toxic to the liver at high doses,
acetaminophen can be safely used for analgesia in cirrhotic patients, though many
hepatologists recommend limiting dosing to 2 g daily (SOR C). Statins can be safely used in
compensated cirrhosis and may improve steatosis (SOR A). Lisinopril and metformin can also
be used safely, hypertension should not be overtreated as cirrhosis progresses to a
decompensated state.

140. A patient born in 1954 requests screening for hepatitis C. His anti-HCV antibody screen
is positive. Which one of the following would be most appropriate at this point?
A. A confirmatory second anti-HCV test in 4–6 weeks
B. A qualitative HCV RNA test
C. Quantitative HCV genotyping
D.Treatment with a dual antiviral regimen pending additional testing
E. Referral for a percutaneous liver biopsy

ANSWER: B
The CDC recommends that all adults 18 years of age or older receive a one-time screening for
hepatitis C virus (HCV). Persons with risk factors for HCV exposure should be screened
periodically, based on risk level. If the anti-HCV antibody screen is positive, then a qualitative
HCV RNA test is the next step (SOR C). Prior to initiating treatment, a quantitative HCV
RNA and genotype testing is necessary (SOR A). In addition, assessing the degree of fibrosis
will provide information regarding the urgency of treatment. Percutaneous liver biopsy is
generally the preferred evaluation after obtaining quantitative and genotype results, all of
which can guide treatment decisions.

ABFM 2017
72-A 52-year-old male sees you for the first time for a health maintenance visit. He previously
tested positive on a hepatitis C antibody test performed by another physician. He seeks your
opinion regarding follow-up testing. Which one of the following tests would you suggest at
this point?
A) -Fetoprotein
B) Hepatitis B antigen
C) Hepatitis C RNA
D) Ultrasonography of the liver
E) A liver biopsy

ANSWER: C
Screening for hepatitis C virus (HCV) with an anti-HCV antibody test is recommended for all
adults at high risk of infection, and one-time screening is recommended in adults born
between 1945 and 1965. If the anti-HCV antibody test result is positive, current infection
should be confirmed with a qualitative HCV RNA test.
Stool Analysis & Cultures
None
Dizziness
Pretest2019
Q1-A 33-year-old woman is seeing you with a chief complaint of “dizziness.” Upon further
characterization, she describes a “spinning” sensation and a sense of “falling forward.” Based
on this description, which of the following terms should be used to characterize her
complaint?
a. Vertigo
b. Orthostasis
c. Presyncope
d. Dysequilibrium
e. Light-headedness
The answer is a. “Dizziness” is a subjective symptom, often meaning different things to
different people. It is imperative that this complaint be better characterized to develop an
appropriate differential diagnosis and treatment plan. Vertigo is a rotational sensation, in
which the room spins around the patient. Patients will often say that the room is “spinning” or
that they feel a sense of falling forward or backward. Orthostasis refers to light-headedness
upon arising, common with orthostatic hypotension. Presyncope is a feeling of impending
faint. Disequilibrium is a sensation of unsteadiness, or a loss of balance. Light-headedness is
often vaguely described as a “floating” sensation.

Q1.1The patient described in the above question reports that this is not the first time she’s had
this symptom. She says that the symptoms are often precipitated when she rolls over in bed.
Symptoms occur a few seconds after a change in head movement, and last a few minutes per
spell. This time, her symptoms have been occurring for 3 days. Which of the following would
you recommend as first-line treatment?
a. Oral meclizine
b. Oral diazepam
c. Oral valproic acid
d. Intratympanic corticosteroid injections
e. Physical therapy directed at the condition
The answer is e. The patient described has benign positional vertigo (BPV). While symptom
relief can occur with medications, treatment involves physical therapy protocols(including the
Epley maneuver or Brandt-Daroff exercises). Oral meclizine and diazepam may be helpful to
treat other causes of vertigo, including Meniere Syndrome, acute labyrinthitis, vestibular
neuritis, or traumatic vertigo. Migraine prophylactic therapies including β-blockers or
valproate may be used to treat migrainous vertigo, but would be less helpful here.
Intratympanic corticosteroids are used for refectory cases of Meniere Syndrome
Q2-A 42-year-old woman is seeing you to follow-up with a new complaint of “dizziness.” She
reports that symptoms first began several months ago. At that time, she reported a subjective
hearing loss and a ringing in her left ear only.
Symptoms were mild, and her physical examination was normal, so you elected to follow her.
Since that time, her symptoms have progressed to include dizziness and some facial
numbness. Which of the following is her most likely diagnosis?
a. Vestibular neuronitis
b. Benign positional vertigo
c. Acoustic neuroma
d. Meniere disease
e. Cerebellar tumor

The answer is c.
Acoustic neuromas typically present with unilateral tinnitus and hearing loss. The symptoms
are constant and slowly progressive. They are among the most common intracranial tumors.
Most are unilateral, with benign lesions arising within the internal auditory canal, and
gradually growing to involve the cerebellopontine angle. Nonclassic presentations are fairly
common, and any individual with a unilateral or asymmetric sensorineural hearing loss should
be evaluated. Vestibular neuronitis presents with an acute onset of severe vertigo lasting
several days, with symptoms improving over several weeks. Benign positional vertigo
typically involves symptoms with position changes only.
Meniere disease presents with discrete attacks of vertigo lasting for several hours, associated
with nausea and vomiting, hearing loss, and tinnitus. A cerebellar tumor would typically
present with disequilibrium as opposed to tinnitus.

Q3-In the evaluation of a 55-year-old man complaining of dizziness, you perform the
DixHallpike (Nylen-Barany) maneuver several times. You had the patient sit on the edge of the
examining table and lie down suddenly with the head hanging 45 degrees backward and
turned to either side. With this maneuver, the vertigo was reproduced immediately and
symptoms did not lessen regardless of repetition.
The direction of the nystagmus changed with changing the direction that the head is turned, and
the symptoms were of mild intensity. Which of the following is the most likely cause of the
vertigo?
a. Stroke
b. Vestibular neuronitis
c. Benign positional vertigo
d. Meniere disease
e. Acoustic neuroma

The answer is a.
The Dix-Hallpike maneuver, described in the question, is often useful to distinguish central
from peripheral causes of vertigo. With a peripheral cause of vertigo, the latency time for the
onset of symptoms of vertigo or nystagmus is 3 to 10 seconds, the symptoms are severe, and
the direction of the nystagmus is fixed. In addition, repeating the maneuver lessens the
symptoms. With a central cause of vertigo, there is no latency to onset of symptoms, no
lessening of symptoms with repeat maneuvers, the direction of the nystagmus changes, and the
symptoms are of mild intensity. Of the above answers, all are peripheral causes of vertigo,
except the correct answer, stroke.

Q4-You are caring for a 26-year-old man with vertigo. You have diagnosed him with a
peripheral vestibular disorder, and are considering treatment options.
Which of the following would be the first-line therapy? a. NSAIDs
b. Antihistamines
c. Antiemetics
d. Antibiotics
e. Benzodiazepines

The answer is b.
Once diagnosed with a peripheral vestibular disorder, antihistamines are the first- line therapy
for symptomatic relief. They suppress the vestibular end-organ receptors and inhibit activation
of the vagal response. Meclizine (Antivert), 25 mg orally every 4 to 6 hours and
diphenhydramine (Benadryl), 50 mg orally every 4 to 6 hours are commonly recommended
choices. Antiemetics may be used if nausea and vomiting are prominent symptoms.
Benzodiazepines may be helpful in symptom reduction, but are usually second-line agents.
NSAIDs and antibiotics are not helpful.
ABFM2020
104. A 63-year-old female presents to your office with a sudden onset of lightheadedness and
mild nausea that she first noted when getting out of bed this morning. She has had repeated
episodes of a spinning sensation when tilting her head up or down. Her symptoms have been
so severe that she could not go to work today. She has a history of essential hypertension that
is well controlled on hydrochlorothiazide. She has not had any headache, hearing loss,
tinnitus, or recent illness or trauma. She has a temperature of 36.8°C (98.2°F), a blood
pressure of 136/80 mm Hg, a heart rate of 80 beats/min, a respiratory rate of 12/min, and an
oxygen saturation of 96% on room air. You perform the Dix-Hallpike maneuver with her right
ear down and in the dependent position and note a latent torsional, upbeating nystagmus.

The most appropriate intervention at this time would be


A. Prolonged upright positioning
B. Canalith repositioning procedures
C. Vestibular function testing
D. Vestibular suppressant medication
E. MRI of the brain
ANSWER: B
This patient presents with symptoms consistent with right benign paroxysmal positional
vertigo (BPPV) and torsional, upbeating nystagmus provoked by the Dix-Hallpike maneuver.
Diagnostic criteria include both patient history and physical examination findings. Symptoms
suggesting BPPV include an acute onset of brief episodic vertigo triggered by positional
changes relative to gravity. In the 2017 clinical practice guidelines on BPPV, the American
Academy of Otolaryngology–Head and Neck Surgery Foundation strongly recommended
accurate diagnosis
60 of posterior semicircular canal BPPV by performing the Dix-Hallpike maneuver (B
recommendation). The guidelines also strongly recommend treatment with a canalith
repositioning procedure (A recommendation).
Positional restrictions, particularly postprocedural restrictions, are not recommended due to
insufficient evidence (A recommendation). Vestibular testing should also be avoided in
patients who meet diagnostic criteria for BPPV and who do not have other vestibular
symptoms such as hearing loss or tinnitus (C recommendation). Vestibular suppressant
medication such as antihistamines and benzodiazepines should be avoided due to potential
risks and lack of evidence to suggest that these medications are as effective as repositioning
procedures (B recommendation). Radiographic imaging such as MRI of the brain or CT of the
head should not be obtained in patients who meet diagnostic criteria for BPPV and who do not
have other signs or symptoms of neurologic pathology (C recommendation)
ABFM2019
Q1-A 43-year-old male who works in a warehouse sees you because of dizziness. He first
noticed mild dizziness when he rolled over and got out of bed this morning. He had several
more severe episodes that were accompanied by nausea, and on one occasion vomiting
occurred after he tilted his head upward to look for items on the higher shelves at work. You
suspect benign paroxysmal positional vertigo, so you perform the Dix-Hallpike maneuver as
part of the examination.
Which one of the following findings during the examination would confirm the diagnosis?
A) Nystagmus when vertigo is elicited
B) Vertigo that occurs immediately following the test-related head movement
C) Persistence of vertigo for 5 minutes following the test-related head movement
D) A drop in systolic blood pressure of >10 mm Hg when supine

ANSWER: A
Benign paroxysmal positional vertigo (BPPV) originates in the posterior semicircular canal in
the majority of patients (85%–95% range reported). The Dix- Hallpike maneuver, which
involves moving the patient from an upright to a supine position with the head turned 45° to
one side and the neck extended 20° with the affected ear down, will elicit a specific series of
responses in these patients.
Following a latency period that typically lasts 5–20 seconds but sometimes as long as 60
seconds, the patient will experience the onset of rotational vertigo. The objective finding of a
torsional, upbeating nystagmus will be associated with the vertigo. The vertigo and nystagmus
typically increase in intensity and then resolve within 1 minute from onset.

.
ABFM2018
Q2-An otherwise healthy 64-year-old male comes to your office accompanied by his wife
because of tinnitus that has affected both ears for the last 3 years. It has been most
troublesome at bedtime. His wife says that he is becoming irritable and depressed because he is
bothered by the buzzing in his ears many times during the day. His only medication is
allopurinol (Zyloprim) for the prevention of gout. The most likely identifiable cause of this
patient’s tinnitus is:
A) medication
B) Meniere’s disease
C) temporomandibular joint dysfunction
D) sensorineural hearing loss
E) impacted cerumen

ANSWER: D
Although tinnitus is idiopathic, sensorineural hearing loss is the most common identified
cause. It can also be caused by other otologic, vascular, neoplastic, neurologic, pharmacologic,
dental, and psychological factors. Almost all patients with tinnitus should undergo audiometry
with tympanometry, and some patients require neuroimaging or assessment of vestibular
function with electronystagmography. Counseling may also improve the chances of successful
subsequent treatment. Several medications can cause tinnitus, but allopurinol is not one of
them.
Eyelid & Lacrimal disorders

ABFM2020
175. A12-year-oldfemaleisbroughttoyourofficebecauseofalesiononherleftlateral upper eyelid
(shown below).
The lesion started 3 days ago as a small, red “pimple,” and since then it has increased in size
and is tender. She does not have any fever, tearing, or conjunctival irritation. No treatment has
been attempted. Which one of the following would be the most appropriate initial
management?
A. Warm, damp compresses for 10 minutes four times daily
2. Topical erythromycin ophthalmic ointment (Ilotycin)
3. Topical hydrocortisone 0.5%
4. Systemic antibiotics with coverage for staphylococci
5. Incisional drainage using a sterile 18- or 20-gauge needle
ANSWER: A
This patient presents with an external hordeolum or stye. Warm, damp compresses for 10
minutes four times a day would be the best initial management. Topical erythromycin
ointment and topical hydrocortisone cream are not indicated for treatment of a stye. Although
staphylococci are commonly involved in this process, antibiotics are not recommended unless
there is evidence of adjoining cellulitis. Warm compresses allow for spontaneous drainage and
resolution. Anti-inflammatory medications are not recommended for hordeolum externum
management, however they could become necessary if it becomes a chalazion. If the
hordeolum has not resolved in about 1 week, incision may be necessary.

ABFM2019
Q1. A 67-year-old male presents to your office for evaluation of chronic redness, flaking, and
discomfort of his eyelids. Additionally, his eyes feel irritated, dry, and sandpapery at times. He
has had difficulties with these symptoms on and off throughout his life but they have
worsened lately. He has not had any vision changes and does not wear contact lenses.
On examination his eyelids appear red and mildly swollen with yellow crusting at the bases of
the eyelashes. You note bilateral mild conjunctival injection. Visual acuity is intact, as are
pupil reactions and extraocular movements. Which one of the following treatments is
appropriate first-line therapy for this condition?
A. Warm compresses and gentle cleansing with a mild shampoo
B. Sodium sulfacetamide eye drops
C. Topical betamethasone
D. Oral acyclovir (Zovirax)
E. Oral cephalexin (Keflex)
ANSWER: A
This patient has blepharitis, a chronic inflammation of the eyelids. Seborrhea is a common
cause in older adults. In younger patients including children, colonization with Staphylococcus
may be a contributing factor. Meibomian gland dysfunction is often part of this condition,
contributing to a reduced quality of tear films, which leads to dry eyes and irritation. Other
diagnoses to consider in this patient include conjunctivitis, preseptal cellulitis, and Sjögren’s
syndrome. Conjunctivitis typically involves the conjunctiva and an eye discharge but less
involvement of the eyelids is present. Cellulitis is an acute rather than chronic condition and
involves more pain and swelling. Sjögren’s syndrome causes dry eye but not inflammatory
changes of the lid. The initial treatment of blepharitis consists of lid hygiene using warm
compresses to remove dried secretions and debris. Mild shampoo can help in this process and
aid in keeping the bacterial colonization load down. In severe or recalcitrant cases a topical
antibiotic ointment may be applied to the lids. Oral antibiotics can be considered for more
severe cases.
Eyelid & Lacrimal disorders
ABFM2020
175. A12-year-old female is brought to your office because of a lesion on her left lateral upper
eyelid (shown below). The lesion started 3 days ago as a small, red “pimple,” and since then it
has increased in size and is tender. She does not have any fever, tearing, or conjunctival
irritation. No treatment has been attempted. Which one of the following would be the most
appropriate initial management?
A. Warm, damp compresses for 10 minutes four times daily
6. Topical erythromycin ophthalmic ointment (Ilotycin)
7. Topical hydrocortisone 0.5%
8. Systemic antibiotics with coverage for staphylococci
9. Incisional drainage using a sterile 18- or 20-gauge needle
ANSWER: A
This patient presents with an external hordeolum or stye. Warm, damp compresses for 10
minutes four times a day would be the best initial management. Topical erythromycin
ointment and topical hydrocortisone cream are not indicated for treatment of a stye. Although
staphylococci are commonly involved in this process, antibiotics are not recommended unless
there is evidence of adjoining cellulitis. Warm compresses allow for spontaneous drainage and
resolution. Anti-inflammatory medications are not recommended for hordeolum externum
management, however they could become necessary if it becomes a chalazion. If the
hordeolum has not resolved in about 1 week, incision may be necessary.

ABFM2019
Q1. A 67-year-old male presents to your office for evaluation of chronic redness, flaking, and
discomfort of his eyelids. Additionally, his eyes feel irritated, dry, and sandpapery at times. He
has had difficulties with these symptoms on and off throughout his life but they have
worsened lately. He has not had any vision changes and does not wear contact lenses.
On examination his eyelids appear red and mildly swollen with yellow crusting at the bases of
the eyelashes. You note bilateral mild conjunctival injection. Visual acuity is intact, as are
pupil reactions and extraocular movements.Which one of the following treatments is
appropriate first-line therapy for this condition?
F. Warm compresses and gentle cleansing with a mild shampoo
G. Sodium sulfacetamide eye drops
H. Topical betamethasone
I. Oral acyclovir (Zovirax)
J. Oral cephalexin (Keflex)

ANSWER: A
This patient has blepharitis, a chronic inflammation of the eyelids. Seborrhea is a common
cause in older adults. In younger patients including children, colonization with Staphylococcus
may be a contributing factor. Meibomian gland dysfunction is often part of this condition,
contributing to a reduced quality of tear films, which leads to dry eyes and irritation. Other
diagnoses to consider in this patient include conjunctivitis, preseptal cellulitis, and Sjögren’s
syndrome. Conjunctivitis typically involves the conjunctiva and an eye discharge but less
involvement of the eyelids is present. Cellulitis is an acute rather than chronic condition and
involves more pain and swelling. Sjögren’s syndrome causes dry eye but not inflammatory
changes of the lid. The initial treatment of blepharitis consists of lid hygiene using warm
compresses to remove dried secretions and debris. Mild shampoo can help in this process and
aid in keeping the bacterial colonization load down. In severe or recalcitrant cases a topical
antibiotic ointment may be applied to the lids. Oral antibiotics can be considered for more
severe cases.

Q2- A 75-year-old male presents to your office with the growth shown below that has
developed under his eye over the past several months. He says the growth is painless but
appears to be slowly enlarging. Which one of the following is the most likely diagnosis?

A. Actinic keratosis
B. Fibrous papule of the face
C. Nodular basal cell carcinoma
D. Sebaceous hyperplasia
E. Seborrheic keratosis

ANSWER: C
This patient has a nodular basal cell carcinoma, which has a pearly papular appearance with
telangiectasia. Basal cell carcinoma is the most common cutaneous malignancy. The incidence
increases with age and occurs most commonly in skin types 1 and 2.

The tumors occur most frequently on the face, scalp, ears, and neck, and less frequently on the
torso and extremities. While locally destructive, basal cell carcinoma rarely metastasizes.
The skin becomes slightly rough with actinic keratosis, and a slight scale forms gradually.
Sebaceous hyperplasia begins as elevated papules that eventually become dome-shaped and
umbilicated. It consists of small tumors made up of sebaceous glands. Fibrous papule of the
face is a variant of angiofibroma. It is usually 1–5 mm in diameter and most often appears on
the nose. Seborrheic keratoses are benign skin neoplasms that are tan or black, well
circumscribed, andhave a stuck-on appearance.
Q2- A 75-year-old male presents to your office with the growth shown below that has
developed under his eye over the past several months. He says the growth is painless but
appears to be slowly enlarging.
Which one of the following is the most likely diagnosis?
F. Actinic keratosis
G. Fibrous papule of the face
H. Nodular basal cell carcinoma
I. Sebaceous hyperplasia
J. Seborrheic keratosis

ANSWER: C
This patient has a nodular basal cell carcinoma, which has a pearly papular appearance with
telangiectasia. Basal cell carcinoma is the most common cutaneous malignancy. The incidence
increases with age and occurs most commonly in skin types 1 and 2. The tumors occur most
frequently on the face, scalp, ears, and neck, and less frequently on the torso and extremities.
While locally destructive, basal cell carcinoma rarely metastasizes.
The skin becomes slightly rough with actinic keratosis, and a slight scale forms gradually.
Sebaceous hyperplasia begins as elevated papules that eventually become dome-shaped and
umbilicated. It consists of small tumors made up of sebaceous glands. Fibrous papule of the
face is a variant of angiofibroma. It is usually 1–5 mm in diameter and most often appears on
the nose. Seborrheic keratoses are benign skin neoplasms that are tan or black, well
circumscribed, andhave a stuck-on appearance.
Otitis media
Pretest2019
Q1. The mother of a 9-month-old infant brings him in for irritability. The child has been fussy
and has not been sleeping well for 2 days. His highest temperature has been 100°F, and he has
had a clear runny nose and cough. On examination, the child is crying and irritable. Which
physical examination finding, by itself, is insufficient to diagnose acute otitis media?
a. Opaque tympanic membrane
b. Bulging tympanic membrane
c. Impaired tympanic membrane mobility
d. Erythematous tympanic membrane
e. Purulent discharge in the ear canal
The answer is d.
A reddened tympanic membrane, by itself, is not a sufficient finding to diagnose acute otitis
media. It may be due to increased intravascular pressure associated with crying. More reliable

1
findings include an opaque tympanic membrane (indicating a purulent effusion), a bulging
tympanic membrane, and impaired tympanic membrane mobility. When all three of those
characteristics are present, the positive predictive value is very high. Purulent discharge in the
ear canal may indicate a tympanic membrane perforation, and in the face of an otherwise
normal canal is more indicative of acute otitis media than otitis externa.

Q2-. You are seeing a 4-year-old male child 2 weeks after being diagnosed with left acute
otitis media. He completed his therapy, and is afebrile, acting well, and apparently back to
normal. On examination, he has a persistent effusion in theleft ear. There is no erythema,
purulence, or hearing loss. Which of the following is the most appropriate next step?

a. Reassurance and reevaluation in 2 to 4 weeks


b. Ten-day course of a second-line antibiotic
c. Regular use of a decongestant and reevaluation in 2 weeks
d. Regular use of an antihistamine and reevaluation in 2 weeks
e. Referral to an otolaryngologist
The answer is a.
Effusions may take up to 3 months to resolve. Antibiotics are not indicated for persistent
effusions in the absence of acute otitis media. Effusions persisting beyond 3 months require
evaluation by an otolaryngologist. Decongestants or antihistamines have never been
documented to help effusions, and a Cochrane review found no clinical benefit for their use. In
fact, subjects treated with decongestants or antihistamines had 11% more side effects than
nontreated subjects
Q3- You are seeing a 6-year-old patient whose mother brought him in for severe ear pain and
fever. On examination, he is febrile with a temperature of 102.5°F, and his right tympanic
membrane is shown below:Which of the following
would be the best initial treatment?

a. A weight-adjusted dose of Tylenol


b. A weight-adjusted course of amoxicillin
c. A weight-adjusted course of amoxicillin-clavulanate
d. A weight-adjusted 3-day course of azithromycin
e. A weight-adjusted 5-day course of azithromycin

The answer is c.
The picture represents acute otitis media. The child should be treated with a first- line
antibiotic. In most cases, amoxicillin is used as first-line therapy. However, in patients with
severe illness (moderate to severe otalgia and/or fever > 102°F), therapy should be initiated
with high-dose amoxicillin-clavulanate (90 mg/kg/d of amoxicillin in two divided doses).
Azithromycin is often used as a first-line choice in 1-day, 3-day, or 5-day doses, but it should
be reserved as a second-line therapy.

28. An 85-year-old nursing home patient with dementia who has bilateral hearing aids has
been slightly more confused over the past 2 weeks according to the staff. He is also
speaking at a louder volume than normal. He does not have any pain, but an examination
shows impacted cerumen in both ears.
Which one of the following would be most appropriate in the management of this patient?
A. No therapy
B. Irrigation of the ears with cold water
C. Use of olive oil drops D. Removal of the cerumen using peroxide
E. Restraining the patient and attempting manual removal of the cerumen

ANSWER: D
Removal of cerumen should be attempted when the patient has symptoms such as pain,
tinnitus, hearing loss, or itching. Removal of the impaction is also indicated in patients who
are not able to communicate about their symptoms, such as patients with developmental delay
or dementia, a nonverbal patient who has had recent behavioral changes, or children with
fever or speech delay.
Cerumen impaction resulting in hearing loss can cause reversible cognitive impairment in
older persons with dementia. Treatment options include irrigation with warm water,
cerumenolytic agents such as carbamide peroxide otic, or manual removal if the patient is
cooperative and if the procedure can be completed without the use of restraints. The use of
cold water, olive oil drops, ear candling, or cotton-tipped swabs should be avoided.
ABFM2018
An 85-year-old nursing home patient with dementia who has bilateral hearing aids has been
slightly more confused over the past 2 weeks according to the staff. He is also speaking at a
louder volume than normal. He does not have any pain, but an examination shows impacted
cerumen in both ears.
Which one of the following would be most appropriate in the management of this patient?
A. No therapy
B. Irrigation of the ears with cold water
C. Use of olive oil drops
D. Removal of the cerumen using peroxide
E. Restraining the patient and attempting manual removal of the cerumen

ANSWER: D
Removal of cerumen should be attempted when the patient has symptoms such as pain,
tinnitus, hearing loss, or itching. Removal of the impaction is also indicated in patients who
are not able to communicate about their symptoms, such as patients with developmental delay

or dementia, a nonverbal patient who has had recent behavioral changes, or children with
fever or speech delay. Cerumen impaction resulting in hearing loss can cause reversible
cognitive impairment in older persons with dementia. Treatment options include irrigation
with warm water, cerumenolytic agents such as carbamide peroxide otic, or manual removal if
the patient is cooperative and if the procedure can be completed without the use of restraints.
The use of cold water, olive oil drops, ear candling, or cotton-tipped swabs should be avoided.

Q1-An otherwise healthy 3-year-old child with no allergies is found to have otitis media with
effusion in the right ear. Which one of the following would you recommend?
A) No treatment, and follow-up in 3 months
B) Amoxicillin
C) Oral antihistamines
D) Nasal corticosteroids
E) Tympanostomy tube placement
ANSWER: A
This child has otitis media with effusion, and the recommended course of action is to follow up
in 3 months. Medications, including decongestants, antihistamines, antibiotics, and
corticosteroids, are not recommended

ABFM2015
Q2-A 3-year-old male is brought to the urgent-care clinic on a Monday morning by his mother
with a 1-day history of complaining of ear pain. The child’s mother says she has not noticed
any fever during this time. He is up to date on all immunizations and has no previous history
of ear infections or history of recent illness. The history is negative for medication allergies.
On examination the child has a temperature of 38.2°C (100.8°F) and seems to be
uncomfortable. When you examine his ears you note moderate bulging of the tympanic
membrane in both ears. All other findings are normal.According to the guidelines published
by the American Academy of Pediatrics,
which one of the following would be the most appropriate initial management?

A)Amoxicillin, 40–50 mg/kg, for 10 days


B)Amoxicillin, 80–90 mg/kg, for 10 days
C)Amoxicillin/clavulanate (Augmentin), 90 mg/kg/day of amoxicillin and 6.4 mg/kg/day
of clavulanate, divided into two doses, for 7 days
D)Cefdinir, 14 mg/kg/day for 10 days
E)Ciprofloxacin (Cipro), 10–20 mg/kg for 7 days
ANSWER: B
The American Academy of Pediatrics (AAP) recommends antibiotic therapy for children 6
months of age or older with severe signs and symptoms of acute otitis media (AOM),
including moderate or severe otalgia or otalgia for more than 48 hours, or a temperature ³39°C
(102°F), whether the AOM is unilateral or bilateral (SOR B). Children younger than 24
months without severe symptoms should receive antibiotic therapy for bilateral AOM,
whereas older children or those with unilateral AOM can be offered the option of observation
and follow-up. The usual treatment for AOM is amoxicillin, but an antibiotic with additional
$-lactamase coverage, such as amoxicillin/clavulanate, should be given if the child has
received amoxicillin within the past 30 days, has concurrent purulent conjunctivitis, or has a
history of AOM unresponsive to amoxicillin (SOR C). Penicillin-allergic patients should be
treated with an alternative antibiotic such as cefdinir, cefuroxime, cefpodoxime, or
ceftriaxone.
ABFM2014
Q3-An 18-month-old male is brought to your office by his mother. The patient is tugging at
both ears and has a temperature of 39.0°C (102.2°F). You diagnose bilateral acute otitis media
for the third time in the last 6 months. The most recent infection was 3 weeks ago and
resolution of the infection was documented after 10 days of treatment with amoxicillin.
Which one of the following antibiotic regimens would be most appropriate at this time?

A)Amoxicillin, 45 mg/kg/day for 10 days


B)Amoxicillin, 90 mg/kg/day for 10 days
C)Amoxicillin, 90 mg/kg/day for 10 days followed by prophylactic treatment with amoxicillin
for 6 months
D)Amoxicillin/clavulanate (Augmentin), 90 mg/kg/day for 10 days
E)Amoxicillin/clavulanate, 90 mg/kg/day for 10 days followed by prophylactic treatment with
amoxicillin for 6 months

ANSWER: D
Although high-dose amoxicillin (90 mg/kg/day) is recommended as the antibiotic of choice for
acute otitis media (AOM) in the nonallergic patient, amoxicillin/clavulanate is recommended
if a child has received antibiotic therapy in the previous 30 days. Prophylactic antibiotics are
not recommended, as harms outweigh benefits. Tympanostomy tubes are an option if a child
has had three episodes of AOM in the past 6 months or four episodes in the past year with at
least one episode in the past 6 months
ABFM2020
Red Eye
si
189. A 36-year-old female presents to your office with a 24-hour history of redness in her
right eye. It is associated with mild pain but no drainage. On examination her visual acuity is
20/20 bilaterally, her pupillary reflex is normal, extraocular movements are intact, and there is
no discharge noted. There is a focal area of hyperemia of the episcleral blood vessels noted
along the medial aspect of the eye. Fluorescein staining is normal.
This patient’s presentation is most consistent with which one of the following?
1. Bacterial conjunctivitis
2. Viral conjunctivitis
3. Episcleritis
4. Iritis
5. Keratitis

ANSWER: C
An acute red eye is a common presentation in primary care and it is critical to differentiate
serious causes from benign causes. Episcleritis is a self-limited condition that can be
idiopathic and presents with mild discomfort and focal hyperemia. Conjunctivitis is typically
associated with a discharge that is clear in viral cases and mucopurulent in bacterial cases.
Iritis is associated with significant pain, a poorly reactive pupil, diminished vision, and
photophobia. This patient does not have changes in visual acuity, photophobia, or severe pain
as seen in keratitis, which would also cause an abnormal fluorescein stain showing corneal
ulceration.

Pretest2019
Q1- You are seeing a 26-year-old male patient complaining of a red eye who says, “I think I
have pink eye.” He reports redness, irritation, tearing, watery discharge, and fairly intense eye
pain. Which of his reported symptoms is more suggestive of something other than
conjunctivitis?
a. Redness
b. Irritation
c. Tearing
d. Discharge
e. Pain

The answer is e.
Conjunctivitis is the most common cause of red eye seen in the primary care office.
Symptoms of conjunctivitis include increased redness, irritation, tearing, discharge, and/or
itching.
The character of the eye discharge is sometimes useful in distinguishing bacterial from viral
conjunctivitis, with bacterial causes associated with purulent discharge and viral causes
associated with more watery discharge. However, one meta-analysis recently failed to find
evidence that discharge character is diagnostically useful. Eye pain is suggestive of a more
serious problem, possibly acute angle closure glaucoma, uveitis, scleritis, keratitis, a foreign
body, or a corneal abrasion.

Q2- You are seeing a 20-year-old college student who reports that her left eye became pink
over the last 24 hours. She is otherwise healthy and takes no medications except oral
contraceptives. She reports redness, irritation, tearing, discharge, and itching. Which of her
symptoms are more specific for an allergic etiology for her condition?

a. Single eye involvement


b. Irritation
c. Tearing
d. Discharge
e. Itching

The answer is e.
Of the symptoms of conjunctivitis, itching is more specific for allergic conditions. Irritation,
tearing, and discharge are more general symptoms, and not useful in differentiating allergic
conjunctivitis from other causes. Allergic conjunctivitis is more characteristically bilateral;
therefore, single-eye involvement in this case would not point to allergic conjunctivitis.

Q3- You are caring for a 3-year-old boy who goes to daycare while his parents are at work.
His mother brought him to see you because the daycare will not take him back until he’s had a
doctor evaluate his eye symptoms. He developed an acute redness of the left eye, associated
with runny nose, cough, and increased irritability. On examination, his eye is red and watery.
The discharge is clear, and he has mild eyelid edema. Which of the following is the most
common cause for his condition?

a. Coxsackie virus
b. Parainfluenza virus
c. Adenovirus
d. Rhinovirus
e. Herpesvirus
The answer is c.
Viral conjunctivitis is most commonly caused by members of the Adenovirus family. It can
be transmitted through ocular and respiratory secretions and less commonly from fomites on
towels or equipment. Typically it starts in one eye, and spreads to the other after a few days.
The natural course is self-limiting, lasting around 10 to 14 days. It is highly contagious, and
people diagnosed should avoid close contact with others for up to 2 weeks. Supportive
treatment is indicated. Although topical antibiotics have been prescribed to try to prevent
bacterial superinfection, there is no good evidence that it makes any significant impact.

Q4- You are seeing a 32-year-old nurse who was treated in an urgent care for bacterial
conjunctivitis. Despite the appropriate use of ciprofloxacin ophthalmic solution over the last 4
days, her purulent discharge and erythema have not improved. What should be the next step in
treatment of this patient?

a. Ciprofloxacin ointment
b. Polymyxin-trimethoprim ophthalmic solution
c. Oral ciprofloxacin
d. Oral sulfamethoxazole-trimethoprim
e. Immediate ophthalmologic referral
The answer is d.
Bacterial conjunctivitis is most commonly caused by Streptococcus and Staphylococcus.
However, there are increasing reports of conjunctivitis caused by methicillinresistant S aureus
(MRSA). MRSA conjunctivitis manifests as bacterial conjunctivitis resistant to conventional
therapy, and is treated with the same drugs used to treat MRSA in other parts of the body.
Cultures should be obtained when MRSA is suspected. It is likely that the other oral or topical
antibiotics listed would not cure MRSA, and an ophthalmology referral is not necessary unless
treatment is unsuccessful.

Q5-You are seeing a 4-year-old boy whose mother brought him in to be assessed. He’s been
irritable and feverish for 3 days, with runny nose, slight cough, and ear pain. Today, he woke
up with bilateral eye redness and discharge. After evaluation and examination, you diagnose
otitis media and bilateral conjunctivitis. You are treating his otitis media with high-dose
amoxicillin therapy. Which of the following is true regarding treatment of his conjunctivitis?
a. No additional treatment is necessary.
b. Treatment with a topical antibiotic ophthalmologic solution is indicated.
c. Treatment with a topical antibiotic ophthalmologic ointment is indicated.
d. Treatment with a topical antiviral ophthalmologic solution is indicated.
e. Treatment with a topical corticosteroid is indicated.
The answer is a.
The patient described in the question has conjunctivitis-otitis media syndrome. It is a common
condition in which children with otitis media also have a purulent ocular discharge and
bilateral conjunctivitis. It responds to treatment for the otitis media, and therefore no additional
treatment for the conjunctivitis is necessary.
Q6-You are caring for a 21-year-old college junior who comes to see you for a red eye. He
noticed the problem after waking up this morning, and it caused him significant concern. He
denies eye pain, loss of vision, discharge, or trauma. He has never had an episode like this
before. He reports that he was very ill yesterday with nausea and intense vomiting for several
hours, but is feeling better today. On examination, you note a localized and sharply
circumscribed bright red patch on his left eye. Which of the following statements is true?

a. Trauma is the likely cause.


b. Underlying hypertension is the likely cause.
c. An unrecognized bleeding disorder is the likely cause.
d. Increased intrathoracic pressure is the likely cause.
e. Referral to an ophthalmologist is indicated.

The answer is d.
The patient in this question has a subconjunctival hemorrhage. In this condition, the redness of
the eye is localized and sharply circumscribed, with the underlying sclera not being visible.
There is generally no discharge, no pain, and no vision disturbance. These hemorrhages can be
spontaneous, but can also arise from trauma, hypertension, bleeding disorders, or from
straining (as with severe coughing, retching, or straining during defecation). In this case, the
intrathoracic pressure increase from patient’s vomiting is the likely cause. Referral to an
ophthalmologist is only indicated if the hemorrhage is from trauma, or if it does not
spontaneously resolve in 2 to 3 weeks. If there are recurrent episodes, consideration of other
causes should be included in the differential.
ABFM2019
Q1-A 30-year-old female sees you for a routine health maintenance visit. She has myopia and
says she is considering LASIK eye surgery and wants your advice . You tell her that LASIK is
associated with
A. A moderate reduction in problems with glare
B. A moderate reduction in problems with dry eyes
C. Satisfactory improvement of vision in almost all patients
D. Prevention of presbyopia

ANSWER: C
LASIK corrective vision surgery has become increasingly common over the last 20 years. A
laser is used to cut a flap the size of a contact lens consisting of corneal epithelium and stroma.
This flap is repositioned and heals without sutures . It is important to counsel patients on
realistic expectations. Vision following the procedure may not be as clear as with glasses or
contact lenses and some individuals still require external correction. Up to 40% of patients
experience dry eyes following the surgery (SOR B). These symptoms may be worse in patients
with chronic pain syndromes such as fibromyalgia, migraine, and irritable bowel syndrome
(SOR C). Glares, halos, and starbursts may affect up to 20% of patients following LASIK. This
may be especially bothersome at night (SOR B ). LASIK does not correct age-related
presbyopia (SOR C). Reading glasses may be necessary if this develops in certain patients.
Overall, however, most patients are satisfied with their results and only 3% are unhappy with
their vision following surgery (SOR C).

ABFM2018
Q2-A 62-year-old Asian female presents to your office with pain and redness in her left eye
that started last night. She does not wear contact lenses. The pain has become more severe and
she now has a headache, light sensitivity, and mild nausea. Examination of the eyes reveals
diffuse conjunctival injection on the left. Her pupils are 4 mm bilaterally but the left one reacts
poorly to light. Her visual acuity is 20/30 on the right and 20/100 on the left.
Which one of the following would be most appropriate at this time?

A) Polymyxin B/trimethoprim ophthalmic drops (Polytrim)


B) Prednisolone ophthalmic drops (Omnipred)
C) An erythrocyte sedimentation rate and C-reactive protein level
D) MRI of the brain with contrast
E) Emergent evaluation by an ophthalmologist
ANSWER: E
This patient has symptoms and examination findings that are concerning for acute angle-
closure glaucoma. Her risk factors include her age, sex, and Asian ancestry. The examination
findings include conjunctival redness, corneal edema, a poorly reactive mid-dilated pupil,
decreased vision, severe eye pain, headache, and nausea. This condition needs to be evaluated
and treated emergently to preserve vision. The examination is not consistent with infectious
conjunctivitis, which generally does not cause severe pain, headache, or decreased pupillary
response. Conditions such as scleritis or episcleritis may present with similar features, but the
pupillary response may help differentiate them from glaucoma. Referral to an ophthalmologist
would still be most prudent. This patient’s presentation is not consistent with a vasculitis or
multiple sclerosis.

ABFM2017
Q3- When evaluating a 52-year-old farmer for complaints of possible visual loss, headaches,
and a suspected corneal lesion, you note the funduscopic findings shown on the following
page. These findings are highly suggestive of
A) optic neuritis
B) glaucoma
C) papilledema
D) optic atrophy
E) rubeosis

ANSWER: B
This funduscopic picture shows a 70% ―cupping‖ or abnormal enlargement of the light-
colored area around the central vascular area in the disc, which is a phenomenon that highly
suggests increased intraocular pressure or glaucoma. Primary care physicians, especially
family physicians, who often examine the eyes for other symptoms must be aware of these
early changes in order to allow patients to obtain treatment to reverse or ameliorate the disease
process
ABFM2016
Q4- A 63-year-old female presents with a complaint of a painful, red right eye. She says that
her vision has also been blurry. She has also noted a discharge. The patient wears contact
lenses. When you examine the patient you note photophobia. The pupils are equal, round, and
reactive to light. You also note unilateral diffuse injection. Fluorescein staining reveals focal
corneal uptake. Which one of the following is the most likely diagnosis?

A) Corneal abrasion
B) Subconjunctival hemorrhage
C) Uveitis
D) Acute angle-closure glaucoma
E) Herpes zoster ophthalmicus

ANSWER: A
Common causes of red eye include infectious conjunctivitis, allergies, corneal abrasion,
keratoconjunctivitis, subconjunctival hemorrhage, uveitis, blepharitis, iritis, acute angle-closure
glaucoma, and herpes zoster ophthalmicus. Viral infections typically cause conjunctivitis with
mild pain and no loss of vision. The problem is usually unilateral in the beginning and a watery
to serous discharge may be noted. Adenovirus is the most common cause. Acute bacterial
conjunctivitis has a similar presentation and may include eyelid edema and a purulent
discharge. Allergic conjunctivitis is usually bilateral and painless, with intense itching, and a
stringy or ropy watery discharge.
Herpes zoster ophthalmicus is associated with a vesicular rash, keratitis, and uveitis. The rash
is preceded by pain and a tingling sensation. Findings include conjunctivitis and dermatomal
involvement, which are usually unilateral.With corneal abrasion there is usually a history of an
injury involving a foreign object. Signs and symptoms include severe eye pain; red, watery
eyes; photophobia; and a foreign body sensation. Vision is usually normal and pupils are equal
and reactive to light. Symptoms of uveitis include a red eye, loss of vision, and photophobia. It
is associated with many autoimmune diseases, including reactive arthritis, ankylosing
spondylitis, and inflammatory bowel disease. Acute angle-closure glaucoma causes a
significant loss of vision, with dilated pupils that don’t react normally to light. Symptoms
include severe pain and watery eyes, with halos around lights. Patients may have nausea and
vomiting. This form of glaucoma often has an acute onset.
ABFM2015
Q5-You see a 4-year-old male in your office for evaluation of persisting fever, rash, and red
eyes. In a discussion with his father you learn that the child has had temperatures in the 99°F–
102°F range for 6 days, along with what the father describes as “pink eye.” Today the child
broke out in a rash on his chest and back and also has cracked red lips. On examination you
confirm that he has bilateral nonpurulent conjunctival injection and a generalized
maculopapular rash, as wellas erythema of his hands and feet.

Which one of the following is recommended at this time to evaluate for cardiac complications?
A) An EKG
B) Transthoracic echocardiography
C) Cardiac CT
D) Magnetic resonance (MR) coronary angiography
E) A radionuclide myocardial perfusion scan

ANSWER: B
This patient meets the criteria for Kawasaki disease, also known as mucocutaneous lymph
node syndrome. It is an acute type of vasculitis that predominantly affects small and medium-
size vessels and is the most common cause of acquired coronary artery disease in childhood.
Diagnostic criteria include fever for at least 5 days and at least 4 of the 5 principal clinical
features:
• changes of the oral cavity and lips
• polymorphous rash
• bilateral nonpurulent conjunctivitis
• changes in the extremities (erythema followed by desquamation)
• cervical lymphadenopathy
Coronary abnormalities, including coronary aneurysms, are the most concerning sequelae of
Kawasaki disease and may occur in the first week. For this reason early cardiac evaluation is
recommended, with transthoracic echocardiography being the preferred initial imaging.
Radionuclide imaging can be useful in assessing cardiac perfusion in patients found to have
persisting echocardiographic findings. MRI coronary angiography can be used to assess
response to treatment over time. Intravenous immunoglobulin and corticosteroids reduce the
risk of coronary abnormalities and should be administered as soon as the disease is suspected.
Q6-A 78-year-old female presents with a red eye. She reports drainage and pain in her left eye
sinceshe woke up today, but no photophobia. Examination of the eye shows conjunctival
erythemaand a mucopurulent discharge. The pupil is normal in size and reactive to light.
Which one of the following should prompt immediate referral to anophthalmologist?
A) Bilateral eye redness
B) A corneal abrasion noted on fluorescein staining
C) Copious mucopurulent drainage from the eye
D) Bright red blood noted under the conjunctiva
E) Reduction of visual acuity

ANSWER: E
Reduced visual acuity may be a symptom of acute angle-closure glaucoma and requires
immediate referral to an ophthalmologist. Copious mucopurulent drainage from the eye is a
sign of infectious conjunctivitis, most likely bacterial, and bilateral eye redness is typically
seen with allergic conjunctivitis. Allergic or infectious conjunctivitis and small corneal
abrasions can be managed by the family physician. Bright red blood under the conjunctiva is
consistent with a subconjunctival hemorrhage that will typically resolve without intervention.

ABFM2014
Q7-A 29-year-old female presents with redness of her left eye. She has just returned from a
summer beach vacation with her children and woke up with a red eye. Your examination
reveals a watery discharge, a hyperemic conjunctiva, and a palpable preauricular lymph node.
Her cornea is clear on fluorescein staining. Which one of the following is most appropriate for
this patient?

A) Reassurance only
B) Culture-guided antibiotic therapy
C) Quinolone eyedrops
D) Corticosteroid/antibiotic eyedrops
E) Urgent ophthalmologic referral
ANSWER: A
Viruses cause 80% of infectious conjunctivitis cases and viral conjunctivitis usually requires no
treatment. Bacterial conjunctivitis is associated with mattering and adherence of the eyelids.
Topical antibiotics reduce the duration of bacterial conjunctivitis but have no effect on viral
conjunctivitis. Allergic conjunctivitis would be more likely if the patient reported itching.
Antibiotics or corticosteroids would not be helpful in this patient and would not prevent
complications. The majority of cases of viral conjunctivitis are caused by adenoviruses, which
cause pharyngeal conjunctival fever and epidemic keratoconjunctivitis. Pharyngeal
conjunctival fever is characterized by high fever, pharyngitis, and bilateral eye inflammation.
Keratoconjunctivitis occurs in epidemics, and is associated with a watery discharge,
hyperemia, and ipsilateral lymphadenopathyin >50% of cases

Q8- A 24-year-old male presents with a 1-week history of right eye redness. He says his eye
hurts, especially with light exposure. He reports no history of trauma but recalls his 2-year-old
daughter having “pink eye” about a month ago. He has a previous history of ankylosing
spondylitis.

On examination his conjunctiva appears injected and he has a sluggishly reacting pupil. No
discharge is noted. Reduced anterior spine flexion is noted on examination of the back.
Fluoroscein staining of the cornea is negative
.
Which one of the following is the most appropriate next step to manage this patient’s eye
condition?
A) Artificial tears
B) Ocular antibiotics
C) Ocular corticosteroids
D) Oral acetazolamide
E) Ophthalmic olopatadine (Patanol
ANSWER: C
Uveitis is inflammation of the uveal tract and can affect any or all of its components, including
the iris. It is the most common extra-articular manifestation of ankylosing spondylitis (AS),
seen in up to 60% of patients with AS. Iritis presents with a painful red eye with conjunctival
injection, photophobia, and a sluggishly reacting pupil. A hazy-appearing anterior chamber
results from the iris producing an inflammatory exudate. Treatment includes topical
corticosteroids, but oral or parenteral corticosteroids and NSAIDs are also effective. Reduced
anterior spine flexion (a positive modified Schober test) results from the skeletal manifestations
of AS. A “bamboo spine” is classically seen on lumbar radiographs. Oral or ocular antibiotics,
artificial tears, ophthalmic olopatadine, and oral acetazolamide are ineffective. Ophthalmology
referral is recommended (SOR B)
Q9- A 4-year-old male sees you for pre-kindergarten screening. On corneal light reflex testing,
the light reflex in the patient’s right eye is in the center of the pupil. In the left eye it is located
below the pupil, over the inferior-lateral portion of the iris.
This clinical finding is associated with a congenital palsy of which one of the following cranial
nerves?
A)Third
B)Fourth
C)Fifth
D)Sixth
E)Seventh
ANSWER: B
In a corneal light reflex test, the patient’s attention is attracted to a target while a light is
directed at the eyes. In normally aligned eyes the light reflex will be located in the center of
each pupil. In patients with esotropia the reflex will be over the lateral portion of the iris in the
affected eye. In exotropia the light reflex is over the medial iris, in hypertropia it is over the
inferior iris, and in hypotropia it is over the superior iris. The finding observed in this child,
hypertropia, will occur with a congenital palsy involving the superior oblique muscle, which is
innervated by the fourth cranial nerve

2-A 61-year-old female presents to your office with a sudden painless loss of vision in her
right eye. Her past medical history includes both hypertension and type 2 diabetes mellitus.
Which one of the following would make you suspect retinal vein occlusion as the cause of her
sudden visual loss?
A) An afferent pupillary defect in the contralateral eye
B) Right eye redness
C) Tortuous retinal veins on funduscopic examination
D) Macular drusen on funduscopic examination

ANSWER: C
The signs of retinal vein occlusion typically include sudden painless loss of vision or distortion
of vision. Redness is not typical and should cause the clinician to suspect an alternate
diagnosis. Tortuous and dilated retinal veins are the most common finding on funduscopic
examination. Patients also often have multiple cotton-wool spots, although these are not
specific to retinal vein occlusion. An afferent pupillary defect often occurs on the affected side.
Diabetes mellitus and hypertension are both risk factors for retinal vein occlusion, increasing
the likelihood in this patient.

You might also like